k_0$ and positive integer $n$.<|endoftext|>
TITLE: Example of nef divisor with Iitaka dimension 0
QUESTION [6 upvotes]: Does anyone know examples of a numerically non trivial nef divisor with Iitaka dimension 0? (Unfortunately, this question might be trivial as right now I can't think of any effective divisors with Iitaka dim 0 other than exceptional divisors)
REPLY [4 votes]: This is not exactly what was asked, but I think it is close to the spirit of the question:
Mumford's example of a non-ample divisor that is positive on every curve gives an example of a numerically non-trivial nef divisor with Iitaka dimension $-1$.
Here is a sketch, you can find a complete proof in many places, or fill in the gaps yourself:
Let $B$ be a curve of genus at least $2$. Then there exists $\mathscr E$, a locally free sheaf on $B$ of rank $2$ and degree $0$ such that $\operatorname{Sym}^m(\mathscr E)$ is stable for all $m>0$. (Proof: HW).
Let $X=\mathbb P(\mathscr E)$ and $D$ the divisor corresponding to $\mathscr O_{\mathbb P(\mathscr E)}(1)$. Then $D\cdot C>0$ for any effective curve $C\subseteq X$. (Proof: HW. Hint: use the stability assumption).
So, $D$ is nef, in fact as nef as it can be $\overset{..}\smile$.
However, $D$ is not ample, because $D^2=0$. (Proof: $\deg\mathscr E=0$).
This is the usual reason this example is mentioned, that is, that being positive on curves is not enough for being ample. But it actually gives an example of a numerically non-trivial nef divisor that does not have any effective representative, because if $D$ was represented by an effective divisor, then it would be positive on it, which would contradict that $D^2=0$. (And, in fact in that case, $D$ would be ample. In other words, any non-ample divisor on a surface that is positive on every curve must have an empty linear system).<|endoftext|>
TITLE: Constant Martin kernel and amenability
QUESTION [8 upvotes]: Consider a finitely supported random walk on a discrete group G such that the support generates $G$ as a semigroup. The Martin kernels are then non-negative functions on the product $G \times M$ where $M$ denotes the Martin boundary of the random walk. Does anyone know an example with $G$ amenable such that there does not exist a point m in the Martin boundary for which the function $K( . , m)$ is constant $1$? And does anyone know an example of a non-amenable group for which there is an element $m$ in the Martin boundary such that the corresponding Martin kernel $K( . ,m)$ is constant $1$?
REPLY [3 votes]: This is a partial answer to your second question. If the function 1 is minimal, then every bounded harmonic function is constant. To reformulate, if there exists a point $m$ in the minimal Martin boundary such that $K(\cdot, m)$ is constant 1,then the Poisson boundary is trivial, which cannot happen if the group is non-amenable. So you have to look at non-minimal points.
EDIT (taking into account the comments of R W)
However, for finitely supported measures, in several classes of groups (such as hyperbolic groups and relatively hyperbolic groups with respect to virtually abelian subgroups), the Martin boundary is minimal.
So in particular, for such examples of groups, the answer is no. You thus have to look for groups such that the minimal boundary is not the whole Martin boundary (such as the examples given by R W).<|endoftext|>
TITLE: The multipartition of the dual of a $C_m \wr S_n$ module
QUESTION [5 upvotes]: I know that the irreducible modules of $C_m \wr S_n$ over $\mathbb{C}$ are parametrised by m-multipartitions. The parts of the multipartition are indexed by the elements of $C_m$.
My question now is: If $V$ is an irreducible module over $\mathbb{C}$ with multipartition $\underline{\lambda} = (\lambda_0, \lambda_1, ..., \lambda_{m-1})$, what is the multipartition of $V^*$?
I feel like it should be a permutation of $\underline{\lambda}$ corresponding to the map $i \mapsto m-i$, but I cannot find that statement anywhere. It should be fairly easy to prove but I have not managed to do so yet.
If the proof turns out to be too complicated, any source would suffice.
Thanks in advance!
REPLY [3 votes]: I assume that by a multipartition you mean here that $\lambda_i$ is a partition of $k_i$ and $\sum_i k_i = n$. In this case the correspondence you described is indeed the duality correspondence. I believe the easiest way to see this is via Clifford Theory: Consider the short exact sequence $$1\to C_m^n\to G\to S_n\to 1$$ where $G=C_m\wr S_n$.
Let $\zeta$ be an $m$-th root of unity and let $g$ be a generator of $C_m$. Every irreducible representation of $C_m^n$ is of the form $$\rho_{t_1,\ldots,t_n}\big((g^{a_1},\ldots,g^{a_n})\big)=\zeta^{\sum_i t_ia_i}$$
where $t_i\in \{0,\ldots, m-1\}$.
Every irreducible representation is $S_n$-conjugate to a unique representation in which $t_1\leq t_2\ldots \leq t_{n-1}$. Fix such a tuple. For $i\in \{0,\ldots, m-1\}$ write $k_i:= |\{j| t_j=i\}|$. The stabilizer of $\rho_{t_1,\ldots, t_n}$ in $S_n$ will then be isomorphic to $S_{k_0}\times\cdots\times S_{k_{m-1}}$. If $(\lambda_i,\ldots, \lambda_m)$ is a multipartition in the sense that $\lambda_i$ is a partition of $k_i$, then the corresponding tensor product of Specht modules $\mathbb{S}_{\lambda_0}\otimes\cdots\otimes \mathbb{S}_{\lambda_{m-1}}$ is an irreducible representation of $S_{k_0}\times\cdots\times S_{k_{m-1}}$. By letting $C_m^n$ act via the character $\rho_{t_1\ldots,t_n}$ on this vector space you get a representation $V$ of $$H:=C_m^n\ltimes (S_{k_1}\times\cdots\times S_{k_m}).$$ By taking the induced representation $\text{Ind}^G_H V$ you get a representation of $G$. Clifford Theory asserts that this is an irreducible representation of $G$, and that you get all the irreducible representations of $G$ this way.
Now for the duality: it holds that $$(\text{Ind}^G_H V)^* \cong \text{Ind}^G_H (V^*).$$
Since all the irreducible representations of $S_{k_0}\times\cdots\times S_{k_{m-1}}$ are self dual, you are left with the same representation of this group. However, $C_m^n$ acts now via the dual character of $\rho_{t_1,\ldots, t_n}$, which is $\rho_{m-t_1,\ldots, m-t_n}$. When you calculate the numbers $k'_i$ which correspond to this representation you will get now that $k'_i=k_{m-i}$.
So the multipartition which corresponds to the dual representation is indeed $(\lambda_0,\lambda_{m-1},\ldots, \lambda_1)$<|endoftext|>
TITLE: Ultrafilter on the ordinal $\omega^\omega$
QUESTION [7 upvotes]: For any ultrafilter $\mathcal{U}$ on $\omega$ and any finite $k$ we can construct tensor power $\mathcal{U}^{\otimes k}$ which is ultrafilter on $\omega^k$. Does there exist some natural extension of this construction for the ordinal $\omega^\omega$?
Edit: my suggestion:
$$
\mathcal{U}^{\otimes\omega}=\{B\subset\omega^\omega~|~\{k<\omega~|~B\cap\omega^k\in\mathcal{U}^{\otimes k}\}\in\mathcal{U}\}
$$
But is it good idea?
REPLY [11 votes]: The relevant general construction is the sum of a family $\{\mathcal V_i:i\in I\}$ of an indexed family of ultrafilters, with respect to an ultrafilter $\mathcal U$ on the index set $I$. If $\mathcal V_i$ is an ultrafilter on $X_i$, then the sum is the ultrafilter $\mathcal W$ on the disjoint union $\bigsqcup_{i\in I}X_i$ defined by
$$
\mathcal W=\{A:\{i\in I:A\cap X_i\in\mathcal V_i\}\in\mathcal U\}.
$$
In your situation, taking $\mathcal V_i$ to be $\mathcal U^{\otimes i}$, you get a sum ultrafilter on $\bigsqcup_{i\in\omega}\omega^i$, which can be identified with the ordinal $\omega^\omega$ to produce the ultrafilter $\mathcal U^{\otimes\omega}$ in the question.<|endoftext|>
TITLE: What is the largest possible probability that a random matrix over $\mathbb{F}_2$ is non-singular?
QUESTION [7 upvotes]: Suppose $A(p, n)=(a_{ij}(p))_{i, j \leq n}$ is an $n\times n$ random matrix over $\mathbb{F_2}$, with all its entries being i.i.d. and such that $P(a_{ij}(p) = 1) = p$, where $p$ is some real number from $[0; 1]$. What is the largest possible probability, that $A(p, n)$ is non-singular and with what $p$ is it reached?
Note, that $A(p, n)$ is non-singular iff $\det(A(p, n)) = 1$.
Solution for $n=1$:
$\det(A(p, 1)) = 1$ with probability $p$. The maximum of $\det(A(p, 1))$ is $1$ and it is reached with $p = 1$.
Solution for $n = 2$:
$\det(A(p, 2)) = 1$ with probability $2p^2(1 - p^2)$. The maximum of $P(\det(A(p, 2))=1)$ is $\frac{1}{2}$ and it is reached with $p = \frac{1}{\sqrt{2}}$.
However, I would like to know some sort of general formula (or at least asymptotics).
After I failed to solve this problem using determinants, I tried to prove this using the fact that a square matrix is non-singular iff its rows are linearly dependent. As there exists only one non-zero element in $\mathbb{F_2}$, we can write linear dependence of the vector system $\{v_i\}_{i \leq n}$ in $\mathbb{F}_2^n$ as $\forall S \subset \{1, ... , n\}$ such that $S \neq \emptyset$ we have $\sum_{i \in S} v_i \neq \overline{0}$. I know the probability that a given set of vectors with i.i.d. random entries Bernoulli distributed with parameter $p$ $\{v_i\}_{i \leq k}$ over $\mathbb{F}_2^n$ satisfy $\sum_{i = 1}^k v_i \neq \overline{0}$ is $(1 - \frac{p((1 - 2p)^k - 1)}{1 - 2p})$. However, I do not know how to proceed further in this direction.
This question on MSE
REPLY [8 votes]: For $n\rightarrow\infty$ the probability ${\cal P}_\infty$ that $A(p,n)$ is nonsingular becomes independent of $p\in(0,1)$, given by
$${\cal P}_\infty=\prod_{i=1}^\infty(1-2^{-i})=0.2887880951$$
See theorem 3.2 in Properties of random matrices and applications (2007).
For finite $n$, there is a result that could be instructive, which is the expectation value $E(p,n)$ of the number of linear dependencies among the rows of $A(p,n)$. This is given by theorem 4.5,
$$E(p,n)=2^{-n}\sum _{j=1}^n \binom{n}{j} \left(1+(1-2 p)^j\right)^n.$$
A plot of $E(p,n)$ as a function of $p$ becomes flatter and flatter with increasing $n$ (see below for $n$ up to 20), consistent with the understanding that for large $n$ the probability that the matrix is singular no longer depends on $p$.<|endoftext|>
TITLE: Can I build infinitely many polytopes from only finitely many prescribed facets?
QUESTION [16 upvotes]: Given a finite set of convex $d$-dimensional polytopes $\mathcal P$, for some $d\ge 2$.
Question: Is it true that there are only finitely many different convex $(d+1)$-dimensional polytopes whose facets are solely (scaled and rotated versions of) polytopes in $\mathcal P$?
Some clarifications
A face of a polytope is the intersection of the polytope with a touching hyperplane, so subdividing facets does not count here.
In general, two $(d+1)$-polytopes shall be considered as different if they differ not just in scale and orientation.
By the usual rigidity arguments, given the shape of facets and their connections, the metric of the polytope is uniquely determined. Hence, if we can built different polytopes, they will be combinatorially different as well.
Example
There are only finitely many polyhedra that can be built from any finite set of regular polygons, but as far as I know, this result is by enumeration (see, e.g. Johnson solids).
REPLY [6 votes]: Not an answer, but an "almost counterexample" for $d=2$ that came to my mind when I saw that question. Starting with a regular dodecahedron, we can successively insert circular "belts" of hexagons, in a way that the whole polyhedron is convex at each stage. The illustrations should give an idea. The shapes of the hexagonal facets can come very close to each other (in the appropriate sense), though it is clear that there can only be so many of a precise shape.<|endoftext|>
TITLE: What is the convex hull of the quaternionic symmetries of the 3 dimensional cube?
QUESTION [10 upvotes]: It is well known that there are exactly five 3-dimensional regular convex polyhedra, known as the Platonic solids.
In 1852 the Swiss mathematician Ludwig Schlafli found that there are exactly six regular convex 4-polytopes (the generalization of
polyhedra to 4 dimensions) and that, for dimensions 5 and above, there are only three!
The six regular 4-polytopes are:
NAME VERTEXES EDGES FACES CELLS
Hypertetrahedron 5 10 10 5
Hypercube 16 32 24 8
Hyperoctahedron 8 24 32 16
24-cell 24 96 96 24
Hyperdodecahedron 600 1200 720 120
Hypericosahedron 120 720 1200 600
The easiest ones to be described are the first two:
a model for the hypertetrahedron may be obtained as the convex hull of the canonical basis in $\mathbb R^5$ (hence a
4-dimensional object), while a model for the hypercube is the Cartesian product $[0, 1]\times[0, 1]\times[0, 1]\times[0,
1]$.
As in the case of 3 dimensions, the dual of a regular 4-polytope is also a regular 4-polytope and it turns out that the six regular
4-polytopes found by Schlafli are related to each other via duality as follows.
Hypetetrahedron <-> Itself
24-cell <-> Itself
Hypercube <-> Hyperoctahedron
Hyperdodecahedron <-> Hypericosahedron
This means that one needs only describe the 24-cell and the hypericosahedron for all of them to be known. In other words:
Hypertetrahedron = convex hull of the canonical basis in 5 dimensions
Hypercube = [0,1]x[0,1]x[0,1]x[0,1]
Hyperoctahedron = dual of the hypercube
Hyperdodecahedron = dual of the hypericosahedron
24-cell ???
Hypericosahedron ???
The description of the last two 4-polytopes above may be obtained by considering the quaternions $\mathbb H$.
Viewing $\mathbb R^3$ within $\mathbb H$ via the map
$$(x,y,z)\mapsto xi+yj+zk, $$
it is well known that every quaternion
$q$, with $\Vert q\Vert=1$, gives a rotation $R_q$ on $\mathbb R^3$ via the formula
$$
R_q(v) = qvq^{-1}, \quad \forall v \in \mathbb R^3.
$$
In fact the correspondence $q\mapsto R_q$ is a two-fold covering of $SO(3)$ by the unit sphere in $\mathbb H$.
Letting $P_{20}$ be the icosahedron in $\mathbb R^3$, consider the quaternionic symmetries of $P_{20}$, which I will write as
$\mathbb {HS}(P_{20})$, defined to be the
set of all unit quaternions $q$ such that $R_q$ leaves $P_{20}$
invariant. In symbols
$$
\mathbb {HS}(P_{20}) =\{q\in \mathbb H: \Vert q\Vert=1,\ R_q(P_{20})=P_{20}\}.
$$
Well, the convex hull of $\mathbb {HS}(P_{20})$ in $\mathbb R^4$ turns out to be a model for the hypericosahedron!
Since the symmetries of a regular polyhedron are the same as the symmetries of its dual, it is clear that the symmetries
of $P_{12}$, the dodecahedron, gives nothing new: the convex hull of $\mathbb {HS}(P_{12})$ is just another model for the
hypericosahedron.
Passing to the (self dual) tetrahedron, call it $P_4$, the convex hull of $\mathbb {HS}(P_{4})$ gives a model for the
remaining 4-polytope, namely the 24-cell, completing the description of the six Schlafli's 4-polytopes.
Question: What is the convex hull of the quaternionic symmetries of the 3 dimensional cube?
If I am not mistaken, this 4-polytope has 48 vertexes and 144 edges, so it is not in Schlafli's list and hence cannot be regular.
EDIT: Yes I was mistaken about the number of edges which is in fact 336 according to M. Winter's answer below!
REPLY [14 votes]: This is the disphenoidal 288-cell, which is the dual of the bitruncated 24-cell.
This is also mentioned in the "Geometry" section of the Wikipedia article on the 288-cell.
It has 48 vertices, and 336 edges. However, 144 of these are of the shortest length, and I suppose you have counted these.
The symmetry group of the tetrahedron is "the half" of the symmetry group of the cube (as the tetrahedron is the 3-dimensional demicube).
You already know that the tetrahedral symmetries give you the 24-cell.
In the same way, the 24-cell is "the half" of the disphenoidal 288-cell: the latter is the convex hull of the union of a 24-cell and its dual (which is a 24-cell as well, but differently oriented).<|endoftext|>
TITLE: Does every f.g. group have a minimal presentation?
QUESTION [12 upvotes]: Call a group presentation $\langle X \,\|\,R \rangle$ minimal if no relator from $R$ is a consequence of the remaining relators, i.e., no $r \in R$ belongs to the normal closure of $R\setminus \{r\}$ in the free group $F(X)$.
Question: Does every finitely generated group have a minimal presentation (with $X$ finite)?
Remark: evidently every finite presentation $\langle X \,\|\,R \rangle$ has a minimal sub-presentation $\langle X \,\|\,R' \rangle$ (where $R' \subseteq R$ has the same normal closure in $F(X)$ as $R$), so the question really concerns infinitely presented groups.
It is possible to construct infinite presentations which do not have minimal sub-presentations. Indeed, let $F=F(a,b)$ be the free group on $\{a,b\}$. One can choose "sufficiently independent" elements $w_1,w_2,\dots \in F$ so that the presentation $$\langle a,b \,\|\, w_i^2,w^2_{i+1}w_i, ~i \in \mathbb{N} \rangle$$ has no minimal sub-presentation. Here $R$ consists of the elements $w_1^2,w_2^2,\dots$, and $w_2^2w_1,w_3^2w_2,\dots$.
However, the above group also has the presentation $\langle a,b \,\|\,w_1,w_2,\dots \rangle$, which may be minimal.
REPLY [9 votes]: The answer is no.
In order to see this, you may combine Theorem 3.9 and Remark 5.3 of [1]. A counter-example is given by the nilpotent-by-Abelian group $B$ of Equation (3.2). Further examples are provided by Remark 5.15.
[1] R. Bieri, Y. de Cornulier, L. Guyot and R. Strebel, "Infinite presentability of groups and condensation", 2014.<|endoftext|>
TITLE: Irreducible global Galois representation with weights 0, 1, 3?
QUESTION [7 upvotes]: Fix a prime number $p$. Can there exist a continuous irreducible representation $\mathrm{Gal}(\overline{\mathbb{Q}}/\mathbb{Q})\to \mathrm{GL}_3(\mathbb{Q}_p)$ that is unramified at almost all primes, is de Rham at $p$ and whose Hodge-Tate weights at $p$ are 0, 1 and 3?
REPLY [6 votes]: Here are two arguments for why such a representation $\rho$ cannot exist.
Automorphic argument: Fontaine and Mazur have conjectured that any irreducible $n$-dimensional geometric representation $\rho$ of $Gal(\overline{\mathbf{Q}} / \mathbf{Q})$ comes from a cuspidal automorphic representation $\pi$ of $GL_n(\mathbf{A}_{\mathbf{Q}})$, and that "local-global compatibility at $\infty$" should hold, which amounts to saying that the Archimedean component $\pi_\infty$ should be determined by the Hodge–Tate weights of $\rho$ – up to a certain explicit shift, the multiset of Hodge–Tate weights of $\rho$ is the Harish-Chandra parameter of $\pi_\infty$. However, the possibilities for the representations $\pi_\infty$ which can show up as Archimedean components of automorphic representations are pretty restricted, so $(0, 1, 3)$ isn't possible. (This is essentially the argument sketched in David Hansen's comment from 2010 that you linked to.)
Motivic argument: Fontaine and Mazur also made a (separate) conjecture that any such $\rho$ is the $p$-adic realisation of a pure motive over $\mathbf{Q}$. By the comparison isomorphism of Faltings–Tsuji relating étale and de Rham cohomology, this implies that the Hodge–Tate weights of $\rho$ give the graded pieces of a pure Hodge structure. Since a pure Hodge structure has a weight $w$ and an action of complex conjugation which switches the $(p, w-p)$ and $(w-p, p)$ parts, this means the set of weights must be symmetric around $w/2$.<|endoftext|>
TITLE: A zeta function for the Klein Four group?
QUESTION [5 upvotes]: Let $G$ be a finite group, $S \subset G$ a generating set, $|g|:=|g|_S=$ word-length with respect to $S$. Let $\phi(g,h)=|g|+|h|-|gh| \ge 0$ be the "defect-function" of $S$. The set $\mathbb{Z}\times G$ builds a group for the following operation:
$$(a,g) \oplus (b,h) = (a+b+\phi(g,h),gh)$$
On $\mathbb{N}\times G$ is the "norm": $|(a,g)| := |a|+|g|$ additive, which means that $|a \oplus b| = |a|+|b|$. Define the multiplication with $n \in \mathbb{N_0}$ to be:
$$ n \cdot a := a \oplus a \oplus \cdots \oplus a$$
(if $n=0$ then $n \cdot a := (0,1) \in \mathbb{Z} \times G$).
A word $w := w_{n-1} w_{n-2} \cdots w_0$ is mapped to an element of $\mathbb{Z} \times G$ as follows:
$$\zeta(w) := \oplus_{i=0}^{n-1} (m^i \cdot (0,w_i))$$
where $m := \min_{g,h\in G, \phi(g,h) \neq 0} \phi(g,h)$.
We let $|w|:=|\zeta(w)|$ and $w_1 \oplus w_2:=\zeta(w_1)\oplus \zeta(w_2)$
Then we have $|w_1 \oplus w_2| = |w_1|+|w_2|$.
For instance for the Klein four group $\{0,a,b,c=a+b\}$ generated by $S:=\{a,b\}$, we get sorting the words $w$ by their word-length:
$$0,a,b,c,a0,aa,ab,ac,b0,ba,bb,bc,c0,ca,cb,cc,a00,a0a,a0b,a0c$$
corresponding to the following $\mathbb{Z}\times K_4$ elements $\zeta(w)$:
$$(0,0),(0,a),(0,b),(0,c),(2,0),(2,a),(2,b),(2,c),(2,0),(2,a),(2,b),(2,c),(4,0),(4,a),(4,b),(4,c),(4,0),(4,a),(4,b),(4,c)$$
corresponding to the the following "norms" of words $|w| = |\zeta(w)|$:
$$0,1,1,2,2,3,3,4,2,3,3,4,4,5,5,6,4,5,5,6$$
Let $a_n, n\ge 0$ be the sequence of numbers generated by the Klein four group.
1) Is
$$\sum_{n=1}^\infty \frac{1}{a_n^s} = \sum_{n=1}^\infty \frac{n+1}{n^s} = \zeta(s-1) + \zeta(s)$$
where $\zeta$ denotes the Riemann zeta function?
I have checked this with SAGE math up to a certain degree and it seems plausible, however I have no idea how to prove it.
2) Is every $a_n$ the product of primes $p=a_k$ for some $k\le n$?
3) Let $\pi_{K_4}(n) = |\{ k : \text{$a_k$ is prime, $k \le n$}\}|$ be the prime counting function of the sequence. What is the approximate relationship to the usual prime counting function $\pi(n)$?
REPLY [2 votes]: Consider a word $w=w_0w_1\cdots w_{n-1}$ with each $w_i\in\{0,a,b,c\}$.
Following your notation,
$$\lvert w\rvert=\lvert\zeta(w)\rvert=\left\lvert\bigoplus_{i=0}^\infty m^i\cdot(0,w_i)\right\rvert=\sum_{i=0}^{n-1}\lvert 2^i\cdot(0,w_i)\rvert=\sum_{i=0}^{n-1} 2^i\lvert w_i\rvert$$
where $\lvert0\rvert=0$, $\lvert a\rvert=1$, $\lvert b\rvert=1$, $\lvert c\rvert=2$.
We now consider the generating function
$$F_n(x)=\sum_wx^{\lvert w\rvert}$$
where the sum is over all words $w=w_0w_1\cdots w_{n-1}$.
Then
\begin{align*}
F_n(x)&=\sum_{w_0}\sum_{w_1}\cdots\sum_{w_{n-1}}x^{|w_0|+2|w_1|+\cdots+2^{n-1}|w_{n-1}|}\\
&=\left(\sum_{w_0}x^{|w_0|}\right)\left(\sum_{w_1}x^{2|w_1|}\right)\cdots\left(\sum_{w_{n-1}}x^{2^{n-1}|w_{n-1}|}\right)\\
&=(1+2x+x^2)(1+2x^2+x^4)(1+2x^4+x^8)\cdots(1+2x^{2^{n-1}}+x^{2^n})\\
&=(1+x)^2(1+x^2)^2(1+x^4)^2\cdots(1+x^{2^{n-1}})^2\\
&=(1+x+x^2+\cdots+x^{2^n-1})^2\\
&=1+2x+\cdots+(2^n-1)x^{2^n-2}+2^nx^{2^n-1}+(2^n-1)x^{2^n}+\cdots+x^{2^{n+1}-2}.
\end{align*}
What this shows is that among the first $4^n$ terms of the sequence:
\begin{align*}
&0\text{ appears }1\text{ times},\\
&1\text{ appears }2\text{ times},\\
&2\text{ appears }3\text{ times},\\
&\cdots\\
&2^n-2\text{ appears }2^n-1\text{ times},\\
&2^n-1\text{ appears }2^n\text{ times},\\
&2^n\text{ appears }2^n-1\text{ times},\\
&\cdots\\
&2^{n+1}-4\text{ appears }3\text{ times},\\
&2^{n+1}-3\text{ appears }2\text{ times},\\
&2^{n+1}-2\text{ appears }1\text{ times}.\\
\end{align*}
This resolves question 1.
Since this seems to be purely a combinatorics question, I would not expect the second and third questions to have particularly interesting answers.
I will remark that
$$\pi_{K_4}(4^n)=\sum_{p\leq2^{n+1}-2}(2^n-\lvert p-2^n+1\rvert)p$$
so you could use some analytic number theory to get asymptotics for $\pi_{K_4}(n)$ if you wanted.<|endoftext|>
TITLE: How to properly verify that $E\times E'$ has no non-trivial effective divisors with Kodaira dimension zero
QUESTION [5 upvotes]: Let $E$ and $E'$ be elliptic curves over $\mathbb{C}$. I am pretty confident that the only effective divisor $D\subset E\times E'$ with Kodaira dimension zero is the trivial divisor.
How to prove this in a simple manner?
REPLY [7 votes]: If $D$ has Iitaka dimension zero, then $\dim H^0 ( n E) =1$ for all $n$, because if it were any larger than $\dim H^0(k ne) \geq k+1$.
If we have an automorphism $\sigma$ with $[ n \sigma (D)] = [n D]$ in the divisor class group, we would have two linearly independent sections unless in fact $\sigma(D)= D$.
For $\sigma$ translation by an $n$-torsion point, for any $D$, $\sigma(D)- D$ is $n$-torsion, and so $[n \sigma(D) ] =[nD]$. (To see this, one can use the exponential exact sequence $H^1( A, \mathcal O_X) \to H^1(A, \mathcal O_X^\times) \to H^2(A, \mathbb Z)$. Translation acts trivially on the first and last terms, meaning that $\sigma (\sigma(D)-D) =\sigma(D)-D$ and thus $\sigma^n=1$ implies $\sigma(D)- D$ is $n$-torsion.)
So to have Kodaira dimension $0$, $D$ would have to be invariant under translation by $n$-torsion for all $n$. Thus, it must be either empty or Zariski dense, and therefore it is empty.<|endoftext|>
TITLE: functors $\text{Vect} \to \text{Vect}$ that preserve filtered and sifted colimits
QUESTION [9 upvotes]: I'm considering various functors from the category $\text{Vect}$ of real vector spaces to itself, and would like to know that they preserve filtered colimits and possibly even sifted colimits. The functors I'm interested in send $V$ to the power $V^k$, the tensor power $V^{\otimes k}$, the exterior power $\Lambda^k(V)$, and the free vector space $F(V)$ on the underlying set of $V$. I have proved by hand that some of these preserve filtered colimits or sifted colimits, but I'm looking for references and/or conceptual arguments.
Of course, I'd also like to know if some of these don't preserve filtered and/or sifted colimits.
(I doubt it matters that I'm working over the reals.)
REPLY [9 votes]: Maybe the following tools can help.
$G: \mathcal{C}_1 \times \cdots\times \mathcal{C}_k\to \mathcal{D}$ preserves sifted colimits separately in each variable if and only if it preserves sifted colimits. Indeed, given a diagram $p: K \to \mathcal{C}_1\times\cdots \times \mathcal{C}_k$ with $K$ sifted, observe that $p= (q_i)\circ \delta$ where $(q_i): \prod_{i=1}^kK \to \prod_{i=1}^k\mathcal{C}_i$ is the product of $q_i=\mathrm{proj}_i\circ p$ and $\delta$ is the diagonal. Since $K$ is sifted, $\delta$ is final, and the colimit may be computed over $K^{\times k}$ instead of over $K$. But then we can apply the hypothesis $k$ times to see that this colimit is preserves by $G$.
The diagonal map $\mathcal{C} \to \mathcal{C}^{\times j}$ preserves all colimits.
The forgetful functor $\mathbf{Vect} \to \mathbf{Set}$ preserves all sifted colimits (since it preserves filtered colimits and reflexive coequalizers by inspection).
The free vector space functor preserves all colimits, being a left adjoint.
Combining (4) and (3) gives the "F(V)" example you wanted, so let's move on to the others.
Notice that the tensor power functor $V \mapsto V^{\otimes n}$ is a composite of the diagonal and the tensor product functor, which preserves all colimits separately in each variable, so it preserves sifted colimits. Moreover, there is an action of $\Sigma_n$ on $V^{\otimes n}$ so this functor refines to $\mathbf{Vect} \to \mathbf{Fun}(\mathrm{B}\Sigma_n, \mathbf{Vect})$. This also preserves sifted colimits since colimits in functor categories are computed pointwise.
It's also true that the product $V \mapsto V^{\times n}$, as a functor to vector spaces with a $\Sigma_n$ action, preserves sifted colimits, since the cartesian product of sets preserves all colimits of sets in each variable, and sifted colimits of vector spaces may as well be computed on the underlying set, so the same reasoning applies (even though the cartesian product functor does not preserve all colimits of vector spaces separately in each variable).
So now, given a functor $\mathbf{Fun}(\mathrm{B}\Sigma_n, \mathbf{Vect})\to \mathbf{Vect}$ that preserves sifted colimits, we can apply it to the tensor power functor to get new functors. For example, we can take $\Sigma_n$-coinvariants (i.e. symmetric powers). We can tensor with the sign representation and then take $\Sigma_n$-coinvariants (which gives the exterior power). Any Schur functor works too, since we're just tensoring over $\mathbb{C}[\Sigma_n]$ (or $\mathbb{R}[\Sigma_n]$ over the reals) with the corresponding representation, and that preserves colimits.<|endoftext|>
TITLE: Quirky, non-rigorous, yet inspiring, literature in mathematics
QUESTION [18 upvotes]: In contrast with such lucid, pedagogical, inspiring books such as Visualizing Complex Analysis by Needham and Introduction to Applied Mathematics by Strang, I've had the pleasure of coming across the non-rigorous, thought-provoking/stimulating, somewhat quirky works of Heaviside on operational calculus and divergent series; of Ramanujan on series, in particular, his use of his master theorem/formula (as explicated by Hardy); and the relatively unknown posthumous notes of Bernard Friedman on distributions and symbolic/operational calculus Lectures on Applications-Oriented Mathematics.
These works just blindside you. You think, "What the hey?" and slowly they grow on you and you start to understand them after further research using other texts, translating the terminology/concepts, and working out details. You're left with a deeper understanding and appreciation of the originality and applicability of the work--much like Hardy professed the day after he received Ramanujan's letter, no doubt.
(Friedman's Applied Mathematics, in contrast, is of a very different nature and an immediately enlightening intro to its topics.)
Any similar experiences with other mathematical works?
(This question is not research-level per se and may be more appropriate for MSE, but certainly the works cited have inspired and continue to inspire much advanced research into the related topics, and the question falls in a similar category to MO-Q1 and the MO-Qs that pop up in the Related section of that question).
REPLY [3 votes]: I enjoyed reading the book 'Quantization, Classical and Quantum Field Theory and Theta Functions' by Andrej Tyurin very much. It is certainly not rigorous but it was very inspiring for me.<|endoftext|>
TITLE: Reference request: proof of Ramanujan's Cos/Cosh Identity
QUESTION [10 upvotes]: The Ramanujan Cos/Cosh Identity, as stated here, is
$$\left[1+2\sum_{n=1}^{\infty}\frac{\cos n\theta}{\cosh n\pi}\right]^{-2}+
\left[1+2\sum_{n=1}^{\infty}\frac{\cosh n\theta}{\cosh n\pi}\right]^{-2}=
\frac{2\Gamma^4\left(\frac34\right)}{\pi}.$$
I am looking for a proof, preferably from a reputable source. I hoped I would find something in Ramanujan's Notebooks, but have so far found no mention of it.
REPLY [24 votes]: I expand my comment into an answer.
The key here is the Fourier series for the elliptic function $\operatorname {dn} (u, k) $ given as $$\operatorname {dn} (u, k) =\frac{\pi} {2K}\left(1+4\sum_{n=1}^{\infty} \frac{q^n} {1+q^{2n}}\cos\left(\frac{n\pi u} {K} \right) \right) $$ where $K$ is the complete elliptic integral of first kind corresponding to modulus $k$ and $q=\exp(-\pi K'/K) $ is the nome corresponding to modulus $k$.
Let $\theta=\pi u/K$ then we have $$\operatorname {dn} \left(\frac{K\theta} {\pi}, k\right) =\frac{\pi} {2K}\left(1+2\sum_{n=1}^{\infty}\frac{\cos n\theta} {\cosh (n\pi K'/K)} \right) $$ Putting $K'=K$ so that $k=1/\sqrt{2}$ and $K=\dfrac{\Gamma^2(1/4)}{4\sqrt{\pi}}$ we get $$1+2\sum_{n=1}^{\infty}\frac{\cos n\theta} {\cosh n\pi} =\frac{2K}{\pi}\operatorname {dn} \left(\frac{K\theta} {\pi}, \frac{1}{\sqrt{2}}\right)$$ Let the above expression be denote by $A$ and the expression obtained from it by replacing $\theta$ with $i\theta$ be denoted by $B$. Then we have to show that $$\frac{1}{A^2}+\frac{1}{B^2}=\frac{2\Gamma^{4}(3/4)}{\pi}$$ Note that we have $$\Gamma(1/4)\Gamma (3/4)=\sqrt{2}\pi$$ and therefore $$\frac{2K} {\pi} =\frac{\sqrt{\pi}}{\Gamma ^2(3/4)}$$ and we have then $$A^{-2}+B^{-2}=\frac{\Gamma ^{4}(3/4)}{\pi}\left(\operatorname {dn} ^{-2}\left(\frac{K\theta}{\pi}\right)+\operatorname {dn} ^{-2}\left(\frac{iK\theta}{\pi}\right)\right)$$ The expression in parentheses is easily seen to be $2$ if we note that $$\operatorname {dn} (iu, k) =\frac{\operatorname {dn} (u, k')} {\operatorname {cn} (u, k')} $$ and here $k=k'=1/\sqrt{2}$.<|endoftext|>
TITLE: Is the lexicographic ordering on the unit square perfectly normal?
QUESTION [6 upvotes]: It's known that order topology is completely normal, so the lexicographic ordering on the unit square is also completely normal. It's also known that the lexicographic ordering on the unit square is not metrizable. I am interested in whether it is perfectly normal. (A space is perfectly normal if for any two disjoint nonempty closed subsets, there is a continuous function $f$ to $[0,1]$ that "precisely separates" the two sets, meaning that the two closed sets are $f^{-1}(0)$ and $f^{-1}(1)$.) And how do we prove it?
REPLY [3 votes]: For a compact Hausdorff $X$ it is equivalent that $X$ is hereditarily Lindelöf or that $X$ is perfectly normal. Sketch: $X$ is hereditarily Lindelöf implies that every open set is an $F_\sigma$ (as $X$ is regular) so dually every closed set is a $G_\delta$. OTOH if $X$ is perfectly normal, every open set is an $F_\sigma$ so $\sigma$-compact and Lindelöf, making all open sets Lindelöf and $X$ hereditarily Lindelöf again.
$[0,1]^2$ in the lexicographic order topology has a discrete subset $[0,1]\times \{\frac12\}$ so is definitely not hereditarily Lindelöf, so not perfectly normal either.<|endoftext|>
TITLE: Applications of quantum representations of the mapping class group to quantum computers
QUESTION [5 upvotes]: Quantum representations of the mapping class group of a surface are certain representations constructed from the data of a TQFT and described, for example, in and 1 and 2.
The following sources 3 and 4 imply that there are applications of quantum representations of the mapping class group to quantum computing.
What are some of these applications? Which concrete mathematical questions (e.g. from low dimensional topology or quantum algebra) have applications/are relevant to quantum computing?
REPLY [3 votes]: The context is topological quantum computation, where quantum information is stored nonlocally in a physical system, so that it is protected from decoherence by local sources of noise. The nonlocal degree of freedom is a socalled non-Abelian anyon, a particle-like excitation which is described by a (2+1)-dimensional topological quantum field theory. Arxiv:1705.06206 provides a survey of the mathematics of topological quantum computing, with a list of conjectures and open problems.
I should add, as a physicist, that I am uncertain whether any of these mathematical questions are relevant in the quest to actually build and operate a quantum computer. The key challenge there is to identify a physical system that has these exotic particles. The fractional quantum Hall effect was a primary candidate for several decades, but this system has now been largely abandoned in favor of superconducting systems, where the energy gaps can be much larger (allowing for operation in a realistic temperature range). Microsoft is heavily invested in the design of a topological quantum computer using anyons in the Ising universality class (Majorana fermions). These are "trivial" from a mathematical perspective, since they only implement a Clifford algebra and do not provide access to the full unitary group. There are no realistic options for Fibonacci anyons (which would cover all unitary operations).<|endoftext|>
TITLE: $KO_*$ groups of $\mathbb{R}P^\infty$, "Snaiths" theorem for $KO$
QUESTION [5 upvotes]: I posted this question some days ago at math.stackexchange, but didn't receive an answer.
I have two questions:
I wonder whether anyone has taken the time to compute $KO_*(\mathbb{R}P^\infty)$?
The standard tools to compute these Groups in the complex case rest on the requirement for the cohomology theories $E$ to be complex orientable. Naturally, I looked up whether something like real orientable cohomology theories exist in the literature but found out that $KO$ is not real-oriented. Anyway, there is a way to "circumvent" Snaiths theorem for the spectrum $K$ if one is only interestd in the algebra of cooperations, in the sense that one can show that
$$K_*(\mathbb{C}P^\infty) \xrightarrow{i_*} K_*K$$ is an injection of rings, where $i$ is induced from the inclusion $\mathbb{C}P^\infty \simeq BU(1) \hookrightarrow BU$. In fact, one only needs to invert the Bott element $\beta$ to turn it into an isomorphism, so it is a localization. This can be concluded from
Robert M. Switzer. Algebraic topology—homotopy and homology. Classics in Mathematics. Springer-Verlag, Berlin, 2002. Reprint of the
1975 original [Springer, New York; MR0385836 (52 #6695)].
17.33, which states
$K_*K$ is generated over $\mathbb{Z}[u,u^{-1},v^{-1}]$ by the polynomials $\{p_1,p_2,\ldots\}$.
By a process reminiscent of
J. F. Adams. Stable homotopy and generalised homology. University of
Chicago Press, Chicago, Ill.-London, 1974. Chicago Lectures in Mathematics.
p. 44 we can describe the relations of the generators of $\beta_i$ of $K_*(\mathbb{C}P^\infty) = K_* \{\beta_0 , \beta_1 , \ldots \}$ such that
$$\beta_1\beta_n = n \beta_n +(n+1)\beta_{n+1}$$
and by setting
$$\binom{x}{i} = \frac{x(x-1)\cdots (x-(i-1))}{i!} \in \mathbb{Q}[x]$$
with $x:=\beta_1$ one can see that
$K_*(\mathbb{C}P^\infty)\otimes \mathbb{Q}$ is the polynomial algebra $K_* \otimes \mathbb{Q}[x]$ over $K_*\otimes \mathbb{Q} = \mathbb{Q} [t,t^{-1}]$ and $K_*(\mathbb{C}P^\infty)$ can be identified with the subalgebra of $K_* \otimes \mathbb{Q}[x]$ generated by $\binom{x}{i}$ for $i=0,1,2, \ldots$,
where we set $\binom{x}{0}=1$.
While snaiths theorem works on the spectrum level and the aforementioned result follows, I wonder whether a similar result holds in the real case, i.e.
$$ KO_*(\mathbb{R}P^\infty)[\alpha^{-1}] \cong KO_*KO$$ for some element $\alpha \in KO_*(\mathbb{R}P^\infty)$?
REPLY [15 votes]: There are many ways to do this. One elementary approach is to use the Adams spectral sequence
$Ext_A(H^*ko \wedge RP^\infty, F_2) \cong Ext_{A(1)}(H^*RP^\infty,F_2) \Rightarrow ko_*(RP^\infty) $
and invert the Bott map. An $A(1)$ resolution giving the answer (since $E_2 = E_\infty$) can be seen at the bottom of the page
http://www.rrb.wayne.edu/art/index.html
We see the groups, starting with $KO_0$, are $0, Z/2, Z/2, Z/2^\infty, 0, 0, 0, Z/2^\infty$ and repeat by Bott periodicity, with the evident action of the coefficients $KO_*$. (This action follows from the homological algebra of the $Ext$ calculation.)<|endoftext|>
TITLE: Physical interpretation of the Manifold Hypothesis
QUESTION [15 upvotes]: Motivation:
Most dimensionality reduction algorithms assume that the input data are sampled from a manifold $\mathcal{M}$ whose intrinsic dimension $d$ is much smaller than the ambient dimension $D$. In machine learning and applied mathematics circles this is typically known as the manifold hypothesis.
Empirically, this is observed to be true for many kinds of data including text data and natural images. In fact, Carlsson et al. found that the high-dimensional space of natural images has a two-dimensional embedding that is homeomorphic to the Klein bottle [1].
Question:
Might there be a sensible physical interpretation of this phenomenon? My current intuition, from the perspective of dynamical systems, is that if we can collect large amounts of data for a particular process then this process must be stable. Might there be a reason why stable physical processes would tend to have low-dimensional phase spaces?
Might there be a better physical perspective for interpreting this phenomenon?
References:
Carlsson, G., Ishkhanov, T., de Silva, V. et al. On the Local Behavior of Spaces of Natural Images. Int J Comput Vis 76, 1–12. 2008.
Charles Fefferman, Sanjoy Mitter, and Hariharan Narayanan. TESTING THE MANIFOLD HYPOTHESIS. JOURNAL OF THE AMERICAN MATHEMATICAL SOCIETY
Volume 29, Number 4, October 2016, Pages 983–1049. 2016.
Bastian Rieck, Markus Banagl, Filip Sadlo, Heike Leitte. Persistent Intersection Homology for the Analysis of Discrete Data. Arxiv. 2019.
D. Chigirev and W. Bialek, Optimal manifold
representation of data : an information theoretic approach, in Advances in Neural Information Processing Systems 16 161–168, MIT Press, Cambridge MA. 2004.
T. Roweis and L. K. Saul. Nonlinear dimensionality reduction by locally linear embedding. Science, 290(5500):2323–2326, 2000.
Henry W. Lin, Max Tegmark, and David Rolnick. Why does deep and cheap learning work so well? Arxiv. 2017.
REPLY [5 votes]: Q: Might there be a reason why stable physical processes would tend to have low-dimensional phase spaces.
Yes. One reason is physical processes have dissipation. E.g., turbulence is "known" to be chaotic dynamics on a low dimensional manifold (i.e., strange attractor) in the infinite dimensional phase space (of $L^2$ velocity fields). Even its dimension can be estimated. See for example:
Doering, Charles R., and John D. Gibbon. "Note on the Constantin-Foias-Temam attractor dimension estimate for two-dimensional turbulence." Physica D: Nonlinear Phenomena 48.2-3 (1991): 471-480.
.<|endoftext|>
TITLE: Global obstructions for being a quotient of a rank $d$ vector bundle
QUESTION [9 upvotes]: In this recent question (which now has an answer), Richard Thomas asked whether any projective $k$-scheme $X$ of (local) embedding dimension $d(X)$ can be embedded in a smooth $k$-scheme of dimension $d(X)$. If $i \colon X \hookrightarrow Y$ is such an embedding, then in particular we get a surjection $i^*\Omega_Y \twoheadrightarrow \Omega_X$. My (so far unsuccessful) strategy was to obstruct such a surjection from existing.
For a coherent sheaf $\mathscr F$, write $d_x(\mathscr F) = \dim_{\kappa(x)} \mathscr F_x \otimes_{\mathcal O_{X,x}} \kappa(x)$ and
$$d(\mathscr F) = \max \left\{d_x(\mathscr F)\ |\ x \in X\right\}.$$
Question. If $X$ is a quasi-projective $k$-scheme, and $\mathscr F$ a coherent sheaf, does there exist a surjection $\mathscr E \twoheadrightarrow \mathscr F$ from a locally free sheaf of rank $d(\mathscr F)$?
Already if $X = \mathbf A^n$ this seems false to me; for example there should exist finite modules $M$ with $d(M) = 2$ that cannot be generated by $2$ elements (here I am using the Quillen–Suslin theorem that a finite projective module on $\mathbf A^n$ is free). But I don't know so many ways to prove that something is not generated by $2$ elements, except for a local obstruction $d_x(\mathscr F) > 2$.
I think it should be possible to give a negative answer to Thomas's question along these lines, by exhibiting a finite flat cover $\pi \colon X \to \mathbf A^n$ such that $\pi_*\Omega_X$ does not admit a surjection from a vector bundle of rank $\deg(\pi) \cdot d(\Omega_X)$. A great answer would incorporate something like this, but I would already be very happy with some global obstruction to surjecting from a vector bundle of a given rank.
REPLY [12 votes]: Let me explain a simple example.
Let $C \subset \mathbb{P}^3$ be a twisted cubic curve. It is a locally complete intersection of codimension 2, hence its ideal $I_C$ is locally generated by two sections. Let me show that there are no surjections $E \twoheadrightarrow I_C$ from a locally free sheaf $E$ of rank 2.
Indeed, assume such a surjection exists. Its kernel is a reflexive sheaf of rank 1, hence is a line bundle, so we have an exact sequence
$$
0 \to L \to E \to I_C \to 0.
$$
Restricting to $C$ we obtain an exact sequence
$$
0 \to \det N^* \to L\vert_C \to E\vert_C \to N^* \to 0,
$$
where $N^*$ is the conormal bundle. But $N^*$ is locally free of rank 2, hence the surjection $E\vert_C \to N^*$ is an isomorphism, hence the middle arrow is zero, hence
$$
\det N^* \cong L\vert_C.
$$
But the adjunction fromula shows that $\det N^* \cong \mathcal{O}_C(-10)$, and this line bundle does not restrict from $\mathbb{P}^3$ (because 10 is not divisible by 3). This contradiction proves that no surjection from $E$ as above exists.
Of course, the same argument works for many other lci of codimension 2.<|endoftext|>
TITLE: Plane partitions with equal margins
QUESTION [16 upvotes]: A plane partition of $n$ is an table of integers $A=(a_{ij})$ which add up to $n$ and non-increase in rows and columns. For example,
$$A= \begin{matrix} 331 \\
32 \ \ \\
11 \ \
\end{matrix}
$$
is a plane partition of $(3+3+1)+(3+2)+(1+1)=14$.
One can view plane partitions as an arrangement of cubes stacked in the corner, see more on Wikipedia.
Define margins to be 1-dim projections of cubes on all coordinate axis. These margins are triples $(\lambda,\mu,\nu)$ of partitions of $n$. For example, for $A$ as above we have $\lambda = (7,5,2)$, $\mu=(7,6,1)$ and $\nu=(7,4,3)$.
Question: What is the smallest $n$ for which there exist two different plane partitions of $n$ with the same margins $(\lambda,\mu,\nu)$?
Note: I know there are two different plane partitions of $2100$ with equal margins. This is a consequence of $p_2(n)
TITLE: Convolution algebra associated to a finite dimensional algebra
QUESTION [5 upvotes]: Given a finite dimensional $k$-algebra $A$ (we can assume it is given by a connected quiver with relations). One can form its trivial extension $T(A)$ (see for example https://math.stackexchange.com/questions/229412/trivial-extension-of-an-algebra ), which is a Frobenius algebra and thus it has a coalgebra structure, see for example the book by Kock on Frobenius algebras and 2D topological quantum field theories.
Now given any $k$-coalgebra $C$ and $k$-algebra $A$, we can form the convolution algebra $W_{C,A}={\rm Hom}_k(C,A)$.
This gives rise to (at least) two new algebras from a given algebra $A$, namely $W_{T(A),A}$ and $W_{T(A),T(A)}$ using that $T(A)$ has a coalgebra structure as a Frobenius algebra. I have no real experience with this topic, so sorry in case my questions are stupid.
Question 1: Have either of the algebras $W_{T(A),A}$ or $W_{T(A),T(A)}$ been considered/described before? Are they quiver algebras and if so, can their quivers be described?
Question 2: What do those two algebras look like in the concrete example where $A=kQ$ is a path algebra (maybe of Dynkin type)?
Question 3: In case those questions are non-trivial, can those algebras be calculated by a computer algebra system like GAP/QPA?
REPLY [3 votes]: Question 3: The algebra constructions $W_{C,A}$ or even $W_{T(A),A}$ are not available in QPA. There are no structures made for co-algebras in QPA.<|endoftext|>
TITLE: Approximations to $\pi$
QUESTION [12 upvotes]: Is there a way to efficiently solve the following problem besides brute-force calculation?
Fix $n\in\mathbb{N}$ (say $n=100$). Find the integers $p,q,r,s$ with $0\leq p,q,r,s\leq n$ such that
$$\pm\frac{p}{q}\pm\sqrt{\frac{r}{s}}$$
most closely approximates $\pi$.
Some cases can be handled by finding the continued fraction expansion of $(\pi-\frac{p}{q})^2$ for various $p,q$. Playing with this method I found the approximation $4-\sqrt{\frac{14}{19}}$, which is really quite good, but may not be best for $n=20$. Note that the solution will be unique (for a given $n$).
REPLY [6 votes]: This is not exactly an answer to the stated question, but it's too long for a comment. Rather than the form given in the question, one could represent a number in the form $\frac{a + b \sqrt{d}}{c}$, where $d$ is a squarefree positive integer,
and this form lends itself to finding good approximations to $\pi$ using lattice reduction.
Fix a bound $n$. For each squarefree $d$ with $1 \leq d \leq n$,
choose a constant $X \approx n^{2} \sqrt{d}$ and create the lattice in $\mathbb{R}^{4}$ spanned by
$$ v_{1} = \begin{bmatrix} 1 \\ 0 \\ 0 \\ X \pi \end{bmatrix}, v_{2} = \begin{bmatrix} 0 \\ 1 \\ 0 \\ -X \end{bmatrix}, v_{3} = \begin{bmatrix} 0 \\ 0 \\ 1 \\ -X \sqrt{d} \end{bmatrix}. $$
A short vector in this lattice with respect to the $\ell_{2}$ norm is a linear combination $a v_{1} + bv_{2} + cv_{3}$ and because the fourth coordinate of these vectors are so large, this forces $\frac{a+b \sqrt{d}}{c}$ to be a close approximation to $\pi$.
Finding the shortest vector in a lattice is a hard problem, even in small dimensional lattices. However, one can get within a constant multiple of the true minimum using the LLL-algorithm. With this, the above algorithm would run in time $O(n \log^{3} n)$ and find "some good solutions", but isn't guaranteed to find the optimal representation (even in this modified form).
I ran this with $n = 10^{6}$ and $X = n^{2} \sqrt{d} \log(n)$ and obtained (after about a minute and a half)
$$
\pi \approx \frac{-327031 + 7075 \sqrt{224270}}{962406}.
$$
The approximation differs from the truth by about $8 \cdot 10^{-22}$.<|endoftext|>
TITLE: What happens to eigenvalues when edges are removed?
QUESTION [9 upvotes]: I am stuck at the following :
Let $G$ be a graph and $A$ is its adjacency matrix.
Let the eigenvalues of $A$ be $\lambda_1\le \lambda_2\leq \cdots \leq \lambda_n$.
If we remove some edges from the graph $G$ and form the graph $H$ keeping the number of vertices same, is there any result how the smallest eigenvalue of $H$ is related to the smallest eigenvalue of $G$?
I know Cauchy Interlacing Theorem which gives the relation between eigenvalues of a graph and its induced subgraph when some vertices are removed.
I want to know what happens when edges are removed keeping the number of vertices same. Can someone help please?
The question stands as:
Let $G$ be a graph and $A_G$ is its adjacency matrix.
Let the eigenvalues of $A_G$ be $\lambda_1\le \lambda_2\leq \cdots \leq \lambda_n$.
Let $H$ be a subgraph of $G$ which has $n$ vertices as $G$ but some edges have been removed from $G$ to form $H$.$A_H$ is its adjacency matrix.
Let the eigenvalues of $A_H$ be $\mu_1\le \mu_2\leq \cdots \leq \mu_n$.
Is $\mu_1\ge \lambda_1$ or $\mu_1\le \lambda_1$?
If someone can give any reference like book or paper , I will be grateful.
REPLY [15 votes]: The smallest eigenvalue can go up or down when an edge is removed.
For "down": $G=K_n$ for $n\ge 3$.
For "up": Take $K_n$ for $n\ge 1$ and append a new vertex attached to a single vertex of the original $n$ vertices. Now removing the new edge makes the smallest eigenvalue go up.
Both of these follow from the fact that the smallest eigenvalue of a connected graph with $n\ge 2$ vertices is $\le -1$ with equality iff the graph is complete.
It has something to do with whether the two corresponding eigenvector entries have the same or opposite sign, but I don't know if that relationship can be made precise.
REPLY [3 votes]: Here's a statement from the book "Spectra of Graphs" by Brouwer and Haemers concerning the largest eigenvalue of the adjacency matrix. It implies that $\lambda_n \geq \mu_n$.
Proposition 3.1.1 The value of the largest eigenvalue of a graph does not increase when vertices or edges are removed.<|endoftext|>
TITLE: Presentation of $H^2(\overline{M}_{0,n},\mathbb{Z})$ as an $S_n$-module?
QUESTION [6 upvotes]: Let $\overline{M}_{0,n}$ be the moduli space of genus zero curves with $n$ marked points. Let $I=\{\{S,S^c\}|S\subset\{1,\dots,n\},|S|\geq2,
|S^c|\geq2\}$ be the set of partitions of $\{1,\dots n\}$ into two subsets, each has at least two elements.
Keel shown that the cohomology group $H^2(\overline{M}_{0,n},\mathbb{Z})$ is generated by boundary divisor classes $\delta_{\{S,S^c\}}$, where $\{S,S^c\}\in I$.
(Here $\delta_S$ is the divisor in $\overline{M}_{0,n}$, consisting of curves with two components, marked by $S$ and $S^c$, and their further degenerations.)
Thus we have a presentation $$\mathrm0\to K\to\bigoplus_{\{S,S^c\}\in I}\mathbb{Z}\cdot\delta_{\{S,S^c\}}\to H^2(\overline{M}_{0,n},\mathbb{Z})\to 0.$$
Keel shown that the kernel $K$ is generated by equations $$\sum_{i,j\in S;k,l\notin S}\delta_{\{S,S^c\}}=\sum_{i,k\in S;j,l\notin S}\delta_{\{S,S^c\}}=\sum_{i,l\in S;k,j\notin S}\delta_{\{S,S^c\}},$$ for any four distinct elements $\{i,j,k,l\}\subset\{1,\dots,n\}$. These give $2\cdot{n\choose 4}$ such equations. But the rank of $K$ is only $\frac{n(n-3)}{2}$. ($\#I=2^{n-1}-n-1$, $\mathrm{rank}H^2(\overline{M}_{o,n},\mathbb{Z})=2^{n-1}-{n\choose2}-1$), so these relations are very much dependent..
The question is, would there be a good presentation of $K$, my goal is to calculate the group cohomology $$H^1(S_n,H^2(\overline{M}_{0,n},\mathbb{Z}))?$$
REPLY [2 votes]: This is a bit too long for a comment.
If you take the relation
$$
\sum_{i,j\in S;k,l\notin S}\delta_{\{S,S^c\}}=\sum_{i,k\in S;j,l\notin S}\delta_{\{S,S^c\}}
$$
and add to both sides $\sum\limits_{i,j,k\in S;l\notin S}\delta_{\{S,S^c\}}$, you get
$$
\sum_{i,j\in S;l\notin S}\delta_{\{S,S^c\}}=\sum_{i,k\in S;l\notin S}\delta_{\{S,S^c\}}.
$$
Of course, this is totally reversible, so you can take these elements instead. (This essentially recovers the presentation of cohomology of $\overline{M}_{0,n}$ when this space is constructed as a De Concini-Procesi wonderful model of the Coxeter hyperplane arrangement of type A.)
Now we note that for fixed $i,l$, it is enough to choose $n-3$ equations among the $(n-2)(n-3)/2$ equations indexed by different choices of $j,k$ to force all of them to hold, and the symmetric group action on those seems rather straightforward; I suspect that this would be quite useful for your purposes.
[Things would be even simpler if you could fix $l$ once and forever (that is, restrict to $S_{n-1}$ inside $S_n$), but I suspect you do not want to do that.]<|endoftext|>
TITLE: An analogue of the exponential function by replacing infinite series with improper integral
QUESTION [16 upvotes]: For every positive real number $x$ we define $$E(x)= \int_0^{\infty} x^t/t!\,\mathrm dt$$
where $t!=\Gamma(t+1)$. This is motivated by classical exponential function.
Is this function well defined (the problem of convergence)? Is there a real analytic extention of $E$ to all real numbers? What about a holomorphic extention to complex numbers? How can we compare $E(x+y)$ with $E(x)$ and $E(y)$? What kind of differential equation can be satisfied by $E$? Is $E$ one to one on real numbers?What can be said about its possible inverse?
REPLY [31 votes]: This is particular case of a classic integral studied by Ramanujan. See Chapter 11 in Hardy's book, "Ramanujan: Twelve Lectures on Subjects Suggested by His Life and Work", where it is shown that
$$
\int_{-\xi}^\infty\frac{x^t}{\Gamma(1+t)}\,dt+\int_0^\infty t^{\xi-1}e^{-xt}\left(\cos\pi \xi-\frac{\sin\pi \xi}{\pi}\ln t\right)\frac{dt}{\pi^2+\ln^2t}=e^x, \quad (x\ge 0, \xi\ge 0).
$$
From this it follows that your integral can be represented as an integral of elementary functions as follows
$$
\int_0^\infty\frac{x^t}{\Gamma(1+t)}\,dt=e^x-\int_0^\infty \frac{e^{-xt}\,dt}{t(\pi^2+\ln^2t)},\quad (x\ge 0).
$$
Also, see Fransen-Robinson constant
$$
C=\int_0^\infty\frac{dt}{\Gamma(t)}= 2.8077702420285...
$$
REPLY [19 votes]: (Some obvious properties of $E$; too long for a comment, though).
The holomorphic extension of $E$ to $\mathbb{C} \setminus (-\infty, 0]$ (in fact, to the entire Riemann surface of the complex logarithm) is given by $$E(x) = \int_0^\infty \frac{\exp(t \log x)}{\Gamma(t+1)}\, dt,$$ where $\log$ denotes the principal branch of the complex logarithm. This follows from a standard application of Morera's theorem, involving Fubini's theorem, the estimate $$|\exp(t \log x)| = \exp(t \log |x|) = |x|^t \le a^t + b^t$$ when $a \le |x| \le b$, and integrability of $(a^t + b^t) / \Gamma(t + 1)$ over $(0, \infty)$.
In particular, for $x > 0$ we have $\log(-x + 0 i) = \log x + i \pi$, and hence
$$ E(-x+0i) = \int_0^\infty e^{i \pi t} \frac{x^t}{\Gamma(t+1)} \, dt $$
is not real-valued in any neighbourhood of $0$. Thus, there is no real-analytic extension of $E$ to $(-\epsilon, \infty)$ (as already follows from Carlo Beenakker's comment).
By dominated convergence theorem, the integral can be differentiated under the integral sign, so
$$E^{(n)}(x) = \int_0^\infty \frac{x^{t - n}}{\Gamma(t+1 - n)}\, dt = \int_{-n}^\infty \frac{x^t}{\Gamma(t+1)}\, dt .$$
This does not seem to lead to any interesting differential equation.
Since $E'(x) > 0$, clearly $E$ is increasing on $(0, \infty)$, with $E(0) = 0$ and $E(\infty) = \infty$.
One can easily find the Laplace transform of $E(x)$: when $\operatorname{Re} \xi > 1$, we have
$$ \int_0^\infty e^{-\xi x} E(x) dx = \int_0^\infty \frac{1}{\xi^{t + 1}} \, dt = \frac{1}{\xi \log \xi} . $$
(EDIT: This was meant to be an extended comment only, but since it has received a number of upvotes, let me add further remarks, inspired by Nemo's answer.)
The Laplace transform $\mathcal{L} E$ of $E$ has a simple pole at $\xi = 1$ with residue $1$, and a branch cut along $(-\infty, 0]$. Since it decays (barely) sufficiently fast at infinity, one can (carefully) write the usual inversion formula and then deform the contour of integration to the Hankel contour to find that
$$ E(x) = e^x - \frac{1}{\pi} \int_0^\infty e^{-t x} \operatorname{Im} (\mathcal{L} E(-t + 0i)) dt .$$
This leads to the formula given in Nemo's answer: since $$\mathcal{L} E(-t + 0i) = -\frac{1}{t \log(-t + 0 i)} = -\frac{1}{t (\log t + i \pi)} \, ,$$ we obtain
$$ E(x) = e^x - \int_0^\infty \frac{e^{-t x}}{t (\pi^2 + \log^2 t)} \, dt .$$
As a consequence, $e^x - E(x)$ is completely monotone, and
$$ E(x) = e^x - \frac{1 + o(1)}{\log x} $$
as $x \to \infty$. Further terms can be obtained in a similar way.
The function $E(x)$ itself is the Mellin transform of $1 / \Gamma(t + 1)$. Thus, $1 / \Gamma(t + 1)$ can be written as the inverse Mellin transform:
$$ \frac{1}{\Gamma(t + 1)} = \frac{1}{2 \pi i} \int_{c + i \mathbb{R}} t^{-1 - x} E(x) dx , $$
or, equivalently,
$$ \frac{1}{\Gamma(t)} = \frac{1}{2 \pi i} \int_{c + i \mathbb{R}} t^{-x} E(x) dx . $$
The definition of $E(x)$ looks a little bit like Mellin–Barnes integral, but the contour is wrong.
Finally, the (fractional) integral of $E(x)$ of order $\alpha$ is given by
$$ I_\alpha E(x) = \frac{1}{\Gamma(\alpha)} \int_0^x E(t) (x - t)^{\alpha - 1} dt = \int_0^\infty \frac{x^{t + \alpha}}{\Gamma(t + 1 + \alpha)} \, dt , $$
and so
$$ I_\alpha E(x) = \int_\alpha^\infty \frac{x^t}{\Gamma(t + 1)} \, dt . $$
This agrees with the expression for the derivatives of $E$ (which correspond to negative integer $\alpha$).<|endoftext|>
TITLE: Conjectures and open problems in representation theory
QUESTION [6 upvotes]: Are there very famous open problems or conjectures in representation theory, or in enumerative geometry, like the volume conjecture in topology?
REPLY [6 votes]: The Clemens conjecture in enumerative geometry: a general quintic threefold has only finitely many rational curves in each positive degree.
REPLY [6 votes]: There are many open, and seemingly deep, conjectures in modular representation theory (or block theory) in connection with enumerating representation-theoretic invariants:
a start of a list might be : Brauer's $k(B)$-problem, the Alperin-McKay Conjecture, the Alperin Weight Conjecture, Dade's conjectures, Isaacs-Navarro conjecture. Gabriel Navarro has several recent survey papers discussing these and other conjectures.
In a different part of (modular) representation theory, with perhaps a more geometric flavor, there are problems such as the Lusztig Conjecture (now known to be false in its original formulation), and work of Geordie Williamson.
As noted by Julian Kuelshammer, Representation Theory is a vast subject, and it might be helpful to point out which specific areas you are most interested in (I only mention two facets of the subject which are most familiar to me).<|endoftext|>
TITLE: Looking for a "cute" justification for a Catalan-type generating function
QUESTION [8 upvotes]: The Catalan numbers $C_n=\frac1{n+1}\binom{2n}n$ have the generating function
$$c(x)=\frac{1-\sqrt{1-4x}}{2x}.$$
Let $a\in\mathbb{R}^+$. It seems that the following holds true
$$\frac{c(x)^a}{\sqrt{1-4x}}=\sum_{n=0}^{\infty}\binom{a+2n}nx^n.$$
QUESTION. Why?
REPLY [3 votes]: Let $C_a(x)=\frac{c(x)^a}{\sqrt{1-4x}}$ and $B_a(x) =\sum_{n=0}^{\infty}\binom{a+2n}nx^n.$
The identity
$c(x)=1+xc(x)^2$ implies $C_{a+1}(x)= C_{a}(x)+x C_{a+2}(x).$
The recursion for the binomial coefficients implies
$B_{a+1}(x)= B_{a}(x)+x B_{a+2}(x)$.
If we show that $B_a(x)=C_a(x)$ holds for $a=1$ then it holds for all positive integers.
This follows from $B_1(x)=\frac{1}{2} \sum_{n=0}^{\infty}\binom{2+2n}{n+1}x^n=
\frac{1}{2x}(B_0(x)-1)=C_1(x).$<|endoftext|>
TITLE: Algorithm for computing external angles for the Mandelbrot set
QUESTION [12 upvotes]: Let $M$ be the Mandelbrot set: there exists a unique series
$$
\psi(z) := z + \sum_{m=0}^{+\infty} b_m z^{-m} = z - \frac{1}{2} + \frac{1}{8} z^{-1} - \frac{1}{4} z^{-2} + \cdots
$$
which defines a conformal bijection between the complement $\{z\in\mathbb{C} : |z|>1\}$ of the closed unit disk in $\mathbb{C}$ and the complement of the Mandelbrot set. This function $\psi$ is sometimes known as the “Jungreis function”, see the answers to this question for more. The argument $\arg(\psi^{-1}(w))$ for a point $w\not\in M$ is called the external angle of $w$.
There exists an easy way to “almost” compute $\arg(\psi^{-1}(w))$, or $\psi^{-1}(w)$ itself for that matter: indeed, if we let $p_0(w) := w$ and $p_{i+1}(w) := p_i(w)^2 + w$, then
$$
p_n(\psi(z)) = z^{2^n} + o(1)
$$
as $z\to\infty$, so $\psi^{-1}(w)$ can be “almost” computed as the limit of the $(2^n)$-th root of $p_n(w)$ as $n\to+\infty$. The reason for the “almost” is that, while this indeed allows for computation of the modulus $|\psi^{-1}(w)|$, it leads to an indetermination between $2^n$ values on the argument. The formula
$$
\psi^{-1}(w) = w \mskip3mu \prod_{n=1}^{+\infty} \left(1 + \frac{w}{p_{n-1}(w)^2}\right)^{1/2^n}
$$
is no better (it also requires computing $(2^n)$-th roots and one cannot simply take the principal determination; my understanding is that one needs to find a determination of $\Big(1 + \frac{w}{p_{n-1}(w)^2}\Big)^{1/2^n}$ that is continuous outside of $M$, which seems computationally intractable).
So, is there a way to lift this square root indeterminacy and compute external arguments for arbitrary $w\not\in M$? Is there an algorithm that does this in a reasonably efficient way (which excludes, e.g., trying to trace external rays outwards towards infinity)?
I was unable to find anything relevant in the literature. There is a 1986 paper by Douady titled “Algorithms for computing angles in the Mandelbrot set” which seems promising, but it seems to concerns the computation for points of $M$, not points outside $M$. This web page about the Mandel program actually discusses the issue (in the section called “Computation of the external argument”), but the description is vague (e.g., where it speaks of a “modified” function $\arg(z/(z-c))$), and the conclusion that “the discontinuities are moved closer to the Mandelbrot set” is not too promising.
REPLY [5 votes]: I have implemented some algorithms based on Kawahira's paper, which as presented goes $\theta \to c \not\in M$, but can be adapted to go $c \to \theta$. $\theta$ is conveniently expressed in turns as a binary expansion. When tracing inwards, one peels off the most-significant bit (aka angle doubling) each time the ray crosses a dwell band (integer part of normalized iteration count increases by 1). The trick when tracing outwards is to prepend bits when crossing dwell bands, depending if the outer cell was entered from its left or right inner cell. A picture may make it clearer:
The exterior grid is generated from the fractional part of the smoothed iteration count, and the argument of the final (first to escape) $z$ iterate. One can see that approaching the $\frac{1}{2}$ bond point, the cells alternate left/right corresponding to the binary expansion $.(01)$ or $.(10)$. The argument of the first iterate to escape (typically floating point), within the cell of the starting point, can be used to get a few more least significant bits for the accumulated angle, but the prefix is found by accumulating bits one by one when tracing the ray outwards.
Source code:
m_d_exray_in.c $\theta \to c$, machine double precision
m_r_exray_in.c $\theta \to c$ in arbitrary (dynamically changed as necessary) precision
m_d_exray_out.c $c \to \theta$, machine double precision
m_r_exray_out.c $c \to \theta$, arbitrary (but fixed, dynamic is possible but still TODO) precision
m-exray-in.c command line driver program showing usage of the library functions
m-exray-out.c command line driver program showing usage of the library functions
git clone https://code.mathr.co.uk/mandelbrot-numerics.git
However, tracing external rays to/from dwell $n$ takes $O(n^2)$ time (even ignoring that higher $n$ needs higher precision which costs more), which may make it too slow in practice. I am also looking for faster methods since some years, but I haven't found any yet. See my related question: fast algorithms for external angle computations<|endoftext|>
TITLE: The "contrary" of an isomorphism
QUESTION [6 upvotes]: Roughly, my question is: is there a standard name for functions which one might characterise as the "contrary" of an isomorphism?
Here is a more precise version of my question. Working model-theoretically, consider the following definition.
Definition. Let $\mathscr{L}$ be a relational signature, and let $\mathcal{A}, \mathcal{B}$ be $\mathscr{L}$-structures. An $\mathscr{L}$-anti-embedding $\pi : A \longrightarrow B$ is any injection such that:
$(a_1, \ldots, a_n) \in R^\mathcal{A}$ iff $(\pi(a_1), \ldots, \pi(a_n)) \notin R^\mathcal{B}$, for each $n$-place predicate $R \in \mathscr{L}$ and all $a_1, \ldots, a_n \in A$
A bijective $\mathscr{L}$-anti-embedding is an $\mathscr{L}$-anti-isomorphism. An $\mathscr{L}$-anti-isomorphism from a structure to itself is an $\mathscr{L}$-anti-automorphism.
My question is: are there standard names for the kinds of functions I just defined? Indeed, are they discussed anywhere?
I have only encountered one such function "in the wild". I was reading Thomas Forster on Church-Oswald set theory; he called his $\{\in\}$-anti-automorphism an "antimorphism". (NB that, in the Church-Oswald setting, self-membered sets are perfectly ok.)
Update: Googling the phrase "anti-isomorphism" teaches me that, in group theory, an anti-isomorphism is standardly defined as a bijection $\pi : \mathcal{G} \longrightarrow \mathcal{H}$ such that $\pi(x \cdot^\mathcal{G} y) = \pi(y) \cdot^\mathcal{H} \pi(x)$. That's obviously a totally different idea. So I should use a different name! If there is no standard name, I welcome suggestions!
REPLY [4 votes]: Tim has drawn my attention to this (Thank you, Tim!) Perhaps i should provide what Old Lags do in this kind of setting, namely provide some ancient history.
The context is Quine's NF. Let us say the dual of a formula of the language of set theory is the result of replacing all occurrences of $\in$ by `$\not\in$'. It's routine to show that the dual of any axiom of NF is a theorem of NF. Thus if we take $<$M,$\in>$ a model of NF and consider M equipped with the complement (in MxM) of the membership relation we get another model of NF! Are these two structures elementarily equivalent? Not reliably! Might they be isomorphic? If they are, then the isomorphism is an antimorphism. Of course - since this is NF - there is the possibility that (not only might there be an antimorphism but) the antimorphism is a set of the model! It's unknown if this can happen. The best i can do is show that every model of NF is elementarily equivalent (with respect to stratifiable formul{\ae}) to one with two (set) permutations $\sigma$ and $\tau$ satisfying
both
$$(\forall x y)(x \in y \leftrightarrow \sigma(x) \not\in \tau(y))$$
and
$$(\forall x y)(x \in y \leftrightarrow \tau(x) \not\in \sigma(y))$$
Arranging for $\sigma = \tau$ is beyond me at the moment. The word
`antimorphism' is a coinage of your humble correspondent, tho' i can't give you chapter and verse.
Sorry if some readers find this perhaps off-piste, but this is the context of Tim's question, and might be helpful.
Thomas Forster www.dpmms.cam.ac.uk/~tf<|endoftext|>
TITLE: Index of Dirac operator and Chern character of symmetric product twisting bundle
QUESTION [10 upvotes]: I am having trouble understanding a couple of lines of computation from Theorem 13.30 in Besse's Einstein Manifolds text
We are twisting the spinor bundle (on Einstein 4-manifold) $\Sigma$ with an auxiliary bundle $S^3\Sigma^-$. We form the Dirac operator $\mathscr{D}^D$ formed by twisting the Levi-Civita connection on $\Sigma$ with a copy $D$ acting on $S^3\Sigma^-$ (and composing with the Clifford action acting trivially on $S^3\Sigma$). Besse evaluates the index of this operator using the APS theorem as an integral over Chern and Pontrjagin classes.
I understand that the $(1-\frac{1}{24}p_1)$ terms comes from the $\widehat{A}$-genus of the manifold, and further the signature theorem, $\tau=\frac13 \int_M p_1(M) $. However, I do not understand the evaluation of the Chern character of the twisting bundle as $(4-10c_2)$ and the subsequent evaluation in terms of Euler characteristic and signature.
Any insight would be greatly appreciated.
Edit: I am still not sure about the final calculation relating the index to the Euler character and signature, however, working backwards it seems we require $c_2(\Sigma^-)=\frac12e(M)-\frac14p_1(M)$ (or perhaps something a bit different if there are lower degree terms which could multiply with with the $\frac{1}{24}p_1$ term from the $\widehat{A}-$genus), where $e(M)$ is the Euler class of $M$.
Edit#2: Solved by Michael Albanese.
REPLY [9 votes]: Your first question can be answered by using the splitting principle.
If $V \to X$ is a complex vector bundle of rank two, then $c_1(S^3V) = 6c_1(V)$ and $c_2(S^3V) = 11c_1(V)^2 + 10c_2(V)$.
Proof: By the splitting principle, there is a map $p : Y \to X$ such that $p^*$ is injective on integral cohomology and $p^*V \cong L_1\oplus L_2$, so $p^*(S^3V) \cong S^3(p^*V) \cong S^3(L_1\oplus L_2)$. In general, we have $S^n(E_1\oplus E_2) \cong \bigoplus_{i+j=n} S^i(E_1)\otimes S^j(E_2)$, so
\begin{align*}
&\, S^3(L_1\oplus L_2)\\
\cong&\, S^3(L_1)\otimes S^0(L_2)\oplus S^2(L_1)\otimes S^1(L_2)\oplus S^1(L_1)\otimes S^2(L_2)\oplus S^0(L_1)\otimes S^3(L_2)\\
\cong&\, L_1^3\oplus L_1^2\otimes L_2\oplus L_1\otimes L_2^2\oplus L_2^3.
\end{align*}
It follows that $c_1(S^3(L_1\oplus L_2)) = 6c_1(L_1) + 6c_1(L_2) = 6c_1(L_1\oplus L_2)$. So $$p^*c_1(S^3V) = c_1(p^*S^3V) = c_1(S^3(L_1\oplus L_2)) = 6c_1(L_1\oplus L_2) = 6c_1(p^*V) = p^*(6c_1(V)).$$ By the injectivity of $p^*$, we have $c_1(S^3V) = 6c_1(V)$.
Similarly, one can compute that
\begin{align*}
c_2(S^3(L_1\oplus L_2)) &= 11c_1(L_1)^2 + 11c_1(L_2)^2 + 32c_1(L_1)c_1(L_2)\\
&= 11(c_1(L_1)+c_1(L_2))^2 + 10c_1(L_1)c_1(L_2)\\
&= 11c_1(L_1\oplus L_2)^2 + 10c_2(L_1\oplus L_2)
\end{align*}
and hence $c_2(S^3V) = 11c_1(V)^2 + 10c_2(V)$. $\quad\square$
In this case, $\Sigma^-$ is an $SU(2)$ bundle and hence $c_1(\Sigma^-) = 0$. So we see that $c_1(S^3\Sigma^-) = 0$ and $c_2(S^3\Sigma^-) = 10c_2(\Sigma^-)$. Therefore
$$\operatorname{ch}(S^3\Sigma^-) = \operatorname{rank}(S^3\Sigma^-) + c_1(S^3\Sigma^-) + \frac{1}{2}(c_1(S^3\Sigma^-)^2 - 2c_2(S^3\Sigma^-)) = 4 - 10c_2(\Sigma^-).$$
To find $c_2(\Sigma^-)$, or $c_2(\Sigma^+)$, we can proceed as follows.
As $\Sigma^{\pm}$ is an $SU(2)$-bundle which is a lift of the $SO(3)$-bundle $\Lambda^{\pm}$, there is a relationship between $c_2(\Sigma^{\pm})$ and $p_1(\Lambda^{\pm})$, namely $p_1(\Lambda^{\pm}) = -4c_2(\Sigma^{\pm})$; see Appendix E of Instantons and Four-Manifolds by Freed and Uhlenbeck, also this question. So now we just need to know $p_1(\Lambda^{\pm})$, but this is given by $\pm 2e(M) + p_1(M)$; see Chapter $6$, Proposition $5.4$ of Metric Structures in Differential Geometry by Walschap. Therefore
$$c_2(\Sigma^{\pm}) = \mp\frac{1}{2}e(M) - \frac{1}{4}p_1(M).$$
Note, as $M$ is assumed to be spin, its signature is a multiple of $16$ by Rohklin's Theorem, so $\frac{1}{4}p_1(M)$ is an integral class. As the signature of $M$ is even, so is the Euler characteristic, and hence $\frac{1}{2}e(M)$ is also an integral class.
Finally, as $c_2(\Sigma^-) = \frac{1}{2}e(M) - \frac{1}{4}p_1(M)$, we see that
\begin{align*}
\int_M(10c_2(\Sigma^-) - 4)\left(1 - \frac{1}{24}p_1(M)\right) &= \int_M 10c_2(\Sigma^-) + \frac{1}{6}p_1(M)\\
&= \int_M 5e(M) - \frac{5}{2}p_1(M) + \frac{1}{6}p_1(M)\\
&= \int_M 5e(M) - \frac{7}{3}p_1(M)\\
&= 5\chi(M) - 7\tau(M)
\end{align*}
as claimed.<|endoftext|>
TITLE: Existence of a multiplication bifunctor for the category of groups
QUESTION [5 upvotes]: For $\mathsf{Grp}$ the category of groups, a bifunctor $M: \mathsf{Grp} \times \mathsf{Grp}\to \mathsf{Grp}$ is a multiplication bifunctor if:
$M(C_n,C_m) \simeq C_{nm}$,
$M(C_1,G) \simeq M(G,C_1) \simeq G$,
for every group $G$ and every $n,m>0$, with $C_n$ the cyclic group of $n$ elements.
Question: Is there a multiplication bifunctor for the category of groups?
(or for the subcategory of countable groups, or of finite groups)
Stronger question: Is there a multiplication bifunctor providing a monoidal structure?
This post is a multiplicative analogous of that additive one.
REPLY [8 votes]: No. $C_1$ is a retract of $C_2$, so $M(C_2,C_1)\simeq C_2$ would have to be a retract of $M(C_2,C_2)\simeq C_4$, which it isn't.<|endoftext|>
TITLE: On the relation between categorification and chromatic redshift
QUESTION [11 upvotes]: In the introduction to the paper Higher traces, noncommutative motives, and the categorified Chern character, Hoyois, Scherotzke and Sibilla write the following.
An important insight emerging from topology is that climbing up the chromatic ladder is related to studying invariants of spaces that are higher-categorical in nature.
The only example that I am aware of is the theory of categorified vector bundles, and its relation with chromatic-level-$2$ cohomology theories, including elliptic cohomology, and the algebraic K-theory spectrum of complex K-theory. See for instance this nLab page and the references therein.
Question. Are there other examples of this phenomenon, and is there a conceptual reason why categorification should be related to chromatic redshifting?
REPLY [6 votes]: The paper https://arxiv.org/abs/1312.5699 by Torlief Veen involves arbitrarily large chromatic redshifts via higher topological Hochschild homology. I know of no other examples like that.<|endoftext|>
TITLE: How are number theory and C*-algebras connected?
QUESTION [9 upvotes]: I came across this research profile where under Research Overview, it states that
These days C*-algebra theory is a very active area of mathematical research in its own right, and enjoys deep connections with other areas of mathematics such as symbolic dynamics, ergodic theory, group theory and even number theory.
My question is how C*-algebras and number theory are connected and if this is an active area of research?
REPLY [6 votes]: I think the most prominent example of this is Connes' work on the Riemann hypothesis from a C*-algebra perspective.<|endoftext|>
TITLE: Horizontal categorification: Two questions
QUESTION [9 upvotes]: According to the nlab, horizontal categorification is a process in which a concept is realized to be equivalent to a certain type of category with a single object, and then this concept is generalized to the same type of categories with an arbitrary number of objects. The prototypical example is the concept of a group, which horizontally categorifies to the concept of a groupoid. It is well-known that in certain parts of algebraic topology groupoids are much more convenient than groups. Similarly, monoids categorify to categories, rings to linear categories, etc.
I have two questions about this. Let C be a concept which horizontally categorifies to a concept D.
1) In many examples, C and D have been known and developed independently from each other. Or at least, D was not introduced as the horizontal categorification of C, but rather this connection was realized afterwards. For example, I am pretty sure that k-linear categories were not introduced as a horizontal categorificiation of k-algebras; instead they were introduced because of the abundance of examples of (large) k-linear categories which appear in everyday mathematics. Although representation theory seems to be in a current progress of a generalization from k-algebras to small k-linear categories, the concept of a k-linear category was already there before that. Are there examples where D was developed with the purpose to categorify C, say in order to solve some problems which deal with C but which are not solvable with C alone? Perhaps C*-categories (categorifying C*-algebras) could provide such an example, but I am not familiar with the history of this concept. And maybe there are other examples as well?
2) In all the examples I know of, it is trivial that C has a horizontal categorification and that it is D. Are there any more interesting examples where, when you look at C, it is not even clear how to interpret C as a type of category with one object? I would like to see examples where the connection between C and D is deep and surprising. These examples should illustrate why horizontal categorification is an important and useful concept in practice.
I could also ask a more provocative question: if any mathematician working outside of category theory reads the nlab article in its current form, why should he/she even care, since, after all, both concepts C and D have been there already without the categorification process?
REPLY [2 votes]: Often when a horizontal categorification is introduced, the author has intended examples, which the author gives as the motivation. It thus makes it a little tricky to determine when a structure has been motivated explicitly by horizontal categorification. However, here are a few examples where (1) the concept had not previously appeared in the literature, and (2) the authors make it clear horizontal categorification was intended (though this precise language may not appear).
In Street's Elementary cosmoi I, the concept of extension system is introduced, which is explicitly intended to be to closed categories what bicategories are to monoidal categories. Intuitively, the analogue of closed structure is proved by right-extensions.
Two more examples are the multi- and poly-bicategories of Cockett–Koslowski–Seely's Morphisms and modules for poly-bicategories, which are horizontal categorifications of multicategories and polycategories respectively.
My favourite example of a surprising horizontal categorification is that of symmetric monoidal categories. A symmetric bicategory (not to be confused with a symmetric monoidal bicategory) is a bicategory with an involution. Explicitly, a bicategory $\mathcal K$ is symmetric when equipped with a biequivalence $\mathcal K \simeq \mathcal K^{\mathrm{op}}$ that is a bijection on objects. A symmetric monoidal category is precisely a one-object symmetric bicategory for which the involution is the identity. This is interesting in two ways: the generalisation from symmetry to involution is a highly non-obvious step; and one is forced to make a generalisation to the original structure in order to horizontally categorify. There is an analogous concept of closed symmetric bicategory, which is a symmetric bicategory with left- and right-extensions. (Closed) symmetric bicategories are defined and studied in May–Sigurdsson's Parametrized Homotopy Theory.
I think these examples give some indication that studying horizontal categorifications in their own right can lead to interesting structures that may then be motivated by concrete examples.<|endoftext|>
TITLE: Are all maps $\mathbb{R}^2 \to \mathbb{R}^2$ with fixed singular values affine?
QUESTION [12 upvotes]: Let $f:\mathbb{R}^2 \to \mathbb{R}^2$ be a smooth map whose differential has fixed distinct singular values $0<\sigma_1<\sigma_2$ and an everywhere positive determinant (which is the product $\sigma_1\sigma_2$).
Must $f$ be affine?
My assumption is equivalent to $df_x \in \text{SO}(2) \cdot \text{diag}(\sigma_1,\sigma_2) \cdot \text{SO}(2)$ for every $x \in \mathbb{R}^2$.
If we were only allowing a copy of $\text{SO}(2)$ from one of the sides of $ \text{diag}(\sigma_1,\sigma_2)$, then the answer would be positive. (This reduces to the case of isometries).
Similarly, if we had $\sigma_1=\sigma_2$, the answer would also be positive.
REPLY [19 votes]: Answer modified on 1 February 2020:
It's not true 'locally' in the sense that non-affine $f$'s satisfying this system of PDE can be constructed on some open sets in $\mathbb{R}^2$. This first order, determined PDE system is hyperbolic, so there are many local solutions. However, it turns out (see below) that all $C^3$ solutions with domain equal to $\mathbb{R}^2$ are affine. (The proof I give below does not work for solutions of lower regularity.)
Let $D\subset\mathbb{R}^2$ be a $1$-connected open domain on which there exists a $C^3$ mapping $f:D\to\mathbb{R}^2$ whose differential $\mathrm{d}f$ has constant, distinct singular values $0<\sigma_1<\sigma_2$. Because $D$ is simply connected, one can choose an orthonormal frame field $E_1,E_2$ on $D$ such that, at each point $p\in D$, the image vectors $F_i(p) = \mathrm{d}f\bigl(E_i(p)\bigr)$ are orthogonal and satisfy $|F_i(p)|=\sigma_i$.
Let $\omega = (\omega_1,\omega_2)$ be the dual coframing on $D$, which is of regularity type $C^2$. The $1$-forms $\eta_i = \sigma_i\,\omega_i$ for $i=1,2$ have the property that $(\eta_1)^2+(\eta_2)^2$, being the $f$-pullback of the flat metric on $\mathbb{R}^2$, must also be a flat metric.
Let $\omega_{12}$ be the connection $1$-form associated to the coframing $\omega$, i.e., it satisfies the structure equations
$$
\mathrm{d}\omega_1 = -\omega_{12}\wedge\omega_2
\qquad\text{and}\qquad
\mathrm{d}\omega_2 = \omega_{12}\wedge\omega_1\,.\tag1
$$
Write $\omega_{12} = -\kappa_1\,\omega_1 + \kappa_2\,\omega_2$. The function $\kappa_i$ is the curvature of the $E_i$-integral curve. Since $\omega_{12}$ is $C^1$, so are the functions $\kappa_i$. A straightforward computation shows that the $1$-form $\eta_{12}$ that satisfies the corresponding structure equations
$$
\mathrm{d}\eta_1 = -\eta_{12}\wedge\eta_2
\qquad\text{and}\qquad
\mathrm{d}\eta_2 = \eta_{12}\wedge\eta_1\,.\tag2
$$
is given by
$$
\eta_{12} = -(\sigma_1/\sigma_2)\,\kappa_1\omega_1
+ (\sigma_2/\sigma_1)\,\kappa_2\omega_2\,.
$$
Since $\sigma_1\not=\sigma_2$, the conditions $\mathrm{d}\omega_{12} = \mathrm{d}\eta_{12}=0$ (which hold because the domain metric and the $f$-pullback of the range metric are both flat) are equivalent to
$$
0 = \mathrm{d}(\kappa_i\,\omega_i) = \bigl(\mathrm{d}\kappa_i - {\kappa_i}^2\,\omega_{3-i}\bigr)\wedge\omega_i\,\qquad i = 1,2.\tag3
$$
Proposition: If $D = \mathbb{R}^2$, then $\kappa_1 \equiv \kappa_2 \equiv 0$, and $f$ is an affine map.
Proof: Suppose that, say, $\kappa_1$ be nonzero at some point $p\in\mathbb{R}^2$ and consider the value of $\kappa_1$ along the $E_2$ integral curve through $p$, which, since $E_2$ has unit length, is necessarily complete. Let $p(s)$ be the flow of $E_2$ by time $s$ starting at $p = p(0)$. Then (3) implies that the function $\lambda(s) = \kappa_1\bigl(p(s)\bigr)$ satisfies $\lambda'(s) = \lambda(s)^2$. Consequently,
$$
\kappa_1\bigl(p(s)\bigr) = \frac{\kappa_1\bigl(p(0)\bigr)}{1-\kappa_1\bigl(p(0)\bigr)s}.
$$
Hence $\kappa_1$ cannot be continuous along this integral curve, which is a contradiction. Thus, $\kappa_1$ and, similarly, $\kappa_2$ must vanish identically when $D = \mathbb{R}^2$. In particular, $\mathrm{d}\omega_i = 0$, from which one easily concludes that $f$ is affine. QED
More interesting, locally, is what happens near a point where $\kappa_1\kappa_2\not=0$. (There is a similar analysis when one of $\kappa_i$ vanishes identically that can safely be left to the reader, but see the note at the end.) One might as well assume that $\kappa_1\kappa_2$ is nowhere vanishing on $D$. Then one can write
$$
\kappa_1\,\omega_1 = \mathrm{d}u
\qquad\text{and}\qquad
\kappa_2\,\omega_2 = \mathrm{d}v
$$
for some $C^2$ functions $u$ and $v$ on $D$, uniquely defined up to additive constants.
Writing $\omega_1 = p\,\mathrm{d}u$ and $\omega_2 = q\,\mathrm{d}v$ for some non-vanishing functions $p$ and $q$, one finds that the structure equations (1), with $\omega_{12} = -\mathrm{d}u + \mathrm{d}v$, yield the equations
$$
p_v = - q \qquad\text{and}\qquad
q_u = -p.
$$
In particular, note that $p_v$ is $C^1$ and $p_{uv}-p = 0$.
Conversely, if $p$ be any nonvanishing $C^2$ function on a domain $D'$ in the $uv$-plane that satisfies the hyperbolic equation $p_{uv}-p=0$ and is such that $p_v$ is also nonvanishing on $D'$, then the $1$-forms
$$
\omega_1 = p\,\mathrm{d}u,\quad \omega_2 = -p_v\,\mathrm{d}v,\qquad
\omega_{12} = -\mathrm{d}u+\mathrm{d}v\tag4
$$
satisfy the structure equations of a flat metric, and so do
$$
\eta_1 = \sigma_1\,p\,\mathrm{d}u,\quad \eta_2 = -\sigma_2\,p_v\,\mathrm{d}v,\qquad
\eta_{12} = -(\sigma_1/\sigma_2)\,\mathrm{d}u+(\sigma_2/\sigma_1)\,\mathrm{d}v.\tag5
$$
Indeed, one now sees that the $1$-forms
$$
\begin{aligned}
\alpha_1 &= \cos(u{-}v)\,p\,\mathrm{d}u +\sin(u{-}v)\,p_v\,\mathrm{d}v\\
\alpha_2 &= \sin(u{-}v)\,p\,\mathrm{d}u -\cos(u{-}v)\,p_v\,\mathrm{d}v
\end{aligned}
$$
are closed, and therefore can be written in the form $\alpha_i = \mathrm{d}x_i$ for some $C^3$ functions $x_i$ on $D'$.
$$
(\mathrm{d}x_1)^2 + (\mathrm{d}x_2)^2 = (\alpha_1)^2 + (\alpha_2)^2 = (\omega_1)^2 + (\omega_2)^2
$$
and, hence, they define a $C^3$ submersion $x = (x_1,x_2):D'\to\mathbb{R}^2$ that pulls back the standard flat metric on $\mathbb{R}^2$ to the metric $(\omega_1)^2 + (\omega_2)^2$ on $D'$.
Similarly, setting $\rho = \sigma_2/\sigma_1$ and
$$
\begin{aligned}
\beta_1 &= \cos(u/\rho{-}\rho v)\,\sigma_1\,p\,\mathrm{d}u
+\sin(u/\rho{-}\rho v)\,\sigma_2\,p_v\,\mathrm{d}v\\
\beta_2 &= \sin(u/\rho{-}\rho v)\,\sigma_1\,p\,\mathrm{d}u
-\cos(u/\rho{-}\rho v)\,\sigma_2\,p_v\,\mathrm{d}v,
\end{aligned}
$$
one finds that $\mathrm{d}\beta_i = 0$ and hence there exist $C^3$ functions $y_i$ on $D'$ such that $\beta_i = \mathrm{d}y_i$. Set $y = (y_1,y_2)$.
Restricting to a subdomain $D''\subset D'$ on which $x$ is $1$-to-$1$ onto its image $D = x(D'')$ yields a domain on which $x^{-1}:D\to D''$ is a $C^3$ diffeomorphism. Now set $f = y\circ x^{-1}:D\to\mathbb{R}^2$, and one has a $C^3$ solution of the original PDE system.
This completely determines the structure of the 'generic' local $C^3$ solutions.
The case when one of the $\kappa_i$, say, $\kappa_1$, vanishes identically (so that the corresponding integral curves are straight lines) and the other is nonvanishing can easily be reduced to the normal form
$$
\omega_1 = \mathrm{d}u,\qquad \omega_2 = \bigl(p(v)-u\bigr)\,\mathrm{d}v,\qquad
\omega_{12} = \mathrm{d}v\tag6
$$
where, now, $p$ is a $C^2$ function of $v$, and the rest of the analysis goes through essentially unchanged.
REPLY [7 votes]: I would like to propose a simple local example:
Consider the map in polar coordinates, $\mathbb C\to \mathbb C$ that takes a complex number $z=e^{2\pi i \theta}r$ to $e^{(\sigma_1/\sigma_2)\cdot 2\pi i \theta}r\sigma_2$.
(apologies for the previous wrong example, I confused in it singular values with eigenvalues...)<|endoftext|>
TITLE: Real rootedness of a polynomial
QUESTION [21 upvotes]: Let's consider $m$ and $n$ arbitrary positive integers, with $m\leq n$, and the polynomial given by:
$$ P_{m,n}(t) := \sum_{j=0}^m \binom{m}{j}\binom{n}{j} t^j$$
I've found with Sage that for every $1\leq m \leq n \leq 80$ this polynomial has the property that all of its roots are real (negative, of course).
It seems these roots are not nice at all. For example for $m=3$ and $n=10$, one has $$P(t) = 120t^3 + 135 t^2 + 30t+1$$ and the roots are:
$$ t_1 = -0.8387989...$$
$$ t_2 = -0.2457792...$$
$$ t_3 = -0.0404217...$$
Is it true that all roots of $P_{m,n}(t)$ are real?
REPLY [24 votes]: If you have two polynomials $f(x)=a_0+a_1x+\cdots a_mx^m$ and $g(x)=b_0+b_1x+\cdots+b_nx^n$, such that the roots of $f$ are all real, and the roots of $g$ are all real and of the same sign, then the Hadamard product
$$f\circ g(x)=a_0b_0+a_1b_1x+a_2b_2x^2+\cdots$$
has all roots real. This was proven originally in
E. Malo, Note sur les équations algébriques dont toutes les racines
sont réelles, Journal de Mathématiques Spéciales, (4), vol. 4 (1895)
One can make stronger statements, such as the result by Schur that says that $\sum k!a_kb_k x^k$ will only have real roots, under the same conditions. Schur's theorem combined with the fact that $\{1/k!\}_{k\geq 0}$ is a Polya frequency sequence, implies Malo's theorem.
I'm not sure what the best reference to learn the theory of real rooted polynomials, and the associated operations that preserve real rootedness is. One textbook I know that discusses some of these classical results is Marden's "Geometry of Polynomials".
REPLY [16 votes]: According to the representation for Jacobi polynomials https://en.wikipedia.org/wiki/Jacobi_polynomials#Alternate_expression_for_real_argument
$$
P^{(0,n-m)}_m(x)=\sum_{j=0}^m \binom{m}{j}\binom{n}{j}\left(\frac{x-1}{2}\right)^{j}\left(\frac{x+1}{2}\right)^{m-j}
$$
OPs polynomial equals
$$
P_{m,n}(t)=(1-t)^mP^{(0,n-m)}_m\left(\frac{1+t}{1-t}\right).
$$
Since zeroes of Jacobi polynomials are real valued, all roots of the polynomial $P_{m,n}(t)$ are also real valued (see D.Dominici, S.J.Johnston, K.Jordaan, Real zeros of
hypergeometric polynomials).<|endoftext|>
TITLE: Theory of integration for functions from $\mathbb{Z}_{p}$ to $\mathbb{Z}_{q}$ for distinct primes $p,q$
QUESTION [6 upvotes]: Let $p$ and $q$ be prime numbers. When $p=q$, Mahler's Theorem gives a complete description of $C\left(\mathbb{Z}_{p};\mathbb{Z}_{p}\right)$, the space of continuous functions from $\mathbb{Z}_{p}$ to $\mathbb{Z}_{p}$. I'm wondering (possibly in vain) if there might be a comparable classification of $C\left(\mathbb{Z}_{p};\mathbb{Z}_{q}\right)$ when $p$ and $q$ are distinct.
I ask only because I've been doing $p$-adic harmonic analysis, but have found myself having to brave the wilds of $L^{\infty}\left(\mathbb{Z}_{p};\mathbb{C}_{q}\right)$, the space of all $f:\mathbb{Z}_{p}\rightarrow\mathbb{C}_{q}$ so that:$$\sup_{\mathfrak{z}\in\mathbb{Z}_{p}}\left|f\left(\mathfrak{z}\right)\right|_{q}<\infty$$
Pontryagin duality lets me do Fourier analysis on $L^{\infty}\left(\mathbb{Z}_{p};\mathbb{C}\right)$; for $p=q$, on the other hand, I can use things like the volkenborn integral, or the amice transform / mazur-mellin transform—$p$-adic distributions, in general. The problem is, without a structure theorem like Mahler's for the $p\neq q$ case, though I can define “integration” on $L^{\infty}\left(\mathbb{Z}_{p};\mathbb{C}_{q}\right)$ by elements of its dual space (continuous functionals $\varphi:L^{\infty}\left(\mathbb{Z}_{p};\mathbb{C}_{q}\right)\rightarrow\mathbb{C}_{q})$, I don't see a way to do useful computations for the specific, non-abstract functions that I'm trying to fourier analyze.
So, I guess what I'm really asking is: how do you take the "integral" or "fourier transform" of such a function?
Any thoughts? Reference recommendations? Etc.?
REPLY [4 votes]: ${\mathbb Z}_p$ and ${\mathbb Z}_q$ are homeomorphic; hence so are $C({\mathbb Z}_p,{\mathbb Z}_p)$ and $C({\mathbb Z}_p,{\mathbb Z}_q)$.<|endoftext|>
TITLE: Classification of the functors on the category of cyclic groups
QUESTION [8 upvotes]: Let $\mathsf{Grp}$ be the category of groups and let $\mathsf{Cyc}$ be the subcategory of cyclic groups.
As seen in the posts here and there (and their answers), a functor $F: \mathsf{Cyc} \to \mathsf{Cyc}$ is a very structured/restrictive notion, we are then lead to wonder whether there exists such a functor which is non-equivalent to the identity or the trivial functor, or if there is a such functor with $F(C_1) \not \simeq C_1$. As pointed out by Martin Brandenburg and Jeremy Rickard, $C_1$ is a retract of $F(C_1)$, so that $F(C_1)$ must be a retract of $F^2(C_1)$, and more generally, $F^n(C_1)$ is a retract of $F^{n+1}(C_1)$, which means that $F^{n+1}(C_1)$ is isomorphic to a semidirect product $F^n(C_1) \ltimes N_n$; now $F^{n+1}(C_1)$ is a cyclic group, so the semidirect product is in fact a direct product and moreover $gcd(|F^n(C_1)|,|N_n|) = 1$.
Question: What are the functors on the categroy of cyclic groups?
Remark: $Aut(-)$ is not such a functor because $Aut(C_8) \simeq C_2 \times C_2$ (and $Aut^2(C_8) \simeq S_3$).
In his answer, Neil Strickland provides examples of functors $F$ with $F(C_1) \not \simeq C_1$ and with $F^2(C_1) \not \simeq F(C_1)$, but with $F^3(C_1) \simeq F^2(C_1)$.
Bonus question: Is there a functor $F: \mathsf{Cyc} \to \mathsf{Cyc}$ such that $F^{n+1}(C_1) \not \simeq F^n(C_1)$ for all $n$?
Remark: If so, the sequence $(F^n(C_1))_n$ cannot be periodic (for $n$ large enough), because then (as shown above) $F^{n+1}(C_1) \simeq F^{n}(C_1) \times N_n$ with $|N_n|>1$ for all $n$.
REPLY [9 votes]: I'll use additive notation, and I'll assume that you are only considering finite cyclic groups. Let $\mathbf{Cyc}_p$ be the category of cyclic $p$-groups. Given $i,j\geq 0$ we can define $Q(p;i,j)\colon\mathbf{Cyc}\to\mathbf{Cyc}_p$ by $Q(p;i,j)(A)=\{a\in p^iA\colon p^ja=0\}$. We can also define a constant functor $C(p;i)\colon\mathbf{Cyc}\to\mathbf{Cyc}_p$ by $C(p;i)(A)=\mathbb{Z}/p^i$. Now suppose we have a collection of functors $F_p$, one for each prime $p$, each of the form $Q(p;i,j)$ or $C(p;i)$, and that only finitely many of the functors $F_p$ are constant. Then the group $F(A)=\prod_pF_p(A)$ is cyclic for all $A$, so we get a functor $F\colon\mathbf{Cyc}\to\mathbf{Cyc}$. I don't know if that gives all possible functors, but it certainly gives a reasonably rich supply of them.
As a very specific example, the functor $F(A)=(A/2A)\times(\mathbb{Z}/3)$ is non-constant with $F(0)\neq 0$.
UPDATE: Here's a more exotic example. If $X$ is a based set of size $1$ or $3$, there is a unique group structure for which the basepoint is the identity. If $A$ is cyclic of order $1$ or $7$ then we can impose an equivalence relation with $a\sim a^2\sim a^4$ for all $a$, and then $A/\sim$ has size $1$ or $3$ with basepoint $0$ and so has a group structure. This construction gives a functor on the category of groups of order $1$ or $7$, and we can compose with $A\mapsto A/7$ to get a functor $\mathbf{Cyc}\to\mathbf{Cyc}_3$. I am sure that there are many variations on this theme.<|endoftext|>
TITLE: On the iterated automorphism groups of the cyclic groups
QUESTION [12 upvotes]: Let $C_n$ be the cyclic group of order $n$. Its automorphism group $Aut(C_n)$ is a group of order $\varphi(n)$ isomorphic to $(\mathbb{Z}/n\mathbb{Z})^{\times}$ the multiplicative group of integer modulo $n$. This last group is abelian but not always cyclic, the first non-cyclic being $Aut(C_8) \simeq C_2 \times C_2$. Moreover the iterated automorphism groups $Aut^m(C_n)$ are not always abelian, as $Aut^2(C_8) \simeq S_3$.
By watching the table here for the group structure of $(\mathbb{Z}/n\mathbb{Z})^{\times}$, we cannot expect an easy classification for the set of groups $Aut(C_n)$, but (Q1) what about the following set? $$\{Aut^m(C_n) \ | \ n \ge 1, m \ge 0 \}$$
If it is also non-easy, (Q2) what about the set of groups $Aut^m(C_n)$ which are isomorphic to $Aut^{m+1}(C_n)$? For $n \le 15$, it is exactly $\{C_1,S_3,D_8 \}$ as shown by the following table:
$$\scriptsize{ \begin{array}{c|c}
G &C_1&C_2&C_3&C_4&C_5&C_6&C_7&C_8&C_9&C_{10}&C_{11}&C_{12}&C_{13}&C_{14}&C_{15} \newline
\hline
Aut(G) &-&C_1&C_2&C_2&C_4&C_2&C_6&C_2^2&C_6&C_4&C_{10}&C_2^2&C_{12}&C_6&C_2 \times C_4 \newline
\hline
Aut^2(G) &-&-&C_1&C_1&C_2&C_1&C_2&S_3&C_2&C_2&C_4&S_3&C_2^2&C_2&D_8 \newline
\hline
Aut^3(G) &-&-&-&-&C_1&-&C_1&-&C_1&C_1&C_2&-&S_3&C_1&- \newline
\hline
Aut^4(G) &- &- &-&-&-&-&-&-&-&-&C_1&-&-&-&-
\end{array} }$$
Let $\alpha(n)$ be the smallest $m \ge 0$ such that $Aut^m(C_n) \simeq Aut^{m+1}(C_n)$. Then:
$$\scriptsize{ \begin{array}{c|c}
n &1&2&3&4&5&6&7&8&9&10&11&12&13&14&15 \newline
\hline
\alpha(n) &0&1&2&2&3&2&3&2&3&3&4&2&2&3&2
\end{array} }$$
Surprisingly, for $n<15$ (but not for $n=15$), $\alpha(n)$ is exactly the number of iterations of the Carmichael lambda function $\lambda$ needed to reach $1$ from $n$ (OEIS A185816). (Q3) Why?
A specific OEIS sequence has just been created (A331921) providing the value of $\alpha(n)$ for $n<32$; in this case, $\alpha(n) \le 5$ and $\{\mathrm{Aut}^5(C_n) \ | \ n < 32 \} = \{C_1,S_3,D_8,D_{12},\mathrm{PGL}(2,7) \}$. A usual laptop cannot compute $\alpha(n)$ for $n \ge 32$ (you are welcome to contribute to this sequence), we just know that $\alpha(32) \ge 6$. Here is the sequence $|Aut^n(C_{32})|$ for $n \le 6$:
$$\scriptsize{ \begin{array}{c|c}
n &0&1&2&3&4&5&6&7&8 \newline
\hline
|Aut^n(C_{32})| & 2^5&2^4&2^4&2^6&2^{7}3&2^{9}3&2^{11}3&?&? \newline
\hline
\text{IdGroup}(Aut^n(C_{32})) & [32,1]&[16,5]&[16,11]&[64,138]&[384,17948]&[1536,?]&[6144,?]&[?,?]&[?,?]
\end{array} }$$
We can also consider the sequence of $n$ such that $\exists m \ge 0$ with $Aut^m(C_n) \simeq C_1$:
$1, 2, 3, 4, 5, 6, 7, 9, 10, 11, 14, 18, 19, 22, 23, 27, 38, 46, 47, 54, 81, \dots$ (A117729).
It seems to be a (strict) subsequence of A179401, so (Q4) why $Aut^m(C_n) \simeq C_1$ $\Rightarrow$ $\varphi^2(n) = \lambda^2(n)$?
It is an open problem whether the sequence $Aut^n(G)$ stabilizes (see here). For a given finite group $G$ there are three possibilities:
(1): $\exists m \ge 0 $ such that $Aut^{m+1}(G) \simeq Aut^{m}(G)$,
(2): not (1) and $\exists m \ge 0$ such that $\exists r>0$ with $Aut^{m+r}(G) \simeq Aut^{m}(G)$,
(3): not (1) and not (2), i.e., $\forall m \ge 0$ and $\forall r>0$ then $Aut^{m+r}(G) \not \simeq Aut^{m}(G)$.
The case (1) means that the sequence $(Aut^m(G))_m$ is constant for $m$ large enough, (2) means that it is periodic non-constant for $m$ large enough, and (3) means never periodic. The existence of finite groups in cases (2) or (3) is open. (Q5) Can we rule out the cases (2) and (3) for the cyclic groups?
If not, let redefine $\alpha(n)$ as the smallest $m \ge 0$ such that $\exists r>0$ with $Aut^{m}(C_n) \simeq Aut^{m+r}(C_n)$ if $C_n$ is in cases (1) or (2), and $\infty$ in the case (3).
REPLY [6 votes]: It seems to me that the structure of ${\rm Aut}^{m}(C_{n})$ also depends heavily on the prime factorization of $n$, and I don't really see any reason to expect the answer to Q1 to be any more tractable than determining the structure of ${\rm Aut}(C_{n})$.
For example (just to illustrate) , if we choose a prime $p$, greater than $3$, and then we take a pair of primes $q_{1}$ and $q_{2}$ so that each $q_{i}-1$ is divisible by $p$, but not by $p^{2}$, and we set $n = q_{1}q_{2}$, then ${\rm Aut}^{2}(C_{n})$ has a direct factor isomorphic to
${\rm GL}(2,p)$, and is not solvable. Furthermore the involvement of ${\rm PSL}(2,p)$ in ${\rm Aut}^{m}(C_{n})$ persists for $m >2$.<|endoftext|>
TITLE: Are exterior algebras intrinsically formal as associative dg algebras?
QUESTION [7 upvotes]: (Cross-posted from mathematics stackexchange.)
Fix a finite dimensional vector space $V$ over a field of characteristic zero, and let $R=Sym(V[1])$ be the free graded commutative algebra generated by $V$ in cohomological degree $-1$, but thought of as a formal associative dg algebra (we forget that it is commutative).
Then it is easy to see that (edit: any $A_{\infty}$-deformation of) $R$ is almost formal, meaning that in a minimal $A_{\infty}$-model, the higher products eventually vanish. Indeed, $R$ is concentrated in non-positive cohomological degrees, so all products of an $A_{\infty}$-structure $\mu_{n} : R^{\otimes n} \rightarrow R[2-n]$ have to vanish for $n>>0$.
Question 1: Is $R$ intrinsically formal, so that for any deformation, $\mu_{n}$ can be assumed to be zero for $n>2$?
Question 2: What if $R=Sym(V[-1])$, so that $R$ is concentrated cohomologically in non-negative degrees? (I'm thinking of the cohomology ring of a torus, but as an associative algebra, rather than a commutative algebra.) Can there be non-trivial $A_{\infty}$-structures on $R$, or is $R$ intrinsically formal?
REPLY [8 votes]: Here are examples of nontrivial $A_\infty$-structures extending the product on $\operatorname{Sym}(\mathbb R^3[-1])$ and $\operatorname{Sym}(\mathbb R^5[-1])$, respectively. Below the fold, I have kept my original answer, which got the main ideas right but almost all degrees wrong.
Fix coordinates $\xi_1,\xi_2,\xi_3$ on $\mathbb R^3[-1]$ and set
$$
\mu_3(f_1,f_2,f_3) = \xi_2\xi_3 (\partial_{\xi_1}f_1)(\partial_{\xi_1}f_2)(\partial_{\xi_1}f_3)\ .
$$
This defines a $A_\infty$-structure: Every term in $[\mu_3,\mu_3](f_1,\dots,f_5) = \sum \mu_3(\dots,\mu_3(\dots),\dots)$ contains $\xi_1^2 = 0$ and hence vanishes; similarly, $[\mu_3,\mu_2](f_1,\dots,f_4)$ vanishes if one of the $f_i$ is a multiple of $\xi_2$ or $\xi_3$, and it also vanishes if one of them is the identity since $[-,-]$ preserves normalized Hochschild cochains. Thus the only expression we have to check is
\begin{align*}
[\mu_2,\mu_3](\xi_1,\xi_1,\xi_1,\xi_1) &= \color{red}{-}\xi_1\mu_3(\xi_1,\xi_1,\xi_1) -\mu_3(\xi_1^2,\xi_1,\xi_1) \\
&\ + \mu_3(\xi_1,\xi_1^2,\xi_1) - \mu_3(\xi_1,\xi_1,\xi_1^2) \\
&\ + \mu_3(\xi_1,\xi_1,\xi_1)\xi_1\\
&= -\xi_1\xi_2\xi_3 + \xi_2\xi_3\xi_1 = 0
\end{align*}
(The red sign comes from the Koszul sign rule.)
This $A_\infty$-algebra has a nontrivial Massey product $0\notin \langle\xi_1,\xi_1,\xi_1\rangle = \xi_2\xi_3 + \xi_1A + A\xi_1$ and hence is not formal.
Essentially the same argument applies to the $A_\infty$-structure on $\mathbb R^5[-1]$ defined by the only nontrivial bracket
$$
\mu_3(f_1,f_2,f_3) = \xi_2\xi_3\xi_4\xi_5 (\partial_{\xi_1}f_1)(\partial_{\xi_1}f_2)(\partial_{\xi_1}f_3)\ .
$$
Recall that we can associate to any (cohomologically) graded algebra $(A,\mu_A)$ the Hochschild cochains
$$
C^n(A) = \prod_{p+q = n}\operatorname{Hom}^p(A^{\otimes q},A)
$$
which come equipped with a differential defined by precomposing with the multiplication of cyclic neighbours in the tensor product. As the totalization of a double complex, it also comes with a canonical filtration, namely
$C^n_{\ge m}(A) = \prod_{{\substack{p+q=n\\q\ge m}}} \operatorname{Hom}^p(A^{\otimes q},A)$. The skew-symmetrization of the pre-Lie structure $f\circ g(c_1,\dots,x_{m+n-1}) = \sum_{i=1}^m \pm f(x_1,\dots,g(x_i,\dots,x_{i+n-1}),\dots,x_{m+n-1})$ defines a filtered shifted dgla structure on $C^*(A)$, and $A_\infty$-structures with trivial differential $(A,0,\mu_A,\mu_3,\dots)$ correspond to Maurer-Cartan elements $(\mu_3,\dots)$ in $C^2_{\ge 3}(A)$.
Now consider any such element $\mu$; its component of lowest filtration defines a degree $2$ cohomology class in the associated graded, since $\mathrm d\mu = -\frac{1}{2}[\mu,\mu]$ lies in higher filtration (as $[C^p_{\ge m}(A),C^q_{\ge n}(A)]\subset C^{p+q-1}_{\ge m+n-1}(A)$). If this cohomology class is $0$, i.e. $\mu = \mathrm d\eta$ up to terms of higher filtration, we can act by the gauge symmetry $\mu\mapsto e^{-\eta}\circ\mu = \mu - \mathrm d\eta\, + (\text{higher filtration})$ to obtain an equivalent Maurer-Cartan element which lies in higher filtration. Since the Hochschild cochains are complete with respect to the canonical filtration, the limit of this process is well-defined, i.e. if all obstructions vanish we obtain that our Maurer-Cartan element is gauge equivalent to $0$.
If $A = \operatorname{Sym}(V)$ is the commutative algebra on a graded vector space $V$, we can compute $C^*(A)$ with its filtration explicitly: We have $C^*(A) \simeq \operatorname{Ext}_{A\otimes A^{op}}(A,A)$, and since the multiplication map $A\otimes A^{op}\to A$ is $\operatorname{Sym}$ of the codiagonal $V\oplus V\to V$, we obtain a Koszul resolution $A\simeq (\operatorname{Sym}(V\oplus V\oplus V[1]),\mathrm d)$. Explicitly, we choose a basis $x_i$ of $V$; then the resolution is generated by elements $x_i,x_i'$ in degree $|x_i|$ and $y_i$ in degree $|x_i|-1$, with $\mathrm dy_i = x_i' - x_i''$. (This is where we use characteristic $0$.) It follows that $C^*(\operatorname{Sym}(V))\simeq \widehat{\operatorname{Sym}}(V\oplus V^*[-1])$, where the filtration is given by the degree in the $V[-1]$-variables, and the hat indicates that we take the completion with respect to this filtration.
Now let us specialize to the two cases in your question, namely $V$ is concentrated in degree $-1$ or $+1$. In the second case, $C^*(A)$ is concentrated in nonnegative degrees, and $HH^2(A) := H^2(C^*(A))\cong \operatorname{Sym}^* V^*\otimes \Lambda^2 V$, with the relevant part in $\operatorname{Sym}$-filtration $\ge 3$. In the first case, $V\oplus V^*[-1]$ lives in degrees $-1$ and $2$, and the deformation group is given by $HH^2_{\ge 3}(A) = \prod_{p\ge 3} \Lambda^{2p-2}V\otimes \operatorname{Sym}^p V^*$ and thus also does not vanish. Indeed, Kontsevich's formality theorem shows that Maurer-Cartan elements of $\hbar C^*_{\ge 3}(A)[[\hbar]]$ are in bijection with those of its cohomology, i.e. an infinitesimal deformation can be extended to a formal deformation iff it satisfies the Maurer-Cartan equation in in the cohomology. In some cases, the formal infinite series involve only finitely many terms and therefore give rise to actual deformations, as is the case in the two examples above.<|endoftext|>
TITLE: Does minimal degree $n$ imply a $K_n$ minor
QUESTION [10 upvotes]: Is it true that any finite graph has a $K_n$ minor, where $n$ is a minimal vertex degree?
REPLY [22 votes]: More generally, it is a classic result (independently due to Kostochka and Thomason) that minimum degree $(\alpha+o(1))n \sqrt{\log n}$ suffices to force a $K_n$ minor, where $\alpha$ is an explicit constant. Conversely, there are random graphs with minimum degree $\Omega(n\sqrt{\log n})$ that do not contain a $K_n$ minor. See here to access the paper by Thomason.
Update. Alon, Krivelevich, and Sudakov have recently given a new short proof of the Kostochka-Thomason result.<|endoftext|>
TITLE: Extent of “unscientific”, and of wrong, papers in research mathematics
QUESTION [97 upvotes]: This question is cross-posted from academia.stackexchange.com where it got closed with the advice of posting it on MO.
Kevin Buzzard's slides (PDF version) at a recent conference have really unsettled me.
In it, he mentions several examples in what one would imagine as very rigorous areas (e.g., algebraic geometry) where the top journals like Annals and Inventiones have published and never retracted papers which are now known to be wrong. He also mentions papers relying on unpublished results taken on trust that those who announced them indeed have a proof.
He writes about his own work:
[...] maybe some of my work in the p-adic Langlands philosophy relies
on stuff that is wrong. Or maybe, perhaps less drastically, on stuff
which is actually correct, but for which humanity does not actually
have a complete proof. If our research is not reproducible, is it
science? If my work in pure mathematics is neither useful nor 100
percent guaranteed to be correct, it is surely a waste of time.
He says that as a result, he switched to formalizing proofs completely, with e.g. Lean, which guarantees correctness, and thus reusability forever.
Just how widespread is the issue? Are most areas safe, or contaminated? For example, is there some way to track the not-retracted-but-wrong papers?
The answer I accepted on academia.stackexchange before the closure gives a useful general purpose method, but I'd really appreciate more detailed area-specific answers. For example, what fraction of your own papers do you expect to rely on a statement "for which humanity does not actually have a complete proof" ?
REPLY [44 votes]: As Kevin Buzzard himself admits in his answer, he somewhat exaggerated his point for effect.
However, I'd submit that if you were unsettled by his talk, then that's a good thing. I don't think that the proper reaction is to look for reassurance that mathematics really is fine, or that the problems are restricted to some easily quarantined corner.
Rather, I think the proper reaction is to strive for a more accurate view of the true state of the mathematical literature, and refuse to settle for comforting myths that aren't based on reality. Some of the literature is rock-solid and can stand on its own, much more of it is rock-solid provided you have access to the relevant experts, and some of it is gappy but we don't really care. On the other hand, some small percentage of it is gappy or wrong and we do care, but social norms within the mathematical community have caused us to downplay the problems. This last category is important. It is a small percentage, but from a scholarly point of view it is a serious problem, and we should all be aware of it and willing to acknowledge it. If, every time someone brings it up, we try to get them to shut up by repeating some "propaganda" that makes us feel good about mathematics, then we are not solving the problem but perpetuating it.
Some related concerns were raised over 25 years ago by Jaffe and Quinn in their article on Theoretical Mathematics. This generated considerable discussion at the time. Let me quote the first paragraph of Atiyah's response.
I find myself agreeing with much of the detail of the Jaffe–Quinn argument, especially the importance of distinguishing between results based on rigorous proofs and those which have a heuristic basis. Overall, however, I rebel against their general tone and attitude which appears too authoritarian.
My takeaway from this is that Jaffe and Quinn made many valid points, but because this is a sensitive subject, dealing with societal norms, we have to be very careful how we approach it. Given the way that the mathematical community currently works, saying that someone's work has gaps and/or mistakes is often taken to be a personal insult. I think that if, as a community, we were more honest about the fact that proofs are not simply right or wrong, complete or incomplete, but that there is a continuum between the two extremes, then we might be able to patch problems that arise more efficiently, because we wouldn't have to walk on eggshells.<|endoftext|>
TITLE: Vanishing of Hochschild homology of a category
QUESTION [7 upvotes]: Let $A$ be a dg- or $A_{\infty}$-category (with $\mathbb{Z}$-graded Hom sets, over a field of characteristic $0$). Let $HH_*(A)$ be the Hochschild homology of $A$.
Suppose that $HH_n(A)=0$ for all $n \in \mathbb{Z}$. Does this imply that $A$ is the zero category?
If not, then what assumptions can I add to $A$ to make this true (e.g. smoothness, properness, ect)?
Remark: I have heard the heuristic that Hochschild homology can be viewed as "differential forms" on the "spectrum" of the non-commutative category $A$ (this heuristic is presumably motivated by the Hochschild-Kostant-Rosenberg theorem). Hence it's natural to expect that $A$ vanishes if it admits no non-zero differential forms.
Note also that if $A$ is a (possibly non-commutative) $k$-algebra, then one can check that $HH_0(A) =0$ iff $A=0$.
REPLY [13 votes]: This precise question was phrased as the vanishing conjecture in Hochschild homology and semiorthogonal decompositions. But we now know that there exist so called (quasi)phantom categories, which give counterexamples. These are categories with vanishing Hochschild homology, and vanishing (resp. torsion) Grothendieck group. As they are admissible subcategories of derived categories of smooth projective varieties, they have all nice properties you could want for their dg enhancements. An overview of some constructions:
Gorchinskiy, Sergey; Orlov, Dmitri, Geometric phantom categories, Publ. Math., Inst. Hautes Étud. Sci. 117, 329-349 (2013). ZBL1285.14018.
Böhning, Christian; Graf von Bothmer, Hans-Christian; Katzarkov, Ludmil; Sosna, Pawel, Determinantal Barlow surfaces and phantom categories, J. Eur. Math. Soc. (JEMS) 17, No. 7, 1569-1592 (2015). ZBL1323.14014.
Galkin, Sergey; Katzarkov, Ludmil; Mellit, Anton; Shinder, Evgeny, Derived categories of Keum’s fake projective planes, Adv. Math. 278, 238-253 (2015). ZBL1327.14081.
Galkin, Sergey; Shinder, Evgeny, Exceptional collections of line bundles on the Beauville surface, Adv. Math. 244, 1033-1050 (2013). ZBL1408.14068.
Böhning, Christian; Graf von Bothmer, Hans-Christian; Sosna, Pawel, On the derived category of the classical Godeaux surface, Adv. Math. 243, 203-231 (2013). ZBL1299.14015.
Alexeev, Valery; Orlov, Dmitri, Derived categories of Burniat surfaces and exceptional collections, Math. Ann. 357, No. 2, 743-759 (2013). ZBL1282.14030.
What is interesting is that Hochschild cohomology can detect their non-vanishing, and all kinds of interesting behavior regarding deformation theory arises, see
Kuznetsov, Alexander, Height of exceptional collections and Hochschild cohomology of quasiphantom categories, J. Reine Angew. Math. 708, 213-243 (2015). ZBL1331.14024.<|endoftext|>
TITLE: The isometry group of a product of two Riemannian manifolds
QUESTION [5 upvotes]: Under what conditions is the isometry group of a product of two Riemannian manifolds the product of the isometry groups of each one of the components?
One counterexample is a product of two isometric manifolds, since the isometry contains the involution which cannot be a product. I was wondering if there are some general criterion.
REPLY [2 votes]: You may find this answer of mine of some interest. There, I explain a special case when one can prove that $I(M \times N) \cong I(M) \times I(N)$ with simple differential geometric arguments involving sectional curvatures. And then (in the edit) I sketch a proof of a much more general result using some holonomy techniques taken from Kobayashi & Nomizu. In a nutshell, this result says that if you have a product of some irreducible Riemannian manifolds and at most one flat Riemannian manifold, then the isometry group of the product is the product of the isometry groups of the factors plus permutations of isometric factors (no compactness or even completeness assumption required). One can also try to drop the irreducibility assumption, but then to say something interesting about the isometry group and how it relates to those of the factors, one needs to add the assumptions of completeness and simply connectedness (at least I don't see how to proceed otherwise). Here is the resulting statement:
Proposition. Let $M = M_1 \times \ldots \times M_k$ be a Riemannian product of complete simply connected Riemannian manifolds. We have an obvious embedding of Lie groups $i \colon I(M_1) \times \ldots \times I(M_k) \hookrightarrow I(M)$. The following are equivalent:
$i$ is a local isomorphism, i. e. it gives an isomorphism on the identity components of the above groups: $I^0(M_1) \times \ldots \times I^0(M_k) \simeq I^0(M)$;
$\dim I(M) = \dim I(M_1) + \cdots + \dim I(M_k)$;
At most one of $M_i$'s has a nontrivial Euclidean de Rham factor.
Moreover, if these conditions are satisfied, then $i$ is an isomorphism if and only if there does not exist $i \ne j$ such that $M_i$ and $M_j$ share isometric de Rham factors. (If all $M_i$'s are irreducible, it means that no two of them are isometric.)
In order to prove this, just de Rham decompose each factor, stack all Euclidean subfactors together, and apply the second proposition from my answer linked above.<|endoftext|>
TITLE: Using irreducible characters of the orthogonal group as basis for homogeneous symmetric polynomials
QUESTION [10 upvotes]: The irreducible characters of the orthogonal group $O(2N)$ are given by
$$ o_\lambda(x_1,x_1^{-1},...x_N,x_N^{-1})=\frac{\det(x_j^{N+\lambda_i-i}+x_j^{-(N+\lambda_i-i)})}{\det(x_j^{N-i}+x_j^{-(N-i)})}$$
I was playing with them as basis for the space of homogeneous symmetric polynomials. I wanted to write the function
$$p_3=\sum_{i=1}^N (x_i^3+x_i^{-3})$$
as a linear combination of $o_\lambda$'s.
I started with $N=2$ and in that case I found that
$$ p_3=o_{(3)}-2o_{(2,1)}+o_{(1)}.$$
However, I then tried with $N=4$ and in that case I found that
$$ p_3=o_{(3)}-o_{(2,1)}+o_{(1,1,1)}.$$
I was expecting the coefficients to be the same, i.e. to be independent of $N$. (Independence of $N$ indeed holds when using characters of the unitary group, in which case the coefficients are the characters of the permutation group.)
Have I made some mistake or are the coefficients indeed dependent on $N$?
REPLY [3 votes]: (I have looked more carefully at this theory of "universal characters" mentioned by Stanley and am updating this answer according to what I learned. All of this was contained in the answer by Stanley, I am just unpacking it for the sake of those like me and the OP who may be confused.)
The following material is in the paper Young-diagrammatic methods for the representation theory of the classical groups of type $B_n$, $C_n$, $D_n$ by Koike and Terada, also in works by King (such as Modification Rules and Products of Irreducible Representations of the Unitary, Orthogonal, and Symplectic Groups, Journal of Mathematical Physics 12, 1588, 1971) and a very readable 1938 book by Murnaghan (The Theory of Group Representations).
$\DeclareMathOperator\Or{O}\DeclareMathOperator\U{U}$So functions $o_\lambda$ correspond to irreducible characters of $\Or(2N)$ only when $\ell(\lambda)\le N$. The set of such actual characters forms a basis for the space of symmetric functions on variables $\{x_1,x_1^{-1},\dotsc,x_N,x_N^{-1}\}$. However, the coefficients in the expansion of power sums in general depend on $N$.
On the other hand, partitions with $\ell(\lambda)>N$ do not define irreducible representations, so $o_\lambda$ is not an actual character. In that case they are replaced by another symmetric function, $o_\lambda\to o_{\widetilde{\lambda}}$ with a modified partition $\widetilde{\lambda}$.
When these functions are used, the large-$N$ expansions continue to hold for small $N$. In that sense they are 'universal'.
When a Schur function $s_\mu$ is decomposed in terms of $o_\lambda$, which corresponds to the branching rule of $\U(2N)\supset \Or(2N)$, one has $\ell(\lambda)\le \ell(\mu)$, so if $\ell(\mu)\le N$ only actual characters appear. This is not true if $N<\ell(\mu)\le 2N$, and then universal characters must also be used. This is not discussed in the classic book "Representation theory" by Fulton and Harris, for example, where only the case $\ell(\mu)\le N$ appears.
The modified partition $\widetilde{\lambda}$ is defined as follows. For $O(2N)$, let $m=2\ell(\lambda)-2N$. Then remove from the Young diagram of $\lambda$ a total of $m$ adjacent boxes, starting from the bottom of the first column and keeping always at the boundary of the diagram. In this way the changes to be implemented are, in sequence, $\lambda'_1\to\lambda'_2-1$, then $\lambda'_2\to\lambda'_3-1$, etc. until the procedures stops at some column $c$. If $m$ is too large and there are not enough boxes to accommodate this procedure, or if the remaining diagram is not a partition, then $o_\lambda=0$; otherwise $o_\lambda=(-1)^{c-1}o_{\widetilde{\lambda}}$.
For example, the universal decomposition for $p_4$ is
$$p_4=o_4-o_{31}+o_{211}-o_{1111}+o_0.$$
If $\lambda=(1,1,1,1)$ and $N=6$, we must remove $8-6=2$ boxes, in which case we get $c=1$ and $\widetilde{\lambda}=(1,1)$. Hence, for $O(6)$ we have $o_{1,1,1,1}=o_{1,1}$ and the universal relation reduces to $p_4=o_4-o_{31}+o_{211}-o_{11}+o_0$. When $N=4$, we must remove $8-4=4$ boxes from $\lambda=(1,1,1,1)$. We end up with no boxes at all, so $o_{1,1,1,1}=o_\emptyset$ for $O(4)$. We must remove $6-4=2$ boxes from $\lambda=(2,1,1)$, arriving at $o_{2,1,1}=o_2$. The relation reduces to $p_4=o_4-o_{31}+o_{2}$. Finally, take $N=2$. We cannot remove $8-2=6$ boxes from $(1,1,1,1)$, so $o_{1,1,1,1}=0$ for this group; when we remove $4-2=2$ boxes from $(3,1)$, the result is not the diagram of a partition, so $o_{3,1}=0$ for this group; removing $6-2=4$ boxes from $(2,1,1)$ leads to the empty partition, and the removing procedure ends in the second column so $c=2$, hence $o_{2,1,1}=-o_\emptyset$ for this group. Thereby the relation reduces to $p_4=o_4$.
Unfortunately, the modification rule is incorrectly stated in the recent "The Random Matrix Theory of the Classical Compact Groups" (Cambridge University Press, 2019), by Elizabeth Meckes.<|endoftext|>
TITLE: 3 term van der Waerden with large step size
QUESTION [7 upvotes]: Let $P(n)$ be the statement "any $n$ coloring of $\mathbb{N}$ contains a monochromatic progression $a, a+d, a+2d$ such that $d>a$".
For which $n$ is $P(n)$ true?
It's easy to see that $P(2)$ is true by a simple modification of the color focusing argument that is used in the traditional proof of van der Waerden's theorem. However, this argument does not seem to generalize to more colors, or at least not very easily.
It's also easy to see that a similar statement is not true for progressions of length $4$, even in the $2$ color case: just color $[2^n,2^{n+1}-1]$ red if $n$ is even and blue if $n$ is odd. If $2^n 2$. To see this, it suffices to show that $P(3)$ is false, because if we start with a $3$-coloring that witnesses the failure of $P(3)$, then we can always add a few more colors, say by using each of the new colors on just one or two numbers each, to obtain a witness to the failure of $P(n)$ for larger $n$.
Here is a coloring that shows $P(3)$ is false, inspired by your example for $2$ colors and length-$4$ progressions. Let $F_n$ denote the $n^{th}$ term of the Fibonacci sequence (with $F_1 = 1$ and $F_2 = 2$), and then
color $m$ red if $m \in [F_{3k},F_{3k+1})$ for some $k$,
color $m$ green if $m \in [F_{3k+1},F_{3k+2})$ for some $k$,
color $m$ blue if $m \in [F_{3k+2},F_{3k+3})$ for some $k$.
To see that this coloring really is a counterexample, let's suppose that $a$, $a+d$, and $a+2d$ all have the same color, and that $a < d$. Notice that this implies
$$\frac{3}{2} < \frac{a+2d}{a+d} < 2.$$
This does not immediately give us a contradiction, but it does tell us that $a+d$ and $a+2d$ must both lie in a single interval of the form $[F_n,F_{n+1})$.
(This is because $2(F_n-1) < F_{n+3}$ for all $n$. This isn't too hard to prove -- for large $n$ it follows from the fact that $F_{n+3}/F_n \approx \varphi^3 \approx 4.2$.) Once we know that $F_n \leq a+d < a+2d < F_{n+1}$, we get that
$$a = 2(a+d) - (a+2d) > 2F_n - F_{n+1} = 2F_n - (F_n+F_{n-1}) = F_n - F_{n-1} = F_{n-2}.$$
This tells us that if $a$ is to have the same color as $a+d$ and $a+2d$, then we must also have $a \in [F_n,F_{n+1})$. But now this is absurd: we have
$\frac{a+2d}{a} > 3$
while $\frac{F_{n+1}}{F_n} \leq 2$ for all $n$.<|endoftext|>
TITLE: Does the homotopy category of spaces admit a weak generating set?
QUESTION [11 upvotes]: As a follow-up to this question, let $\mathcal C$ be a category and $\mathcal S \subseteq \mathcal C$ a class of objects. Say that $\mathcal S$ is weakly generating if the functors $Hom_{\mathcal C}(S,-)$ are jointly conservative, for $S \in \mathcal S$. That is, a map $X \to Y$ in $\mathcal C$ is an isomorphism if and only if it induces a bijection $Hom_{\mathcal C}(S,X) \to Hom_{\mathcal C}(S,Y)$ for each $S \in \mathcal S$.
Question 1: Does the homotopy category of spaces admit a small generating set? (For example, as Simon Henry asks, do finite CW complexes work? How about the spheres?)
Of course, by Whitehead's theorem, the homotopy category of pointed connected spaces admits a generating set given by the spheres. But I'm not sure about unpointed spaces.
Note that the singleton set comprising the contractible space $\ast$ is a generator in the $\infty$-category of spaces, since $X \to Y$ is an equivalence if and only if $Map(\ast, X) \to Map(\ast,Y)$ is an equivalence. But passage to the the homotopy category discards the higher homotopy of the mapping spaces.
Question 2: More generally, if $\mathcal C$ is an accessible $\infty$-category, then does the homotopy category $h\mathcal C$ admit a small generating set? What if we assume that $\mathcal C$ is presentable?
Again, if $\mathcal C$ is $\kappa$-accessible, then the class $\mathcal C_\kappa$ of $\kappa$-compact objects forms a generating set in $\mathcal C$, but it's not clear if it forms a generating set in $h\mathcal C$. In fact, I think that Question 2 (in the "presentable" case) is equivalent to Question 1: if the answer to Question 1 is affirmative, so that $\mathcal S$ is a generating set for the homotopy category of spaces and $\mathcal T$ is a generating set for $\mathcal C$, then the set of spaces $S \ast T$ for $S \in \mathcal S, T \in \mathcal T$ forms a generating set for $h\mathcal C$. Here $\ast$ denotes copowering.
One result in this direction is Rosicky's Theorem, which says (in model-independent language) that if $\mathcal C$ is a presentable $\infty$-category, then the canonical functor $h\mathcal C \to Ind_\kappa(h\mathcal C_\kappa)$ is essentially surjective and full for some $\kappa$. For my purposes, it would suffice to know that this functor is conservative for some $\kappa$.
REPLY [13 votes]: This paper by Kevin Carlson and Dan Christensen says that the answer to question one is no: No set of spaces detects isomorphisms in the homotopy category, arXiv:1910.04141.<|endoftext|>
TITLE: Undetermined Banach-Mazur games in ZF?
QUESTION [22 upvotes]: This question was previously asked and bountied on MSE, with no response. This MO question is related, but is also unanswered and the comments do not appear to address this question.
Given a topological space $\mathcal{X}=(X,\tau)$, the Banach-Mazur game on $\mathcal{X}$ is the (two-player, perfect information, length-$\omega$) game played as follows:
Players $1$ and $2$ alternately play decreasing nonempty open sets $A_1\supseteq B_1\supseteq A_2\supseteq B_2\supseteq ...$.
Player $1$ wins iff $\bigcap_{i\in\mathbb{N}} A_i=\emptyset$.
ZFC implies that there is a subspace of $\mathbb{R}$ with the usual topology whose Banach-Mazur game is undetermined; on the other hand, it's consistent with ZF+DC (and indeed adds no consistency strength!) that no subspace of $\mathbb{R}$ does this ("every set of reals has the Baire property").
However, when we leave $\mathbb{R}$ things get much weirder. My question is:
Does ZF alone prove that there is some space $\mathcal{X}$ whose Banach-Mazur game is undetermined?
Controlling the behavior of all possible topological spaces in a model of ZF is extremely hard for me, and I suspect the answer to the question is in fact yes. In fact, I recall seeing a pretty simple proof of this; however, I can't track it down or whip up a ZF-construction on my own (specifically, everything I try ultimately winds up being a recursive construction killed by having too many requirements to meet in the given number of steps).
REPLY [10 votes]: This is only a partial answer. ZF + DC + 'every Banach-Mazur game is determined' is inconsistent.
Let $X$ be the set of all functions of the form $f: \alpha \rightarrow \{0,1\}$, with $\alpha$ an ordinal such that for any ordinals $\beta < \gamma$ with $\omega \cdot \gamma + \omega \leq \alpha$, the sets $(n \in \mathbb{N} : f(\omega \cdot \beta + n) = 1)$ and $(n \in \mathbb{N} : f(\omega \cdot \gamma + n) = 1)$ are distinct.
Let $\tau$ be the topology on $X$ generated by sets of the form $U_f = (g \in X : g \supseteq f)$, and let $\mathcal{X} = (X,\tau)$.
Claim: Player $1$ does not have a winning strategy for the Banach-Mazur game on $\mathcal{X}$.
Proof of claim: For any strategy $S$ for player $1$. Let $T$ be the tree of all initial segments of plays against $S$ such that each play by player $2$ is of the form $U_f$ for some $f \in X$ of length $\omega \cdot \alpha$ for some $\alpha$. If this tree fails to be pruned, then player $1$ has in some play of the game played a set $V$ such that for any $U_f \subseteq V$, $f$ enumerates every element of $2^{\mathbb{N}}$ extending $\sigma$ for some $\sigma \in 2^{<\omega}$. This would imply that $2^{\mathbb{N}}$ can be well-ordered, allowing us to construct a Bernstein set, contradicting our assumptions. Hence this must be a pruned tree of height $\omega$, so by dependent choice it has a path. Let $g$ be the union of all $f$ such that $U_f$ is on that path somewhere. By construction, $g \in X$, so we have that the strategy where player $2$ blindly plays the moves in this path wins against $S$.
So, it must be the case that player $2$ has a winning strategy $S$. For any $f \in X$ with length $\omega \cdot \alpha$ for some $\alpha$, let $T_f$ be the strategy for player $1$ that plays $U_f$ on player $1$'s zeroth move and on player $1$'s $n+1$st move, if player $2$ played $V$, then player $1$ plays $V \cap \bigcup_{\sigma \in 2^n} U_{f\frown \sigma\frown 0}$, if this is non-empty, and otherwise player $1$ plays $V \cap \bigcup_{\sigma \in 2^n} U_{f\frown \sigma\frown 1}$. (Note that since the union of these two is $V$, one of these must be non-empty.) Since $S$ is a winning strategy, for any $f\in X$ of length $\omega \cdot \alpha$ for some $\alpha$, the play of $T_f$ against $S$ must result in a nonempty intersection. By construction, for any $g$ and $h$ in that intersection, $g(\omega \cdot \alpha + n) = h(\omega \cdot \alpha +n)$ for all $n<\omega$, and the set $(n \in \mathbb{N} : h(\omega \cdot \alpha + n) = 1)$ must be distinct from $(n \in \mathbb{N} : f(\omega \cdot \beta + n) = 1)$ for any $\beta < \alpha$.
Therefore $S$ gives us a uniform procedure for picking a real not on a given well-ordered list of reals. By iterating this gives us a well-ordering of the reals. Therefore we can construct a Bernstein set from $S$ and we have that the Banach-Mazur game on that set is not determined, which contradicts our assumption. Therefore ZF + DC proves that there is an undetermined Banach-Mazur game.
Right now I don't see how to use a failure of DC to build an undetermined game.<|endoftext|>
TITLE: Do there exist any variational principles on the space of braids (or knots)?
QUESTION [5 upvotes]: This is very speculative question and I do not know where to start looking up the literature, or if what I am looking for is even mathematically possible/meaningful.
Q: I am interested in finding out whether any variational or action principles have been derived anywhere in the physics or mathematical literature (e.g. in study of geometry of configuration spaces etc.) on the space of braids (or knots).
Explicitly, some example where a Lagrangian is a function of a braid representation, and it is minimized/extremized under some constraints. The resulting equation would be differential equation whose solution gives a trajectory in the space of braids.
REPLY [4 votes]: O'Hara introduced knot energies, and a Möbius invariant case was studied by Freedman-He-Wang. For prime knots, Zheng-Xu He subsequently showed that there exists a smooth minimizer (up to Möbius transformation), and now it is known that critical points are analytic.
For braids, one can look at various variational principles by looking at metrics on Teichmuller space of the punctured sphere. The Teichmuller metric is a Finsler metric, with unique minimizing geodesics for pseudo-Anosov braids. The Weil-Petersson metric is a Riemannian metric which is incomplete, but for which pseudo-Anosovs also have unique minimizing representatives.
The minimizing geodesic then gives a canonical braid representative up to conjugacy (by choosing the representatives to have fixed center of mass and constant moment). One can also find unique representatives for braids up to choosing basepoint surfaces (e.g. $n$ points lying on the $x$-axis in $\mathbb{R}^2$).
For the 3-punctured sphere, the Teichmuller metric is the hyperbolic plane, and one can easily compute the geodesic.
One can also look at solutions to the planar $n$-body problem to get braid realizations. Richard Montgomery first proved the existence of solutions to the 3-body problem.<|endoftext|>
TITLE: Conjectured primality test for specific class of $N=4kp^n+1$
QUESTION [9 upvotes]: Can you provide a proof or counterexample for the following claim?
Let $P_m(x)=2^{-m}\cdot((x-\sqrt{x^2-4})^m+(x+\sqrt{x^2-4})^m)$ .
Let $N= 4kp^{n}+1 $ where $k$ is a positive natural number , $ 4k<2^n$ , $p$ is a prime number and $n\ge3$ . Let $a$ be a natural number greater than two such that $\left(\frac{a-2}{N}\right)=-1$ and $\left(\frac{a+2}{N}\right)=-1$ where $\left(\frac{}{}\right)$ denotes Jacobi symbol. Let $S_i=P_p(S_{i-1})$ with $S_0$ equal to the modular $P_{kp^2}(a)\phantom{5} \text{mod} \phantom{5} N$. Then $N$ is prime if and only if $S_{n-2} \equiv 0 \pmod{N}$ .
You can run this test here.
I have verified this claim for $k \in [1,500]$ with $p \leq 97$ and $n \in [3,50]$ .
Further generalization of the claim
A
Let $P_m(x)=2^{-m}\cdot((x-\sqrt{x^2-4})^m+(x+\sqrt{x^2-4})^m)$ .
Let $N= 2kp^{n} + 1 $ where $k$ is a positive natural number , $ 2k<2^n$ , $p$ is a prime number and $n\ge3$ . Let $a$ be a natural number greater than two such that $\left(\frac{a-2}{N}\right)=-1$ and $\left(\frac{a+2}{N}\right)=-1$ where $\left(\frac{}{}\right)$ denotes Jacobi symbol. Let $S_i=P_p(S_{i-1})$ with $S_0$ equal to the modular $P_{kp^2}(a)\phantom{5} \text{mod} \phantom{5} N$. Then $N$ is prime if and only if $S_{n-2} \equiv -2 \pmod{N}$ .
You can run this test here.
B
Let $P_m(x)=2^{-m}\cdot((x-\sqrt{x^2-4})^m+(x+\sqrt{x^2-4})^m)$ .
Let $N= 2kp^{n} - 1 $ where $k$ is a positive natural number , $ 2k<2^n$ , $p$ is a prime number and $n\ge3$ . Let $a$ be a natural number greater than two such that $\left(\frac{a-2}{N}\right)=1$ and $\left(\frac{a+2}{N}\right)=-1$ where $\left(\frac{}{}\right)$ denotes Jacobi symbol. Let $S_i=P_p(S_{i-1})$ with $S_0$ equal to the modular $P_{kp^2}(a)\phantom{5} \text{mod} \phantom{5} N$. Then $N$ is prime if and only if $S_{n-2} \equiv -2 \pmod{N}$ .
You can run this test here.
REPLY [7 votes]: The "if and only if" statement fails for
$$[p,n,k,a] \in \{ [3, 4, 1, 100], [3, 4, 1, 225], [3, 6, 13, 2901] \}$$ and many others. In these cases, $N$ is not prime, but the congruence $S_{n-2}\equiv 0\pmod{N}$ still holds.<|endoftext|>
TITLE: Notation for "the" left adjoint functor
QUESTION [13 upvotes]: As far as I know, there is no "official" notation for the left adjoint of a functor $F : \mathcal{C} \to \mathcal{D}$ if it exists. I have seen the notation $F^*$ sometimes, but this looks only nice when $F$ is already written as $F_*$, which is not practical. (This notation is then motivated by direct and inverse image functors. And it seems to be quite common for adjunctions between preorders aka Galois connections.) Similarly, I have seen the notation $F_*$ for the right adjoint of $F$, which only looks nice when $F$ is already written as $F^*$. I have also seen the notation $F^{\dagger}$ for the right adjoint, which looks nice, but then how would you denote the left adjoint if it exists? Perhaps ${}^{\dagger} F$? I don't want to start a debate here what is a good notation or not, since this is subjective anyway and is not suited for mathoverflow. I would like to know:
Are there any textbooks, influential papers or monographs on category theory which have introduced a notation for the left adjoint of $F$? Is there any notation which has been used by multiple authors?
Just to avoid any misunderstanding: Of course there is the official notation $F \dashv G$ when $F$ is left adjoint to $G$, but $\dashv$ is a relation symbol. I am interested in a function symbol (which makes sense since left and right adjoints are unique up to canonical isomorphism if they exist).
REPLY [4 votes]: P. Gabriel uses $F_\lambda$ for the left adjoint and $F_\rho$ for the right adjoint. See for instance p. 344 of the article "Covering spaces in representation theory" by K. Bongartz and P. Gabriel (Invent. Math. 65, 1982) (EuDML).<|endoftext|>
TITLE: Automorphism groups of odd order
QUESTION [11 upvotes]: This is inspired by this question. Is there a description of finite groups without automorphisms of order $2$?
REPLY [4 votes]: New version (existence hinted in previous version): If $G$ is a non-trivial finite (solvable) group of odd order with $\Phi(G) = 1$, then $G$ has an automorphism of order $2$.
It is well-known and easy to check that $F = F(G)$ is a product of minimal normal subgroups of $G$, each an elementary Abelian $p_{i}$-group for some prime $p_{i}$.
Also, $F$ is well-known to be complemented in $G$ in this case (I give a proof for completeness:
Choose a proper subgroup $H$ of $G$ minimal subject to $G = FH$ (such exists because $1 \neq F \not \leq \Phi(G)$). Then $(H \cap F) \leq \Phi(H)$ by minimality of $H$. Also $F \cap H$ is normal in $\langle H,F \rangle = G$, since $F$ is Abelian and $F \lhd G$. If $F \cap H \neq 1$, then there is a maximal subgroup $M$ of $G$ with $G = (F \cap H)M$ since $\Phi(G) = 1$. Then $H = (F \cap H)(M \cap H)$ by Dedekind's modular law. But then $H = H \cap M \leq M$ since $F \cap H \leq \Phi(H)$. But then $G = (F \cap H)M \leq M$, contrary to the fact that $M$ is maximal).
Now $G = FH$ for some subgroup $H$ of ${\rm Aut}(F)$, and the product is semidirect. Thus $G$ is isomorphic to a subgroup of the holomorph $ X = F{\rm Aut}(F)$ (the semidirect product of $F$ with its automorphism group). Here, we have $G \cong F{\rm Aut}_{G}(F)$, where ${\rm Aut}_{G}(F)$ is the subgroup of ${\rm Aut}(F)$ induced by the conjugation action of $G$ on $F$.
Now let $t$ be the central element of ${\rm Aut}(F)$ of order $2$ which inverts $F$ elementwise (note that $t$ is indeed central in ${\rm Aut}(F)$, because $\alpha(f)^{-1} = \alpha(f^{-1})$ for every $\alpha \in {\rm Aut}(F)$). Then $F\langle t \rangle$ normalizes every subgroup of $X$ containing $F$, so normalizes $F{\rm Aut}_{G}(F) \cong G$.
Now $|(F{\rm Aut}_{G}(F))(F \langle t\rangle)| = 2|F{\rm Aut}_{G}(F)|$, so that $t$ induces an automorphism of order $2$ of $F{\rm Aut}_{G}(F) \cong G$ (recall that $t$ already inverts $F$ elementwise). Note that $F{\rm Aut}_{G}(F)$ is of index $2$ in $(F{\rm Aut}_{G}(F))(F \langle t\rangle)$, so is normal in the latter group.<|endoftext|>
TITLE: Almost transferred model structures
QUESTION [6 upvotes]: Let $F : \mathcal{C} \leftrightarrows \mathcal{D} : U$ be a Quillen adjunction between cofibrantly generated model categories. The model structure on $\mathcal{D}$ is called transferred if $U$ preserves and reflects weak equivalences and fibrations. It follows that cofibrations of $\mathcal{D}$ are generated by $F(\mathrm{I})$, where $\mathrm{I}$ is a set of generating cofibrations of $\mathcal{C}$.
Now, assume that cofibrations of $\mathcal{D}$ are generated by $F(\mathrm{I})$ and that $U$ reflects fibrant objects (or even all fibrations). Is the model structure on $\mathcal{D}$ necessarily transferred in this case? If these conditions hold and the transferred model structure exists, then it necessarily coincides with the given one. Thus, the question can be reformulated as follows. Is there a model category $\mathcal{D}$ satisfying conditions given above such that the transferred model structure on $\mathcal{D}$ does not exist?
REPLY [5 votes]: This is too long for a comment, so I'm putting it here. I can think of three places to look for such an example. First, you could check out Example 2.8 in this paper I wrote with Michael Batanin: Bousfield Localization and Eilenberg-Moore Categories, arXiv:1606.01537.
In that example, we proved that the transferred model structure doesn't exist, so you could check if there is a different model structure on the category of operads valued in Ch($\mathbb{F}_2)$ that has the property you want.
Second, it is well-known that the transferred model structure on commutative differential graded algebras does not exist in characteristic $p>0$. So, CDGA($\mathbb{F}_p)$ doesn't have a transferred model structure. It does have a model structure discovered by Donald Stanley: Determining Closed Model Category Structures, (1998) (link),
and you could check if Stanley's model structure has the property you want. Seems plausible.
Third, you could try to construct such an example using the techniques of Reid Barton at this answer:
Counter-example to the existence of left Bousfield localization of combinatorial model category
You'd want to pick $C$ and $D$ to be very small, like Reid did. I would try to work this out, but I just don't have time (and probably won't all semester).<|endoftext|>
TITLE: Cyclic action on Kreweras walks
QUESTION [22 upvotes]: A Kreweras walk of length $3n$ is a word consisting of $n$ $A$'s, $n$ $B$'s, and $n$ $C$'s such that in any prefix there are at least as many $A$'s as $B$'s, and at least as many $A$'s as $C$'s. For example, with $n = 3$, one Kreweras walk is: $w = AABBCACCB$. These are the same as walks in $\mathbb{Z}^2$ from the origin to itself consisting of steps $(1,1)$, $(-1,0)$, and $(0,-1)$ which always remain in the nonnegative orthant (treat $A$'s as $(1,1)$ steps, $B$'s as $(-1,0)$ steps, and $C$'s as $(0,-1)$ steps). Kreweras in 1965 proved that the number of Kreweras walks is $\frac{ 4^n(3n)!}{(n+1)!(2n+1)!}$ (OEIS sequence A006335). Many years later, in the 2000's, the Kreweras walks became a motivating/foundational example in the theory of "walks with small steps in the quarter plane" as developed by Mireille Bousquet-Mélou and her school. They also are related to decorated planar maps and in particular are a key ingredient in recent breakthrough work relating random planar maps to Liouville quantum gravity.
I discovered a very interesting cyclic action on Kreweras walks, which apparently had not been noticed previously. Let me refer to this action as rotation. To perform rotation on a Kreweras walk $w$, first we literally rotate the word $w =(w_1,w_2,...,w_{3n})$ to $w' = (w_2,w_3,...,w_{3n},w_1)$. With the above example of $w$, we get $w' = ABBCACCBA$. This is, however, no longer a valid Kreweras walk. So there will be a smallest index $i$ such that $(w'_1,...,w'_i)$ has either more $B$'s than $A$'s, or more $C$'s than $A$'s. Then we create another word $w''$ by swapping $w'_i$ and $w'_{3n}$ (which is always an $A$). For example, with the previous example, we have $i = 3$ (corresponding to the second $B$ in the word), and we get $w'' = ABACACCBB$. It's not hard to see that the result is a Kreweras walk, which we call the rotation of the initial Kreweras walk. The sequence of iterated rotations of our initial $w = AABBCACCB$ example looks like
$$ 00 \; AABBCACCB \\
01 \; ABACACCBB \\
02 \; AACACCBBB \\
03 \; ACACABBBC \\
04 \; AACABBBCC \\
05 \; ACABBACCB \\
06 \; AABBACCBC \\
07 \; ABAACCBCB \\
08 \; AAACCBCBB \\
09 \; AACCBABBC \\
...$$
In particular, $3n = 9$ applications of rotation yields the Kreweras walk which is the same as our initial $w$ except that the $B$'s and $C$'s have swapped places. If we did another $9$ applications we would get back to our initial $w$.
Conjecture: For a Kreweras walk of length $3n$, $3n$ applications of rotation always yields the Kreweras walk which is the same as the initial walk except with the $B$'s and $C$'s swapped (so $6n$ applications of rotation is the identity).
(So my question is obviously: is my conjecture right?) I've thought about this conjecture a fair amount with little concrete progress. I've done a fair amount of computational verification of this conjecture: for all $n \leq 6$, and for many thousands more walks with various $n \leq 30$.
Where this action comes from: The Kreweras walks of length $3n$ are in obvious bijection with the linear extensions of a poset $P$, namely, $P=[n] \times V$, the direct product of the chain on $n$ elements and the 3-element ``$V$'' poset with relations $A < B$, $A < C$. I became aware of this poset thanks to this MO answer of Ira Gessel to a previous question of mine, which cited this paper of Kreweras and Niederhausen in which the authors prove not just a product formula for the number of linear extension of $P$, but for the entire order polynomial of $P$. The rotation of Kreweras walks as I just defined it is nothing other than the famous (Schützenberger) promotion operation on linear extension of a poset (see this survey of Stanley's for background on promotion). There are few non-trivial classes of posets for which the behavior of promotion is understood (see section 4 of that survey of Stanley's), so it is very interesting to discover a new example. In particular, all known examples are connected to tableaux and symmetric functions, etc.; whereas this Kreweras walks example has quite a different flavor.
Some thoughts: The analogous rotation action on words with only $A$'s and $B$'s (i.e., Dyck words) is well-studied; as explained in section 8 of this survey of Sagan's on the cyclic sieving phenomenon, it corresponds to promotion on $[2]\times[n]$, and in turn to rotation of noncrossing matchings of $[2n]$. There is a way to view a Kreweras walk as a pair of noncrossing partial matchings on $[3n]$ (basically we form the matching corresponding to the $A$'s and $C$'s, and to the $A$'s and $B$'s). But this visualization does not seem to illuminate anything about the rotation action (in particular, when we rotate a walk, one of the noncrossing partial matchings simply rotates, but something complicated happens to the other one).
As mentioned earlier, there is a bijection due to Bernardi between Kreweras walks and decorated cubic maps, but I am not able to see any simple way that this bijection interacts with rotation.
On a positive note, it seems useful to write the $3n$ rotations of a Kreweras walk in a cylindrical array where we indent by one each row, as follows:
$$
\begin{array} \,
A & A & B & b & C & A & C & C & B \\
& A & b & A & C & A & C & C & B & B \\
& & A & A & C & A & C & c & B & B & B \\
& & & A & c & A & C & A & B & B & B & C \\
& & & & A & A & C & A & B & B & b & C & C \\
& & & & & A & c & A & B & B & A & C & C & B \\
& & & & & & A & A & B & b & A & C & C & B & C \\
& & & & & & & A & b & A & A & C & C & B & C & B \\
& & & & & & & & A & A & A & C & C & B & c & B & B \\
& & & & & & & & & A & A & C & C & B & A & B & B & C
\end{array}
$$
In each row I've made lowercase the $B$ or $C$ that the initial $A$ swaps with. We can extract from this array a permutation which records the columns in which these matches occur (where we cylindrically identify column $3n+i$ with column $i$). In this example, the permutations we get is $p = [4,3,8,5,11,7,10,9,15] = [4,3,8,5,2,7,1,9,6]$. The fact that this list of columns is actually a permutation (which I don't know how to show) is equivalent to the assertion that the position of the $A$'s after $3n$ rotations is the same as in the initial Kreweras walk. Furthermore, this permutation $p$ determines position of $A$'s. Namely, the $A$'s are exactly the $p(i)$ for which $p(i) < i$. In our example, these are $2$, $1$, and $6$, corresponding to $i = 5,7,9$. Also, you can see how the $3n$ rotations "permute" the position of the $A$'s from $p$ as well. To do that, write down a new permutation $q$ from $p$: $q$ is the product of transpositions $q = (3n, p(3n)) \cdots (2, p(2)) \cdot (1, p(1))$. Then $q$ exactly tells us how the $A$'s are permuted. In our example, as we process the transpositions of $q = (9,6)(8,9)(7,1)(6,7)(5,2)(4,5)(3,8)(2,3)(1,4)$ right-to-left on the positions $\{1,2,6\}$ of $A$'s we see $1 \to 4 \to 5 \to 2$; $2 \to 3 \to 8 \to 9 \to 6$; and $6 \to 7 \to 1$. Note that the $A$'s end up changing places, and that they each are involved in a different number of swaps. Another thing worth noting is that the permutation $p$ does not determine the Kreweras walk (even after accounting for the $B \leftrightarrow C$ symmetry).
In spite of these observations, the lack of any connection to algebra (e.g., the representation theory of Lie algebras), and the lack of any good "model" for these words, makes it really hard to reason about how they behave under rotation.
EDIT:
Let me add one example which may indicate some subtlety. Let's define a $k$-letter Kreweras word of length $kn$ to be a word consisting of $n$ A's, $n$ B's, $n$ C's, $n$ D's, etc. for $k$ different letters such that in any prefix there are at least as many $A$'s as $B$'s, at least as many $A$'s as $C$'s, at least as many $A$'s as $D$'s, etc. So $3$-letter Kreweras words are the Kreweras walks discussed above, and $2$-letter Kreweras words are the Dyck words. We can define rotation for $k$-letter Kreweras words in exactly the same manner: literally rotate the word, find the first place the inequalities are violated, swap this place with the final $A$ to obtain a valid word (and this corresponds to promotion on a certain poset).
For the case $k=2$, note that $kn$ applications of rotation to a $k$-letter Kreweras word of length $kn$ results in a word with the $A$'s in the same position (because this is just rotation of noncrossing matchings). For the case $k=3$, apparently $kn$ applications of rotation results in a word with the $A$'s in the same position (because apparently the $B$'s and $C$'s switch). But for $k > 3$, it is not true necessarily that $kn$ applications of rotation results in a word with the $A$'s in the same position. For instance, with $k=4$ and $n=3$, starting from the word $w=AADACCDCBDBB$, 12 rotations gives us:
$$ 00 \; AADACCDCBDBB \\
01 \; ADACCDABDBBC \\
02 \; AACCDABDBBCD \\
03 \; ACADABDBBCDC \\
04 \; AADABDBBCDCC \\
05 \; ADABDBACDCCB \\
06 \; AABDBACDCCBD \\
07 \; ABDAACDCCBDB \\
08 \; ADAACDCCBDBB \\
09 \; AAACDCABDBBD \\
10 \; AACDCABDBBDC \\
11 \; ACDAABDBBDCC \\
12 \; ADAABDBBDCCC $$
where the $A$'s do not end up in the same positions they started in. So something kind of subtle has to be happening in the case $k=3$ to explain why they do.
REPLY [6 votes]: Martin Rubey and I solved my conjecture.
The basic idea of the proof is as follows. First to a Kreweras word $w$ we associate what we call its bump diagram, which is just the union of the two noncrossing partial matchings of $\{1,2,...,3n\}$ associated to $w$ (the one for the A's and B's and the one for the A's and C's), drawn as a graph in the obvious way. For example, with $w=AABBCACCB$ its bump diagram is
We also think of this diagram as a set of ordered pairs ('arcs'); in this example that set is
$$\{ (1,4),(1,8),(2,3),(2,5),(6,7),(6,9)\} $$
We extract a permutation $\sigma_w$ of $\{1,2,...,3n\}$ from the bump diagram as follows.
For $i=1,2,...,3n$, we take a trip from position $i$. We start traveling from position $i$ along the unique arc ending at $i$ (if $w_i=B$ or $C$), or the "shorter arc" beginning at $i$ (if $w_i=A$), and we continue until we reach a "crossing" of arcs. When we hit a crossing of arcs $(i,k)$ and $(j,\ell)$ with $i \leq j < k < \ell$ (note that we allow the case $i=j$), we follow the following "rules of the road":
if we were heading towards $i$, then we turn right and head towards $\ell$;
if we were heading towards $\ell$, then we turn left and head towards $i$;
otherwise we continue straight to where we were heading.
When we finish our trip at position $j$ then we define $\sigma_w(i) := j$.
For example, to compute $\sigma_w(3)$: we start traveling along the arc $(2,3)$ heading towards $2$; then we come to the crossing of $(2,3)$ and $(2,5)$ and we turn right, heading towards $5$; then we come to crossing of $(1,4)$ and $(2,5)$ and we turn left, heading towards $1$; then we come to the crossing of $(1,4)$ and $(1,8)$ and we turn right, heading towards $8$; then we come to the crossing of $(1,8)$ and $(6,9)$, but we just continue straight to $8$; and so we finish our trip at $8$. So $\sigma_w(3)=8$.
Or to compute $\sigma_w(7)$: we start traveling along the arc $(6,7)$ heading towards $6$; then we come to the crossing of $(6,7)$ and $(6,9)$ and we turn right, heading towards $9$; then we come to the crossing of $(1,8)$ and $(6,9)$ and we turn left, heading towards $1$; and then we come to the crossing of $(1,4)$ and $(1,8)$, but we just continue straight to $1$; and so we finish our trip at $1$. So $\sigma_w(7)=1$.
We could compute the whole permutation is $\sigma_w = [4,3,8,5,2,7,1,9,6]$.
You might notice that this example $w$ is the same as the original post and that this permutation $\sigma_w$ is the same as the "permutation" $p$ defined in terms of the cylindrical rotation array.
Indeed, this always happens (that the trip permutation is the same as the permutation from the cylindrical rotation array). It follows from the main lemma behind the overall proof, which is
Lemma. If $w'$ is the rotation of $w$, then $\sigma_{w'} = c^{-1} \sigma_w c$ where $c= (1,2,3,...,3n)$ is the "long cycle."
As a remark, these trip permutations come from the theory of plabic graph (cf. Section 13 of Postnikov's paper https://arxiv.org/abs/math/0609764).
Since $\sigma_w$ does not completely determine $w$, to finish the proof we need to keep track of a little more data. For that purpose, we define $\varepsilon_w=(\varepsilon_w(1),...,\varepsilon_w(3n))$, a sequence of $3n$ letters which are B's or C's, defined by
$$ \varepsilon_w(i) := \begin{cases} w_{\sigma(i)} &\textrm{if $w_{\sigma(i)}\neq A$} \\ w_{\sigma(\sigma(i))} &\textrm{if $w_{\sigma(i)}= A$}. \end{cases} $$
Similarly to the previous lemma, we can show
Lemma. If $w'$ is the rotation of $w$, then $\varepsilon_{w'} = (\varepsilon_w(2),\varepsilon_w(3),...,\varepsilon_w(3n),-\varepsilon_w(1))$ with the convention that $-B=C$ and vice-versa.
The above lemmas easily imply that the $3n$th rotation of $w$ is obtained from $w$ by swapping B's and C's.
Martin and I will post a preprint to the arXiv with all the details soon.
EDIT: The paper is now on the arXiv at https://arxiv.org/abs/2005.14031.<|endoftext|>
TITLE: A classification of $G_{\delta\sigma}$ zero-dimensional spaces?
QUESTION [6 upvotes]: Among separable metrizable spaces:
Cantor set is the unique compact zero-dimensional space without isolated points.
$\mathbb Q$ is the unique countable space without isolated points
$\mathbb R \setminus \mathbb Q$ is the unique zero-dimensional, $G_\delta$-space with no compact neighborhood.
$\mathbb Q ^\omega$ is the unique zero- dimensional, first category $F_{\sigma\delta}$-space with the property that no nonempty clopen subset is a $G_{\delta\sigma}$-space.
Question. Is there a simple classification of zero-dimensional $G_{\delta\sigma}$-spaces which have no compact neighborhoods?
The simplest examples would be $\mathbb Q$, $\mathbb R\setminus \mathbb Q$, and $\mathbb Q\times (\mathbb R\setminus \mathbb Q)$. Are there many others?
Is there a nice characterization of $\mathbb Q\times (\mathbb R\setminus \mathbb Q)$?
REPLY [5 votes]: This paper by Van Mill from 1981 gives a characterisation of $\Bbb Q \times \Bbb P$ (where $\Bbb P$ is a common notation for the irrationals) in Thm 5.3:
If $X$ is separable metrisable and zero-dimensional, $\sigma$-complete and nowhere complete and nowhere $\sigma$-compact then $X \simeq \Bbb Q \times \Bbb P$.
Where by complete I mean topologically complete (i.e. in this context: completely metrisable) and a nowhere-$P$ space is one where no non-empty open subset has property $P$, so $\Bbb P$ and $\Bbb Q$ are nowhere locally compact, e.g.)
I think $\Bbb Q \times C$ (with $C$ the Cantor set) is another example for your list.
A lot of information can be found in van Engelen's PhD-thesis from 1985: homogeneous zero-dimensional absolute Borel sets, where he shows there are are $\omega_1$ many homeomorphism types of subsets of $C$ that are both $F_{\sigma \delta}$ and $G_{\delta\sigma}$, also separately written up here. In his thesis he also gave the first characterisation of $\Bbb Q^\omega$ (now relegated to the appendix of it). Look up Van Engelen's work from around that time for related results.<|endoftext|>
TITLE: Differential forms on standard simplices via Whitney extension vs diffeological structure
QUESTION [7 upvotes]: The standard simplices $\Delta^n \subset \{\mathbf{x}\in\mathbb{R}^{n+1}\mid x_0 + \ldots + x_n =1 \} =: \mathbb{A}^n$ carry two natural sorts of smooth differential forms:
Those differential forms on the interior of $\Delta^n$ that extend smoothly to a neighbourhood of $\Delta^n$ in $\mathbb{A}^n$ (this definition is used for instance by Dupont in his book Curvature and Characteristic Classes). This is implicitly using Whitney's extension theorem, I think.
Consider $\Delta^n$ as a diffeological subspace of $\mathbb{A}^n$, where a function $\phi\colon \mathbb{R}^k \to \Delta^n$ is a plot if and only if the composite $\mathbb{R}^k \to \Delta^n \hookrightarrow \mathbb{A}^n$ is smooth. A differential form $\omega$ on $\Delta^n$ then consists of the data of a differential form $\phi^*\omega$ for each plot $\phi$ with a compatibility condition when a plot factors through another via a smooth map between Euclidean spaces.
The first of these is more of a "maps out" viewpoint, and probably corresponds to a natural smooth space structure defined via smooth real-valued functions. The second is a "maps in" viewpoint. Note that the $D$-topology arising from the diffeology in 2. above is the standard topology on the simplex. We then get a cochain complex of differential forms of each type, as exterior differentiation can be defined in the more-or-less obvious way in each case.
My question is: how do these relate? Is one a subcomplex of the other? Or are they quasi-isomorphic, via a third cochain complex?
The motivation is that Dupont's simplicial differential forms on semisimplicial manifolds $X_\bullet$ look like they should be differential forms on the fat geometric realisation considered as a diffeological space, since a simplicial differential form is more or less descent data for the sheaf of differential forms and the 'cover' $\coprod_{n\geq 0} \Delta^n\times X_n \to ||X_\bullet||$, assuming the first definition above. However, if his differential forms on $\Delta^n$ (or more precisely on $\Delta^n\times X_n$) aren't diffeological differential forms, they don't give a form on $||X_\bullet||$ as a diffeological space. I guess all we really need is a map of cochain complexes from the first to the second given above.
REPLY [3 votes]: The two chain complexes are isomorphic for any $n≥0$.
Fix some $n≥0$, $k≥0$ and consider $k$-forms on the $n$-simplex.
I will use the notations $Ω_e^k$ and $Ω_d^k$ for forms of type 1 and 2 respectively.
I also use the notation $Δ_d^n$ for the diffeological $n$-simplex
so that $\def\Hom{\mathop{\rm Hom}} \def\R{{\bf R}} \def\d{\,{\rm d}} Ω_d^k=\Hom(Δ_d^n,Ω^k)$.
First, we have a canonical map $ι\colon Ω_e^k→Ω_d^k$ that
sends $ω∈Ω_e^n$ to the map $Δ_d^n→Ω^k$
that sends a smooth map $φ\colon S→Δ_d^n$ to the $k$-form $φ^*ω∈Ω^k(S)$.
Secondly, the map $ι$ is injective,
which follows from the following two observations.
By the Yoneda lemma the restriction of $ι(ω)\colon Δ_d^n→Ω^k$ to the sheaf represented by the open interior of $Δ_d^n$ equals the restriction of $ω$ to the open interior of $Δ^n$.
Furthermore, any two forms in $Ω_e^k$ whose restrictions to the open interior of $Δ^n$ coincide must be equal (by continuity).
Thirdly, the map $ι$ is surjective.
This can be shown using a two-step construction:
in the first step we construct a certain element $ω$ of $Ω_e^k$
starting from some given element $ψ$ of $Ω_d^k$
and in the second step we show that $ι(ω)=ψ$.
First, let's briefly examine the simplest nontrivial case $d=n=1$.
We have a form $ψ∈Ω_d^1(Δ_d^1)=Ω_d^1([0,1])$,
and we already know its restriction $f(x) \d x$ to the open interval $(0,1)$.
In particular, $f$ is a smooth function on $(0,1)$.
Pulling back $ψ$ along the plot $\R→\R$ ($x↦x^2$)
yields some 1-form $g(x) \d x$, where for any $x≠0$ we have $g(x)=2xf(x^2)$
and $g$ is a smooth function on $(-1,1)$.
Since $g$ is odd, we have $g(0)=0$,
so the even function $h$ given by $h(x)=g(x)/(2x)$ is smooth on $(-1,1)$
and we have $h(x)=f(x^2)$ for all $x≠0$.
Set $f(0)=h(0)$.
We claim $f$ is smooth on $(-1,1)$.
Indeed, $h'(x)=2xf'(x^2)$ for $x≠0$,
and since $h'$ is odd, we have $h'(0)=0$
and $h'(x)/(2x)$ is a smooth function on $(-1,1)$
whose value at $x=0$ is $f'(0)$.
We repeat this step by induction, proving $f$ has derivatives of all orders at 0.
The general case is nothing else than a multivariable paremetrized
version of the above argument.
More precisely, we argue as follows.
For the first step, suppose we are given some $ψ∈Ω_d^k$.
Given some point $x∈Δ^n$, we would like to define $ω(x)$.
Denote by $d≥0$ the codimension of the stratum of $Δ^n$ that contains $x$.
Parametrize the simplex $Δ^n$ using $n$ coordinates
in such a way that $x_1=⋯=x_d=0$
and the other $n-d$ coordinates of $x$ are strictly positive,
so that the points $y$ of the stratum containing $x$ are defined by the relations $y_i≥0$ for $1≤i≤d$.
Decompose $$ω=∑_I g_I ∏_{i∈I} \d x_i$$ into its individual components with respect to this coordinate system.
Pick one such component $g_I ∏_{i∈I} \d x_i$; we would like to define $g_I$ as a smooth function on some open neighborhood $U$ of the given stratum of $Δ^n$, which is parametrized by the remaining $n-d$ coordinates $x_{d+1}$, …, $x_n$.
Consider the smooth map $U→Δ^n$ that (in the coordinates introduced above)
sends $x_i↦x_i^2$ for $1≤i≤d$ and $x_i↦x_i$ for $i>d$.
This defines a morphism $τ\colon U→Δ_d^n$, so we have a form $τ^*ψ∈Ω^k(U)$.
Now take the coefficient $h$ of $τ^*ψ$ before $∏_{i∈I} dx_i$.
Take the partial derivative of $h$ with respect to all coordinates $x_i$
such that $i∈I$ and $i≤d$.
Divide the resulting function by $2^{\#(I∩\{1,…,d\})}$.
This is the function $g_I$.
This argument also proves that the resulting $k$-form $ω$ is smooth.
For the second step, we have to show that $ι(ω)=ψ$.
Reusing the notation of the previous paragraph,
consider some arbitrary plot $φ\colon S→Δ_d^n$ such that $φ(s)=x$ for some $s∈S$.
The first $d$ coordinates of $φ$ must be nonnegative in a neighborhood $U$ of $s$.
Since $φ$ is differentiable, the first derivatives of these $d$ coordinates must vanish at the point $s$.
Thus, taking the square root of each of the first $d$ coordinates produces a smooth map $ε\colon U→\R^n$ such that $φ=τε$, where the map $τ$ was defined in the previous paragraph.
Now $φ^*ι(ω) = ε^* τ^* ι(ω) = ε^* τ^*ψ = φ^*ψ$,
where $τ^*ι(ω)=τ^*ψ$ by definition of $ω$.<|endoftext|>
TITLE: Empty interior of union of cosets?
QUESTION [5 upvotes]: The following question arises from trying to understand Lemma 1.3(ii) of arXiv:math/0405063. I believe a particular case of the proof (and in fact I think the proof is essentially equivalent to this claim) is:
Let $G$ be a locally compact group. Let $C,D$ be cosets (not assumed open, closed etc.) each of which has empty interior. Then $C\cup D$ also has empty interior.
This is not try in general topology, of course: let $C,D$ be the rational, respectively, irrationals, in $\mathbb R$. However, I cannot decide if being a coset rules out this sort of example. Is the claim true, and if so, what is a proof?
REPLY [4 votes]: This is false. Take the (compact abelian) group $G=(\mathbf{Z}/2\mathbf{Z})^\mathbf{N}$ and let $H$ be a dense subgroup of index 2 (there are many, since $G$ has only countably many closed subgroups of index 2 but has $2^c$ subgroups of index $2$, and clearly a subgroup of index 2 is either closed or dense). Then $G=H\cup (G\smallsetminus H)$ and both $H$ and its coset $G\smallsetminus H$ have empty interior.<|endoftext|>
TITLE: Scattering theory for Coulomb potential
QUESTION [5 upvotes]: Both physical and mathematical theories of quantum scattering seem to be well developed in the case when the potential (or a more general perturbation of the Laplacian) decays fast enough at infinity and satsifies some regularity assumptions. For example under such rather general assumptions existence of $S$-matrix is proved in Ch. XIV of Hormander's "The Analysis of Linear Partial Differential Operators II".
However the Coulomb potential does not satisfy these assumptions of fast decay at infinity. Moreover my impression is that this is not just a technical issue, but the whole theory should look different for the Coulomb potential.
I am wondering if there is any notion of $S$-matrix for the Coulomb potential and how it looks like.
A reference would be helpful. Apologies if the question is not advanced enough- I am not an expert.
REPLY [5 votes]: The $1/r$ Coulomb potential needs to be regularized, typically this is done by studying the Yukawa potential $e^{-\alpha r}/r$ and taking the limit $\alpha\rightarrow 0$ at the end. A recent critical examination of this procedure can be found in Regularization of the Coulomb scattering problem (2004).<|endoftext|>
TITLE: Maximizing $\iiint|(x-z)\times(y-z)|d\mu d\mu d\mu$ over probability measures on the unit circle
QUESTION [5 upvotes]: What probability measure(s) maximize the quantity $\iiint_{\mathbb{S}^1}|(x-z)\times(y-z)|d\mu(x)d\mu(y)d\mu(z)$?
The answer appears to be uniform measure, since informally it appears better to have more triangles in the support of $\mu$ which the function $|(x-z)\times(y-z)|$ computes the area of.
Is there an argument that shows this is true generally for $\mathbb{S}^{d-1}$?
Edit: Thanks to @fedja for the answer to the main question and the request for clarification. While it could be possible to replace the area of triangles with volumes of simplices (and so on), what was meant by generalization was most immediately the question of whether uniform measure maximizes $$\iiint_{\mathbb{S}^2}|(x-z)\times(y-z)|d\mu(x)d\mu(y)d\mu(z).$$
REPLY [6 votes]: Yes, it is true for the circle (though the reason is not quite the one you suggested). We shall consider the discrete version of the problem, which is to put some odd number $n\ge 3$ of points (think of them as of being assigned the probability of $1/n$ each) on the circle so that the sum of triangle areas is maximized. Then an optimal configuration exists by compactness. We just want to show that it should be an equispaced distribution. Since you can obtain any measure on the circle as a weak limit of such discrete measures (and the integrand is continuous, so a small displacement of the measure does not change the integral much), you'll get the required optimality of the uniform measure as a limiting statement.
Take any of the points $e$ and enumerate all other points by the numbers $1,\dots,n-1$ clockwise starting from $e$ (so the whole system is $e,e_1,\dots,e_{n-1}$. Fix all points except $e$ and think a bit of how the total double area depends on the position of $e$. You'll realize quite soon that it is just some constant plus $v_e\times e$ where
$$v_e=\sum_{1\le k< m \le n-1}(e_m-e_k).$$
If you now consider for fixed $\ell<\frac n2$ all vectors $e_m-e_k$ with [$(\ell\le k< m\le n-\ell$) and ($k=\ell$ or $m=n-\ell$)], you'll see that $v_e=\sum_{\ell<\frac n2}A_\ell(e_{n-\ell}-e_\ell)$ with some $A_\ell>0$. You will be able to increase the sum of (even oriented) areas if $e$ is not perpendicular to $v_e$, so in the optimal configuration we must always have $\langle e,v_e\rangle=0$ for every choice of $e$.
Now recall the famous "drive around the circle without running out of gas" problem (more precisely, its particular case when all gas stations have the same amount of fuel sufficient for driving $\frac 1n$ of the entire circle). The conclusion of it is that you can choose $e$ so that the arc distance to $e_k$ is at most $\frac k n$ of the whole circle for every $k=1,\dots,n-1$, which makes all scalar products $\langle e,e_{n-\ell}-e_\ell\rangle$ with $\ell<\frac n2$ non-positive with the only chance of the equality $\langle e,v_e\rangle=0$ when the arc distance from $e$ to $e_k$ is exactly $\frac kn$ for all $k=1,\dots,n-1$.
As to higher dimensions, can you, please, specify first what you mean by "this"? (there are several ways to try to generalize the setup, so the word seems rather ambiguous to me :-) )<|endoftext|>
TITLE: Is it possible to multiply two series to get as a result all composite numbers?
QUESTION [6 upvotes]: I was toying with the following problem:
Is it possible to find two infinite integer sequences $(a_n), (b_n)>0$ such that $\sum_{n=1}^{\infty}\frac{1}{(a_n)^s}\cdot \sum_{n=1}^{\infty}\frac{1}{(b_n)^s}=\sum_{n=1}^{\infty}\frac{1}{(c_n)^s}$ for every $s>1$? Here $c_n$ denotes the $n$-th composite number.
I can show that without loss of generality, $1\in a_n$ and for every $x$ with $\Omega(x)=2, x\in (b_n)$ but this did not help much.
Can someone provide an answer to this problem?
REPLY [6 votes]: This is the answer of Greg Martin, with the correction of Mark Sapir, and details added.
Write $\Omega(n)$ for the number of prime factors of $n$ (counted with multiplicity), and $\Omega_{\operatorname{odd}}(n)$ for the number of odd prime factors of $n$ (counted with multiplicity), so $\Omega(n) = \Omega_{\operatorname{odd}}(n) + v_2(n)$ (where $v_2$ is the valuation at $2$).
Example. Let $A = \{1,2,4,8,\ldots\} = 2^{\mathbf Z_{\geq 0}}$, and let
$$B = \{n\ |\ \Omega(n) = 2\} \cup \{n \text{ odd}\ |\ \Omega(n) \geq 3\}.$$
For $n \in \mathbf Z_{>0}$, the number of representations $n = a \cdot b$ with $a \in A$ and $b \in B$ is $1$ if $n$ is composite, and $0$ otherwise.
Proof. Given $n \in \mathbf{Z}_{>0}$ composite (i.e. $\Omega(n) \geq 2$), define $k \in \mathbf Z_{\geq 0}$ as follows:
If $\Omega_{\operatorname{odd}}(n) \geq 2$, set $k = v_2(n)$.
If $\Omega_{\operatorname{odd}}(n) = 1$, set $k = v_2(n) - 1$.
If $\Omega_{\operatorname{odd}}(n) = 0$, set $k = v_2(n) - 2$.
In cases 2 and 3, note that $k \geq 0$ since $\Omega(n) \geq 2$. Then set $a = 2^k$ and $b = \tfrac{n}{a}$. Then $n = a \cdot b$, and clearly $a \in A$. We also have $b \in B$:
In case 1 above, $b$ is odd with $\Omega(b) \geq 2$;
In case 2 above, $b$ is even with $\Omega(b) = 2$;
In case 3 above, $b = 4$.
This shows existence of the desired decomposition. For uniqueness, assume $n = a \cdot b$ with $a \in A$ and $b \in B$. Let $m = v_2(n)$. Then $\Omega_{\operatorname{odd}}(b) = \Omega_{\operatorname{odd}}(n)$, so
If $\Omega_{\operatorname{odd}}(n) \geq 2$, then $\Omega_{\operatorname{odd}}(b) \geq 2$, which by definition of $B$ forces $b$ odd, hence $a = 2^m$.
If $\Omega_{\operatorname{odd}}(n) = 1$, then $\Omega_{\operatorname{odd}}(b) = 1$, which by definition of $B$ forces $b$ even and $\Omega(b) = 2$, hence $a = 2^{m-1}$.
If $\Omega_{\operatorname{odd}}(n) = 0$, then $\Omega_{\operatorname{odd}}(b) = 0$, which by definition of $B$ forces $b = 4$, hence $a = 2^{m-2}$.
This shows that $(a,b)$ must be as constructed above. Finally, since all elements of $B$ are composite, any integer of the form $n = a \cdot b$ with $a \in A$ and $b \in B$ is composite. $\square$<|endoftext|>
TITLE: Is the complement of a zero-dimensional subset of the plane path-connected?
QUESTION [10 upvotes]: Let $X$ be a zero-dimensional subset of the plane $\mathbb R ^2$. Is $\mathbb R ^2\setminus X$ necessarily path-connected? I feel the answer must be yes but I need a reference. If it helps, assume $X$ is nowhere dense.
REPLY [14 votes]: If the zero-dimensional set $X$ is not closed, then the answer is "no".
To construct a suitable example, take any open bounded neighborhood $U\subset\mathbb R^2$ of zero, whose boundary $\partial U$ does not contain a topological copy of $[0,1]$. For example, for $U$ we can take a bounded connected component of the complement of the union of two suitable pseudoarcs in the plane. Then the set $$X=\mathbb R^2\setminus\{\vec a+\tfrac1n\partial U:\vec a\in\mathbb Q^2,\;n\in\mathbb N\}$$ will have the desired property: it is zero-dimensional and its complement $\mathbb R^2\setminus X$ does not contain a copy of $[0,1]$ (by the Baire Theorem) and hence is not path-connected.<|endoftext|>
TITLE: A new cardinal characteristic (related to partitions)?
QUESTION [11 upvotes]: In this post I will discuss some cardinal characteristic of the continuum, related to partitions of $\omega$ and would like to know if it is equal to some known cardinal characteristic.
By a partition of $\omega$ I understand a cover of $\omega$ by pairwise disjoint nonempty subsets. A partition $\mathcal P$ is called finitary if $\sup_{P\in\mathcal P}|P|$ is finite.
A family $\mathfrak P$ of partitions of $\omega$ is called directed if for any two partitions $\mathcal A,\mathcal B\in\mathfrak P$ there exists a partition $\mathcal C\in\mathfrak P$ such that each set $S\in\mathcal A\cup\mathcal B$ is contained in some set $C\in\mathcal C$.
Let $\mathfrak P$ is a family of partitions of $\omega$. An infinite subset $D\subset\omega$ is called $\mathfrak P$-discrete if for any partition $\mathcal P\in\mathfrak P$ there exists a finite set $F\subset D$ such that for any $P\in\mathcal P$ the intersection $P\cap (D\setminus F)$ contains at most one point.
Let $\kappa$ be the smallest cardinality of a directed family $\mathfrak P$ of finitary partitions of $\omega$ admitting no infinite $\mathfrak P$-discrete set $D\subset\omega$.
It can be shown that $\mathfrak b\le\kappa\le\mathfrak c$ (the upper bound follows from the observation that any maximal directed family of finitary partitions has no infinite discrete set, see Proposition 6.5 in this preprint).
Problem 1. Is $\kappa$ equal to some known cardinal characteristic of the continuum?
Problem 2. Is $\kappa=\mathfrak c$ in ZFC?
Problem 3. Find lower and upper bounds on $\kappa$ (which are better than $\mathfrak b\le\kappa\le\mathfrak c$).
Added in Edit. The lower bound $\sup_{U\in\beta\omega}\pi(U)\le\kappa$, suggested by Todd Eisworth can be improved to $\mathfrak s\le \kappa$. One can also prove that $\max\{\mathfrak b,\mathfrak s,\}\le\mathfrak j\le\kappa\le\mathrm{non}(\mathcal M)$ and hence $\kappa$ is not equal to $\mathfrak c$. The cardinal $\mathfrak j$ is discussed in this MO-post.
REPLY [7 votes]: This is not an answer, but hopefully it's a helpful observation:
(1) If $U$ is an ultrafilter on $\omega$ and $\mathcal{P}$ is a finitary partition of $\omega$, then there is $A\in U$ such that $A\cap P$ contains at most one element for each $P\in\mathcal{P}$.
(As if each piece of the partition has cardinality at most $n$, then there is a $k\leq n$ such that the union of pieces with size exactly $k$ is in $U$. Now split this union up into $k$ pieces in the obvious way, and one of these is in $U$.)
(2) Given an ultrafilter $U$, let $\tau(U)$ be the least cardinal $\tau$ such that some subfamily of $U$ of cardinality $\tau$ fails to have an infinite pseudo-intersection. (We do not require the pseudo-intersection to be in $U$, so $\aleph_1\leq\tau(U)\leq\mathfrak{c}$.)
Observation:
If $U$ is an ultrafilter on $\omega$, then $\tau(U)\leq\kappa$.
Proof. Given a family $\mathfrak{P}$ of finitary partitions of $\omega$ (directed or not), we fix for each $P\in\mathfrak{P}$ a set $A_P\in U$ meeting each element of $P$ in at most one point. If $|\mathfrak{P}|<\tau(U)$ then we can find an infinite pseudo-intersection $X$ for the collection $\{A_P:P\in\mathfrak{P}\}$, and $X$ is $\mathfrak{P}$-discrete.$_\square$
I don't know anything about the cardinals $\tau(U)$. I note that at one point Blass and Shelah claimed to have model containing both simple $P_{\aleph_1}$ and simple $P_{\aleph_2}$ points, but Alan Dow discovered an error in the paper, and I'm not sure if it has ever been repaired. (The existence of a simple $P_{\aleph_1}$-point implies $\mathfrak{b}=\mathfrak{u}=\aleph_1$, while the simple $P_{\aleph_2}$ point is an ultrafilter $U$ with $\tau(U)=\aleph_2$. In such a model, $\kappa$ would be strictly greater than $\mathfrak{b}$.)
Clearly this is all tied up with the topology of $\beta\omega$, so I suspect much more is known by the experts.<|endoftext|>
TITLE: Cutting the unit square into pieces with rational length sides
QUESTION [5 upvotes]: The following questions seem related to the still open question whether there is a point(s) whose distances from the 4 corners of a unit square are all rational.
To cut a unit square into n (a finite number) triangles with all sides of rational length. For which values of n can it be done if at all?
To cut a unit square into n right triangles with all sides of rational length. For which values of n can it be done, if at all?
Remark: If one can find a finite set of 'Pythagorean rectangles' (rectangles whose sides and diagonal are all integers) that together tile some square (of integer side), that would answer this question.
3.To cut a unit square into n isosceles triangles with all sides of rational length. For which values of n can it be done, if at all?
Now, one can add the requirement of mutual non-congruence of all pieces to all these questions. Further, one can demand rationality of area of pieces or replace the unit square with other shapes (including asking for a triangulation of the entire plane into mutually non-congruent triangles all with finite length rational length sides)...
Note: From what has been shown by Yaakov Baruch in the discussion below, cutting the unit square into mutually non-congruent rational sided-right triangles can be done for all n>=4. Indeed, he has shown n=4 explicitly; for higher n, one can go from m non-congruent pieces to m+1 pieces by recursively cutting any of the m right triangular pieces n by joining its right angle to the hypotenuse to cut it into two smaller and mutually similar but non-congruent pieces. That basically settles questions 1 and 2 - the non-congruent pieces case. However, if we need all pieces to be non-congruent and non-similar, the n=4 answer has no obvious generalization to higher n.
References:
1. https://nandacumar.blogspot.com/2016/06/non-congruent-tiling-ongoing-story.html?m=1
On dissecting a triangle into another triangle
REPLY [5 votes]: @YaakovBaruch's beautiful construction for $n=4$:
Edit by Yaakov Baruch. A partition with 8 isoceles triangles (I'm sure not minimal):<|endoftext|>
TITLE: Which plane curves can be harmonically parametrized?
QUESTION [5 upvotes]: In this question, a “(closed oriented plane) curve” $\Gamma$ will mean a continuous map $f \colon \mathbb{U} \to \mathbb{C}$ where $\mathbb{U} := \{z\in\mathbb{C} : |z|=1\}$ is the unit circle, modulo right-composition by orientation-preserving homeomorphisms $\varphi\colon\mathbb{U}\to\mathbb{U}$. Any such $f$ (i.e. any of the $f\circ\varphi$ defining the curve) will be called a “parametrization” of the curve $\Gamma$.
I'm willing to add any reasonable regularity conditions on $\Gamma$ if they help in answering the question, e.g., piecewise $C^1$ or even $C^\infty$.
Say that a curve $\Gamma$ is harmonically parametrizable iff it admits a parametrization $f \colon \mathbb{U} \to \mathbb{C}$ (see above) such that the Fourier coefficients $(c_k(f))_{k\in\mathbb{Z}}$ of $f$ are zero for all $k<0$: this then allows us to see $f$ as the restriction to $\mathbb{U} = \partial\Delta$ of a continuous function $F$ on the closed unit disk $\overline{\Delta}$ that is holomorphic on the open unit disk $\Delta$ (namely, $F$ is the Poisson integral of $f$, or equivalently $F(z) = \sum_{k=0}^{+\infty} c_k z^k$; see also this question).
More generally, if $k_0\in\mathbb{Z}$ say that $\Gamma$ is $k_0$-harmonically parametrizable iff it admits a parametrization $f \colon \mathbb{U} \to \mathbb{C}$ that $c_k(f)=0$ for all $k0$, being $k_0$-harmonically parametrizable seems to say something about the moments of $\Gamma$ for the harmonic measure vanishing, and this is probably less interesting.)
PS: This question seems related to a kind of converse to the one I'm asking (I want to know if there exists a parametrization change which becomes harmonic, that other question is about what happens upon such a change).
REPLY [2 votes]: First of all, this is a purely topological problem. Let $\gamma:U\to C$ be a curve.
Your question is when this curve can be reparameterized so that the new parameterization is by boundary values of
an analytic function in the unit disk.
It is necessary that $\gamma$ extends to
a topologically holomorphic map of the unit disk to $C$ (topologically holomorphic map, a. k. a. polymersion, is a map which is topologically equivalent to $z\mapsto z^n$ near every point.
This condition is also sufficient. Indeed, once we have a
topologically holomorphic map, we can pull back the complex analytic structure to the disk, and then use the Uniformization theorem. So for every topologically
holomorphic map $f$ there is a homeomorphism $\phi$ of the disk such that $f\circ\phi$ is holomorphic.
Second, this topological problem is solved in the paper (under some smoothness conditions on the curve):
C. Curley and D. Wolitzer, Btranched immersions of surfaces, Michigan Math. J. 33 (1986) 131-144.
Unfortunately the answer is somewhat complicated and I do not reproduce it here.<|endoftext|>
TITLE: RSK correspondence
QUESTION [8 upvotes]: Up to now, what are the difference ways we know to define RSK correspondence? I already know:
By insertion and recording tableau.
Ball construction or Viennot's geometric construction.
Growth diagram proposed by Sergey Fomin.
Do you know other models?
REPLY [4 votes]: Here, slightly edited, is the first paragraph of Steinberg's paper, An occurrence of the Robinson–Schensted correspondence.
Let $V$ be an $n$-dimensional vector space over an infinite field, $\mathscr F$ the flag manifold of $V$, $u$ a unipotent transformation of $V$, and $\lambda$ the type of $u$, a partition of $n$ whose parts are the sizes of the Jordan blocks for $u$. … The components of $\mathscr F_u$, the variety of flags fixed by $u$, correspond naturally to the standard tableaux of shape $\lambda$. The purpose of this note is to show that the "relative position" of any two components of $\mathscr F_u$ (in general an element of the Weyl group, in the present case an element of $S_n$) is given, in terms of the corresponding tableaux, by the Robinson–Schensted correspondence.<|endoftext|>
TITLE: Asymptotic behavior of a certain sum of ratios of consecutives primes
QUESTION [10 upvotes]: I am looking for the asymptotic growth of the following sum
$$\sum_{k=1}^{n}\frac{p_{k+1}+p_k}{p_{k+1}-p_k}$$
where $p_k$ stands for the prime of index $k$.
Manual computations show, for small values of n, a behavior quite similar to that of the sum over naturals
$$\sum_{k=0}^{n-1}(2k+1)=n^2$$
But more accurate simulations with Python suggest that
$\sum_{k=1}^{n}\frac{p_{k+1}+\,p_k}{p_{k+1}-\,p_k}$ ~ $\frac{2}{e}\,n^2\log\log n$
Is there anybody who can confirm this asymptotic behavior and, if it is correct, give a sketch of a proof?
REPLY [10 votes]: It is elementary to prove that the sum grows at least as fast as $n^2$, and at most as fast as $n^2\log n$. The precise asymptotic behavior depends on the distribution of prime gaps $p_{k+1}-p_k$, on which we only have conjectures (see also my Added section below).
It is clear that
$$\#\{k\leq n: p_{k+1}-p_k>\log n\}<\frac{p_{n+1}}{\log n},$$
hence the contribution of $p_{k+1}-p_k>\log n$ is
$$\sum_{\substack{1\leq k\leq n\\p_{k+1}-p_k>\log n}}\frac{p_{k+1}+p_k}{p_{k+1}-p_k}<\frac{p_{n+1}}{\log n}\cdot\frac{p_{n+1}+p_n}{\log n}=O(n^2).$$
Now, for a fixed even $h\leq\log n$, Hardy and Littlewood conjectured that
$$\#\{k\leq n: p_{k+1}-p_k=h\}\sim\frac{n}{\log n}\cdot 2C_2\cdot D_h,\tag{$\ast$}$$
where
$$C_2:=\prod_{p>2}\left(1-\frac{1}{(p-1)^2}\right)=0.66016\dots\qquad\text{and}\qquad D_h:=\prod_{\substack{p|h\\{p>2}}}\frac{p-1}{p-2}.$$
If we believe in this, then integration by parts gives that the contribution of $p_{k+1}-p_k=h$ is asymptotically $n^2 C_2 D_h/h$. Based on this heuristic, it is reasonable to conjecture that
$$\sum_{k=1}^{n}\frac{p_{k+1}+p_k}{p_{k+1}-p_k}\sim C_2\, n^2\sum_{h\leq\log n}\frac{D_h}{h}.$$
It is straightforward that the Dirichlet series of $D_h$ factors as
$$\sum_{h=1}^\infty\frac{D_h}{h^s}=\zeta(s)F(s),$$
where $F(s)$ is an explicit Euler product converging uniformly in $\Re(s)>3/4$, say. Therefore, heuristically,
$$\sum_{k=1}^{n}\frac{p_{k+1}+p_k}{p_{k+1}-p_k}\sim C\, n^2\log\log n,$$
where $C:=C_2F(1)$. Most likely, the constant $C$ is not equal to $2/e$ as suggested by the original post, but I have not checked this.
Added. It think that the known upper bounds for the left hand side of $(\ast)$ allow one to show, unconditionally, that the sum in question is $O(n^2\log\log n)$. As Lucia kindly pointed out, a result of Gallagher's implies that $C=1$.<|endoftext|>
TITLE: Importance of textbooks in health of a sub-discipline
QUESTION [18 upvotes]: I am interested in published articles, and also more informal writing (blog posts, talk slides etc.) which discuss the importance of textbooks (where this word encompasses research monographs etc.) in the long-term health of a sub-discipline in Mathematics.
Motivation:
I have been thinking of late about how large Mathematics is getting (compared to, say, 50-60 years ago) with many more mathematicians and many more published papers. It also seems to me that at least some areas are becoming increasingly technical.
Furthermore, it can be very hard to follow a field by just reading the original papers: there can be false steps, or incomplete results, which only reach final form after some attempts. Often the pressures of space mean that motivation, or background material, is omitted in articles.
A good textbook can solve all of these problems. It seems to me that especially graduate students, or more established mathematicians seeking to move field, or use results of a different field to their own, face these sorts of problems in the extreme. By contrast, people working in the field probably carry around a lot of the "missing content" in their heads. This then makes me wonder if the lack of textbooks might lead to an ever increasing barrier to entry, and perhaps to sub-disciplines dying out as younger/newer mathematicians do not take up the study.
Hence my question of whether these thoughts have been stated in a longer, more thought out way before.
REPLY [10 votes]: The following is purely my opinion, but the specific question that you posed seems to invite such answers.
Strictly speaking, I'd say that the answer to your question is "Yes", a lack of textbooks in a new area is a barrier to entry. However, my experience over the past 30+ years is that areas don't stay "new" for long, and as things become more solidified, people write introductory (graduate level) textbooks. This may be to promote their vision, or it may simply be because they find the subject beautiful and want to share that beauty with others. To take an older example, Grothendieck's revolution in algebraic geometry created a large barrier for entry, but Hartshorne's book appeared when I was in graduate school, and it provided a way in. Was it perfect? No. Have other books, possibly better introductions, appeared since. Sure. But it was there, and I think it's fair to say that it helped train a generation (or more) of algebraic geoemters and those in allied fields. (I'm in the latter group.)
So "yes", lack of graduate level texts in a area is a barrier to entry. But is it a long-term problem. I'd suggest that the answer is "No", because a new area of mathematics that's thriving tends to acquire such textbooks.<|endoftext|>
TITLE: Logical completeness of Hilbert system of axioms
QUESTION [10 upvotes]: This is really a question about references. The entry in Russian Wikipedia about Hilbert's axioms states, in particular, that completeness of Hilbert's system was proven by Tarski in 1951. The reference is to the Russian encyclopedia of elementary mathematics, to which I don't have access, and I somehow am not able to find any references to this statement in the literature.
I have two questions:
The continuity axioms are not first-order-logic statements. What
does then completeness mean in this situation?
Does anybody know a reference to the original proof by Tarski or any
other proof of this statement, for that matter?
REPLY [13 votes]: The original is Alfred Tarski's book "The completeness of elementary algebra and geometry", which was due to appear in 1940 but never made it into print because of the outbreak of WW2. An edition appeared after all in 1967 (Institut Blaise Pascal, Paris), but is not easy to come by.
Essentially the same argument is presented in Tarski's 1948/51 "A decision method for elementary algebra and geometry", available here.
Tarski uses an axiomatic setup of elementary geometry different (but equivalent) to Hilbert's, using only points (not lines or planes) as primitive terms, and two relation symbols, $B$ and $D$ (ternary and quaternary, respectively). $Bxyz$ signifies that the point $y$ is on the "line" $xz$ between the points $x$ and $z$ ("betweenness"), while $Dxyzw$ means that the distance between $x$ and $y$ equals that between $z$ and $w$ (the "equidistance" relation). Here, $x$, $y$, $z$ (and $w$) are allowed to coincide.
Full continuity is the second order axiom $\forall_{X}\forall_{Y}((\exists_{a}\forall_{x\in X}\forall_{y\in Y}Baxy)\rightarrow(\exists_{b}\forall_{x\in X}\forall_{y\in Y}Bxby))$, where $X$ and $Y$ are variables ranging over sets of points.
First order continuity is weaker, and is expressed as an axiom schema where $X$ and $Y$ are given as $\lbrace x\mid \phi(x)\rbrace$ and $\lbrace y\mid \psi(y)\rbrace$, respectively, for arbitrary first order formulas $\phi(x)=\phi(x,p_{1},\cdots,p_{n})$ and $\psi(y)=\psi(y,p_{1},\cdots,p_{n})$ that are allowed to contain parameters $p_{1},\cdots,p_{n}$.
A special case of first order continuity is the Circle Axiom, by which a line that contains an interior point of a circle (in the same plane) must meet that circle.
Completeness is the statement that any model $\mathfrak A$ of the axioms of elementary $n$-dimensional geometry without continuity is isomorphic to $K^{n}$ (with the obvious interpretations for the $B$ and $D$ relations) for a Pythagorean ordered field $K$ (that is $K\models\forall_{a}\forall_{b}\exists_{c}(a^{2}+b^{2}=c^{2})$), uniquely determined by $\mathfrak A$ up to isomorphism.
Under full continuity, $K$ must be $\mathbb{R}$, under first order continuity $K$ must be real closed, and for the Circle Axiom $K$ merely needs to be Euclidean (i.e., $K\models\forall_{a}\exists_{b}(a=b^{2}\vee -a=b^{2})$).
This is the content of the Representation Theorem, Th. I, (16.15) in "Metamathematische Methoden in der Geometrie" by W. Schwabhäuser, W. Szmielew and A. Tarski, Springer Hochschultext, 1983, an excellent reference for the metamathematics of elementary geometry (in German).
Edit: Let me add a few comments.
The statement above that Tarki's setup is equivalent to Hilbert's is rather imprecise, as noted by Matt F. and others. Tarski works in first order logic, while a formalization of Hilbert's system is at least unclear. (Still, the axioms in Hilbert's axiom groups I-IV can be derived from Tarski's axioms, as shown in the Schwabhäuser, Szmielew, Tarski text).
For the same reason, it is not clear what it would mean for Hilbert's system to be complete (in the modern sense), and I do not claim that "completeness" of Hilbert's system follows from that of Tarski. Hilbert includes a "completeness axiom", to the effect that his "model" of the axioms in groups I-V (where V is archimedeanity) cannot be extended to a "model" with a larger universe.
To add to the confusion, my use of the word Completeness above (in the body of the answer, in reference to the Representation Theorem) was also unfortunate. Tarski has shown that the first order theory of real closed fields is complete (in the modern sense), and that, as a result, the same goes for the theory of $n$-dimensional elementary geometry (based on Tarski's axioms, with the first order continuity axiom schema included).<|endoftext|>
TITLE: Sparse cofinal families
QUESTION [7 upvotes]: Let $\kappa$ be an infinite cardinal and as usual denote by $[\kappa]^\mu$ the set of all subsets of $\kappa$ having cardinality $\mu$:
Call a family $\mathcal{F} \subset [\kappa]^\omega$ sparse if for all $\mathcal{G} \in [\mathcal{F}]^{\omega_1}$ the set $\bigcup \mathcal{G}$ is uncountable.
This is a pretty natural notion that has been rediscovered by various authors and is the combinatorial core of various seemingly unrelated problems in topology, analysis and algebra (see the list of references at the end).
Getting a sparse family isn't that much of a deal. The problem is often getting a big one, or more precisely, a cofinal one, with respect to containment.
It's easy to see that there is a sparse cofinal family in $([\aleph_n]^\omega, \subseteq)$), for every $n<\omega$. This is a simple consequence of the fact that $cf([\aleph_n]^\omega, \subseteq)=\aleph_n$.
QUESTION: Is there (in ZFC) a cardinal $\kappa$ such that $cf([\kappa]^\omega, \subseteq) > \kappa$ and there is a sparse cofinal family on $[\kappa]^\omega$?
This cardinal, if it exists, must be larger than $\aleph_\omega$, because the existence of a sparse cofinal family of countable subsets of $\aleph_\omega$ can be proved to be independent of ZFC, modulo very large cardinals. Indeed (see Todorcevic's book or Blass's article for the proofs):
If the Chang's Conjecture variant $(\aleph_{\omega+1}, \aleph_\omega) \twoheadrightarrow (\aleph_1, \aleph_0)$ holds then there is no sparse cofinal family of countable subsets of $\aleph_\omega$. The consistency of this Chang's Conjecture variant has been proven from (slightly less than) a 2-huge cardinal by Levinski, Magidor and Shelah.
If $\square_{\aleph_\omega}+cf([\aleph_\omega]^\omega, \subseteq)=\aleph_{\omega+1}$ holds then there is a sparse cofinal family of subsets of $\aleph_\omega$.
REFERENCES:
Blass, Andreas, On the divisible parts of quotient groups, Göbel, Rüdiger (ed.) et al., Abelian group theory and related topics. Conference, August 1-7, 1993, Oberwolfach, Germany. Providence, RI: American Mathematical Society. Contemp. Math. 171, 37-50 (1994). ZBL0823.20057.
Kojman, Menachem; Milovich, David; Spadaro, Santi, Noetherian type in topological products, Isr. J. Math. 202, 195-225 (2014). ZBL1302.54008.
Peter Nyikos, Generalized Kurepa and MAD families and Topology, preprint.
Spadaro, Santi, On two topological cardinal invariants of an order-theoretic flavour, Ann. Pure Appl. Logic 163, No. 12, 1865-1871 (2012). ZBL1269.03047.
Todorcevic, Stevo, Walks on ordinals and their characteristics, Progress in Mathematics 263. Basel: Birkhäuser (ISBN 978-3-7643-8528-6/hbk). vi, 324 p. (2007). ZBL1148.03004.
REPLY [4 votes]: Your problem is related to the generalized Chang conjectures of the form $(\kappa^+,\kappa) \twoheadrightarrow (\aleph_1,\aleph_0)$.
Observation: If $(\kappa^+,\kappa) \twoheadrightarrow (\aleph_1,\aleph_0)$ holds, and if $\mathrm{cf}([\kappa]^{\omega},\subseteq) > \kappa$, then $([\kappa]^{\omega},\subseteq)$ has no sparse cofinal family.
So if we'd like to answer your question in the negative (or restrict our search for a positive answer), we should ask when it's possible to have lots of instances of $(\kappa^+,\kappa) \twoheadrightarrow (\aleph_1,\aleph_0)$ holding simultaneously.
A paper relevant to this question is "Global Chang's Conjecture and singular cardinals" by Monroe Eskew and Yair Hayut (available here). In it, they articulate the following conjecture:
Singular Global Chang's Conjecture: For all infinite $\mu < \kappa$ of the same cofinality, $(\kappa^+,\kappa) \twoheadrightarrow (\mu^+,\mu)$.
The word "conjecture" is used here to allude to Chang's Conjecture. No one is suggesting the SGCC as a potential theorem of $\mathsf{ZFC}$, and in fact it's not hard to show that the negation of the SGCC is consistent with $\mathsf{ZFC}$. The suggestion is that the SGCC might be consistent with $\mathsf{ZFC}$, relative to large cardinals. My point is that if this conjecture is consistent, then the answer to your question is no. Based on the observation above,
If this conjecture holds, then there are no cardinals $\kappa$ with the property described in your question. In fact, SGCC implies that the answer to your question is no even when $\omega$ is replaced by an arbitrary infinite cardinal $\mu$.
So the conjecture gives a conditional answer to your question.
In the same paper (Theorem 32), the authors show that a fragment of this conjecture is consistent. This fragment is enough to show, using the observation above, that
If $\alpha < \omega_1$, then it is consistent (relative to two supercompact cardinals) that the answer to your question is no for every $\kappa < \aleph_\alpha$. That is, it is consistent that if $\kappa < \aleph_\alpha$ and $\mathrm{cf}([\kappa]^{\omega},\subseteq) > \kappa$, then $[\kappa]^\omega$ has no sparse cofinal family.
A proof of the observation above:
Recall that $(\kappa^+,\kappa) \twoheadrightarrow (\mu^+,\mu)$ is an abbreviation for the following statement:
$\bullet \ \ $ For every model $M$ for a countable language $\mathcal L$ that contains a predicate $A \subseteq M$, if $|M| = \kappa^+$ and $|A| = \kappa$ then there is an elementary submodel $M'$ of $M$ such that $|M'| = \mu^+$ and $|M' \cap A| = \mu$.
To prove the observation above, suppose $(\kappa^+,\kappa) \twoheadrightarrow (\aleph_1,\aleph_0)$, and suppose that $\mathrm{cf}([\kappa]^{\omega},\subseteq) > \kappa$.
Let $\mathcal C$ be any cofinal family in $[\kappa]^\omega$. Using the Lowenheim-Skolem theorem in the usual way, let $M$ be an elementary submodel of $H(\theta)$ (for some sufficiently big regular cardinal $\theta$) such that $\mathcal C \in M$, $\kappa \subseteq M$, and $|M| = |M \cap \mathcal C| = \kappa^+$. Let $\phi$ be a bijection $M \rightarrow \mathcal C \cap M$. (Note that this $\phi$ cannot be a member of $M$.)
Consider the language consisting of a single relation symbol and a single function, and consider the model
$$(M,\in,\phi,\kappa)$$
in this language. Applying our hypothesis $(\kappa^+,\kappa) \twoheadrightarrow (\aleph_1,\aleph_0)$, there is some $M' \subset M$ such that $(M',\in,\phi,\kappa \cap M')$ is an elementary substructure of $(M,\in,\phi,\kappa)$ with $|M'| = \aleph_1$ and $|M' \cap \kappa| = \aleph_0$.
Let $\mathcal C' = M' \cap \mathcal C$. The restriction of $\phi$ to $M'$ is (by elementarity) a bijection $M' \rightarrow \mathcal C'$. Therefore $|\mathcal C'| = |M'| = \aleph_1$.
If $X \in \mathcal C'$, then $X$ is a countable set in $M'$. Because $M'$ is a model of (enough of) $\mathsf{ZFC}$, every countable set in $M'$ is a subset of $M'$. So $X \subseteq M'$. But $X \subseteq \kappa$ as well, so every $X \in \mathcal C'$ is a subset of $M' \cap \kappa$.
Therefore $\bigcup \mathcal C' = M' \cap \kappa$. Because $|M' \cap \kappa| = \aleph_0$, this means that $\mathcal C'$ is an uncountable subset of $\mathcal C$ whose union is countable. So $\mathcal C$ is not sparse.
Because $\mathcal C$ was an arbitrary cofinal family in $[\kappa]^\omega$, this shows that $[\kappa]^\omega$ does not admit a sparse cofinal family.<|endoftext|>
TITLE: Bounds on the lowest dimension of a faithful representation of a finite group
QUESTION [12 upvotes]: Let $G$ be a finite group. Can one always construct a faithful representation of $G$ over $\mathbb C$ of dimension $\le \sqrt{|G|}$? What would be the order of the minimal such dimension for a "random" group? Are the worst groups (with the largest minimal faithful dimensions) special in some way?
REPLY [10 votes]: I'm writing a paper where I prove that all counterexamples are related to Derek Holt's example: if G is a finite group then either G has a faithful representation over $\mathbb{C}$ of dimension $\leq\sqrt{|G|}$ or $G$ is a $2$-group with center is elementary abelian of order 8 and all irreducible characters of $G$ whose kernel does not contain $Z(G)$ vanish on $G-Z(G)$.
For any of these $2$-groups, the minimal dimension of a faithful representation is $\frac{3}{\sqrt{8}}\sqrt{|G|}$.
I also prove that this minimal dimension is equal to $\sqrt{|G|}$ if and only if $G$ is a $2$-group with center elementary abelian of order either $4$ or $16$ and all irreducible characters of $G$ whose kernel does not contain $Z(G)$ vanish on $G-Z(G)$.
I'd be happy to share my preprint. It would help me to know the motivation for this question.<|endoftext|>
TITLE: Lengths of closed geodesics on a flat vs hyperbolic punctured torus
QUESTION [5 upvotes]: Let $T$ be a torus (oriented closed surface of genus 1), $p\in T$, and $T^* := T - \{p\}$.
Let $\mu$ denote a flat structure on $T$. This can be obtained for example by choosing a uniformization $p_f: T\cong\mathbb{R}^2/\Lambda$ for some lattice $\Lambda$ and descending the standard flat structure on $\mathbb{R}^2$ down to $T$.
The flat structure on $T$ restricts to one on $T^*$. Moreover, if we fix an identification $\mathbb{C}\cong\mathbb{R}^2$, this flat structure gives a complex structure on $T^*$. This complex structure yields a holomorphic universal covering $p_h : \mathbb{H}\rightarrow T^*$, such that the deck transformation group consist of hyperbolic isometries of $\mathbb{H}$, and hence this complex structure determines a hyperbolic structure on $T^*$.
By a paper of Gutkin and Judge (Affine mappings of translation surfaces), it seems to follow from their Theorem 6.5 that the number of (simple) closed geodesics on a flat (punctured) torus (up to translation) of length bounded by $L$ is quadratic in $L$. If the torus is the unit square torus, the leading term turns out to be $\frac{3}{\pi}L^2$.
On the other hand, by a result of McShane and Rivin (A norm on homology of surfaces and counting simple geodesics, as pointed out to me in a previous question), there is a natural bijection between the simple closed geodesics in the flat torus up to translation with those in the corresponding hyperbolic torus (both sets are bijective onto the set of primitive homology classes under the natural map), and that the number of hyperbolic simple closed geodesics of length bounded by L is also quadratic in L.
This suggests the flat geodesic representatives of primitive homology classes might have length which is universally proportional to that of the hyperbolic geodesic representative. Could this be true? If not, what is known about the relationship between these lengths under the natural bijection described above?
References would be appreciated!
REPLY [4 votes]: Yes, this is true. It is shown (either in the paper you cite or the other McShane-Rivin paper) that the length of a simple closed geodesic is quasi-the-same as the combinatorial length ($m+n$) (this is easy, because a simple geodesic stays away from the cusp), and that, in turn, is easily seen to be quasi-the-same as the Euclidean length.<|endoftext|>
TITLE: Vector bundles on $\mathbb{A}^n / G$
QUESTION [23 upvotes]: Let $G$ be a finite group acting linearly on $\mathbb{A}^n$. Do we expect algebraic vector bundles on $X := \mathbb{A}^n/G$ to be trivial? Here by the quotient I mean the singular scheme, not the stack quotient.
What is known:
(1) For finite abelian groups $G$, $X$ is an affine toric variety, and vector bundles on $X$ are trivial by a Theorem of Gubeladze: https://iopscience.iop.org/article/10.1070/SM1989v063n01ABEH003266. In particular, when $G$ is the trivial group, triviality of vector bundles on $\mathbb{A}^n$ is an older result by Quillen-Suslin.
(2) Line bundles on $X$ are trivial. This can be shown by lifting a line bundle on $X$ to a $G$-line bundle on $\mathbb{A}^n$; then $\mathrm{Pic}^G(\mathbb{A}^n)$ is the group of characters of $G$, and since the linear $G$-action has a fixed point $0$, this character will be trivial, hence coming from a trivial line bundle on $X$.
(2') For vector bundles of higher rank the argument in (2) does not work. This has to do in particular with $G$-actions on $\mathbb{A}^N$ not being linearizable in general: https://link.springer.com/chapter/10.1007%2F978-94-015-8555-2_3
(3) The Grothendieck group of vector bundles is $\mathrm{K}_0(X) = \mathbb{Z}$. We prove it in https://arxiv.org/pdf/1809.10919.pdf, Prop. 2.1 indirectly, using comparison with cdh topology of differential forms.
(3') By homotopy invariance of K-groups in the smooth case, $\mathrm{K}^G_0(\mathbb{A}^n) \simeq \mathrm{K}^G_0(\mathrm{Spec}(k))$ which is the Grothendieck ring of $G$-representations; however this does not seem to help.
Is there any more evidence for/against the triviality of vector bundles on $X$? Is this question mentioned anywhere in the literature?
REPLY [9 votes]: In the paper Affine varieties dominated by $\mathbf{C}^2$ Gurjar considers a slightly more general situation, namely an affine normal variety $\mathrm{X}$ with a proper surjective morphism $\mathbf{A}^2\rightarrow\mathrm{X}$. He shows that every line bundle on $\mathrm{X}$ is trivial; together with a result of Anderson (every vector bundle on $\mathrm{X}$ is the direct sum of a trivial bundle and a line bundle) this shows in particular that every vector bundle on $\mathbf{A}^2/\mathrm{G}$ is trivial.<|endoftext|>
TITLE: Derivators and fibred $\infty$-categories
QUESTION [11 upvotes]: In his Cohomological methods in intersection theory, Cisinski writes:
"[...] note however that, by Balzin’s work [Bal19, Theorem 2], it is clear that one can go back and forth between the language of fibred $\infty$-categories and the one of algebraic derivators."
By Bal19 he intends Balzin's Reedy model structures in families. Let me quote the abovementioned Theorem 2.
Thm. 2 in Bal19.
Let $\mathsf{E} \to \mathsf{R}$ be a left model Reedy fibration. Then the induced $\infty$-functor $\mathsf{L}\text{Sect}(\mathsf{R}, \mathsf{E}) \to \text{Sect}(\mathsf{R}, \mathsf{LE})$ is an equivalence of quasicategories.
Even though I sense a connection between derivators, fibred $\infty$-categories and this statement, I cannot make it precise. Could someone help me spelling out what Cisinski means precisely?
REPLY [4 votes]: I am no Denis-Charles but given the other paper you quoted let me think of a sketch, perhaps you will be able to make the right out of it.
Let $\mathcal E \to \mathcal C$ be a Quillen presheaf (model categorical fibres, Quillen pairs as transition functors) where each $\mathcal E(c)$ is stable. This would imply, if we had the cofibrant generation in fibres for example, that for any functor $F: \mathcal D \to \mathcal C$, the category of sections $Sect(\mathcal D,F^*\mathcal E)$ is a stable model category. You can then introduce a $2$-functor $\mathbb D: (Cat/ \mathcal C)^{op} \to CAT$ that sends $F: \mathcal D \to \mathcal C$ to the homotopy category $Ho (Sect(\mathcal D,F^*\mathcal E))$ and study its properties. Something like that could be called a relative (pre)derivator (maybe it already is, forgive my ignorance, then) valued in triangulated categories.
We can also start with the infinity-localisation $L \mathcal E \to \mathcal C$ and study its infinity-categorical sections $Sect(\mathcal D,F^* L \mathcal E)$ for functors $F: \mathcal D \to \mathcal C$. Since a localisation of a stable model category is a stable infinity-category (a statement true even without cofibrant generation due to Proposition 3 mentioned below), $L \mathcal E \to \mathcal C$ is a bicartesian fibration with stable fibres and using Section 5 of HTT (co/limits of sections computed fibrewise) we can verify that each $Sect(\mathcal D,F^* L \mathcal E)$ is stable. Taking their homotopy categories would again yield another $2$-functor $\mathbb D':(Cat/ \mathcal C)^{op} \to CAT$ valued in triangulated categories.
Proposition 3 of the Reedy paper means however that $Ho (Sect(\mathcal D,F^*\mathcal E)) \cong Ho (Sect(\mathcal D,F^* L \mathcal E))$ where the latter is the homotopy category of an infinity-category and the former is that of a model category. In other words, the relative derivators $\mathbb D$ and $\mathbb D'$ are equivalent; the equivalence comes from the fairly canonical map $L Sect(\mathcal D,F^*\mathcal E)) \to Sect(\mathcal D,F^* L \mathcal E)$ (universality of localisation is used here) so it should be trackable along base changes. Thus you can study your relative derivators any way you want, via the model-categorical presentation or the infinity-categorical one.
I suppose you get an algebraic derivator if you replace $\mathcal C$ by $Sch$ (no restriction on the size of $\mathcal C$ was made above) and consider something like $QCoh \to Sch$ for the Quillen presheaf, but you will have to take it from here to see if that fits. Maybe I should add something about such relative derivators to the Reedy paper if life permits.<|endoftext|>
TITLE: Non-constant holomorphic map onto a smooth curve
QUESTION [5 upvotes]: Let $\Gamma$ be a smooth projective curve in $\mathbb{P}^2$ and let $U$ be an open neighborhood of $\Gamma$. Denote by $\Gamma_1,\Gamma_2,\ldots,\Gamma_n$ a finite collection of smooth curves intersecting $\Gamma$ transversally. My question is:
Does there exist a non-constant holomorphic function $\pi: U \setminus \bigcup\limits_{i=1}^n \Gamma_i \to \Gamma \setminus \bigcup\limits_{i=1}^n \Gamma_i$ ?
The situation as in the following picture.
I can build a smooth map $\pi$ easily by partition of unity. Intuitively, $\pi$ seems like the tubular neighborhood projection of $\Gamma$ ? But unfortunately, a holomorphic tubular neighborhood does not exist in general as discussed in Is there any holomorphic version of the tubular neighborhood theorem?. And I realized that even the existence of a non-constant holomorphic map $\pi : U \to \Gamma$ (without removing any curve) is not obvious to me.
Any comment is welcome. Thanks in advance!
REPLY [4 votes]: Let me show that such a map usually doesn't exist (even if we don't remove any additional curves). Consider the case when $\Gamma$ is a curve of degree $\ge 4$. Then one can slightly perturb $\Gamma$ in $\mathbb CP^2$ to a curve $\Gamma'$ that is not isomorphic to $\Gamma$ (so that it stays in a small neighbourhood of $\Gamma$). If the map $\pi$ would exists, we can restrict it to $\Gamma'$, and it will be extendable to the whole $\Gamma'$. So we get a holomorphic (non constant) map $\pi: \Gamma'\to \Gamma$, which is impossible, since $\Gamma$ and $\Gamma'$ are not isomorphic (and by Riemann-Hurwitz for two curves of the same genus $\ge 2$ a non-constant holomorphic map $\Gamma\to \Gamma'$ exits only if the curves are isomorphic).
This argument can be easily generalised to curves $\Gamma$ of degree $3$, i.e. cubics. The case of conics and lines should be analysed separately (and for lines $\Gamma$ such a map sometimes exists, for example, when all $\Gamma_i$'s are lines transverse to the line $\Gamma$ and intersecting at one point).<|endoftext|>
TITLE: Is $\mathbb{F}_{p}(t)^{h}$ an elementary substructure of/existentially closed in $\mathbb{F}_{p}((t))$?
QUESTION [7 upvotes]: It is a well-known fact that the Henselization of the function field $\mathbb{F}_{p}(t)$ in regard to the $t$-adic valuation is $\mathbb{F}_{p}(t)^{alg} \cap \mathbb{F}_{p}((t))$, so of course $\mathbb{F}_{p}(t)^{h}$ embeds into $\mathbb{F}_{p}((t))$, but is it known whether this embedding is elementary in the language of rings $\mathcal{L}_{\text{Ring}}$ or respectively the language of valued fields $\mathcal{L}_{\text{Ring}, \ \mid}$ (this does not matter as the valuation is uniformly definable in henselian valued fields with finite residue field). I would assume that, if this is true, it is not known because we do not know whether $\mathbb{F}_{p}((t))$ is decidable, but do we at least know whether $\mathbb{F}_{p}(t)^{h}$ is existentially closed in $\mathbb{F}_{p}((t))$? This at least should be guaranteed for the perfect hull by Ax-Kochen-Ershov for tame fields.
REPLY [3 votes]: There is apparently not a very short answer to this. It is, I think, really not known whether this extension is elementary and it would be very surprising if it was known, since we do not even know if the theory of $\mathbb{F}_{p}((t))$ is decidable. But there is the following result by Franz-Viktor Kuhlmann in his paper "The Algebra and Model Theory of Tame Valued Fields" (https://arxiv.org/pdf/1304.0194.pdf, Theorem 5.9)
Let $(K,v)$ be a henselian field. Assume that $(L/K,v)$ is a separable subextension of $(K^{c}/K,v)$. Then $(K,v)$ is existentially closed in $(L,v)$. In particular, every henselian inseparable defectless field is existentially closed in its completion.
Here $K^{c}$ stands for the completion in regard to $v$. Now we take a look at $\mathbb{F}_{p}(t)^{h}$, it is obviously henselian. And from that it obviously follows that any henselian valued field is existentially closed in its completion. Of course this does not give a lot of insight on the problem, so let me try and elaborate a little: The proof of Theorem 5.9 makes use of the fact that it is enough to be existentially closed in every finitely generated subfield to be existentially closed. So we take such a subfield $(F,v)$ and go over to its Henselization, which still lies in the completion (because the completion is Henselian). This is of course a subextension hence it is separable. We can see that indeed $(F,v)^{h}=(K(x_{1}, \ldots, x_{n}),v)^{h}$. Then it boils down to show that if we set $K_{i}:=(K(x_{1}, \ldots, x_{i}), v)^{h}$, then $K_{i} \leq_{1} K_{i+1}$, which we can show by realizing that $x$ is the limit of a Cauchy sequence and further results in the linked paper.
This is of course a very rough sketch. It is better to read the paper to grasp the proof in detail.<|endoftext|>
TITLE: Yet another real-rooted polynomial
QUESTION [12 upvotes]: In this entry I asked for the real-rootedness of a polynomial, and two very interesting answers were given: one using Malo's theorem and the other a clever rewriting of the expression using Jacobi Polynomials which are known to be real-rooted.
Now, in the middle of a reasoning I had to somehow prove that the following polynomials are real-rooted:
$$Q_{m,n}(t) = \sum_{j=0}^m \binom{m}{j}\binom{n}{j} \binom{t-j}{m+n+1}$$
defined for $m\leq n$, integers.
Observe that where in the last question one had $t^j$, here one has a $\binom{t-j}{m+n+1}$.
Of course, it has been verified numerically for the first small cases and it is true and, in fact, it seems to be true that all the roots are $\leq m+n$.
It's possible to prove that many of the roots are integers, which suggests one may "factor out" something like $\binom{t-m}{n+1}$ out of the expression. The problem is that the remaining factor does not seem to be friendly at all.
This leads me to the following questions:
1) Any ideas to prove the real-rootedness of this thing?
2) (Much wider and maybe not useful for this concrete problem but interesting on its own) There's a theorem of Schur (see here Theorem 1) that gives a sufficient condition for a polynomial to be real rooted. I want to know if there's any kind of generalization in the case one changes the basis $\{1,x,x^2,\ldots\}$ for example with $\{\binom{x}{0},\binom{x}{1},\binom{x}{2},\ldots\}$.
REPLY [13 votes]: First, we write the polynomial in hypergeometric form
$$
Q_{m,n}(t)=\frac{(-1)^{m+n+1} \Gamma (m+n-t+1) }{\Gamma (-t) \Gamma (m+n+2)}\, _3F_2\left({-m,-n,m+n-t+1\atop 1,-t};1\right).
$$
Applying Thomae's transformation we get (see eq. 1.3 in W. N. Bailey, Contiguous Hypergeometric Functions of the Type 3F2(1)):
$$
Q_{m,n}(t)=\frac{(-1)^{n+1} m! \Gamma (m+n-t+1) }{\Gamma (m+n+2) \Gamma (m-t)}\, _3F_2\left({-m,n+1,-m-n+t\atop 1,-m};1\right).
$$
Here the polynomial is understood as $\displaystyle{\lim_{\epsilon\to 0}{}_3F_2\left({-m,n+1,-m-n+t\atop 1,-m+\epsilon};1\right) }$, so $-m$ in the numerator and denominator of the hypergeometric function can not be cancelled.
The hypergeometric polynomial in the last expression can be expressed in terms of Hahn polynomials, which are discrete orthogonal polynomials (see M. Ismail, Classical and Quantum Orthogonal Polynomials in One Variable):
Thus
$$
Q_{m,n}(t)=(-1)^{n+1}\frac{ (m-t)_{n+1} }{(m+1)_{n+1}}\,Q_m(m+n-t;0,n-m,m).
$$
It is known that zeroes of Hahn polynomials are real and simple. In particular, one can see that some roots are indeed integers due to the factor $(m-t)_{n+1} $, in agreement with OP's observation.<|endoftext|>
TITLE: Better trigonometrical inequalities for $\zeta(s)$?
QUESTION [8 upvotes]: The inequality $$3 + 4 \cos \theta + \cos 2 \theta \geq 0$$ plays a key role in the proof of the classical zero-free region of the Riemann zeta function. Are there other inequalities of the form
$$\sum_{i=0}^k a_i \cos b_i \theta \geq 0,\;\;\;\;\;a_\geq 0$$
such that $a_{i_0} = \sum_{i\ne i_0} a_i$ for some $0\leq i_0\leq k$ and $a_{0} < \frac{3}{4} a_{i_0}$, $b_0=0$?
REPLY [17 votes]: Assuming the $b_i$ are all distinct (or at least non-zero for $i \neq 0$), this is not possible. (Otherwise there are trivial examples, e.g. $1 + 2 \cos(0 \theta)+ \cos(0 \theta) \geq 0$ or $1 + 4 \cos \theta + \cos(2\theta) + 2 \cos(0 \theta) \geq 0$.)
Suppose that $\sum_{i=0}^k a_i \cos b_i \theta \geq 0$. Since $a_{i_0} = \sum_{i \neq i_0} a_i$, this implies that whenever $\cos b_{i_0} \theta = -1$, one must have $\cos b_i \theta = +1$ for all other $i$. In particular, the other $b_i$ must be integer multiples of $2b_{i_0}$. We now have
$$ a_0 + a_{i_0} \cos b_{i_0} \theta + \sum_{i \neq 0, i_0} a_i \cos b_i\theta \geq 0$$
with the $b_i$ in the sum nonzero integer multiples of $2b_{i_0}$. Performing a Taylor expansion around $\theta = \pi / b_{i_0}$ to second order, we conclude that
$$ - a_{i_0} \frac{b_{i_0}^2}{2} + \sum_{i \neq 0, i_0} a_i \frac{b_i^2}{2} \geq 0$$
and hence (since $b_i^2 \geq 4 b_{i_0}^2$ and $b_{i_0} \neq 0$)
$$ \sum_{i \neq 0, i_0} a_i \leq \frac{1}{4} a_{i_0}$$
or equivalently
$$ a_0 \geq \frac{3}{4} a_{i_0}.$$
Thus one cannot have $a_0 < \frac{3}{4} a_{i_0}$. This argument also shows that up to rescaling and other trivial rearrangements, Mertens' inequality $3 + 4 \cos(\theta)+\cos(2\theta) \geq 0$ is the unique inequality that attains $a_0 = \frac{3}{4} a_{i_0}$.
At a more metamathematical level, if there were a variant of Mertens' trigonometric inequality that gave superior numerical results towards the classical zero free region, I would imagine that this would already have been noticed by now. :-)<|endoftext|>
TITLE: Examples of incorrect arguments being fertilizer for good mathematics?
QUESTION [24 upvotes]: Sometimes (perhaps often?) vague or even outright incorrect arguments can sometimes be fruitful and eventually lead to important new ideas and correct arguments.
I'm looking for explicit examples of this phenomenon in mathematics.
Of course, most proof ideas start out vague and eventually crystallize. So I think the more incorrect/vague the original argument or idea, and the more important the final fruit, the better, as long as there is still a pretty direct connection from the vague idea to the final fruit.
Note: Many "paradoxes" sort-of are like this, but I think aren't what I'm looking for. (William Byers's book "How Mathematicians Think" has several examples and lots of discussion of the important role of paradox in mathematical research.) For example, the relationship between Russell's paradox, Godel's Incompleteness Theorem, and the undecidability of the halting problem (Church; Turing). But I think, unless the paradox has some other aspects of the vague-idea-as-fertilizer phenomenon, that I'm not looking for examples of paradoxes, though I am willing to be convinced otherwise.
Edit: It's been suggested that this a duplicate of this other question, but I really think it is not. I am more interested in examples of outright incorrect (or nearly so) original statements that nonetheless lead to fruitful mathematics, whereas the other question seems to essentially be asking about ideas that start out intuitive, non-rigorous, or ill-defined and then are turned into rigorous arguments but along the same intuitive lines. (And, as I said above, I think I agree with one of the answers there that that is simply much of mathematics.) By comparing the answers to the other question to the three great answers already on this question (knot theory rising because Kelvin thought atoms were knotted strings; Lame's erroneous proof of FLT leading Kummer to develop algebraic integers; Lebesgue's incorrect proof that projections of Borel sets are Borel leading to Suslin's development of analytic sets), one can get a sense of the difference.
REPLY [5 votes]: Obviously König's theorem should appear on this page. König suggested a proof by which the real numbers cannot be well-ordered. Unfortunately, he misunderstood some of the work he relied on, and thence we have this wonderful theorem known as König's theorem or Zermelo–König's theorem:
If $I$ is any set, and for each $i\in I$, $|A_i|<|B_i|$, then $\left|\bigcup_{i\in I}A_i\right|<\left|\prod_{i\in I}B_i\right|$.<|endoftext|>
TITLE: Acyclic group and finite CW-complex
QUESTION [13 upvotes]: Is there a nontrivial example of an acyclic group $G$ such that its corresponding Eilenberg space $K(G,1)$ is homotopy equivalent to a finite CW-complex ?
REPLY [17 votes]: The Higman group with presentation
$$\langle{a,b,c,d}\mid{aba^{-1}b^{-2}},~bcb^{-1}c^{-2},~cdc^{-1}d^{-2},~
dad^{-1}a^{-2}\rangle$$
is perfect, and the 2-complex associated to this presentation
has Euler characteristic 0. Hence this complex is acyclic.
It is in fact aspherical, but it may be simpler to observe that
Higman's group is also an iterated generalized free product with
amalgamations $(A*_{\langle{b}\rangle}B)*_{F(a,c)}(C*_{\langle{c}\rangle}D)$, where $A,B,C$ and $D$ are copies of the Baumslag-Solitar group $BS(1,2)$, generated by $\{a,b\}$, $\{b,c\}$, $\{c,d\}$ and $\{d,a\}$, respectively. We may assemble a 2-dimensional Eilenberg-Mac Lane complex for the Higman group in a similar way.<|endoftext|>
TITLE: A question on simple $P_{\aleph_2}$-points
QUESTION [6 upvotes]: This question is motivated by discussion surrounding this MO question.
An ultrafilter $U$ on $\omega$ is a simple $P_{\aleph_2}$-point if it is generated by a sequence $\langle X_\alpha:\alpha<\omega_2\rangle$ such that
$$\alpha<\beta\Longrightarrow |X_\beta\setminus X_\alpha|<\aleph_0,$$
that is, generated by an $\omega_2$ sequence of subsets of $\omega$ that is decreasing modulo the ideal of finite sets.
Question
Is the existence of a simple $P_{\aleph_2}$-point consistent with $\mathfrak{b}=\aleph_1$?
REPLY [3 votes]: The answer is "yes". Alan Dow pointed out in correspondence that the construction of Blass and Shelah referenced below can be modified to yield such a model. The particular model arises by using finite support to add an $\omega_1$-sequence of Cohen reals, followed by a finite support iteration adding the generating set for the $P_{\aleph_2}$-point using a variant of Mathias forcing. The salient point of the construction is that the Mathias-type forcing (really Mathias forcing with respect to a carefully chosen ultrafilter at each stage) does not add a real dominating the Cohen reals, and so $\mathfrak{b}=\aleph_1$ in the extension.
Referring to the discussion on the question of Banakh that motivated our question, this shows there is a model in which Banakh's cardinal $\kappa$ is strictly greater than $\mathfrak{b}$, as the $P_{\aleph_2}$-point is an ultrafilter $U$ with $\tau(U)=\aleph_2$.
Blass, Andreas; Shelah, Saharon, Ultrafilters with small generating sets, Isr. J. Math. 65, No. 3, 259-271 (1989). ZBL0681.03033.<|endoftext|>
TITLE: The abc-conjecture as an inequality for inner-products?
QUESTION [21 upvotes]: The abc-conjecture is:
For every $\epsilon > 0$ there exists $K_{\epsilon}$ such that for all natural numbers $a \neq b$ we have:
$$ \frac{a+b}{\gcd(a,b)}\,\ <\,\ K_{\epsilon}\cdot \text{rad}\left(\frac{ab(a+b)}{\gcd(a,b)^3}\right)^{1+\epsilon} $$
I have two questions after doing some experiments with SAGEMATH:
1) Is the matrix
$$L_n = \left( \frac{\gcd(a,b)}{a+b}\right)_{1\le a,b \le n}$$
positive definite?
2) Is the matrix:
$$ R_n = \left(
\frac{1}{\text{rad}\left(\frac{ab(a+b)}{\gcd(a,b)^3}\right)}
\right)_{1\le a,b \le n} $$
positive definite?
If both of the questions can be answered with yes, then we would have "mappings"
$$\psi ,\phi: \mathbb{N} \rightarrow \mathbb{R}^n$$
and the abc-conjecture might be stated as an inequality in the inner-product of these mappings:
$$\left< \psi(a),\psi(b) \right>^{1+\epsilon} < K_{\epsilon} \left < \phi(a), \phi(b) \right >$$
which I think would be very interesting.
Edit:
I realized that it is better to ask the following question:
Is
$$R^{(\epsilon)}_n := (\frac{2^{\epsilon}}{\text{rad}(\frac{ab(a+b)}{\gcd(a,b)^3})^{1+\epsilon}})_{1\le a,b\le n}$$
positive definite for all $\epsilon \ge 0$?
If "yes", then we would have:
For all $\epsilon \ge 1$ and all $a \neq b$ the following are equivalent:
$$1) d_R^{(\epsilon)}(a,b) = \sqrt{1-\frac{2^{1+\epsilon}}{\text{rad}(\frac{ab(a+b)}{\gcd(a,b)^3})^{1+\epsilon}}}>d_L(a,b) = \sqrt{1-2\frac{\gcd(a,b)}{a+b}}$$
$$2) \left < \psi^{(\epsilon)}_R(a),\psi^{(\epsilon)}_R(b) \right > < \left < \psi_L(a),\psi_L(b) \right >$$
3) The abc conjecture for $\epsilon \ge 1$ with $K_{\epsilon} = \frac{1}{2^{\epsilon}}$
Related question Two questions around the $abc$-conjecture
Also the metrics $d_R^{(\epsilon)},d_L$ would be embedded in Euclidean space.
Yet another edit:
It seems that
$$\frac{\phi(n)}{n} = \sum_{d|n} \frac{\mu(d)}{\text{rad}(d)}$$
wher $\mu, \phi$ are the Moebius function and the Euler totient function.
From this it would follow using Moebius inversion, that :
$$\frac{1}{\text{ rad}(n)} = \sum_{d|n} \frac{\mu(d)\phi(d)}{d}$$
which could (I am not sure about that) be helpful for question 2).
Edit with proof that $k(a,b)$ is a kernel:
Let
$$k(a,b) := \frac{1}{\frac{ab(a+b)}{\gcd(a,b)^3}} = \frac{\gcd(a,b)^3}{ab(a+b)} = \frac{\gcd(a,b)^2}{ab} \cdot \frac{\gcd(a,b)}{a+b} = k_1(a,b) \cdot k_2(a,b)$$
It is known that:
$$\int_0^1 \psi(at)\psi(bt) dt = \frac{1}{12} \frac{(a,b)^2}{ab} = \frac{1}{12} k_1(a,b).$$
Where $\psi(t) = t - \lfloor t \rfloor - \frac{1}{2}$ is the sawtooth function.
Hence $k_1(a,b)$ is a kernel.
On the other hand, it is known for example by the answer of @DenisSerre, that $k_2(a,b)$ is also a kernel.
Hence the product $k(a,b) = k_1(a,b) \cdot k_2(a,b)$ is also a kernel.
Update:
I found this paper online which is interesting (Set there: $X_a = \{ a/k | 1 \le k \le a \}$ then: $|X_a \cap X_b| = |X_{\gcd(a,b)}| = \gcd(a,b)$ ) and may be of use for the questions above:
https://www.researchgate.net/publication/326212690_On_the_positive_semi-definite_property_of_similarity_matrices
Setting in the paper above $A_i = \{ i/k | 1 \le k \le i \}$ we see that $|A_i \cap A_j| = |A_{\gcd(i,j)}| = \gcd(i,j)$ and $|A_i|=i$.
Since in the paper it is proved that:
1) The Sorgenfrei similarity $\frac{|A_i \cap A_j|^2}{|A_i||A_j|}$ is a (positive definite $\ge0$, symmetric) kernel, we have another proof, that $\frac{\gcd(a,b)^2}{ab}$ is a kernel.
2) The Gleason similarity $\frac{2|A_i \cap A_j|}{|A_i|+|A_j|}$ is a (positive definite $\ge0$, symmetric) kernel, we have another proof, that $\frac{\gcd(a,b)}{a+b}$ is a kernel.
Using the product of these kernels, we get the new kernel $\frac{\gcd(a,b)^3}{ab(a+b)}$.
REPLY [15 votes]: The matrix $L_n$ is positive definite.
Proof. The matrix $G_n$ with entries ${\rm gcd}(a,b)$ is positive definite because of $G=D^T\Phi D$ where $\Phi={\rm diag}(\phi(1),\ldots,\phi(n))$ ($\phi$ the Euler's totient function) and $d_{ij}=1$ if $i|j$ and $0$ otherwise. Then the matrix $H_n$ with entries $\frac1{a+b}$ is positive definite because
$$h_{ij}=\int_0^1 x^{i+j-1}dx$$
and the matrix with entries $x^{i+j-1}$ is positive semi-definite for $x>0$. Finally $L_n=G_n\circ H_n$ (Hadamard product) is positive definite.<|endoftext|>
TITLE: Around the diophantine equation $\frac{a}{2b+3c}+\frac{b}{2c+3a}+\frac{c}{2a+3b}=\text{odd integer}$, over positive integers
QUESTION [7 upvotes]: I am interested to know if a similar theorem that shows this answer of the post
Estimating the size of solutions of a diophantine equation (this MathOverflow, January 5th 2016) is feasible for a different diophantine problem.
Conjecture. Let $a,b$ and $c$ be integers greater or equal than $1$. Then $$\frac{a}{2b+3c}+\frac{b}{2c+3a}+\frac{c}{2a+3b}$$ can never be an odd (positive) integer.
Question. I would like to know if the reasonings in the answers of the linked post also works for the diophantine equation in previous conjecture. Is it possible to prove previous conjecture, or is there any counterexample? Many thanks.
I don't know if this equation is in the literature (as reference I've added the mentioned post and [1]) as a special case of the expression $$\frac{\lambda a}{\beta b+\eta c}+\frac{\lambda b}{\beta c+\eta a}+\frac{\lambda c}{\beta a+\eta b}=N$$
with $N$ a non-zero integer, and for given integers $1\leq \lambda\leq \beta\leq \eta$. Our case, is similar than the linked post with $(\lambda,\beta,\eta)=(1,2,3)$. Our curve can be written (I did the calculation using Wolfram Alpha online calculator)
$$6 a^3 + 9 a^2 b + 4 a b^2 + 6 b^3 + 4 a^2 c + 18 a b c + 9 b^2 c + 9 a c^2 + 4 b c^2 + 6 c^3$$
$$\qquad\qquad\qquad\qquad\qquad\qquad\qquad-n((2 a + 3 b) (3 a + 2 c) (2 b + 3 c))=0$$
from $\frac{a}{2b+3c}+\frac{b}{2c+3a}+\frac{c}{2a+3b}=n$.
I think that this is a good companion of previous post, but if it is in the literature feel free to comment or provide your answer as a reference request and I search and read the statement about previous Conjecture from the literature.
References:
[1] Andrew Bremner and Allan MacLeod, An Unusual Cubic Representation Problem, Annales Mathematicae et Informaticae Volume 43 (2014), pp. 29-41.
REPLY [7 votes]: The problem of this question is qualitatively and quantitatively different in some ways from that considered by Andrew Bremner and myself.
If we take the cubic, with $N$ as a fixed constant, it is possible to show that the related elliptic curve is
\begin{equation*}
E_N:G^2=H^3+((35N+18)H+4(1260N+1441))^2
\end{equation*}
with the formulae linking $(H,G)$ to solutions $a,b,c$ being lengthy, but straightforward to find.
The discriminant is
\begin{equation*}
\Delta=2^{12}5^27^2(5N-3)(1260N+1441)^3(7N^2+15N+9)
\end{equation*}
so that $E_N$ has two components for all $N\ge1$.
This curve is in the standard form for one with $\mathbb{Z}3$ torsion, with points of order $3$ when $H=0$.
The algebra reducing the original cubic to the elliptic curve showed that there are other rational points.
I found
\begin{equation*}
H=\frac{288(18N+31)(36N+43)}{361}
\end{equation*}
giving
\begin{equation*}
G=\pm \frac{4(468N+635)(79056N^2+186120N+115297)}{6859}
\end{equation*}
which just lead to trivial solutions.
What I then found was the very surprising fact that this point is double $(-144,\pm 2660)$ for all $N$.
The positive $G$ value gives the parametric solution
\begin{equation*}
a=12(34992N^2+78120N+40499)
\end{equation*}
\begin{equation*}
b=-279936N^2-1271340N-1096057
\end{equation*}
\begin{equation*}
c=18(10368N^2+54180N+52811)
\end{equation*}
The elliptic curve has positive rank and simple numerical experiments show rank $3$ is fairly common.<|endoftext|>
TITLE: A minimality problem for a class of Banach spaces
QUESTION [6 upvotes]: The following question is related to the previous question Minimality properties of James' space; I post it as a new question since the system does not allow me to add a comment.
Question Consider the following class of non-Hilbertian spaces:
$X_{p,2}=(\sum_{n=1}^\infty \oplus\ell^p_n)_2$, $1\le p\le \infty$, $p\neq 2$.
Is it true that the only infinite dimensional Banach space that is isomorphically embedded into anyone of them is the Hilbert space?
Notice that all these spaces are subspaces of the space $\mathcal{J}$.
REPLY [2 votes]: I think such a space has type 2 and cotype 2 so by Kwapien's theorem it is isomorphic to a Hilbert space.<|endoftext|>
TITLE: What are Harish-Chandra bimodules used for?
QUESTION [12 upvotes]: There are many recent papers on classification of Harish-Chandra bimodules for rational Cherednik algebras and, more generally, non-commutative algebras which are quantizations of symplectic singularities (Losev). What is the meaning of Harish-Chandra bimodules in terms of representation theory of the underlying algebra/its category O? Are Harish-Chandra bimodules related to the classical notion of Harish-Chandra modules?
REPLY [7 votes]: Here is an answer from a mathematician who prefers me to post it here myself:
Harish-Chandra bimodules make sense in a very wide context. Take two
filtered algebras A, A' that quantize the same commutative algebra $C$,
and fix isomorphisms ${\rm gr} A \to C$, ${\rm gr} A^{'} \to C$. Then one can make sense
of the definition of a $HC (A, A^{'})$-bimodule. These are (A, A')-bimodules,
say B, that admit a filtration such that \gr B is a finitely generated
C-module, meaning that the left and right actions on C coincide. It is
not hard to see that if A, A' are $U(g)$ for a simple Lie algebra g, this
coincides with the notion of HC bimodule that I alluded above.
In the context of symplectic singularities, note that you need to have a
Hamiltonian $\mathbb C^*$-action to define the category O. Such an action does not
always exist (e.g. for Kleinian singularities outside of type A). In
this sense, HC bimodules are a substitute for the category O. See for
example Ginzburg https://arxiv.org/pdf/0807.0339.pdf
When you do have categories O, HC bimodules give, via tensor product,
functors between categories O for different quantization parameters. For
example, projective functors in Lie theory are a special case of
tensoring with a HC $U(g)$-bimodule. In this sense, HC bimodules also
generalize the notion of projective functors. Translation functors for
Cherednik algebras are a special case of this. I must warn, however,
that tensoring with a HC bimodule is in general a very bad functor -- it
can kill many things and it is not exact. Nevertheless, these functors
were used by Losev to construct derived equivalences between categories
O for Cherednik algebras https://arxiv.org/pdf/1406.7502.pdf
Also, Harish-Chandra bimodules are much more sensitive to the
quantization parameter than the category O is. Category O always has the
same number of simples = number of fixed points under Hamiltonian torus
action. This is far from being true for HC bimodules. For example, for
type A Cherednik algebras the quantization parameter is a complex number
$c$ (I apologize if I am overexplaining, I don't know how familiar you are
with these). If $c$ is not a rational number with denominator $1 < d \leq n$
($n =$ rank of symmetric group) then the category O is semisimple and
equivalent to reps of $S_n$. This is not true for the category of HC
$H_{c}$-bimodules. For these parameters, the category is still semisimple,
but it is only equivalent to reps of $S_n$ when c is an integer.
Otherwise, it is equivalent to Vec. In this sense, HC bimodules detect
how integral the parameter is. See https://arxiv.org/pdf/1409.5465.pdf
Theorem 1.1 for the case of rational Cherednik algebras (the subgroup
$W_{c}$ essentially detects how far c is from being integral). This was
generalized by Losev to symplectic singularities in
https://arxiv.org/pdf/1810.07625.pdf
One more thing, the simplest example of a HC $A$-bimodule is the regular
bimodule. So one can use HC bimodules to answer questions about, for
example, ideals in $A$ (usually these techniques come from constructing
restriction functors for HC bimodules, similar to the
Bezrukavnikov-Etingof functors for category O and applying them to the
regular bimodule). This was used by Losev for Cherednik algebras in
https://arxiv.org/pdf/1001.0239.pdf (see Thms 1.3.1 and 5.8.1) and for
finite W-algebras in https://arxiv.org/pdf/0807.1023.pdf
Finally, in the context of symplectic resolutions it is believed that HC
bimodules should categorify the homology of the generalized Steinberg
variety. This is of course not true in general (even for Cherednik
algebras for the reasons above -- for some parameters there are simply
not enough irreducibles) but it should be true for integral parameters,
for an appropriate notion of integral. See Braden-Proudfoot-Webster,
https://arxiv.org/pdf/1208.3863.pdf Proposition 6.16 (later in that
paper they show that wall-crossing functors are always tensoring with an
appropriate HC bimodule, Proposition 6.23)<|endoftext|>
TITLE: Does $Π^1_{2n+1} = (Σ^1_{2n+1})^{M_{2n-1}}$?
QUESTION [10 upvotes]: Every $Π^1_1$ formula $φ$ without free second order variables can be converted into a $Σ^1_1$ $ψ$ such that $φ ⇔ ψ^\mathrm{HYP}$, and vice versa. ($\mathrm{HYP}$ is the hyperarithmetical universe, which can be identified with $L_{ω_1^\mathrm{CK}}$.) Essentially, arbitrary $Π^1_1$ statement $⇔$ well-foundedness of some recursive relation '$≺$' $⇔$ existence of a hyperarithmetical set iterating the Turing jump along '$≺$' $⇔$ arbitrary $(Σ^1_1)^\mathrm{HYP}$ statement.
My question is whether, assuming projective determinacy, an analogous correspondence holds for $Π^1_{2n+1}$; and I conjecture $Π^1_{2n+1} = (Σ^1_{2n+1})^{M_{2n-1}}$.
$M_n$ is the minimal iterable inner model with $n$ Woodin cardinals. $M_0$ is the constructible universe $L$, and $M_{-1}$ (not an inner model) would be $L_{ω_1^{\mathrm{CK}}}$. The reals in $M_n$ are precisely those that are $Δ^1_{n+2}$ in a countable ordinal. For even $n$, $M_n$ is $Σ^1_{n+2}$ correct, but this is not the case for odd $n$.
A positive answer to the question should enhance our understanding of $Π^1_{2n+1}$ prewellordering and uniformization. True $Σ^1_{2n}$ statements can be 'graded' (assuming projective determinacy) by the complexity of the least witness. For example, consider a theory $T$ such as ZFC + "there is a supercompact cardinal", and assume that $T$ has sufficiently sound models. At the $Σ^1_2$ level, we can assign $T$ an ordinal based on the least height of a transitive model of $T$; furthermore, there is a $Δ^1_2$ example of such a model. This generalizes to $Σ^1_{2n}$ and models of $T$ that are closed under $M_{2n-3}^\#$. But what kind of witnesses do we have for $Π^1_{2n+1}$ statements?
The first paragraph gives an answer for $Π^1_1$, and we can generalize it as follows. Assuming $M_{2n+1}^\#$ exists, a $Π^1_{2n+3}$ statement $T$ is true iff there is an iterable model $M$ of ZFC + "$2n+1$ Woodin cardinals" such that $M^{\mathrm{Coll}(ω,δ)}⊨T$ where $δ$ is the least Woodin cardinal in $M$. (The use of ZFC in $M$ is essentially arbitrary; also, genericity iterations allow Woodin cardinals to 'absorb' real quantifiers.) Presumably, such an $M$ can be chosen to be $Δ^1_{2n+3}$ and its existence is $(Σ^1_{2n+3})^{M_{2n+1}}$ (but how?) (Also, the least complexity of such $M$ should be connected to $Π^1_{2n+3}$ prewellordering, but the exact connection is unclear to me as the prewellordering is about sets of reals.)
REPLY [10 votes]: Your conjecture is true. We assume PD throughout. The proof I see requires citing a number of facts from inner model theory and descriptive set theory. First, it uses Woodin's theorem characterizing the reals of $M_{2n-1}$ as the set $Q_{2n+1}$ of points in Baire space that are $\Delta^1_{2n+1}$ definable from a countable ordinal. In symbols:
Theorem (Woodin). $\omega^\omega\cap M_{2n-1} = Q_{2n+1}$.
It uses a correctness theorem for the odd levels:
Theorem $M_{2n-1}$ is $\Pi^1_{2n}$-correct.
A set $A\subseteq \omega^\omega$ is $\Pi^1_{2n+1}$-bounded if $\Pi^1_{2n+1} = \exists^{A} \Pi^1_{2n+1}$. We need that $Q_{2n+1}$ is $\Pi^1_{2n+1}$-bounded. In fact, something stronger is true (see Kechris-Martin-Solovay's "Introduction to $Q$-theory"):
Theorem (Kechris-Martin-Solovay). $Q_{2n+1}$ is the largest $\Pi^1_{2n+1}$-bounded subset of Baire space.
We need Moschovakis's "Spector-Gandy theorem for the odd levels" (Moschovakis, Descriptive Set Theory, 6E.7):
Theorem (Moschovakis) $\Pi^1_{2n+1}\cap\omega^\omega = \exists^{\Delta^1_{2n+1}\cap \omega^\omega}\Pi^1_{2n}\cap \omega^\omega$.
Moschovakis's theorem will actually be applied to $Q_{2n+1}$ using the $Q$-Theory Reflection Theorem:
Theorem (Kechris-Martin-Solovay) If $A\subseteq \omega^\omega$ is $\Pi^1_{2n+1}$, then $\exists x\in \Delta^1_{2n+1}\ A(x)$ if and only if $\exists x\in Q_{2n+1}\ A(x)$.
Given these facts, the calculation becomes a straightforward pointclass calculation.
By definition, $\Sigma^1_{2n+1} = \exists^{\omega^\omega}\Pi^1_{2n}$, so $(\Sigma^1_{2n+1})^{M_{2n-1}} = \exists^{\omega^\omega\cap M_{2n-1}}(\Pi^1_{2n})^{M_{2n-1}}$.
Woodin's theorem characterizing $\Pi^1_{2n+1}$ along with the $\Pi^1_{2n}$-correctness of $M_{2n-1}$ imply $\exists^{\omega^\omega\cap M_{2n-1}}(\Pi^1_{2n})^{M_{2n-1}}\cap\omega^\omega = \exists^{Q_{2n+1}}\Pi^1_{2n} \cap\omega^\omega$.
Moschovakis's Spector-Gandy Theorem along with the $Q$-Theory Reflection Theorem yields that $\exists^{Q_{2n+1}}\Pi^1_{2n}\cap \omega^\omega = \exists^{\Delta^1_{2n+1}\cap\omega^\omega}\Pi^1_{2n}\cap \omega^\omega = \Pi^1_{2n+1}\cap \omega^\omega$.
Stringing together a bunch of pointclass identities, one can conclude that $(\Sigma^1_{2n+1})^{M_{2n-1}}\mathrel{\cap} \omega^\omega = \Pi^1_{2n+1} \cap \omega^\omega$.
You might also want to look at Theorem 4.12 of John Steel's paper Projectively Well-Ordered Inner Models. I think you can use the proof to get a more inner model theoretic proof of your conjecture, but Steel's result is closely related and of independent interest.<|endoftext|>
TITLE: Are evil properties really evil
QUESTION [15 upvotes]: I have this question for a moment now, so I think it is time that I sort it out.
I got into category theory and homotopy type theory at the same time, and so I have always read and been told that one should be careful about "evil" properties, i.e. the ones that are not invariant under equivalence of categories. So in order to try and disambiguate the terminology, what I will refer to as "categories" are up to equivalence, and I will call a precategory the objects up to isomorphism. In other words, categories are objects of the category $\operatorname{Ho}(\operatorname{Cat})$, for the folk model structure of $\operatorname{Cat}$, and the precategories are the objects of $\operatorname{Cat}$.
It is my understanding that category theory is in fact about categories and not precategories, and it makes perfect sense, since the point of category theory is to give the proper meaning to isomorphisms. The intuition that I always keep in mind is that it should not matter if I see the same object many times as long as I know that it is the same. In order to really talk about these categories, people have started looking at the appropriate language, which is a language in which evil property do not make sense, or are not expressible. This is a way to get to the theory of categories, or their internal language (I am not sure how to refer to this notion exactly).
However, there are many constructions that are used on precategories, the two main ones that I have in mind are the Reedy categories and the contextual categories (aka C-systems), and some results are proved by making heavy use of these constructions.
My understanding of the situation
The precatgories can be thought of as some sort of a "presentation" of a given category. It is of course not unique since many non-isomorphic can be equivalent, but that's fine, I am used to that in groups or vector spaces : A group can have different presentations. Now the using an "evil" notion would amount to working in a given presentation of a group, so whatever non-evil property you prove for a Reedy category is in fact a property of the category presented by my precategory which is Reedy. In group theory would be analogue to a theorem of the form
"If a group has a finite presentation, then it satisfies $P$"
And then I can freely state that $\mathbb{Z}$ satisfies $P$, since it is finitely presented as $\langle x\rangle$ , although I might as well give the (different) presentation $\langle x_1,x_2,\ldots | x_1=x_2, x_1 =x_3,\ldots\rangle$, which is not finite. It is sort of picking a representative in an equivalence class to prove a property which goes through the quotient. This is something we do a lot, and it gives a perfectly valid proof. So here I can very easily transform the property "being a Reedy category" to "being equivalent to a Reedy category", and I have changed my evil property into a non-evil one. Every non-evil property I can prove for Reedy categories is in fact provable for all the precategory that are categorically equivalent to a Reedy category
Possible limitations
There are two main "problems" that I can see using evil notions in order to prove non-evil ones
Firstly the construction that I get may not be "canonical" or "natural" (again I am not sure what the proper terminology should be here). If I take the example of a Reedy model structure, there might be many equivalent precategory that present the same category and that are Reedy. The Reedy model structure I get using one might not agree with the Reedy model structure I get using the other - even up to the equivalence between them. This is in my intuition much like the fact that every finite dimensional vector space is isomorphic to its dual, but there is necessarily a choice of a basis, leading to really different isomorphisms. Here a choice of a basis is a given presentation. This also makes sense with my previous correction of Reedy to be non-evil : the property "being equivalent to a Reedy precategory" gives an equivalence of categories, and then a specific choice of a presentation, and there might be many ways to "be equivalent to a Reedy precategory", leading to different constructions for the model structure. But again, constructing something non-canonical is not that big of a deal, if we carry out the constructions everywhere. After it is true that all finite dimensional vector space is equivalent to its dual even if there is no canonical way to do it - Why not imagine properties in non-evil properties in categories that are true, but never "non-evily" true (i.e. never true in a canonical way)?
The proof that we get is not in the language of categories (and I think this is basically saying the same thing). It is a proof in the language of precategories. But again it sounds fine to me, with finite dimensional vector spaces we usually say "take a basis", which is not expressible in their internal language. It does not make the fact true, it just makes it non-canonical.
My Question
So firstly, I would like to know if my intuition is correct. If so, the "evil" properties do not seem too evil to me, and we can freely use them as long as we don't claim canonicity. I have seen examples of this in different areas of mathematics, so why not in category theory? If my intuition is incorrect, am I missing something that would make these concepts truly evil?
Then a related question that I have (assuming my understanding is correct) is are there any known examples of a property that is always true but never canonically in category theory? What I mean by that is a theorem that we can prove about categories but using precategories, and that is not provable in the internal language of categories? If we know for some reason that no such thing exists, then I would be convinced to avoid evil properties, as they are unnecessary, but otherwise how to know that we are not missing something by avoiding them?
REPLY [14 votes]: Before I try to answer the question itself, let me make a few preliminary remarks.
Firstly, a minor point: it's not really correct to say that categories are the objects of Ho(Cat), if by Ho(Cat) you mean the homotopy 1-category of Cat; the correct thing to say is that categories are the objects of the 2-category Cat. The point is that in Ho(Cat), whose morphisms are isomorphism classes of functors, you lose the ability to distinguish between multiple isomorphisms between two isomorphic functors.
Secondly, it's perfectly correct, as you say, that there are some operations on things sometimes called "categories" that require treating them up to isomorphism rather than equivalence. But instead of your "precategories", I will call those things "strict categories", to match the nLab and the HoTT Book (of which the latter got the terminology from the former). So Reedy categories and contextual categories are best thought of as strict categories, rather than categories proper.
Thirdly, I would approach the problem of defining a "Reedy non-strict category" somewhat differently. It's true that a given category could be equivalent to a Reedy strict category in more than one way --- but even more than that, it's already true that a given strict category can be Reedy in more than one way! That is, being Reedy is structure on a strict category, not a property of it, and different choices of Reedy structure on the domain can yield different Reedy model structures, even before we start to worry about strictness of categories. So similarly, being Reedy must also be structure of a non-strict category, and once we realize that it is quite natural to say that a Reedy structure on a non-strict category $C$ consists of a specified strict category $C'$, a specified equivalence $C\simeq C'$, and a Reedy structure on $C'$. With this definition, a Reedy structure on a non-strict category induces a Reedy model structure in a canonical and equivalence-invariant way. (With that said, there seems relatively little reason to do such a thing, since most naturally-arising Reedy categories are in fact strict. There's nothing wrong with using strict categories, as long as we don't assume that every category is strict.)
Now, your question seems to be roughly "what is wrong with non-equivalence-invariant constructions on categories?" As you say, this is really the same question as "what is wrong with non-isomorphism-invariant constructions on vector spaces?".
The first answer is that in practice we don't keep track of "which equivalent/isomorphic copy" of an object we are using. For instance, the tangent space of a manifold at a point can be defined in many ways, yielding isomorphic but non-equal vector spaces. If we have a construction on vector spaces that is not isomorphism-invariant, then when someone wants to apply that construction to the tangent space of a manifold, we have to ask "which tangent space?" Or, even worse, if we have a construction on groups that is not isomorphism-invariant and someone wants to apply that construction to the two-element group (or even the one-element group!) we have to ask "which one?" The experience of abstract/structural mathematics over the past century strongly suggests that such constructions should not be considered part of linear algebra or group theory. (You seem to believe that such constructions are used in some areas of mathematics, but I would dispute that; can you give examples of the sort of thing you are thinking of?) Essentially the same is true for categories and equivalences.
The second answer, which to my mind is even more powerful, is that only invariant constructions can be generalized to "variable" objects. Consider for instance the notion of vector bundle over a space. Even though we can choose bases for each individual fiber, if the bundle is nontrivial it may not be possible to choose a family of global sections that form a basis at every point simultaneously. Thus, a construction on vector spaces which depends on the choice of basis cannot be performed on vector bundles. Another way of thinking about this is that a vector bundle can be represented by a functor into the groupoid of vector spaces, so any construction on vector spaces that is to be "composed" with such a functor must itself be functorial on the groupoid of vector spaces, i.e. respect isomorphism.
Exactly the same thing is true for categories, although the relevant kind of "variation" also has to be categorified. There are "categorified spaces" and "bundles of categories" over them such that although each individual "fiber" can be presented by some strict category, the entire bundle cannot be simultaneously so presented. Thus, a construction on categories that doesn't respect equivalence, or equivalently that depends on a choice of strict presentation, cannot be performed on such "bundles of categories".<|endoftext|>
TITLE: Is the product of commuting ultrafilters an ultrafilter?
QUESTION [9 upvotes]: If $U$ is a filter on $X$ and $W$ is a filter on $Y$, their product is the filter $U\times W$ on $X\times Y$ generated by rectangles $A\times B$ where $A\in U$ and $B\in W$.
In certain circumstances (e.g., when $U$ is $|W|$-complete), the product of two ultrafilters $U$ and $W$ is again an ultrafilter. In this situation, $U$ and $W$ must commute in the following sense: for any binary relation $R$, we have
$\forall^U x\ \forall^W y\ (x\mathrel{R} y)$ if and only if $\forall^W y\ \forall^U x\ (x\mathrel{R} y)$. (Here, for $P$ a unary predicate, we write $\forall^U x\ P(x)$ to mean that $\{x\in X : P(x)\}\in U$.)
The question is whether the converse holds.
Question: If two ultrafilters $U$ and $W$ commute, must $U\times W$ be an ultrafilter?
Conceivably, a positive answer is provable in ZFC or assuming GCH. The question has a vacuous positive answer under the assumption that there are no measurable cardinals since this implies all commuting ultrafilters are principal. Put another way, constructing a counterexample would require the use of large cardinals.
Background:
Ultrafilters $U$ and $W$ such that $U\times W$ is ultra were studied by Blass in his thesis. Blass showed that this is equivalent to the statement that $U$ is complete modulo $W$ in the sense that for any sequence $\langle A_i : i\in I\rangle\subseteq U$ defined on a $W$-large set $I$, $\bigcap_{i\in J} A_i\in U$ for some $W$-large set $J$. In particular, despite all appearances, the relation $U$ is complete modulo $W$ is symmetric in $U$ and $W$.
Commuting ultrafilters are related to Kunen's Commuting Ultrapowers Lemma, which says that if $U$ is $|W|$-complete, then $j_U(j_W) = j_W\restriction \text{Ult}(V,U)$ (which is pretty clear) and $j_W(j_U) = j_U\restriction \text{Ult}(V,W)$ (which is nontrivial). Here $j_U : V\to \text{Ult}(V,U)$ denotes the (transitive collapse of the) ultrapower embedding associated to $U$ and $j_U(j_W) = \bigcup_{x\in V} j_U(j_W\restriction x)$. It is a not-so-easy exercise to see that for countably complete ultrafilters $U$ and $W$ commute if and only if $j_U(j_W) = j_W\restriction \text{Ult}(V,U)$. In particular, despite all appearances, the relation $j_U(j_W) = j_W\restriction \text{Ult}(V,U)$ is symmetric in $U$ and $W$.
REPLY [3 votes]: The following may give a hint:
Suppose $U$ is a uniform ultrafilter on $\omega$ and $W$ is an ultrafilter on $\kappa$ such that $W$ commutes with $U$, then $W$ is countably complete.
Otherwise, there exist $\langle A_i \in W: i\in \omega\rangle$ decreasing such that $\bigcap_{i\in \omega} A_i =\emptyset$. Let $R\subset \omega\times \kappa$ be such that $i R \gamma$ iff $\gamma\in A_i$.
It can't be the case that $\forall^W \gamma \forall^U i \ iR\gamma$. Since fix some such $\gamma\in \kappa$, we have $\gamma\in \bigcap_{i\in \omega} A_i$ which is impossible.
By commuting, it can only be that $\neg\forall^U i \forall^W \gamma \ iR\gamma$, hence $\forall^U i \forall^W \gamma \ \neg iR\gamma$. But this is bogus too since fix some such $i\in \omega$, we have $D\in W$ such that for all $\gamma\in D$, $\gamma\not\in A_i$, contradicting with the fact that $A_i\in W$.<|endoftext|>
TITLE: Parity of shuffle permutations
QUESTION [5 upvotes]: A $(p,q)$-shuffle is a permutation $\sigma$ of the set $\{1, \dots, p, p+1, \dots,p+q\}$ such that $$\sigma(1)<\dots<\sigma(p)$$ and $$\sigma(p+1) < \dots<\sigma(p+q)\,.$$
It is known that the number of $(p,q)$-shuffles is ${p+q \choose p}$.
Looking at the case $p+q = 6$ and considering the parity of shuffles, I was surprised to find that
$3$ of the ${6 \choose 1} = 6$ $(1,5)$-shuffles have even parity
$9$ of the ${6 \choose 2} = 15$ $(2,4)$-shuffles have even parity
$10$ of the ${6 \choose 3} = 20$ $(3,3)$-shuffles have even parity
$9$ of the ${6 \choose 4} = 15$ $(4,2)$-shuffles have even parity
$3$ of the ${6 \choose 5} = 6$ $(5,1)$-shuffles have even parity
It's not so difficult to believe that the number of even $(p,q)$-shuffles should equal the number of even $(q,p)$-shuffles. But I was very surprised to see that there are so many more even than odd $(2,4)$-shuffles.
Are any closed-form solutions known for the number of even $(p,q)$-shuffles? If not, is anything known about the asymptotic behaviour?
REPLY [3 votes]: As Philippe suggests in the comments, it is well know that
$$\sum_{\sigma \text{ is a $(a,b)$-shuffle}} q^{\ell(\sigma)}={a+b \choose b}_q$$
where the $q$-binomial coefficient (or Gaussian binomial coefficient) on the right side is defined by
$${n \choose k}_q=\frac{[n]_q! }{[k]_q! [n-k]_q!}$$
where $[n]_q!=[1]_q [2]_q \cdots [n]_q$ and $[n]_q=1+q+\cdots +q^{n-1}$.
Thus the difference between the number of even and odd shuffles is obtained by specializing at $q=-1$. This polynomial is palindromic, so, depending on the parity, one either gets 0 or the middle (and largest) coefficient of the polynomial ${a+b \choose b}_q$.
Precise asymptotics for this coefficient (and others) are given in Theorem 1 of this paper by Melczer, Panova, and Pemantle.<|endoftext|>
TITLE: Kirby calculus on Mazur manifolds
QUESTION [10 upvotes]: I have questions about Akbulut and Kirby's paper Mazur manifolds.
I couldn't figure out the following equality passages:
Any help will be appreciated.
REPLY [9 votes]: Here is a proof with pictures. Observe that blowing up and blowing own doesn't change the 3 manifold,i.e, the boundary. So all these above pictures have same boundary. As a 3 manifolds all of these are isomorphic.<|endoftext|>
TITLE: Factorization and vertex algebra cohomology
QUESTION [12 upvotes]: A chiral algebra on a smooth curve $X$, in the sense of Beilinson-Drinfeld, is a right $D_{X}$-module with a chiral bracket, which is a map $\mathcal{V}^{\boxtimes 2}(\infty\Delta)\rightarrow \Delta_{*}\mathcal{V}$ satisfying certain conditions making it look like a Lie bracket. In fact $\mathcal{V}$ is a Lie algebra with respect to a certain pseudo-tensor structure on $D_{X}$-modules. A pseudo-tensor structure is what one obtains when one forgets the tensor product but remembers the associated multilinear maps. Multi-linear maps from a collection $\{\mathcal{V}_{i\in I}\}$ to $\mathcal{W}$ are defined to be $\mathrm{Hom}_{D_{X^{I}}}(\boxtimes_{i\in I}\mathcal{V}_{i}(\infty\Delta_{\mathrm{big}}),\Delta_{*}\mathcal{W})$. One thus obtains a complex of vector spaces $C^{*}_{CE}(\mathcal{V})$ via the appropriate construction of Chevalley-Eilenberg cochains, which makes sense in a pseudo-tensor category. Notably this formalism does not produce CE chains.
If $X$ is taken to a be a formal disc $D$, then we obtain a vertex algebra. I believe in this case the above construction produces the vertex algebra cohomology (with coefficients in the adjoint module) studied in the work of Bakalov, de Sole, Heluani and Kac (arXiv link). Is this true? If not what is the precise relation?
Now if $\mathcal{V}$ is a chiral algebra, then BD construct its factorization homology. This is constructed as de Rham homology of the associated $D$-module on the Ran space of $X$. In chiral terms I believe it is gotten by taking (the colimit over finite sets $I$) of the Chevalley Cousins complexes on $X^{I}$, cf. these notes by Rozenblyum (on Gaitsgory's webpage).
This can also be performed on a formal disc $D$*, in which case one obtains what one might call Vertex Homology. Has this been studied anywhere? Is there a relation with the work of BdSHK cited above, in particular does the BdSHK complex act on it?
For example, if I'm not mistaken we compute $H^{\mathrm{vtx}}_{0}(V)=V/\operatorname{Res}(V)$, where $\operatorname{Res}(V)$ is the image of all the residues $\operatorname{res}_{z=0}(v(z))$ of the fields, which has a very similar flavour to the complex of BdSH.
*Edit, in accordance with David Ben-Zvi's comments it seems that vertex algebra homology as defined in this question would be better defined over a punctured disc $D^{*}$.
REPLY [8 votes]: If $X$ is taken to a be a formal disc $D$, then we obtain a vertex algebra. I believe in this case the above construction produces the vertex algebra cohomology (with coefficients in the adjoint module) studied in the work of Bakalov, de Sole, Heluani and Kac. Is this true? If not what is the precise relation?
This is not exactly so. If you take $X=\mathbb{A}^1$ and your D-modules are translation equivariant and you consider the translation equivariant CE complex you will get the same complex that we studied.
Chiral algebras on the disc are not the same thing as vertex algebras. If you consider chiral algebras equivariant with respect to the action of the group $Aut_\mathcal{O}$ of changes of coordinates then they are the same as quasi-conformal vertex algebras (notation of Frenkel Ben-Zvi). The $Aut_\mathcal{O}$-equivariant complex as you describe on the punctured disk will be equivalent to ours in the quasi-conformal setting.
This can also be performed on a formal disc $D^*$, in which case one obtains what one might call Vertex Homology. Has this been studied anywhere? Is there a relation with the work of BdSHK cited above, in particular does the BdSHK complex act on it?
Again if you want something that should be called "vertex homology" then you should consider either $Aut_{\mathcal O}$ equivariant objects on the disk or translation equivariant objects on the line.
There are a few statements in this answer that are plain wrong cause I had in mind translation invariants sections on the disk. I'll strike them through.
Factorization homology of a unital chiral algebra vanishes on an open proper subset of a curve. Factorization homology of the restriction of a chiral algebra to an affine proper subset of a curve vanishes. This is the content of Lemma 4.3.4 in Beilinson and Drinfeld's book.
I also believe that David's comments about Factorization homology on the disk are wrong. There's a big difference between the topological setting and the algebraic setting and between $E_2$ algebras and the chiral/factorization algebras of Beilinson and Drinfeld. In particular, the chiral homology of the disk with coefficient on any unital chiral algebra vanishes. This is because chiral homology is not just the CE complex of a Lie algebra on the Ran space, but also it's de Rham complex. In particular as you point out the degree zero homology of this complex is $V/V_{(-1)}V = 0$ and this is because we have the vacuum vector to play around. Therefore the image of $\mathbb{1}$ in chiral homology vanishes and then by the last comment before the mentioned Lemma 4.3.4 the whole complex vanishes.
Chiral homology of a curve is really a global object and you cannot detect any of it by looking at formal disks (see however 2.4.12 in BD in agreement with David's comments). The complex studied in BdSHK is purely a local object, or rather a translation equivariant complex on the line. I am not aware of any direct connection between them.
There are however a couple of instances where you can compute the global chiral homology by local considerations: the case of $X=\mathbb{P}^1$ and the case of an elliptic curve. Both cases because you have global coordinates. In degree $0$ this is the work of Zhu way before chiral homology was defined. In the elliptic curve case we have a preprint with J. van Ekeren describing the nodal curve limit of the first chiral homology group and we are supposed to post a continuation describing the general elliptic curve soon.
Edit: To reflect David's comments. Indeed by $U\subset X$ open they really mean an open affine, so that the example of a disconnected curve is not allowed. The point needed is the vanishing of the de Rham cohomology. They use this lemma heavily in proving the quasi-isomorphism of the complex with supports in 4.4.2--4.4.3.
Commutative vertex algebras are not a counter-example to this statement as the functor of coinvariants on the disk is canonically zero as long as your algebras are unital. Here the very naive notion of coinvariants is clear, the Cousin complex on $D^2$ is already enough to compute the homology in degree $0$ and this group is $V/V_{(-1)}V$ (think of homologies with support at the origin).
As for the $E_2$ algebras business: here is where things get sloppy. As far as I know the folklore is that $E_2$ algebras are essentially the same as topological vertex algebras. With this I have no objection, however notice that the latter is a vertex algebra together with a BRST differential, and that this equivalence is inherently derived. Since the BRST cohomology of a topological vertex algebra is typically stupid (it lies in conformal weight zero and is a commutative algebra) then it's no wonder that the chiral homology viewed as a complex with this extra BRST differential is silly. However the complex without taking this differential into account is still very interesting, you capture irreducible modules and Jacobi elliptic functions on genus $1$ (at least in rational cases) and so forth.
The second point is more subtle: I have seen in many places the analogy drawn between Factorization homology in the topological world and chiral homology in the algebraic setting. I may be wrong and very much outdated in the literature that I can cope with, but I have not seen anything written to prove an actual dictionary. There is no formal way of constructing a factorization algebra (in the topological meaning) coming from a factorization algebra in the algebraic world. Folklore expected that if one takes the complex that computes chiral homology on the Ran space of $X$ á la BD and one takes an analitification of this and then sections with compact support, this is something that looks like a Factorization algebra in the topological setting. As far as I know the problem is that the process of analytification (or tensoring with the analytic de Rham complex) does not commute with tensor products and this breaks down being a Lie/Commutative algebra.
I don't claim that having a topological intuition is wrong, just that at least when trying to translate that intuition into an actual theorem in BD's setting the situation is not so transparent even in the simplest of cases (for example the zeoro-th Hochshild Homology of higher Zhu algebras equals zero-th chiral homology of vertex algebras, this is a theorem that requires a lot of theory and a bunch of further assumptions that have no topological counterparts).<|endoftext|>
TITLE: Weyl map for $SU(n)$
QUESTION [9 upvotes]: Let $G= SU(n)$ and let $\mathbb{T}$ be the maximal torus in $G$ given by diagonal matrices. We have
$$
H^*(G,\mathbb{Q}) \cong \Lambda_{\mathbb{Q}}[x_3, x_5, \dots, x_{2n-1}] \ .
$$
Now consider the Weyl map
$$
p \colon G/\mathbb{T} \times \mathbb{T} \to G \quad , \quad ([g],z) \mapsto gzg^{-1}\ .
$$
The induced map in rational cohomology
$$
p^* \colon H^*(G,\mathbb{Q}) \to H^*(G/\mathbb{T}, \mathbb{Q}) \otimes H^*(\mathbb{T},\mathbb{Q})
$$
is injective. In fact, if we restrict the codomain to fixed points with respect to a certain action of the Weyl group $W$ it becomes an isomorphism (see for example Reeder, On the Cohomology of Compact Lie Groups). The cohomology ring $H^*(\mathbb{T},\mathbb{Q})$ is isomorphic to another exterior algebra and there are also explicit descriptions of $H^*(G/\mathbb{T},\mathbb{Q})$ (see Reeder's paper again for a reference).
Is there a formula describing $p^*(x_{2i+1})$ in terms of any set of natural generators for the codomain?
REPLY [7 votes]: First, let me fix generators for $H^*(SU(n))$ and $H^*(SU(n)/\mathbb T)$: For the first, consider the vector bundle on $\Sigma SU(n)$ with clutching map $\operatorname{id}_{SU(n)}$, i.e. with classifying map $f_n\colon\Sigma SU(n)\simeq \Sigma\Omega BSU(n)\to BSU(n)$, and let $\Sigma x_{2i-1} = f^*c_i$. For the second, let $\pi_k\colon \mathbb T\subset U(1)^k\to U(1)$ be the projection to the $k$-th coordinate and $L_k = SU(n)\times_{T,\pi_k} \mathbb C$ be the associated line bundle, and set $y_k = c_1(L_k)$; since $\bigoplus_{k=1}^n \pi_k$ is the restriction of the defining representation of $SU(n)$ to $\mathbb T$, the sum $E := \bigoplus_{k=1}^n L_k$ is trivial, so that all symmetric polynomials in the $y_k$ vanish since they can be expressed via Chern classes of $E$. The induced map $Q[y_1,\dots,y_n]/Q[y_1,\dots,y_n]^{S_n}\to H^*(SU(n)/\mathbb T)$ is an isomorphism; a basis for this cokernel is given by the monomials $y_1^{\alpha_1}\dots y_n^{\alpha_n}$ with $0\le \alpha_k < k$.
Now think of $p:G/\mathbb T\times\mathbb T\to SU(n)$ as an automorphism $\phi$ of $E\boxtimes\underline{\mathbb C}$; it splits as $\phi = \phi_1\oplus\dots\oplus \phi_n$, where $\phi_k = \operatorname{id}_{L_k}\boxtimes \pi_k$ is the automorphism of $L_k\boxtimes \underline{\mathbb C}$ obtained as the external tensor product of the identity of $L_k$ with the projection $\pi_k:\mathbb T\subset U(1)^k\to U(1)$.
Consider the clutching construction $(E\boxtimes\underline{\mathbb C})^\phi\to G/\mathbb T\times\mathbb T\times S^1$ which is obtained by gluing the two ends of $E\boxtimes\underline{\mathbb C}$ together using $\phi$. Denote by $z_k = \pi_k^*([U(1)]), u = [S^1]$ the obvious cohomology classes in degree $1$, and itdentify them and the $y_k$ with their image in the product. An easy argument shows that $c_1(\mathbb C^{\pi_k}) = uz_k\in H^2(\mathbb T\times S^1)$. By naturality, we have
\begin{align*}
(E\boxtimes\underline{\mathbb C})^\phi &\cong \bigoplus_{k=1}^n(L_k\boxtimes\underline{\mathbb C})^{\phi_k}\\
c_1\big((L_k\boxtimes\underline{\mathbb C})^{\phi_k}\big) &= c_1(L_k) + c_1(\underline{\mathbb C}^{\pi_k}) = y_k + uz_k\\
c\big((E\boxtimes\underline{\mathbb C})^\phi\big) &= \prod_{k=1}^n c\big((L_k\boxtimes\underline{\mathbb C})^{\phi_k}\big)\\
&= \prod_{k=1}^n (1 + y_k + uz_k)\\
&= \underbrace{c(E)}_{= 0} + u\sum_{k=1}^n z_k \sum_{S\subset \{1,\dots,n\}\smallsetminus\{k\}}\prod_{l\in S} y_l
\end{align*}
It is no surprise that this expression vanishes for $u=0$ since we can use the chosen trivialization of $E\boxtimes \underline{\mathbb C}$ to descend $(E\boxtimes \underline{\mathbb C})^\phi$ to $\Sigma(G/\mathbb T\times \mathbb T)$, and the resulting vector bundle has classifying map $f_n\circ \Sigma p$. Chasing through the definitions, we see that
$$
p^* x_{2i-1} = \sum_{k=1}^n\Big[\sum_{\substack{S\subset \{1,\dots,n\}\smallsetminus\{k\}\\|S| = i - 1}}\prod_{l\in S} y_l\Big]\otimes z_k\ .
$$<|endoftext|>
TITLE: Show that this process is not a martingale
QUESTION [9 upvotes]: I am cross-posting this question from MSE since I did not received any answer, furthermore I tried asking some professors in my university but still we could not find an answer.
The most surprising thing is that this exercise was taken from an introductory text on Stochastic Analysis. (Introduction to Stochastic Integration by Hui-Hsiung Kuo)
Assume we have the following stochastic process:
$$X_t=\int_0^t e^{B(s)^2}dB(s)\, ,0\leq t \leq 1$$
where $(B)_{t\geq 0}$ is a Brownian Motion.
I have to show that $X_t$ is not a martingale.
I know that if $t< \frac 1 4$ then $\int_0^t \mathbb E(e^{2B(s)^2})ds < \infty $ and then the process is a martingale, this makes me think that $X_t$ is actually a local martingale, but I don't see how to prove that it's not a proper martingale.
Thanks in advance.
REPLY [7 votes]: Here's an approach that comes from
Li, Xue-Mei, Strict local martingales: examples, Stat. Probab. Lett. 129, 65-68 (2017). ZBL1386.60159, https://arxiv.org/abs/1609.00935. Indeed, she mentions this very example after Corollary 4 (bottom of page 4 in the arXiv version).
Lemma. Suppose $X_t$ is a continuous martingale, and let $\langle X \rangle_t$ be its quadratic variation. Then for every $0 < \alpha < 1$ and every $t$, we have $E[\langle X \rangle_t^{\alpha/2}] < \infty$.
Proof. Let $M_t = \sup_{0 \le s \le t} |X_s|$. By Doob's maximal inequality, for any $x > 0$ we have
$$P(M_t \ge x) \le \frac{1}{x} E|X_t|.$$
So
$$\begin{align*}
E[M_t^\alpha] &= \int_0^\infty P(M_t^\alpha \ge x)\,dx \\&\le 1 + \int_1^\infty P(M_t^\alpha \ge x)\,dx \\& \le 1 + E|X_t| \cdot \int_1^\infty x^{-1/\alpha}\,dx \\&< \infty.\end{align*}$$
Then
by the Burkholder-Davis-Gundy inequality we have$$E[\langle X \rangle_t^{\alpha/2}] \le C_\alpha E[M_t^\alpha] < \infty$$
as desired. $\Box$
Now for the process at hand, we have $\langle X \rangle_t = \int_0^t e^{2 B_s^2}\,ds$. By Hölder's inequality or Jensen's we have $\langle X \rangle_t^\alpha \ge t^{\alpha-1} \int_0^t e^{2 \alpha B_s^2}\,ds$, so by Fubini/Tonelli $E[ \langle X \rangle_t^\alpha] \ge t^{\alpha - 1} \int_0^t E[e^{2 \alpha B_s^2}]\,ds$. The integrand is infinite for all $s > 1/4$, so by the lemma $X_t$ cannot be a martingale for any $t > 1/4$.
I feel like perhaps there should be a more direct way to relate $E[\langle X\rangle_t^{\alpha/2}]$ to $E[X_t]$, perhaps by some clever application of Itô's formula and Hölder's inequality, but I don't quite see how.
REPLY [4 votes]: Let $B_t:=B(t)$. By the Itô formula
$$f(B_1)-f(B_0)=\int_0^1 f'(B_t)\,dB_t+\frac12\,\int_0^1 f''(B_t)\,dt$$
with $f(b):=\int_0^b e^{a^2}da$ (with $\int_0^b:=-\int_b^0$ for $b<0$), we have
\begin{equation*}
X_1=\int_0^1 f'(B_t)\,dB_t
=f(B_1)-\frac12\,\int_0^1 f''(B_t)\,dt
=f(B_1)-\int_0^1 B_t e^{B_t^2}\,dt. \tag{1}
\end{equation*}
From here, it is not hard to see that $EX_1$ does not exist.
Indeed, consider the event
\begin{equation*}
A:=\{B_u\in[b,b+1/b],B_1-B_u<-2/b,M_u>-b\},
\end{equation*}
where $b\to\infty$,
\begin{equation*}
u:=1-1/b^2,
\end{equation*}
\begin{equation*}
M_u:=\min_{t\in[u,1]}(B_t-B_u).
\end{equation*}
Note that on the event $A$ we have $B_u\ge b$, $B_10$ for all $t\in[u,1]$. Therefore and because (by the l'Hospital rule) $f(b)\sim e^{b^2}/(2b)$, it follows from (1) that on $A$
\begin{align*}
X_1&\le\frac{e^{(b-1/b)^2}}{(2+o(1))b}-\int_0^u B_t e^{B_t^2}\,dt \\
&=\frac{e^{b^2-2}}{(2+o(1))b}-\int_0^u B_t e^{B_t^2}\,dt \\
&\le\frac{e^{b^2-2}}{(2+o(1))b}-\int_0^u (\tfrac tu\,b+B_t^u)
\exp\{(\tfrac tu\,b+B_t^u)^2\}\,dt,
\end{align*}
where $B_t^u:=B_t-\frac tu\,B_u$; for the latter, displayed inequality, we recall that $B_u\ge b$ on $A$ and use the fact that $se^{s^2}$ is increasing in real $s$.
The Brownian bridge $(B_t^u)_{t\in[0,u]}$ is a zero-mean (Gaussian) process independent of $(B_u,B_1-B_u,M_u)$; so, $(B_t^u)_{t\in[0,u]}$ is independent of the event $A$.
Introduce also the event
$$C_x:=\{\max_{t\in[0,u]}|B_t^u|\le x\}$$
for real $x>0$, which allows us to use the Fubini theorem to get
\begin{align*}
EX_1\,1_{A\cap C_x}\le\Big(&\frac{e^{b^2-2}}{(2+o(1))b}P(C_x) \\
&-\int_0^u E(\tfrac tu\,b+B_t^u)
\exp\{(\tfrac tu\,b+B_t^u)^2\}1_{C_x}\,dt\Big)\,P(A). \tag{2}
\end{align*}
Because $g_a(b):=(a+b) e^{(a+b)^2}+(a-b)e^{(a-b)^2}$ is convex and even in real $b$ for each real $a\ge0$, we have $g_a(b)\ge g_a(0)=2ae^{a^2}$. Therefore and
because the distribution of the Brownian bridge $(B_t^u)_{t\in[0,u]}$ is symmetric,
\begin{align*}
E(a+B_t^u) \exp\{(a+B_t^u)^2\}1_{C_x} =\tfrac12\,Eg_a(B_t^u)\,1_{C_x} \ge ae^{a^2}\,P(C_x)
\end{align*}
for $a\ge0$. So, by (2),
\begin{align*}
EX_1\,1_{A\cap C_x}&\le\Big(\frac{e^{b^2-2}}{(2+o(1))b}-\int_0^u \tfrac tu\,b
\exp\{(\tfrac tu\,b)^2\}\,dt\Big)\,P(C_x)P(A) \\
&=\Big(\frac{e^{b^2-2}}{(2+o(1))b}-\frac u{2b} \, (e^{b^2}-1)\Big)\,P(C_x)P(A) \\
&=-e^{b^2(1+o(1))}\,P(C_x)P(A).
\end{align*}
On the other hand, letting now $x\to\infty$, we have $P(C_x)\to1$
and, still with $b\to\infty$,
\begin{align*}
P(A)&=P(B_u\in[b,b+1/b])P(B_1-B_u<-2/b,M_u>-b) \\
&\ge P(B_u\in[b,b+1/b])[P(B_1-B_u<-2/b)-P(M_u\le-b)] \\
&=e^{-b^2/(2+o(1))}[P(B_1<-2)-o(1)]=e^{-b^2/(2+o(1))}.
\end{align*}
Thus,
\begin{align*}
EX_1\,1_{A\cap C_x}&\le-e^{b^2(1+o(1))}\,(1-o(1))\,e^{-b^2/(2+o(1))}=-e^{b^2/(2+o(1))}\to-\infty,
\end{align*}
which shows that indeed $EX_1$ does not exist.<|endoftext|>
TITLE: Stability of fractional Sobolev spaces under diffeomorphisms
QUESTION [5 upvotes]: Let $H^s_p(\mathbb{R}^n)$ be the fractional Sobolev space of fractional order $s\in \mathbb{R}$, for $1
TITLE: Classification of $\operatorname{Rep}D(H)$
QUESTION [5 upvotes]: Question
Let $H$ be a finite dimensional complex Hopf algebra and $D(H)$ its quantum double. Can we classify the simple objects in $\operatorname{Rep}D(H)$ if the representations of $H$ are well-understood?
My impression is that this is not yet completely developed. However, the more examples the better! Would you mind sharing papers/resources that deal with the representations of quantum doubles, even those that deal with specific examples?
Example: finite group algebra
For example, if $H$ is the complex group algebra of some finite group $G$, both $\operatorname{Rep}(G)$ and $\operatorname{Rep}D(G)$ are well-enough understood (to my standard). See my previous questions about this
Representations of $D(G)$ as an object in the center of $\operatorname{Rep}(G)$
Classification of $\operatorname{Rep} D(G)$
Interestingly, irreducible representations of $D(G)$ do not restrict to irreducible ones as $G$-reps. Viewing them as objects of $Z(\operatorname{Rep}G)$ reveals hidden structures among irreps of $G$: nontrivial half-braidings arise naturally!
Example: Taft algebra
As another example, the representations of the double of Taft algebras are examined here by Chen.
REPLY [2 votes]: To any skew-pairing $\lambda:U\otimes H\rightarrow k$, one can associate a hopf algebra $D(U, H)$ (built on $U\otimes H$) which is called the generalized quantum double of $U$ and $H$.
(If $H$ is finite dimensional, $U=H^{*cop}$ and $\lambda$ is the usual evaluation map, then, this corresponds to the usual quantum double $D(H)$ introduced by Drinfeld).
In the article On the irreducible representations of generalized quantum doubles, the authors describe the irreducible representations of generalized quantum double hopf algebras $D_{\lambda_f}(U,H)$, where $\lambda_f:U\otimes H\rightarrow k$ is a skew-pairing induced by a surjective hopf algebra map $f:U\rightarrow H^{*cop}$.
One of the main results of the paper (as i understand it) is theorem 1.1 (p.2), which describes the simple objects of $\operatorname{Rep}\big(D_{\lambda_f}(U,H)\big)$, in a way which generalizes the corresponding description of the simple objects of $\operatorname{Rep}D(G)$ (discussed in the questions linked to the OP). Then the paper proceeds in a classification of the generalized quantum double, simple modules, for the case where both $U,H$ are semisimple hopf algebras and there is a surjective map $U\rightarrow H^{*cop}$.
Furthermore:
In The Representations of Quantum Double of Dihedral Groups, the finite dimensional, indecomposable, left $D(kD_n)$-modules are classified, where $D_n$ is the dihedral group of order $2n$ and $k$ is an algebraically closed field, of odd characteristic $p\ |\ 2n$.<|endoftext|>
TITLE: fundamental groups of complements to countable subsets of the plane
QUESTION [14 upvotes]: This question is a follow-up of this MSE post and a comment by Henno Brandsma:
Question 1. Let $S$ be the set of isomorphism classes of fundamental groups $\pi_1(E^2 - C)$, where $C$ ranges over all countably infinite subsets of the Euclidean plane $E^2$. What is the cardinality of $S$?
All what I can say is that $S$ contains at least two non-isomorphic groups:
One is the free group $F_\omega$ of countably infinite rank, the fundamental group of the complement to a closed discrete countably infinite subset of $E^2$, does not matter which one, say, $C={\mathbb Z}\subset {\mathbb R}\subset {\mathbb R}^2$.
The other is $G_{{\mathbb Q}^2}=\pi_1(E^2-C)$, where $C$ is a dense countable subset of $E^2$, again, does not matter which one, for instance, $C={\mathbb Q}^2$. This group is not free since it contains, for instance, the fundamental group of the Hawaiian Earrings. (Actually, it contains $\pi_1$ of every nowhere dense planar Peano continuum.)
The natural expectation is that $S$ is uncountable (more precisely, has the cardinality of continuum: It is clear that the cardinality of $S$ cannot be higher than that).
Edit. Following Yves' suggestion:
Question 2. Let ${\mathbb H}$ denote the Hawaiian Earrings. Is the fundamental group $\pi_1({\mathbb H})$ essentially freely indecomposable? (Here a group $G$ is essentially freely indecomposable if in every free product decomposition $G\cong G_1\star G_2$, one of the free factors $G_1, G_2$ is free of finite rank.) One can also ask for the weaker property of $G=\pi_1({\mathbb H})$, namely, that $G$ does not admit free product decompositions $G\cong G_1\star G_2$ with two uncountable factors.
The only relevant result I could find in the literature is the theorem (due to Higman) which (according to "The combinatorial structure of the Hawaiian earring group" by Cannon and Conner) implies that that every freely indecomposable free factor of $G=\pi_1({\mathbb H})$ is either trivial or infinite cyclic. Maybe Higman's methods prove more, but his paper ("Unrestricted Free Products, and Varieties of Topological Groups", Journal of LMS, 1952) is behind the paywall.
If Q2 (even in the weaker form) has positive answer, then in Q1 at least one can say that $S$ is infinite.
REPLY [4 votes]: Thanks to the comments, my original cardinality bound $\aleph_1\leq |S|\leq \mathfrak{c}$ has been refined to the equality $|S|=\mathfrak{c}$ that I originally suspected.
For Question 1: $S$ has the cardinality of the continuum. It's clear that $|S|\leq \mathfrak{c}$. Below, I'll argue that $|S|$ is at least the cardinality of the set of homeomorphism types of closed nowhere dense subsets of $[0,1]$. Since this set has cardinality $\mathfrak{c}$ (using Pierre PC's comment below), we have $|S|\geq \mathfrak{c}$. For the lower bound, I'll use a construction from this paper.
Consider any infinite closed nowhere dense subset $A\subseteq[0,1]$ containing $\{0,1\}$. Let $\mathcal{I}(A)$ denote the ordered set of components of $[0,1]\backslash A$. For each $I=(a,b)\in\mathcal{I}(A)$, let $$C_I=\left\{(x,y)\in\mathbb{R}^2\mid y\geq 0,\left(x-\frac{a+b}{2}\right)^2+y^2=\left(\frac{b-a}{2}\right)^2\right\}$$ be the semicircle whose boundary is $\{(a,0),(b,0)\}$. Let $$\mathbb{W}_{A}=([0,1]\times\{0\})\cup \bigcup_{I\in\mathcal{I}(A)}C_I$$ with basepoint $(0,0)$. Here is an example where $\mathcal{I}(A)$ has the order type of the linear order sum $\omega^{\ast}+\omega+1+\omega^{\ast}$ where $\ast$ denotes reverse order.
First, notice that $\mathbb{W}_A$ is a one-dimensional Peano continuum (connected, locally path-connected, compact metric space). By picking a single point in the interior of each simple closed curve $C_I\cup (\overline{I}\times \{0\})$, we can see that $\mathbb{W}_A$ is homotopy equivalent to $E^2\backslash C$ for some countably infinite set $C$. The fundamental groups $\pi_1(\mathbb{W}_A)$ ranging over all such $A$ realize continuum-many non-isomorphic groups. Here are the heavy hitting theorems that get the job done.
Eda's homotopy classification of 1-dimensional Peano continuua: two one-dimensional Peano continua are homotopy equivalent if and only if they have isomorphic fundamental groups. I would just like to pause and emphasize how incredible and powerful this result is. When you first hear about it and realize the kind of groups and spaces it applies to, you might get the impression that you are being scammed.
Let $\mathbf{w}(X)$ denote the subspace of $X$ consisting of the points at which $X$ is not semilocally simply connected, i.e. the 1-wild set of $X$. For general spaces, the homotopy type of $\mathbf{w}(X)$ is a homotopy invariant of $X$, but monodromy actions in one-dimensional spaces have discrete graphs (see 9.13 of this paper of mine with H. Fischer). Among spaces whose monodromy actions have discrete graphs, the homeomorphism type of $\mathbf{w}(X)$ becomes a homotopy invariant of $X$ (see 9.15 of the same paper). Thus, among 1-dimensional spaces, $\mathbf{w}(X)$ is a homotopy invariant. This result is kind-of embedded in Eda's work leading up to 1. but the core idea behind what is really going on is fleshed out in Section 9 of the linked paper.
By combining 1. and 2. we have:
Corollary: If one-dimensional Peano continua $X$ and $Y$ have isomorphic fundamental groups, then $\mathbf{w}(X)\cong \mathbf{w}(Y)$. A direct consequence is that the Hawaiian earring group and the free product of the Hawaiian earring group with itself are not isomorphic because two copies of $\mathbb{H}$ adjoined by an arc has two 1-wild points.
Returning back to the spaces $\mathbb{W}_A$, notice that $\mathbf{w}(\mathbb{W}_A)$ is homeomorphic to the Cantor Bendixsion derivative of $A$, i.e. the subspace of non-isolated points of $A$. Every closed nowhere dense set $B\subseteq [0,1]$ is the Cantor Bendixsion derivative of some other $A$. Hence, the 1-wild sets of the $\mathbb{W}_A$ realize all closed nowhere dense subsets of $B\subseteq [0,1]$. By the corollary, each homeomorphism class of a nowhere dense closed subset of $[0,1]$, gives a unique isomorphism class of fundamental group $\pi_1(\mathbb{W}_A)$ and hence a unique isomorphism class of a fundamental group $\pi_1(E^2\backslash C)$ for some countably infinite set $C$. In the comments below, Pierre PC gives a construction of $\mathfrak{c}$-many non-homeomorphic closed nowhere dense subsets of $[0,1]$ (one can confirm this by analyzing the Cantor Bendixson derivatives of the neighborhoods of the described "super limit points"). Hence, $|S|=\mathfrak{c}$.<|endoftext|>
TITLE: Nontrivially fillable gaps in published proofs of major theorems
QUESTION [80 upvotes]: Prelude: In 1998, Robert Solovay wrote an email to John Nash to communicate an error that he detected in the proof of the Nash embedding theorem, as presented in Nash's well-known paper "The Imbedding Problem for Riemannian Manifolds" (Annals of Math, 1956), and to offer a nontrivial fix for the problem, as detailed in this erratum note prepared by John Nash. This topic is also discussed in this MO question.
Of course, any mathematician who has been around long enough knows of many published proofs with significant gaps, some provably irreparable, and some perhaps authored by himself or herself. What makes the above situation striking--and discomforting to many of us--is the combination of the following three factors:
(1) The theorem whose proof is found faulty is a major result that was published in 1950 or after, in a readily accessible source to experts in the field . (I chose the 1950 lower bound as a way of focusing on the somewhat recent past).
(2) The gap detected is filled with a nontrivial fix that is publicly available and consented to by experts in the field (so we are not talking about gaps easily filled, or about gaps alleged by pseudomathematicians, or about false publicly accepted theorems, as discussed in this MO question).
(3) There is an interlude of 30 years or more between the publication of the proof and the detection of the gap (I chose 30 years since it is approximately the age difference between consecutive generations, even though the interlude is 42 years in the case of the Nash embedding theorem).
Question to fellow mathematicians: what is the most dramatic instance you know of where all of the three above factors are present?
REPLY [3 votes]: Not too long to wait to find the mistake, but the following might be interesting:
In 1905, Henri Lebesgue claimed to have proved that if B is a subset of $\mathbb{R}^2$ with the Borel property, then its projection onto a line is a Borel subset of the line.
This is false. In 1912 Souslin and Luzin defined an analytic set as the projection of a Borel set. All Borel sets are analytic, but, contrary to Lebesgue's claim, the converse is false.<|endoftext|>
TITLE: Compact complex surface that admits a Kodaira fibration is Kahler
QUESTION [5 upvotes]: A Kodaira fibration is a compact complex surface X endowed with a holomorphic submersion onto a Riemann surface $\pi: X\to\Sigma$ which has connected fibers and is not isotrivial.
Is there an easy way to see why a compact complex surface that admits a Kodaira fibration is Kahler? I know for a complex compact surface is Kahler if and only if its first Betti number is even. I wonder it's possible to deduce that a compact complex surface that admits a Kodaira fibration has even Betti number?
REPLY [2 votes]: Let $f \colon S \longrightarrow B$ be a Kodaira fibration, and let $F$ be a general fibre. Then by [Kas68, Thm. 1.1] we have $g(B) \geq 2$ and $g(F) \geq 3$.
In particular, $S$ contains no rational or elliptic curves: in fact, such curves cannot neither dominate the base (because $g(B) \geq 2$) nor be contained in fibres (because the fibration is by assumption smooth).
So every Kodaira fibred surface $S$ is minimal and, by the superadditivity of the Kodaira dimension, it is of general type.
In particular, it is not only Kähler but actually algebraic (i.e., projective).
References.
[Kas68] A. Kas: On deformations of a certain type of irregular algebraic surface, American J. Math. 90, 789-804 (1968). ZBL0202.51702.<|endoftext|>
TITLE: Consequences of existence of a certain function from $\omega_1$ to $\omega_1$
QUESTION [14 upvotes]: In his book [1], Paul Larson remarks (Remark 1.1.22) that in L there is a function $h:\omega_1\rightarrow\omega_1$ such that for any countable elementary submodel $X$ of $V_\gamma$ (where $\gamma$ is the first strong limit cardinal), we have the order-type of $X\cap Ord$ is strictly less than $h(X\cap\omega_1)$.
What is known about the relationship between the existence of such a function and large cardinal phenomena?
Larson remarks that the existence of such a function is consistent with many large cardinals, and later in the book sketches a result of Velickovic showing that no such function exists in the presence of a precipitous ideal on $\omega_1$. What more is known? Happy to also learn results concerning other related functions, or be pointed to associated references.
[1] Larson, Paul B., The stationary tower. Notes on a course by W. Hugh Woodin, University Lecture Series 32. Providence, RI: American Mathematical Society (AMS) (ISBN 0-8218-3604-8/pbk). x, 132 p. (2004). ZBL1072.03031.
REPLY [6 votes]: Not sure if this is what you're interested in, but let me take up the question of what large cardinal axioms are known to be consistent with the existence of $h$. The answer is all large cardinal axioms known to have a canonical inner model. Beyond that, the consistency question remains open: indeed, it remains open whether large cardinal axioms beyond the reach of inner model theory could outright imply the existence of a precipitous ideal on $\omega_1$, and thereby refute the existence of $h$.
It seems plausible, however, that $h$ in fact exists in the canonical inner models yet to be discovered. This is because one can construct a function that is almost exactly like $h$ (see the fourth-to-last paragraph below) using a general condensation principle due to Woodin called Strong Condensation, which likely holds in all canonical inner models. If $\kappa$ is a cardinal, Strong Condensation at $\kappa$ states that there is a surjective function $f : \kappa\to H(\kappa)$ of such that for all $M\prec (H(\kappa),f)$, letting $(H_M,f_M)$ be the transitive collapse of $(M,f)$, $f_M = {f}\restriction H_M$.
All the known canonical inner models satisfy Strong Condensation at their least strong limit cardinal. (Strong condensation cannot hold at any cardinals past the first Ramsey cardinal.) Moreover by a theorem of Woodin, under $\text{AD}^+ + V = L(P(\mathbb R))$, for a Turing cone of reals $x$, $\text{HOD}_x$ satisfies Strong Condensation at its least strong limit cardinal. This heuristically argues that Strong Condensation should hold at the least strong limit cardinal in canonical inner models satisfying arbitrarily strong large cardinal axioms, assuming that such models exist. The reason is that the pattern observed in inner model theory to date suggests that these models should locally resemble the $\text{HOD}$s of determinacy models.
Here is the approximation to the existence of $h$ one gets assuming Strong Condensation at the least strong limit cardinal $\gamma$: There is a set $a\subseteq \omega_1$ and a function $g:\omega_1\to\omega_1$ such that for any $N\prec V_\gamma$ with $a\in N$, $g(N\cap \omega_1) > \text{ot}(N\cap \gamma)$. The rest of this answer consists of a proof of this fact.
Fix $f : \gamma\to H(\gamma)$ witnessing Strong Condensation at $\gamma$. The first step of the proof is cosmetic. One uses a theorem of Woodin which states that $f$ is definable over $H(\gamma)$ from the parameter $f \restriction \omega_1$. Let $a\subseteq \omega_1$ code $f\restriction \omega_1$. (It is an easy exercise to show that $f\restriction \omega_1\in H(\omega_2)$.) Then every $N\prec V_\gamma$ with $a\in N$ has the property that $N\cap H(\gamma)\prec (H(\gamma),f)$.
For $\alpha < \gamma$, let $P_\alpha = f[\alpha]$. Note that the $P_\alpha$ are increasing with union $H(\gamma)$, and if $M\prec (H(\gamma),f)$ and $\text{ot}(M\cap \gamma) = \alpha$, then the transitive collapse of $M$ is equal to $P_\alpha$. The structures $P_\alpha$ will play the role of the $L_\alpha$ hierarchy in Larson's proof.
For every $\xi < \omega_1$, let $g(\xi)$ be the least ordinal $\alpha$ such that there is a surjection from $\omega$ to $\xi$ in $P_\alpha$. Suppose $N\prec V_\gamma$ and $a\in N$. We will show that $g(N\cap \omega_1) > \text{ot}(N\cap \gamma)$. Let $M = N\cap H(\gamma)$, so $M\prec (H(\gamma),<)$. Clearly it suffices to show that $g(M\cap \omega_1) > \text{ot}(M\cap \gamma)$. Let $H_M$ be the transitive collapse of $M$. Then $M\cap \omega_1 = \omega_1^{H_M}$ and letting $\beta = \text{ot}(M\cap \gamma)$, $H_M = P_\beta$. Assume towards a contradiction that $\beta \geq g(\omega_1^{H_M})$. By the definition of $g$, there is a surjection from $\omega$ to $\omega_1^{H_M}$ in $P_\beta$. But since $P_\beta = H_M$, this contradicts that $\omega_1^{H_M}$ is uncountable in $H_M$.<|endoftext|>
TITLE: Cobordism and Kirby calculus
QUESTION [12 upvotes]: It may be a simple question but I wonder to ask:
Is it possible to draw a homology cobordism between $3$-manifolds by using the techniques of Kirby calculus?
At least, for instance, Brieskorn spheres?
REPLY [13 votes]: As Golla pointed out that since every smooth $4$-manifold has a handle decomposition, you can draw a Kirby diagram. See the following pretty nice picture from Akbulut's lecture notes (now it is a published book titled $4$-manifolds).
Golla listed that lots of Brieskorn spheres which are known to be bound integral or rational homology balls, i.e., they are all integral or rational homology cobordant to $S^3$.
Following the technique of Akbulut and Larson, I also recently found new Brieskorn spheres bounding rational homology balls: $\Sigma (2,4n+3,12n+7)$ and $\Sigma(3,3n+2,12n+7)$, see the preprint.
It is interesting to note that the following Brieskorn spheres bound rational homology balls but not integral homology balls (these $4$-manifolds must contain $3$-handle(s)):
$\Sigma(2,3,7)$, $\Sigma(2,3,19)$,
$\Sigma(2,4n+1,12n+5)$ and $\Sigma(3,3n+1,12n+5)$ for odd $n$,
$\Sigma(2,4n+3,12n+7)$ and $\Sigma(3,3n+2,12n+7)$ for even $n$.
On the other hand, we know that every closed oriented $3$-manifold is cobordant to $S^3$ due to the celebrated theorem of Lickorish and Wallace.
In the following picture, you can see the explicit cobordism from $\Sigma(2,3,13)$ to $\Sigma(2,3,7)$ which is constituted by adding the red $(-1)$-framed $2$-handle. (The knot pictures are from KnotInfo). Here, blow down the red one to get the right-hand side. (Of course, they are not integral homology cobordant.)
Note that the Brieskorn spheres $\Sigma(2,3,6n+1)$ are obtained by $(+1)$-surgery on the twist knots $(2n+2)_1$, see for example Saveliev's book pg. 49-50. Here, $6_1$ is the stevedore knot (the left knot in the figure) and $4_1$ is the figure-eight knot (the right one).
REPLY [8 votes]: There are many examples of the sort, in effect. As far as I know, Akbulut and Kirby (Mazur manifolds, Michigan Math. J. 26 (1979)) proved that $\Sigma(2,5,7)$, $\Sigma(3,4,5)$, and $\Sigma(2,3,13)$ bound contractible 4-manifolds; their work was then extended by Casson and Harer (Some homology lens spaces which bound rational homology balls, Pacific Math. J. 96 (1981)). Stern, Fintushel-Stern, and Fickle have more examples.
I'm sure that the Akbulut-Kirby (and some of the Casson-Harer) examples were done by Kirby calculus.
Additionally, there are also examples of Brieskorn spheres bounding rational homology 4-balls (but not integral ones, because they have non-zero Rokhlin invariant). The first example was $\Sigma(2,3,7)$, and the rational ball was produced by Fintushel and Stern (A $\mu$-invariant one homology 3-sphere that bounds an orientable rational ball, in Four-manifold theory (Durham, NH, 1982) (1984)) by explicit handle moves. This has been extended further; the latest news I have are from a paper of Akbulut and Larson (Brieskorn spheres bounding rational balls, Proc. Amer. Math. Soc. 146 (2018)), where they provide two infinite families of examples: $\Sigma(2,4n+1,12n+5)$ and $\Sigma(3,3n+1,12n+5)$, as well as $\Sigma(2,3,19)$.
Akbulut's book 4-manifolds (Oxford University Press) contains a wealth of examples along these lines (not many more with Brieskorn spheres, I should think).
Finally, I am not aware of (but would be interested in seeing) explicit, non-trivial examples of (rational or integral) homology cobordisms between Brieskorn spheres.<|endoftext|>
TITLE: Homomorphism induced by the second exterior power of a linear map
QUESTION [9 upvotes]: Consider the map from $M(n, \mathbb Z) \rightarrow M(\binom{n}{2}, \mathbb Z)$ taking a matrix A to its second compound, i.e, $\bigwedge^2 A$.
Restricting this map to the invertible matrices we get a homomorphism of groups from $\mathrm{GL}(n, \mathbb Z)$ to $\mathrm{GL}(\binom{n}{2}, \mathbb Z)$.
How can we determine if a given matrix $B \in \mathrm{GL}(\binom{n}{2}, \mathbb Z)$ is contained in the image of this map?
REPLY [11 votes]: First, let us discuss the same question over an algebraically closed field (e.g., over $\overline{\mathbb{Q}}$). Let $V$ be a vector space of dimension $n$. The question is to understand the image of the homomorphism
$$
\lambda \colon \mathrm{GL}(V) \to \mathrm{GL}(\wedge^2V).
$$
Note that $\mathrm{GL}(\wedge^2V)$ acts naturally on the projective space $\mathbb{P}(\wedge^2V)$, which contains as a subvariety the Grassmannian
$$
\mathrm{Gr}(2,V) \subset \mathbb{P}(\wedge^2V).
$$
Clearly, it is preserved by the action of $\mathrm{GL}(V)$. The converse is also true for $n > 4$, i.e., if $g \in \mathrm{GL}(\wedge^2V)$ is such that
$$
g(\mathrm{Gr}(2,V)) \subset \mathrm{Gr}(2,V),
$$
then $g \in \mathrm{Im}(\lambda)$. This follows immediately from the isomorphism
$$
\mathrm{Aut}(\mathrm{Gr}(2,V)) \cong \mathrm{PGL}(V).
$$
Over $\mathbb{Z}$, I guess, the equality
$$
\det(\wedge^2g) = \det(g)^{n-1}
$$
gives an extra constraint; so besides preserving the Grassmannian one should impose the condition that the determinant is $(n-1)$-st power.<|endoftext|>
TITLE: Hard: One more generator needed for a Z/6 elliptic curve
QUESTION [8 upvotes]: We are searching for rank 8 elliptic curves with the torsion subgroup $\mathbb{Z}/6$ using newly discovered families similar to Kihara's as described in
A. Dujella, J.C. Peral, P. Tadić, Elliptic curves with torsion group $\mathbb{Z}/6\mathbb{Z}$, Glas. Mat. Ser. III 51 (2016), 321-333 doi:10.3336/gm.51.2.03, 1503.03667
and came across a curve
[1,0,1,-728177856117250596635013323700992100749546784263413,7725511368374502384905062271934799362136250437256099440874528514783779254988]
Both Magma Calculator and mwrank return $7$ generators for this curve:
SetClassGroupBounds("GRH");
E:=EllipticCurve([1,0,1,-728177856117250596635013323700992100749546784263413,7725511368374502384905062271934799362136250437256099440874528514783779254988]);
MordellWeilShaInformation(E);
Both Magma and mwrank return $8$ for the upper bound on rank:
E:=EllipticCurve([1,0,1,-728177856117250596635013323700992100749546784263413,7725511368374502384905062271934799362136250437256099440874528514783779254988]);
TwoPowerIsogenyDescentRankBound(E);
8 [ 4, 4, 4, 4, 4 ]
[ 6, 6, 6, 6, 6 ]
mwrank -v0 -p200 -s
[1,0,1,-728177856117250596635013323700992100749546784263413,7725511368374502384905062271934799362136250437256099440874528514783779254988]
Version compiled on Oct 29 2018 at 22:35:09 by GCC 7.3.0
using NTL bigints and NTL real and complex multiprecision floating point
Enter curve: [1,0,1,-728177856117250596635013323700992100749546784263413,7725511368374502384905062271934799362136250437256099440874528514783779254988]
Curve [1,0,1,-728177856117250596635013323700992100749546784263413,7725511368374502384905062271934799362136250437256099440874528514783779254988] : selmer-rank = 9
upper bound on rank = 8
Considering parity, there should be one more generator on the curve.
Is there a way to find it?
We would greatly appreciate any hint leading to the discovery of the extra generator.
A bounty of $100$ has been offered for obtaining it.
Also, if you can compute an extra generator, your name will be published at the bottom of the page here: https://web.math.pmf.unizg.hr/~duje/tors/z6.html
REPLY [17 votes]: A set of points that generate $E(\mathbb{Q})$ modulo torsion is given by
(1955516573881233507049678279 : -86467145649172260650105545143411861089140 : 1),
(49225691888888099223656060329/10201 : 67749663895993353685065159554645568700902610/1030301 : 1),
(61339810590192565389735634 : -440289331793622522908840423931186017125 : 1),
(301884243790342804873202050999/1681 : 164095919303197903219089875947912899634054060/68921 : 1),
(12495717670305680867142229 : -24031745881863415519418908823242701040 : 1),
(48812081421189741670987918753619270029/14228919471376 : -3895612939954697213016286372117889003488190324193605593985/53673248632044722624 : 1),
(5561842419887590167868100830494509281/162696869449 : 9905381606012663087305509196041719017978015930195439090/65624921170340293 : 1),
(-24644413733187137559835573003063695698428162289232517969749039/810893447144357785058346728220801409 : 30847724470076383865716266151756242512110696731502256770076024073253839003102120576612459770/730206486187013450403786627354716551758061149557632577 : 1)
My guess is that you were missing the final one.
We can find the final point by applying $4$-descent to the $2$-covering $C_2$ given by
$$y^2 = 3600489235862039958255625x^4 - 26108156374576368607091450x^3 + 135553629286468859778411799x^2 + 184563701310722380421312754x +
49111306298020667812024521$$
of $E$, which is one of the 511 $2$-coverings returned by running the command TwoDescent(E) magma. Running the command FourDescent(C2) returns 256 $4$-coverings of $E$. One of them is the curve $C_4$ defined by the intersection of the quadrics
$$46500x^2 + 74693xy + 54170xz + 647076xw - 121026y^2 - 196538yz + 862965yw - 212375z^2 - 238791zw + 333744w^2$$
and
$$722768x^2 - 2936122xy + 3336517xz - 2782182xw - 2731148y^2 - 13360024yz - 950117yw - 4385375z^2 + 2688700zw - 199207w^2$$
in $\mathbb{P}^3(\mathbb{Q})$. Running PointsQI(C4,2^11) returns a single point $Q = (2834:53:2444:376)$. We can map $Q$ to a rational point on $E$ using the map given as the second return value of the command AssociatedEllipticCurve(C4:E:=E), and the point we get is the final point above.<|endoftext|>
TITLE: Could groups be used instead of sets as a foundation of mathematics?
QUESTION [28 upvotes]: Sets are the only fundamental objects in the theory $\sf ZFC$. But we can use $\sf ZFC$ as a foundation for all of mathematics by encoding the various other objects we care about in terms of sets. The idea is that every statement that mathematicians care about is equivalent to some question about sets. An example of such an encoding is Kuratowski's definition of ordered pair, $(a,b) = \{\{a\},\{a,b\}\}$, which can then be used to define the cartesian product, functions, and so on.
I'm wondering how arbitrary the choice was to use sets as a foundation. Of course there are alternative foundations that don't use sets, but as far as I know all these foundations are still based on things that are quite similar to sets (for example $\sf HoTT$ uses $\infty$-groupoids, but still contains sets as a special case of these).
My suspicion is that we could instead pick almost any kind of mathematical structure to use as a foundation instead of sets and that no matter what we chose it would be possible to encode all of mathematics in terms of statements about those structures. (Of course I will add the caveat that there has to be a proper class of whichever structure we choose, up to isomorphism. I'm thinking of things like groups, topological spaces, Lie algebras, etc. Any theory about a mere set of structures will be proved consistent by $\sf ZFC$ and hence be weaker than it.)
For concreteness I'll take groups as an example of a structure very different from sets. Can every mathematical statement be encoded as a statement about groups?
Since we accept that it is possible to encode every mathematical statement as a statement about sets, it would suffice to show that set theory can be encoded in terms of groups. I've attempted a formalization of this below, but I would also be interested in any other approaches to the question.
We'll define a theory of groups, and then ask if the theory of sets (and hence everything else) can be interpreted in it. Since groups have no obvious equivalent of $\sf ZFC$'s membership relation we'll instead work in terms of groups and their homomorphisms, defining a theory of the category of groups analogous to $\sf{ETCS+R}$ for sets. The Elementary Theory of the Category of Sets, with Replacement is a theory of sets and functions which is itself biinterpretable with $\sf ZFC$.
We'll define our theory of groups by means of an interpretation in $\sf{ETCS+R}$. It will use the same language as $\sf{ETCS+R}$, but we'll interpret the objects to be groups and the morphisms to be group homomorphisms. Say the theorems of our theory are precisely the statements in this language whose translations under this interpretation are provable in $\sf{ETCS+R}$. This theory is then recursively axiomatizable by Craig's Theorem. Naturally we'll call this new theory '$\sf{ETCG+R}$'.
The theory $\sf{ETCS+R}$ is biinterpretable with $\sf ZFC$, showing that any mathematics encodable in one is encodable in the other.
Question: Is $\sf{ETCG+R}$ biinterpretable with $\sf ZFC$? If not, is $\sf ZFC$ at least interpretable in $\sf{ETCG+R}$? If not, are they at least equiconsistent?
REPLY [31 votes]: The answer is yes, in fact one has a lot better than bi-interpretability, as shown by the corollary at the end. It follows by mixing the comments by Martin Brandenburg and mine (and a few additional details I found on MO). The key observation is the following:
Theorem: The category of co-group objects in the category of groups is equivalent to the category of sets.
(According to the nLab, this is due to Kan, from the paper "On monoids and their dual" Bol. Soc. Mat. Mexicana (2) 3 (1958), pp. 52-61, MR0111035)
Co-groups are easily defined in purely categorical terms (see Edit 2 below).
The equivalence of the theorem is given by free groups as follows: if $X$ is a set and $F_X$ is the free group on X then Hom$(F_X,H)=H^X$ is a group, functorially in H, hence $F_X$ has a cogroup object structure. As functions between sets induce re-indexing functions: $H^X \rightarrow H^Y$ that are indeed group morphisms, morphisms between sets indeed are cogroup morphisms.
Explicitly, $\mu:F_X \rightarrow F_X * F_X$ is the map that sends each generator $e_x$ to $e_x^L * e_x^R$, and $i$ is the map that sends each generators to its inverse.
An easy calculation shows that the generators are the only elements such that $\mu(y)=y^L*y^R$ and hence that any cogroup morphism comes from a function between sets. So the only co-group morphisms are the ones sending generators to generators.
And with a bit more work, as nicely explained on this other MO answer, one can check that any cogroup object is of this form.
Now, as all this is a theorem of $\sf{ETCS}$, it is a theorem of $\sf{ETCG}$ that all the axioms (and theorems) of $\sf{ETCS}$ are satisfied by the category of cogroup objects in any model of $\sf{ETCG}$, which gives you the desired bi-interpretability between $\sf{ETCS}$ and $\sf{ETCG}$. Adding supplementary axioms to $\sf{ETCS}$ (like R) does not change anything.
In fact, one has more than bi-interpretability: the two theories are equivalent in the sense that there is an equivalence between their models. But one has a lot better:
Corollary: Given $T$ a model of $\sf{ETCS}$, then $Grp(T)$ is a model of $\sf{ETCG}$. Given $A$ a model of $\sf{ETCG}$, then $CoGrp(A)$ is a model of $\sf{ETCS}$. Moreover these two constructions are inverse to each other up to equivalence of categories.
Edit: this an answer to a question of Matt F. in the comment to give explicit example of how axioms and theorems of $\sf{ECTS}$ translate into $\sf{ECTG}$.
So in $\sf{ECTS}$ there is a theorem (maybe an axioms) that given a monomorphism $S \rightarrow T$ there exists an object $R$ such that $T \simeq S \coprod R$.
In $\sf{ECTG}$ this can be translated as: given $T$ a cogroup object and $S \rightarrow T$ a cogroup monomorphism* then there exists a co-group $R$ such that $T \simeq S * R$ as co-groups**.
*: It is also a theorem of $\sf{ECTG}$ that a map between cogroup is a monomorphism of cogroup if and only if the underlying map of objects is a monomorphisms. Indeed that is something you can prove for the category of groups in $\sf{ECTS}$ so it holds in $\sf{ECTG}$ by definition.
** : We can prove in $\sf{ECTG}$ (either directly because this actually holds in any category, or proving it for group in $\sf{ECTS}$) that the coproduct of two co-group objects has a canonical co-group structure which makes it the coproduct in the category of co-groups.
Edit 2: To clarify that the category of cogroup is defined purely in the categorical language:
The coproduct in group is the free product $G * G$ and is definable by its usual universal property.
A cogroup is then an object (here a group) equipped with a map $\mu: G \rightarrow G * G$ which is co-associative, that is $\mu \circ (\mu * Id_G) = \mu \circ (Id_G * \mu)$, and counital (the co-unit has to be the unique map $G \rightarrow 1$), that is $(Id_G,0) \circ \mu = Id_G$ and $(0,Id_G) \circ \mu = Id_G$, where $(f,g)$ denotes the map $G * G \rightarrow G$ which is $f$ on the first component and $g$ on the other component, as well as an inverse map $i:G \rightarrow G$ such that $(Id_G ,i ) \circ \mu = 0 $.
Morphisms of co-groups are the map $f:G \rightarrow H$ that are compatible with all these structures, so mostly such that $ (f * f) \circ \mu_H = \mu_G \circ f $.
If you have doubt related to the "choice" of the object $G * G$ (which is only defined up to unique isomorphisms) a way to lift them is to define "a co-group object" as a triple of object $G,G *G,G * G *G$ with appropriate map between them satisfying a bunch of confition (includings the universal property) and morphisms of co-group as triple of maps satisfying all the expected conditions. This gives an equivalent category.<|endoftext|>
TITLE: Can filtered colimits be computed in the homotopy category?
QUESTION [10 upvotes]: For $\mathcal{S}$ the $(\infty,1)$-category of spaces its homotopy category $h\mathcal{S}$ does not have pushouts or pullbacks. Even if it does, they won't always agree with the (homotopy) pushouts or pullbacks in $\mathcal{S}$.
Generally, filtered colimits are much better behaved than general ones. For example filtered colimits in the (Quillen) model category $\mathit{sSet}$ compute homotopy colimits in $\mathcal{S}$.
In light of this I was wondering the following:
Does $h\mathcal{S}$ have filtered colimits and, if so, does the functor $\mathcal{S} \to h\mathcal{S}$ preserve them?
More generally one could ask the same for any presentable $(\infty,1)$-category $\mathcal{C}$; I particularly care about the case of the derived category $\mathcal{D}(R)$ of a ring $R$.
REPLY [5 votes]: As has already been said, the homotopy category does not admit filtered colimits in general, but it’s much worse than that. Even colimits in an $\infty$-category which don’t give rise to colimits in the homotopy category sometimes do give rise to weak colimits. (A weak colimit cocone gives the existence, but not the uniqueness, of the factorizations a colimit cocone gives.) This is the case, for instance, with sequential colimits, as well as those along any free category, at least in spaces.
So one might ask whether at least every filtered colimit in $\mathcal S$ gives rise to a weak filtered colimit in $h\mathcal S$. Alas, this is still not true. In our paper kindly referenced by Tim, Christensen and I give an $\aleph_1$-indexed sequence of spaces whose homotopy colimit is not a weak colimit in the homotopy category, namely the sequence mapping a countable ordinal $\alpha$ to the wedge of $\alpha$ circles. The homotopy colimit is a wedge of $\aleph_1$ circles, and the problem is that a map out of that just requires too much coherence to be constructed out of a cocone over countable wedges in $h\mathcal S$. So there is not much hope for filtered colimits in $h\mathcal S$. I expect the same counterexample would work, though I have no idea how to make the argument, in higher homotopy categories $h_n\mathcal S$.
Regarding “minimal” or “distinguished” weak colimits, the general idea is that you want some weak colimits which are distinguished up to at least non-unique isomorphism, as occurs for cones in triangulated categories. Since “homotopy colimits” of sequences in triangulated categories with countable coproduct a are constructed out of those coproduct together with cones, they are also distinguished in this sense.
It is possible to get at the idea of minimal weak colimit of at least a filtered diagram in a category which may not be triangulated, but which has some set of objects detecting isomorphisms, by asking that $Hom(S,\mathrm{wcolim} D_i)\cong \mathrm{colim} Hom(S, D_i)$ for every $S$ in your isomorphism-detecting set. Such weak colimits are then indeed determined up to isomorphism, and they’re also nice because they see the objects $S$ as compact.
However, this is not to say that such distinguished weak colimits are common! Our diagram from above actually admits no weak colimit which views even $S^1$ as compact in this way. (Though note that some weak colimit always exists-homotopy pushouts give weak pushouts, coproducts exist, and then the usual construction applies.)
If your category actually has a set of compact generators in a model, as for $D(R)$, then a distinguished weak colimit must come from a homotopy colimit. Franke gives an argument, cited in our paper, that on these grounds distinguished weak colimits of uncountable chains should essentially never exist in $D(R)$. The problem is that there’s a spectral sequence converging to homs out of a homotopy colimit indexed by $J$ whose $E_2$ page involves the derived functors $R^n\mathrm{lim}^J$ for all $n$. These derived functors were shown by Osofsky to be non vanishing up through $n$ when $J=\aleph_n$, and the homotopy colimit is a weak colimit only if the spectral sequence collapses, so this probably shouldn’t happen. However Franke doesn’t give an argument that there couldn’t in principle be enough unlikely differentials to produce the collapse.
Christensen and I tried for a while to work with the analogous spectral sequence for spaces, but it seemed to require a proficiency with calculating higher derived limits unsupported by the literature-Osofsky gives a special example, and for all I can tell nobody else has ever calculated a derived limit over $\aleph_n$. So our approach turns out to be entirely different and doesn’t immediately apply to the stable case. Thus I think it’s unknown, though highly doubtful, whether $D(R)$ admits minimal filtered colimits in general.<|endoftext|>
TITLE: When homology isomorphism implies homotopy isomorphism
QUESTION [8 upvotes]: Let's suppose that
$f:X\rightarrow X$ is a continuous map such that
$H_{\ast}(f): H_{\ast}(X)\rightarrow H_{\ast}(X)$ is a homology isomorphism (with integral coefficients)
$X$ is a finite connected CW-complex.
$\pi_{1}(f): \pi_{1}(X)\rightarrow \pi_{1}(X)$ is an isomorphism of fundamental groups.
$\pi_{1}(X)$ is a finitely presented group.
$\pi_{n}(f)=0$ for $n>1$.
the homotopy colimit $$hocolim(X\rightarrow_{f} X\rightarrow_{f} X\dots)$$ is homotopy equivalent to a finite CW-complex.
Does it imply that $f$ has to be a weak homotopy equivalence ?
My guess is that the answer should be no but I don't have a counterexample.
REPLY [15 votes]: Here's a counterexample.
Set $X'=S^1\vee S^2$.
Consider the following map $F':S^2\vee S^2\vee S^2\rightarrow X'$: It maps the first $S^2$ summand to the $S^2$ summand of $X'$ via a map that represents $2\in\pi_{2}S^2$; it maps the second summand once around the $S^1$ factor of $X'$, and maps the third $S^2$ summand to the $S^2$ summand in $X'$ by a map that represents $-1\in\pi_{2}S^2$.
Let $F$ be the composition of $F'$ with the map $S^2\rightarrow S^2\vee S^2\vee S^2$ which collapses 2 different latitudinal circles.
Form $X$ by attaching a 3-cell to $X'$ by the map $F$. Note that $\pi_{1}X=\pi_{1}S^1$, and the inclusion $S^1\hookrightarrow X$ is a homology isomorphism.
The map $f:X\rightarrow X$ which collapses $X$ to its $S^1$ summand satisfies all the requirements. In this case the hocolim in requirement 6 is $\simeq S^1$.<|endoftext|>
TITLE: How to check whether a given matrix is in the image of a representation?
QUESTION [6 upvotes]: Let $G$ be a compact simple Lie group, and let $\rho$ be a (faithful, unitary) irreducible representation thereof of $\mathbb K$-dimension $n$, where $\mathbb K=\mathbb C/\mathbb R/\mathbb H$ if $R$ is real/complex/pseudo-real, respectively. It then follows that there is a subgroup of $SU(n)/SO(n)/Sp(n)$, respectively, isomorphic to $G$. One can think of $\rho$ as a map from $G$ to this subgroup.
How can I check whether a given matrix $M\in SU(n)/SO(n)/Sp(n)$ is in the image of $\rho$? In other words, given one such matrix $M$, how can I decide whether there exists some $g\in G$ such that $\rho(g)=M$?
For the sake of concreteness, say $G=G_2$ is the first exceptional simple group, and let $\rho$ be the representation with highest weight $2\omega_2$ (which is real and $27$-dimensional). This means that for any $g\in G_2$, $\rho(g)$ is a $27$-dimensional orthogonal matrix. If I take some arbitrary $27$-dimensional orthogonal matrix $M$, how can I check whether it can be written as $M=\rho(g)$ for some $g\in G_2$?
Note: I am particularly interested in the case where $M$ is diagonal, but I'd be interested in hearing about the general case as well. In the diagonal case, where everything is abelian, and one can essentially focus on a Cartan subalgebra, I assume one can be quite explicit about the image of $\rho$. In the general case, I wouldn't be surprised if one has to work harder.
REPLY [4 votes]: Suppose that $G$ is a compact Lie group (or, more generally, an algebraic group over some field $F$), ${\mathfrak g}$ is its Lie algebra. You are given a linear representation $\rho: G\to GL(n, F)$. From this, compute the representation of the Lie algebra $\rho': {\mathfrak g}\to End(F^n)$. In fact, the representation $\rho$ is frequently given in terms of the highest weight of $\rho'$. Now, you check section 4.5 "Computing defining polynomials of an algebraic group in terms of its Lie algebra" in
W. de Graaf, "Computing with Linear Algebraic Groups", CRC, 2017.
He describes an algorithm for computing defining polynomial equations $p_i, i=1,...,N$, for $\rho(G)$ in terms of $\rho'({\mathfrak g})$ (more precisely, in terms of a basis for this subalgebra in $End(F^n)$). He even has software (check his webpage https://www.science.unitn.it/~degraaf/) for practical computations of this type.
Lastly, to verify if the given matrix $A\in GL(n, F)$ belongs to $\rho(G)$, evaluate the polynomials $p_i$ on $A$.<|endoftext|>
TITLE: 3-balls with the same boundary in $S^4$ differ up to diffeomorphism
QUESTION [6 upvotes]: I am looking at this recent paper by Budney and Gabai and I am confused by a certain sentence in it. Theorem 4.7 states that if $\Delta_1$ and $\Delta_2$ are two 3-balls smoothly embedded in $S^4$ that are identical near there boundary, then there is a diffeomorphism $\Phi: S^4 \to S^4$ that is the identity on the neighborhood of the boundary of these two balls where they agree and $\Phi(\Delta_1) = \Delta_2$.
The proof of this theorem is one sentence long, and this is the sentence where I am seeking some clarification. The sentence cites two facts:
(1) Regular neighborhoods are unique
(2) $\operatorname{Diff}_0(S^2)$ is connected
and two papers:
(1) J. Cerf, Topologie de certains espaces de plongements, Bull. Soc. Math. France, 89 (1961), 227–380.
(2) R. Palais, Extending diffeomorphisms, Proc. Amer. Math. Soc. 11 (1960), 274–277.
I am not sure exactly what parts of these two references are being used. I believe for the Palais paper, maybe the authors are using Corollary 2 to say that there is some diffeomorphism of $S^4$ taking $\Delta_1$ to $\Delta_2$. I am not sure what is in the Cerf paper (it is written in french and over 100 pages so that has kept me away). However, maybe this is the standard reference for the fact that every diffeomorphism of $S^3$ extends over $B^4$?
I'm also not exactly sure what the statement "regular neighborhoods are unique" means. I suppose it means that any two regular neighborhoods for a smooth submanifold differ up to homotopy rel the submanifold.
I would love it if someone could tell me how to fit these pieces together and understand the proof.
REPLY [5 votes]: The cited (early) work by Cerf proves that, given a submanifold Y in a manifold X, the obvious map Diff(X)->Emb(Y,X) is a locally trivial fibration.
I guess that Budney and Gabai mean the following. By Palais, all embeddings D^3->S^4 are isotopic. Hence, for i=0, 1, the complement C_i of a small
open tubular neighborhood U_i of Delta_i in S^4 is diffeomorphic with the compact 4-ball B^4. One has two disjoint embeddings phi_i, psi_i of B^3 in the boundary S^3 of B^4=C_i, one orientation-preserving, the other orientation-reversing, representing the two sides of Delta_i. It remains to extend the diffeomorphism between C_0 and C_1 through U_0
and U_1. This amounts to find a diffeomorphism f:B^4->B^4
such that f o phi_0=phi_1 and f o psi_0=psi_1. The papers by Palais and Cerf precisely give this. The connexity of Diff_+(S^2), and the unicity of the tubular neighborhood up to isotopy, serve to arrange that
the extension goes well on a small neighborhood of the 2-sphere bounding Delta_0 and Delta_1.<|endoftext|>
TITLE: Request for an exact formula related to a partition in number theory
QUESTION [5 upvotes]: The Frobenius equation is the Diophantine equation $$
a_1 x_1+\dots+a_n x_n=b,$$
where the $a_j$ are positive integers, $b$ is an integer, and a solution $$(x_1, \dots, x_n)$$
must consist of non-negative integers, i.e.
$$
x_j \in \mathbb{N}
$$
as Natural numbers. For negative $b$, there are no solutions.
My question: Is there any known formula that counts the number of solutions, by giving $a_1, \dots, a_n$, $b$, and $n$? Let us call this function as $F (a_1, \dots, a_n; b, n)$, what is known for this:
$$F (a_1, \dots, a_n; b, n)=?$$
For all the $a_j=1$, we can simplify the above Frobenius equation to:
$$
x_1+\dots+x_n=b, \tag{1}$$
where $b \in \mathbb{Z}^+$ is a positive integer.
Here is another simpler question: Is there a general formula for Eq.(1) counting all the possible solutions $$(x_1, \dots, x_n)$$
for given the positive integer $n \in \mathbb{Z}^+$ and $b \in \mathbb{Z}^+$? This should be related to the Partition, but I am not sure the exact forms are known? Say, can we find the total number of soultions as a function $f(n,b)$, and what is
$$
f(n,b)=?
$$
It seems the answer is known:
$$
f(n,b)= \binom{b+n-1}{n-1}.
$$
p.s. Sorry if this question is too simple for number theorists. But please provide me answer and Refs if you already know the answer. Many thanks!
REPLY [4 votes]: You can take generating function $$f(z):=\frac{1}{1-z^{a_1}}\frac{1}{1-z^{a_2}}\cdots \frac{1}{1-z^{a_n}}$$ as in Max Alekseyev's answer and calculate $F (a_1, \dots, a_n; b, n)$ as $$
\frac{1}{2 \pi i} \int_{|s|=\rho} f(s) \frac{d s}{s^{b+1}} \quad (0<\rho<1).
$$
It gives the answer
$$
F (a_1, \dots, a_n; b)=\frac{b^{n-1}}{(n-1) ! a_{1} \ldots a_{n}}+\sum_{k=0}^{n-2} c_{k} b^{k}.
$$
It is a classical applications of contour integration taken from the book "Residues and their applications" by A.O. Gelfond (1966, pp. 98-99, Russian). If $(a_j,a_k)=1$ ($j\ne k$) then all poles (excepting $s=1$) are simple and formula can be simplified:
$$
F (a_1, \dots, a_n; b)=\frac{(-1)^{n-1}}{(n-1) !} \frac{d^{n-1}}{d s^{n-1}}\left[s^{-b-1} \prod_{k=1}^{n} \frac{1-s}{1-s^{a_{k}}}\right]_{s=1}+R
$$
where $|R|
TITLE: Bounding and domination numbers for relation $\leq$ modulo $\omega$-nullsets
QUESTION [7 upvotes]: We say that $A\subseteq \omega$ is a nullset if $$\lim\sup_{n\to \infty} \frac{|A\cap n|}{n+1} = 0.$$
Let $\omega^\omega$ denote the set of functions $f:\omega\to\omega$. We define a pre-ordering relation $\leq^0$ on $\omega^\omega$ by saying that $f\leq^0 g$ if $f(x) \leq g(x)$ for all $x\in\omega\setminus N$ where $N\subseteq \omega$ is a nullset.
Similarly to the bounding number and the dominating number respectively, we define
${\frak b}^0 = \min\{|B|: B\subseteq \omega^\omega \land \forall f\in \omega^\omega\; \exists b\in B(b\not\leq^0 f)\}$, and
${\frak d}^0 = \min\{|D|: D\subseteq \omega^\omega \land \forall f\in \omega^\omega\; \exists d\in D(f\leq^0 d)\}$.
Do we have ${\frak b}^0={\frak b}$? And what about ${\frak d}^0={\frak d}$?
REPLY [4 votes]: The answer seems to be positive according to this paper: Barnabás Farkas, Lajos Soukup: The zero density ideal, cardinal invariants and related forcing problems.1
Theorem 2.3. If $\mathcal I$ is a rare ideal on $\mathbb N$ then $\mathfrak b = \mathfrak b_{\mathcal I}$ and $\mathfrak d = \mathfrak d_{\mathcal I}$.
Just before this theorem the authors mention that the ideal $\mathcal Z_0$ of the sets with zero density is a rare ideal.
A similar result is shown for analytic P-ideals in Corollary 5.5 of More on cardinal invariants of analytic P-ideals by the same two authors (arXiv, eudml). Again, this class of ideals includes $\mathcal Z_0$.
1I wasn't able to find whether the paper was published somewhere, but a preprint is available here (Wayback Machine). The same paper was also mentioned in this answer: Are these two quotients of $\omega^\omega$ isomorphic?<|endoftext|>
TITLE: When is the model structure on functors correct, i.e. when does localization commute with taking functor categories?
QUESTION [7 upvotes]: Let $C$ be a small category and $M$ a model category. Then there are various "global" model structures (projective, injective, Reedy) on the category $Fun(C,M)$ of functors from $C$ to $M$, all with the same (levelwise) weak equivalences. The whole point of having such a model structure is that it should present the $\infty$-category $Fun(C,\tilde M)$, where $\tilde M$ is the $\infty$-category presented by $M$. But I'm not sure when this is actually the case.
Of course, by "the $\infty$-category presented by $M$", I mean $\tilde M = M[W^{-1}]$ is $M$ localized at the weak equivalences in the $\infty$-categorical sense, and similarly "the $\infty$-category presented by $Fun(C,M)$" is the $\infty$-categorical localization $Fun(C,M)[Fun(C,W)^{-1}]$.
Questions:
If $C$ is a small category and $M$ is a model category, then under what conditions do the standard model structures on $Fun(C,M)$ present the $\infty$-category $Fun(C,\tilde M)$, where $\tilde M = M[W^{-1}]$ is the $\infty$-category presented by $M$?
More generally, if $C$ and $M$ are relative categories, then under what conditions does the mapping relative category $\widetilde{Fun(C,M)} = Fun(\tilde C, \tilde M)$ where $\tilde{(-)}$ denotes taking the associated quasicategory?
In a more model-independent direction, when does localization of $\infty$-categories commute with taking functor categories? That is, when does $Fun(C,M[W^{-1}]) = Fun(C,M)[Fun(C,W)^{-1}]$ where $C,M$ are $\infty$-categories and $W \subseteq M$ is a subcategory?
REPLY [10 votes]: If your model structures are assumed to have small limits or colimits, the answer to the question of the title is: always. For any model category $M$ and any small category $C$, inverting levelwise weak equivalences in $Fun(C,M)$ is equivalent to considering the $\infty$-category of functors from $C$ to $M[W^{-1}]$. This is a special case of Theorem 7.9.8 (and Remark 7.9.7) in my book on higher categories. It is even possible to take for $M$ a model $\infty$-category in the sense of Mazel-Gee. In fact, Theorem 7.5.8 gives sufficient conditions on $M$ which are much more general: essentially, the mere existence of a class of well behaved fibrations is good enough (this includes Brown's categories of fibrant objects, but, more generally, a version where we do not assume all objects to be fibrant; in particular, all the variations on semi-model structures are OK) if we assume further properties: we mainly need this extra structure to exist on functor categories $Fun(C,M)$ as well (which is automatic in practice, as explained in Example 7.9.6 and Remark 7.9.7 of loc. cit.). If we restrict ourselves to those $C$ whose nerve is a finite simplicial set (e.g. finite partially ordered sets), this kind of properties is true in a much greater level of generality; see Theorem 7.6.17.
Observe that, if $M$ is good enough in the sense that $\widetilde{Fun(C,M)}\cong Fun(C,\tilde M)$ for any small category $C$, then, it is automatic that, for any subcategory $S\subset C$, the localization of the full subcategory of $Fun(C,M)$ whose objects are those functors sending the maps of $S$ to weak equivalences will autmatically be a model of the $\infty$-category of functors from $C[S^{-1}]$ to $\tilde M$.
All of this is obviously model independent.<|endoftext|>
TITLE: Infinite simple group of exponent a power of 2
QUESTION [6 upvotes]: Let G be a simple group of exponent 2^n for n>1. Is G necessarily finite? If not, what is an example of an infinite simple group of exponent a power of 2?
REPLY [6 votes]: For every large enough $n$, there exist infinite simple groups of exponent dividing $n$.
(I call a group aperiodic if it has no nontrivial finite quotient.) Indeed, by the solution to the restricted Burnside problem the Burnside group $B(2,n)$ is known to be virtually aperiodic; let $B(2,n)^\circ$ be its (unique) minimal finite index subgroup, it is thus finitely generated. For $n$ large enough, $B(2,n)$ is known to be infinite. Hence $B(2,n)^\circ$ is infinite too, hence has a simple quotient, also aperiodic and hence infinite. $\Box$
The above proof produces groups of exponents dividing $n$, probably with some more efforts we can get exponent exactly $n$ but I think it's unimportant. I don't know if one can obtain quasi-finite groups in this context, when the exponent is $2^m$ (quasi-finite means infinite with all proper subgroups finite); these are close variants of Tarski monsters. Also probably one can produce infinitely generated countable examples, and uncountable too, but this requires other/additional arguments. Nevertheless:
$\forall n$, there exists no infinite locally finite simple group of exponent $n$. (Nor even in which every element has order dividing some power of $n$.)
For $n=2^k$ (or more generally $p^aq^b$ with $p,q$ prime, these would be locally solvable and indeed Malcev proved that there is no infinite, locally solvable simple group. In general, this is essentially due to Hartley (1995) (Springerlink behind paywall). Namely he proved that for every finite subset $F$ in a simple locally finite group $G$, there exists a finite subgroup $H$ containing $F$, and a normal subgroup $N$ of $H$ such that $H/N$ is simple and $F$ projects injectively into $H/N$. Since (by classification) finite simple groups of given exponent have bounded order, this implies the result. (Hartley also quotes Meierfrankelfeld for the same result.) The statement is explicit in a 2005 paper by Cutolo-Smith-Wiegold (ScienceDirect), Lemma 4.<|endoftext|>
TITLE: Book on manifolds from a sheaf-theoretic/locally ringed space PoV
QUESTION [11 upvotes]: I'm looking for an introductory (or rather, non-advanced) book on manifolds as locally ringed spaces, i.e., from the algebraic geometric viewpoint. Most introductory texts only introduce manifolds from the differentiable viewpoint; I wonder if a text introducing differential manifolds from an algebraic viewpoint exists.
REPLY [4 votes]: In Introduction to differential geometry (see the review) by R.Sikorski the author introduces the concept of (what is now called) Sikorski space. Sikorski spaces are "affine, reduced differential spaces" and hence they can be approached algebraically by looking at their coordinate rings. Differentiable manifolds are important examples Sikorski spaces. Unfortunately the book was not translated to english. Luckily there are some publications (in english and perhaps in french) by Sikorski in which he explains this very natural concept. One of them is Differential modules.
A book $C^{\infty}$ Differentiable Spaces by Navarro González and Sancho de Salas develop theory of differentiable spaces by first constructing real spectra for smooth algebras and then glue them in order to obtain general spaces. This is analogical to the development of algebraic geometry (scheme theory) by Grothendieck and his school. The book might be a bit advanced.<|endoftext|>
TITLE: Is the imaginary part of $t\mapsto\zeta(1/2+it)$ close to the derivative of its real part?
QUESTION [5 upvotes]: Plotting $t\mapsto\zeta(1/2+it)$ on Wolfram alpha, it seems that the maxima of its real part are close to the zeros of its imaginary part, while the maxima of the latter seem close to the inflection points of the former. Can this be made precise? For example, is there a canonical notion of distance between those two functions that attains only small values?
REPLY [3 votes]: (An extended comment.) The derivative of the real part does not really seem to be close to the imaginary part, as seen in the following picture generated by Mathematica:
Code: Plot[{Re[I Zeta'[1/2 + I t]], Im[Zeta[1/2 + I t]]}, {t, 0, 80}]
The corresponding zeroes of the two functions indeed seem to be reasonably close to each other. This is no surprise, however: $\zeta(\tfrac{1}{2} + i t)$ essentially circles around (mostly in the right half-plane). In each "circle" the distance between a maximum or minimum of the real part and the corresponding zero of an imaginary part is roughly as large as the distance froth the "center" to the real axis. And most "circles" seem to be centered near the real axis.<|endoftext|>
TITLE: Why is the Fourier transform so ubiquitous?
QUESTION [63 upvotes]: Many operations and equivalences in mathematics arise as some sort of Fourier transform. By Fourier transform I mean the following:
Let $X$ and $Y$ be two objects of some category with products, and consider the correspondence $X \leftarrow X \times Y \to Y$. If we have some object (think sheaf, function, space etc.) $\mathcal{P}$ over $X \times Y$ and another, say $\mathcal{F}$ over $X$, assuming the existence of suitable pushpull and tensoring operations, we may obtain another object over $Y$ by pulling $\mathcal{F}$ back to the product, tensoring with $\mathcal{P}$, then pushing forward to $Y$.
The standard example is the Fourier transform of functions on some locally compact abelian group $G$ (e.g. $\mathbb{R}$). In this case, $Y$ is the Pontryagin dual of $G$, $\mathcal{P}$ is the exponential function on the product, and pushing and pulling are given by integration and precomposition, respectively.
We also have the Fourier-Mukai functors for coherent sheaves in algebraic geometry which provide the equivalence of coherent sheaves on dual abelian varieties. In fact, almost all interesting functors between coherent sheaves on nice enough varieties are examples of Fourier-Mukai transforms. A variation of this example also provides the geometric Langlands correspondence
$$D(Bun_T(C)) \simeq QCoh(LocSys_1(C))$$
for a torus $T$ and a curve $C$. In fact, the geometric Langlands correspondence for general reductive groups seems to also arise from such a transformation.
By the $SYZ$ conjecture, two mirror Calabi-Yau manifolds $X$ and $Y$ are dual Lagrangian torus fibrations. As such, the conjectured equivalence
$$D(Coh(X)) \simeq Fuk(Y)$$
is morally obtained by applying a Fourier-Mukai transform that turns coherent sheaves on $X$ into Lagrangians in $Y$.
To make things more mysterious, a lot of these examples are a result of the existence of a perfect pairing. For example, the Poincaré line bundle that provides the equivalence for coherent sheaves on dual abelian varieties $A$ and $A^*$ arises from the perfect pairing
$$A \times A^* \to B\mathbb{G}_m.$$
Similarly, the geometric Langlands correspondence for tori, as well as the GLC for the Hitchin system, arise in some sense from the self-duality of the Picard stack of the underlying curve. These examples seem to show that non degenerate quadratic forms seem to be fundamental in some very deep sense (e.g. maybe even Poincaré duality could be considered a Fourier transform).
I don’t have a precise question, but I’d like to know why we should expect Fourier transforms to be so fundamental. These transforms are also found in physics as well as many other “real-world” situations I’m even less qualified to talk about than my examples above. Nevertheless I have the sense that something deep is going on here and I’d like some explanation, even if philosophical, as to why this pattern seems to appear everywhere.
REPLY [7 votes]: A signature property of the fourier transform is that it converts convolution into multiplication. (This is crucial for real world applications such as image processing especially as in the discrete case the fourier transform can be computed very quickly using the FFT).
Another signficant property of the fourier transform is that it measures how randomly a finite set is distributed via the size of its fourier coefficients.
This is applied in Roth's proof of Szemeredi's as one can show that if all the fourier coefficients of a set are small then the set has approximately the right number of arithmetic progressions of length three.
Similarly, counting solutions to algebraic equations over finite fields uses the fact that the values of polynomials including simple powers are randomly distributed and this can be measured using the fourier transform and exponential sum estimates.<|endoftext|>
TITLE: Is the symplectic quotient $\mu^{-1}(0)/G$ unique up to something?
QUESTION [6 upvotes]: Given a Hamiltonian action of a compact Lie group $G$ on a symplectic manifold $(M,\omega)$, we may choose a moment map $\mu \colon M\to \mathfrak{g}^* $ and obtain the symplectic reduction $M/\!\!/G = \mu^{-1}(0)/G$. This construction clearly depends on the choice of moment map. However, I wonder if it is still unique up to some sort of (very?) weak equivalence in the symplectic category?
REPLY [5 votes]: Given a Hamiltonian group action, moment maps may only differ by constant addition. So you seem to be comparing the reduced spaces at different levels. Let me state the two extreme cases.
When $G$ is a torus, any constant addition to a moment map is also a moment map. In a paper "Birational equivalence in the symplectic category (1989)" by Guillemin and Sternberg, authors showed that reduced spaces at regular levels are related by blowing up and down. I do not know the recent progress though. It might be helpful to read papers citing their paper.
The other extreme case is when $G$ is semisimple. In this case, the moment map, which is $G$-equivariant, is unique by the semisimplicity. Then the reduced space is unique and there is nothing to do.<|endoftext|>
TITLE: How often a random walk with irrational increments is close to 0?
QUESTION [8 upvotes]: Let $\omega$ be an irrational number, and $X$ a random variable taking values $1,-1,\omega,-\omega$ each with probability $1/4$. Let then $X_i$ be iid variables with the same law as $X$ and $S_n=\sum_{i=1}^n X_i,n\in \mathbb N$ be the corresponding random walk.
Is it possible to have a precise asymptotics for $P(|S_n|<\epsilon)$ for $\epsilon>0$? Ultimately I would like to know the behaviour of
$$\sum_{n=1}^\infty n^{-3/2} P(|S_n|<\epsilon)$$ as $\epsilon\to 0$.
I feel like the diophantine properties of $\omega$ are relevant for this asymptotics.
How would you proceed to get such an estimate? Ideally I would like to consider $X$ with any discrete law, with eventually infinitely many atoms.
EDIT: to be clear, I think there are ad-hoc methods to solve this kind of problems, as Mateusz shows below. I want to make sure not to miss any kind of general theory that solves this kind of problems in the theory of random walks.
REPLY [4 votes]: Just an extended comment. Let $X_n$ be the simple random walk in $\mathbb{Z}^2$. Then $$\mu(\{x\}) = \sum_{n = 1}^\infty n^{-3/2} P(X_n = x)$$ is comparable with $$\sum_{n = 1}^\infty n^{-3/2} \times n^{-1} \exp(-|x|^2 / (2 n)) \approx (1 + |x|)^{-3}.$$ So your question boils down to estimating $$\sum_{k \in \mathbb{Z}} \frac{1}{(1 + |k|)^3} \, \mathbb{1}_{(-\epsilon, \epsilon)}(k \omega - \lfloor k \omega\rfloor) . $$ This indeed seems closely related to how well one can approximate $\omega$ with rationals.<|endoftext|>
TITLE: When is a large graph with a given degree sequence likely to be connected?
QUESTION [10 upvotes]: Are there any results on whether a large random graph with a given degree distribution is likely to be connected?
In Erdős-Rényi graphs with $n$ vertices and $m$ edges, we have two sudden transitions (for large $n$):
A giant component appears above the threshold $m/n = 1/2$.
The graph becomes connected above the threshold $m/n = (\ln n)/2$.
There is a result analogous to (1) above by Molloy and Reed for random graphs with a given degree distribution. If $d$ denotes the vertex degree and $\langle \cdot \rangle$ denotes the average, then the quantity of interest is $Q = \langle d^2 \rangle - 2\langle d \rangle$. A giant component suddenly appears above the threshold $Q > 0$.
Question: Is there a result analogous to (2) for random graphs with a fixed degree sequence, in the large graph limit? Is there a quantity that can be computed from the degree distribution, and when it crosses a threshold, the graph suddenly becomes connected (in the $n\rightarrow\infty$ limit)? Let us assume that there are no isolated vertices ($d\ne 0$).
Clarification update: Let me try to give a more precisely specified version of the problem. Suppose we have $n$ vertices. Of these, precisely $n_d = f_d n$ have degree $d$: thus we have a degree sequence
$$(
\overbrace{0,\dots,0}^{\text{$n_0$ times}},\;
\overbrace{1,\dots,1}^{\text{$n_1$ times}},\;
\overbrace{2,\dots,2}^{\text{$n_2$ times}}, \dots).
$$
Choose one simple (labelled) graph with this degree sequence uniformly at random.
What conditions do we need to have on the $f_d$ (the degree distribution), or on $n_d$, so that in the $n \rightarrow \infty$ limit the graph is connected with probability 1?
Clearly, if the $f_0 \ne 0$, then there are isolated vertices and the graph is not connected. Therefore, one condition is that $f_0 = 0$.
REPLY [2 votes]: Second edition
This is a partial answer to the question per the "Clarification Update", but first I'll generalize a little. Suppose that for each $n$ we have a degree sequence $n_0,n_1,n_2,\ldots$, where $n_d=n_d(n)$ means the number of vertices of degree $d$. Also let the number of edges be $m=m(n)$ and the maximum degree be $\varDelta=\varDelta(n)$. Now we take a random simple graph $G=G(n)$ with this degree sequence, each such graph being equally likely. We seek to know if $G$ is connected. Take $n_0=0$ from now on.
This type of random graph has been extensively studied. I'll just make some simple observations using Theorem 2.1 of this paper.
By Theorem 2.1 the expected number of isolated edges is
$$\binom{n_1}{2}\frac{1+O(\varDelta/m)}{2m}$$ if $\varDelta=o(m)$.
Assuming the latter condition, the expected number of isolated edges goes to $0,\infty$ according as $n_1^2/m$ goes to $0,\infty$, respectively. This doesn't imply instantly that $n_1\approx \sqrt{m}$ is the threshold for having an isolated edge, but it is true (use the second moment method).
So now assume $n_1=o(m)$.
I thought a combination of degrees 1 and 2 might be an issue, but the most likely isolated component, a path of two edges, is unlikely if $n_1=o(\sqrt m)$.
(So, if these components are likely, so are isolated edges.)
Now consider isolated cycles. The expected number of isolated cycles of length $k$ is $$\frac{(n_2)_k(1+O(k\varDelta/m))}{2k\,(m)_k},$$ where $(x)_k=x(x-1)\cdots(x-k+1)$, provided $k\varDelta=o(m)$.
Since $n_2\le m$, this never goes to infinity for fixed $k$, but
the sum over an increasing number of $k$ values does go to infinity
if $n_2=(1-o(1))m$. In the other direction, if $n_2=o(m)$ then
the expectation goes to 0 for each $k$ and moreover the terms
appear to be decreasing exponentially as $k$ increases. Here
there is a gap in the proof because $k\varDelta=o(m)$ might not
be true for very large $k$ unless also $\varDelta=O(1)$. This gap can be filled but I won't go
into it. Modulo some things I haven't quite proved, the probability of connectedness goes to 1 if $n_2=o(m)$ and to 0 if $n_2=(1+o(1))m$.
In the intermediate ranger, for example if $n_2=cm$ for $0\lt c\lt 1$, I believe that the distribution of the number of isolated cycles will be Poisson with constant mean.
Beyond this, I'm reluctant to reinvent the wheel because someone
must have done this before except possibly in the case that some
degrees are very low and others very high. There are no component
types that are likely to occur under conditions when isolated edges
or cycles are unlikely to occur. The fact that random
regular graphs of degree at least 3 are almost always connected
was proved by Wormald in the 1970s. I hypothesize that $n_0=0$,
$n_1=o(\sqrt m)$ and $n_2\le cm$ for some $c\le 1$ are necessary
and sufficient conditions for almost sure connectivity.
The question also asks us to consider the case that there constants
$f_0,f_1,\ldots$ independent of $n$ such that $n_d(n)=f_d\,n$ for
all $n,d$. Translating what is above, the condition for connectivity
is $f_0=f_1=f_2=0$. Clearly forcing $f_d$ to be independent of $n$ loses a lot of detail.<|endoftext|>
TITLE: The geometry of the action of the semidirect product
QUESTION [9 upvotes]: I'm going by the maxim
Groups, like men, are known by their actions
This naturally leads one to ask "given groups $G, H$ which act on sets $S, T$ and the semidirect product $G \rtimes H$, how does one visualize the action of $G \rtimes H$? What does it act on? Some combination of $S$ and $T$? ($S \times T$ perhaps?)
I know some elementary examples, likr $D_n \simeq \mathbb Z_n \rtimes \mathbb Z_2$. However, given an unknown situation, I am sure I cannot identify whether it is a semidirect product that is governing the symmetry.
The best responses on similar questions like intuition about semidirect product tend to refer to this as some kind of "direct product with a twist". This is shoving too much under the rug: the twist is precisely the point that's hard to visualize. Plus, not all "twists" are allowed --- only certain very constrained types of actions turn out to be semidirect product. I can justify the statement by noting that:
the space group of a crystal splits as a semidirect product iff the space group is symmorphic --- this is quite a strong rigidity condition on the set of all space groups.
This question on the natural action of the semidirect product identifies one choice of natural space for the semidirect product to act on, by introducing an unmotivated (to me) equivalence relation, which "works out" magically. What's actually going on?
The closest answer that I have found to my liking was this one about discrete gauge theories on physics.se, where the answer mentions:
If the physical space is the space of orbits of $X$ under an action $H$. Ie, the physical space is $P \equiv X / H$. Then, if this space $P$ is acted upon by $G$. to extend this action of $G \rtimes H$ onto $X$ we need a connection.
This seems to imply that the existence of a semidirect product relates to the ability to consider the space modulo some action, and then some action per fiber. I feel that this also somehow relates to the short exact sequence story(though I don't know exact sequences well):
Let $1 \rightarrow K \xrightarrow{f}G \xrightarrow{g}Q \rightarrow 1$ be a short exact sequence. Suppose there exists a homomorphism $s: Q \rightarrow G$ such that $g \circ s = 1_Q$. Then $G = im(f) \rtimes im(s)$. (Link to theorem)
However, this is still to vague for my taste. Is there some way to make this more rigorous / geometric? Visual examples would be greatly appreciated.
(NOTE: this is cross posted from math.se after getting upvotes but no answers)
REPLY [2 votes]: Suppose $K\circlearrowright X$ is an action of $K$ on some set. We have the structure homomorphism $\varphi:K\rightarrow \text{Sym}(X).$
Let $H$ act on $K$ by automorphisms, and suppose these automorphisms can be realized as inner automorphisms within $\text{Sym}(X)$. That is, the action is given by some $\theta:H\rightarrow\text{Sym}(X)$ such that conjugations by $\theta(H)$ leave $K$ invariant. Equivalently, the $H$-action takes stabilizer subgroups of $K\circlearrowright X$ to stabilizers (it may permute them nontrivially).
Then we can construct an action of $K\rtimes H$ on $X$ by $(k,h).x = \varphi(k)\cdot\theta(h).x$, where the product $\varphi(k)\theta(h)$ is just taken in $\text{Sym}(X)$. Here, the multiplication rule for $K\rtimes H$ is
$$(k_1, h_1)\cdot (k_2,h_2) = (k_1 \cdot (h_1.k_2), h_1 h_2). $$
Any action arises in this way, since $H$ acts on $K$ by conjugation in the semidirect product $K\rtimes H$ and therefore it also acts by conjugation in the image under the structure morphism $K\rtimes H \rightarrow \text{Sym}(X)$ of an action of the semidirect product on $X$. This explains when an action of $K$ can be extended to an action of $K \rtimes H$.
It is not clear how to visualize the above. So let's pass to a nice special case.
Given another action of $H$ on a set $Y$, we can extend $H\circlearrowright Y$ to an action $K\rtimes H \circlearrowright Y$ (let the latter act via the quotient $K\rtimes H\twoheadrightarrow H$). Then we can produce an action of $K\rtimes H \circlearrowright Y \times X$. This action descends to the quotient $H\circlearrowright Y$ so that $K$ fixes each fiber $\{y\}\times X$, and $H$ acts by permuting fibers and "twisting" $X$.
If the $H$-action is faithful, the action of any element $(k,h)$ can be nicely separated into an $h$-part and a $k$-part, the $h$-part being uniquely identified by the action on $Y$. Thus given a permutation $\sigma$ of $Y\times X$ we can write that it is of the form $(k,h)$ for a known $h$, and then compute the permutation $(y,x)\mapsto \sigma\left((e,h^{-1}).(y,x)\right)$, which acts the same as $(k,h)\cdot(e,h^{-1}) = (k,e)$.
One gets some examples which appear different, but are isomorphic to these, by choosing different identifications between fibers than $\text{id}_X : \{y_1\}\times X \rightarrow \{y_2\}\times X$. These identifications may be analogous to the connection described in the question.<|endoftext|>
TITLE: Is the separability of the space needed in the proof of the Prohorov's theorem?
QUESTION [6 upvotes]: The Section 5 of the book:
Billingsley, P., Convergence of Probability Measures, 1999,
studies Prohorov's theorem. A short reminder is given below.
Let $\Pi$ be a family of probability measures on $(S,\mathcal{F})$. We call $\Pi$ relatively compact if every sequence of elements of $\Pi$ contains a weakly convergent subsequence. The family $\Pi$ is tight if for every $\epsilon$ there is a compact set $K$ such that $P(K)>1-\epsilon$ for every $P$ in $\Pi$.
The direct half of the Prohorov's theorem is given in the Theorem 5.1: If $\Pi$ is tight, then it is relatively compact.
The converse half of Prohorov's theorem is given in the Theorem 5.2: Supose that $S$ is separable and complete. If $\Pi$ is relatively compact, then it is tight.
My question: In the proof of the Theorem 5.2 (i.e. relatively compact $\Rightarrow$ tight), we use separability and completness of the space $S$. On the other hand, in the proof of the Theorem 5.1 (i.e. tight $\Rightarrow$ relatively compact), I know that we do not need completness of $S$, but I do not know if we do need separability. I didn't find the place where separability is used in the proof of the Theorem 5.1. So my question is do I need or not the separability of the space $S$ in the direct part of the Prohorov's theorem?
Remarks:
I know the proofs of the same theorem that use separability (e.g. Note).
Prohorov's theorem in most books is given as one theorem on Polish spaces, so they assume separability in both halfs. It goes like this usually: Let $S$ be a Polish space and $\Pi$ a collection of probability measures on $S$. Than $\Pi$ is tight if and only if it is relatively compact.
The reason I am asking is that I would like to use the direct half of Prohorov's theorem on the problem I am currently working. Space $S$ in my case is complete but not separable.
Help with this would be great and needed. Thanks in advance.
REPLY [4 votes]: Separability is not necessary. In fact, tightness of a family of Borel probability measures implies relative compactness in the vague/weak-* topology on any completely regular space. For instance, this can be found in volume 4 of Fremlin's Measure Theory. Specifically Proposition 437U (b) shows that tight families are compact in the narrow topology, and 437K (c) shows that for completely regular spaces, the narrow topology agrees with the weak-* topology.
My original answer below answers the wrong question - the question is about whether tight implies relatively compact, rather than the other way.
Let $\kappa$ be a real-valued measurable cardinal, and $\mu : \mathcal{P}(\kappa) \rightarrow [0,1]$ a probability measure vanishing on singletons. Consider $\kappa$ to be a discrete metric space. Then the 1-element family $\{\mu\}$ is compact, because it is a singleton, but it is not tight because all compact subsets of $\kappa$ are finite sets, so have measure zero.
You say that completeness is not necessary, but (unless you are making an extra assumption) it is, for essentially the same reason - there are separable metric spaces with Borel measures on them that are not tight.
REPLY [3 votes]: You are correct that separability is not needed. However, there is also not really any loss of generality in assuming it. For suppose that $\Pi$ is tight. Then for every $n$ there exists a compact set $K_n$ such that $\mu(K_n) > 1-\frac{1}{n}$ for all $\mu \in \Pi$. So if we set $S_0 = \bigcup_{n=1}^\infty K_n$, then $S_0$ is separable and $\mu(S_0) = 1$ for all $\mu \in \Pi$. We can now view $\Pi$ as a set of probability measures on $S_0$, and it is still tight (since the $K_n$ are also compact in $S_0$). The separable case of the theorem then implies that $\Pi$ is weakly relatively compact in $\mathcal{P}(S_0)$, i.e. every sequence in $\Pi$ has a subsequence converging weakly in $\mathcal{P}(S_0)$, and you can easily check that such a subsequence also converges weakly in $\mathcal{P}(S)$. So $\Pi$ is weakly relatively compact in $\mathcal{P}(S)$, as desired.
In other words, once you have a tight family, then all those measures live on a separable subset of $S$ anyway, so the rest of the space is irrelevant and might as well not be there.<|endoftext|>
TITLE: Does Higman's embedding theorem hold inside group varieties?
QUESTION [7 upvotes]: Suppose $\mathfrak{U}$ is a variety of groups. Let's define $F_n(\mathfrak{U})$ as relatively free groups in $\mathfrak{U}$.
Suppose $G \in \mathfrak{U}$ is a finitely generated group. We call $G$ finitely presented in $\mathfrak{U}$ iff $\exists n \in \mathbb{N}$ and finite $A \subset F_n(\mathfrak{U})$ such that $G \cong \frac{F_n(\mathfrak{U})}{\langle \langle A \rangle \rangle}$. We call $G$ recursively presented in $\mathfrak{U}$ iff $\exists n \in \mathbb{N}$ and recursively enumerable $A \subset F_n(\mathfrak{U})$ such that $G \cong \frac{F_n(\mathfrak{U})}{\langle \langle A \rangle \rangle}$.
My question is:
Is it true, that a finitely generated group is recursively presented in $\mathfrak{U}$ iff it is isomorphic to a finitely generated subgroup of a group finitely presented in $\mathfrak{U}$?
This fact is true for the varieties of abelian groups due to linear algebra, proved for the variety of all groups by Higman, and for the Burnside varieties by Olshanski.
However, I do not know, whether it is true in general.
This question on MSE
REPLY [10 votes]: Kharlampovich proved that there exist two finitely based varieties of solvable groups $ {\mathfrak A} \subset {\mathfrak B}$ such that the word problem is not solvable in the groups f.p. in the smaller variety but solvable in the groups that are f.p. in the bigger variety (the result can be found in our joint survey "Algorithmic problems in varieties", see also Kharlampovich, O. G.
The word problem for solvable Lie algebras and groups.
Mat. Sb. 180 (1989), no. 8, 1033–1066, 1150; translation in
Math. USSR-Sb. 67 (1990), no. 2, 489–525). Now if you take a group $G$ that is finitely presented in the smaller variety and has undecidable word problem then $G$ is finitely generated, recursively presented in the class of all groups since the varieties are finitely based, but cannot be embedded into any group $H$ which is finitely presented in the bigger variety. So variety ${\mathfrak B}$ does not have the property in the OP. Varieties with that property are sometimes called Higman varieties. In my comment above I mentioned some known Higman varieties in addition to those in the OP. It is not known if the variety of solvable groups of class $\le 3$ is Higman. In fact all known Higman varieties have been mentioned in the OP and in my comment.<|endoftext|>
TITLE: When is the homotopy category of an accessible $\infty$-category accessible?
QUESTION [9 upvotes]: Let $\mathcal C$ be an accessible $\infty$-category, and let $ho(\mathcal C)$ be its homotopy category. I can think of two "trivial" reasons for $ho(\mathcal C)$ to be accessible:
$ho(\mathcal C) = \mathcal C$;
$ho(\mathcal C)$ is small with split idempotents.
Otherwise, I am aware of very few examples where $ho(\mathcal C)$ is accessible. Indeed, given that $ho(Spaces)$ is very far from accessible, I think I should expect that it is quite rare for $ho(\mathcal C)$ to be accessible.
However, I know of one interesting class of examples where $ho(\mathcal C)$ is "nontrivially" accessible. Let $k$ be a field.
Write $D(k)$ for "derived $\infty$-category of $k$", i.e. the category of chain complexes over $k$ localized ($\infty$-categorically) at the quasi-isomorphisms. This is a presentable $\infty$-category.
Then $ho(D(k))$ is the usual derived category of $k$, which is equivalent to the usual 1-category of graded $k$-vector spaces (though of course the triangulated structure is different), and so is obviously accessible (locally presentable, in fact).
This one class of examples has me second-guessing my expectation that taking homotopy categories rarely preserves accessibility.
Questions:
What are some other examples of accessible $\infty$-categories $\mathcal C$ such that $ho(\mathcal C)$ is also accessible (which do not satisfy (i) or (ii) above)?
Given such an example, is the functor $\mathcal C \to ho(\mathcal C)$ accessible (i.e. does it preserve $\kappa$-filtered colimits for some $\kappa$?)
Can we (partially) characterize this condition, giving necessary and / or sufficient conditions for $ho(\mathcal C)$ and $\mathcal C \to ho(\mathcal C)$ to be accessible?
Is there anything to be said in particular about the case where $\mathcal C$ is stable and presentable, or even more particularly the case where $\mathcal C$ is the derived $\infty$-category of a ring?
More broadly, at the moment it seems very mysterious to me that $ho(D(k))$ comes out to be accessible. I'd appreciate any perspective which makes this fact look less mysterious.
REPLY [2 votes]: Here's a further generalization:
Claim: Let $\mathcal T$ be a triangulated category and with a $t$-structure such that $\mathcal T^\heartsuit$ is has coproducts, which are exact. Suppose there exists $M \in \mathcal T^\heartsuit$ and a monic endomorphism $M \overset r \rightarrowtail M$ in $\mathcal T^\heartsuit$ which is not an isomorphism, and such that $Hom(M,M/r)$ is an $r$-torsion $End(M)$-module. Then $\mathcal T$ is not concrete, and in particular not accessible.
Proof: For most of the proof, we work in $\mathcal T^\heartsuit$. Similar to before, for every ordinal $\alpha$, we define $W_\alpha$ to be the set of finite subsets of $\alpha+1$, and set $M_\alpha$ to be the cokernel of the map $\oplus^{W_\alpha} M \xrightarrow{1-s} \oplus^{W_\alpha} M$, where $s$ carries the $[\alpha_0<\dots< \alpha_n]$ -copy of $M$ to the $[\alpha_1<\dots<\alpha_n]$ copy of $M$ via the map $r$. A straightforward transfinite induction shows that the $[\alpha_0<\dots<\alpha_n]$'th structure map $M \to M_\alpha$ is in $r^{\alpha_0} Hom(M,M_\alpha)$, and in particular the $[\alpha]$th structure map is in $r^\alpha Hom(M,M_\alpha)$.
It is straightforward to see that the natural map $M_\alpha \to M_{\alpha+1}$ actually splits, and its cokernel $Q$ sits in an exact sequence $M/r \to Q \to M_\alpha \to 0$, where the map to $M_\alpha$ is obtained by deleting the last letter of each word (which is always $\alpha$ here), and the copy of $M/r$ corresponds to the generator $[\alpha]$ (moreover, the $\alpha$th structure map $M \to M_\alpha$ is nonzero and factors through $M/r$). We claim now that $r^{\alpha+1}Hom(M,M_\alpha) = 0$. This is proved by induction on $\alpha$ using the following
Lemma: Let $M$ be an object of an abelian category and $r: M \to M$ a map. If $0 \to A \to B \to C$ is exact, and if $Hom(M,C)$ is $r$-torsion, then any map $\phi \in r^{\alpha+1} Hom(M,B)$ factors (necessarily uniquely) through $A$, and the factored map lies in $r^\alpha Hom(M,A)$. Dually, if $A \to B \to C \to 0$ is exact and $Hom(M,A)$ is $r$-torsion, then if $r^\alpha Hom(M,C) = 0$ it follows that $r^\alpha Hom(M,B) = 0$, for $\alpha \geq 1$.
We apply the lemma to the exact sequences $0 \to M_\alpha \to M_{\alpha+1} \to Q \to 0$ and $M/r \to Q \to M_\alpha \to 0$ at the successor steps of a transfinite induction to conclude that indeed $r^{\alpha+1} Hom(M,M_\alpha) = 0$.
Now we conclude as before: the $\alpha$th structure map $M \to M_\alpha$ doesn't factor through the $\beta$th structure map $M \to M_\beta$ for $\beta > \alpha$ because then it would lie in $r^\beta Hom(M,M_\alpha) = 0$, and it is in fact nonzero. Since $M\to M_\alpha$ are weak cokernels in $\mathcal T$, they form a proper class of pairwise-inequivalent weak cokernels out of $M$, so that $\mathcal T$ is not concrete.<|endoftext|>
TITLE: Is it possible to express the functional square root of the sine as an infinite product?
QUESTION [5 upvotes]: Cross-post from MSE.
It is known that the sine can be expressed as an infinite product: $$\sin(x) = x \prod_{n=1}^{\infty} \Big{(} 1 - \frac{x^{2}}{n^{2}{\pi}^{2}} \Big{)} .$$ We can define that functional square root of a function $g(\cdot)$ to be the function $f(\cdot)$ that satisfies $f(f(x)) = g(x)$. The square root of the sine function with respect to function composition has been discussed previously on MO on a number of occasions. For instance, here the formal power series is considered.
I wonder whether the functional square root of the sine also has an infinite product representation. If not, has any research been done on this question?
REPLY [4 votes]: Functional square root of sine is not unique, and it cannot be defined in the whole complex plane (according to a theorem of I. N. Baker). You can define it on $(0,\pi)$ with $f(0)=0$, for example. The resulting function is analytic in the component of Fatou set of sine adjacent to zero on the right (see, for example the first figure
here). But this function $f$ has no zeros in its domain, therefore a representation as infinite product does not make much sense. I mean that $\log f$ can be defined and any breaking of this log into summands gives you an infinite product representation. Of course this answer is related to a true product,
it does not exclude that there is some "formal" product representing this
square root, whatever a "formal product" may mean.<|endoftext|>
TITLE: "Sub-logarithmic" zero-free regions from Deuring-Heilbronn/Linnik's repulsion theorem
QUESTION [6 upvotes]: For each $n\in\mathbb{N}$, let:
$\chi_n\pmod{q_n}$ a real non-principal Dirichlet character ($q_1 < q_2 < \cdots$),
$\beta_n$ the largest real zero of $L(s,\chi_n)$,
$\delta_n := (1-\beta_n)\log(q_n)$.
Let $\chi\pmod{q}$ be a Dirichlet character, and consider $s = \sigma + it$ with $|t| < 1$. In p. 206 of Heath-Brown's "Prime twins and Siegel zeros", it is mentioned that the Deuring-Heilbronn phenomenon implies that there is some absolute constant $C>0$ such that, for each $n\in \mathbb{N}$, the region
$$ \bigg\{ s ~\bigg|~ \sigma \geq 1 - \frac{C\log(\delta_n^{-1})}{\log(q)},\ |t| < 1 \bigg\} $$
has no zeros (besides $\beta_1,\ldots,\beta_n$) of $L(s,\chi)$. (At least, that is how I interpreted the assertion "$r_0 \gg L^{-1}\log \eta$" at the mentioned page). Assuming this statement, it follows that:
(Sub-logarithmic zero-free regions (ZFR)) If there exists a sequence of Siegel zeros $\beta_n$ with $\delta_n \to 0$, then all the other zeros $\sigma + i\gamma$ of $L(s,\chi)$ with $|\gamma| < 1$ for Dirichlet characters $\chi\pmod{q}$ satisfy:
$$ \sigma < 1 - \frac{1}{o(\log(q))}. $$
It appears to me that this (or slight variations of this) statement is often used in the literature [the only example I have in mind at the moment is Remark 1 in p. 515 (p. 6 in the link) of Granville & Stark's $ABC$ implies no "Siegel zeros" for $L$-functions of characters of negative discriminant, where it is mentioned that $\delta_n \to 0$ implies $\frac{L'}{L}(1,\chi_n) = (1-\beta_n)^{-1} + o(\log(q_n))$].
However, I am having trouble following the deduction of these "sub-logarithmic ZFRs" from the Deuring-Heilbronn phenomenon alone. Using the Deuring-Heilbronn (Linnik's repulsion theorem) as in Théorème 16, Sec. 6 of Bombieri's "Le grande crible", we get that there are absolute constants $c_1,c_2 >0$ such that, fixing $n\in\mathbb{N}$, it holds:
$$ \sigma < 1 - c_1\log\left(c_2\frac{\delta_n^{-1}}{\log(q_n q)/\log(q_n)}\right) \cdot \frac{1}{\log(q_n q)} $$
(I am just taking $T = q_n q$ in Théorème 16). Assuming we have an infinite sequence $q_n \to +\infty$ with $\delta_n \to 0$, for a given $q\in\mathbb{N}$ we may take $q_k \leq q < q_{k+1}$, so that $\log(q_{k+1} q)/\log(q_{k+1})< 2$, and hence:
$$ \sigma < 1 - c_1\frac{\log(\frac{c_2}{2}\delta_{k+1}^{-1})}{\log(q_{k+1} q)}. $$
It appears, then, that to derive the "sub-logarithmic" ZFRs, it is necessary to have $\log(q_{k+1}) \ll \log(q_k)$ as $k\to \infty$, i.e.: the gaps between the conductors of consecutive exceptional characters need to be polynomially bounded, even if we assume $\delta_n \to 0$.
I do not think my conclusion is correct (e.g., I believe I misinterpreted some aspect of Heath-Brown's paper), but I have not been able to get rid of this condition on the growth of the $q_n$. In short, my question is the following:
Q. Is it really possible to derive "sub-logarithmic" ZFRs from Linnik's repulsion theorem (i.e., without additional growth conditions on the $q_n$)?
REPLY [4 votes]: Let $\chi$ be a non-principal real Dirichlet character modulo $q$. Let
$$\beta_0=1-\frac{1}{\eta\log q}$$
be a real zero of $L(s,\chi)$ satisfying $\eta\geq 100$ for convenience (Heath-Brown's condition is $\eta\geq 3$). Let $\rho=\beta+i\gamma$ be any zero of $L(s,\chi)$ such that $\rho\neq\beta_0$ and $|\gamma|\leq 1$. We strengthen Heath-Brown's claim $r_0\gg L^{-1}\log\eta$ to (note that $L=\log q)$
$$1-\beta\gg\frac{\log\eta}{\log q}.$$
We shall deduce this from Theorem 2 of Jutila's 1977 paper "On Linnik's constant", which is also Heath-Brown's reference. Let us write $\delta$ for the left hand side. If $\delta>1/60$, then the statement is trivial by $\eta\ll q$. So we shall assume that $\delta\leq 1/60$. Then, Jutila's theorem yields readily (using $D\leq 2q$) that
$$1-\beta_0\geq\frac{1}{10}\cdot\frac{q^{-3\delta}}{\log q}.$$
In other words, $\eta\leq 10 q^{3\delta}$. By the assumption $\eta\geq 100$, this implies that $\eta\leq q^{6\delta}$, or equivalently that
$$\delta\geq\frac{1}{6}\cdot\frac{\log\eta}{\log q}.$$
We have verified Heath-Brown's claim.<|endoftext|>
TITLE: Hecke algebra relation versus $\operatorname{SL}_2$ trace relation
QUESTION [7 upvotes]: The quadratic relation in the (type $A$) Hecke algebra is $(T-t)(T+t^{-1}) =0$, which can be rewritten as
$$
T-T^{-1} = t-t^{-1}$$
Suppose $A \in \operatorname{SL}_2(\mathbb{Q})$ with eigenvalues $a,a^{-1}$. Then it isn't hard to show the following identity $$A + A^{-1} = (a+a^{-1}) \mathrm{Id}$$
If $i^2=-1$ then we can "convert" between these two formulas by saying $T=iA$ and $t = ia$.
Is there a conceptual reason for this?
REPLY [3 votes]: Interesting observation! Unfortunately, this seems to be a coincidence.
The original, algebraically motivated definition of the Hecke algebra gives the quadratic relation as
$$(T-q)(T+1)=0.$$
For $SL_2$, the Hecke algebra has a faithful two dimensional representation. The eigenvalues of $T$ are $q$ and $-1$, and the quadratic relation simply becomes the Cayley--Hamilton theorem.
But any 2x2 matrix $A$ also satisfies its characteristic equation
$$(A-\lambda_1)(A-\lambda_2)=0,$$
where $\lambda_1,\lambda_2$ are the eigenvalues of $A$. If $A$ is nonsingular, then the rescaling
$A\mapsto \sqrt{\lambda_1\lambda_2}A$ transforms this into
$$(A-a)(A-a^{-1})=0,$$
where $a=\sqrt{\frac{\lambda_1}{\lambda_2}}$.
Thus your observation reduces to the fact that $i$ is a "nice" value of $\sqrt{\lambda_1\lambda_2}$ for the Hecke algebra. But in fact the niceness here is rather circular. As mentioned above, the more motivated definition of the Hecke algebra is not symmetric. To reach the modern presentation, we symmetrize the definition by replacing $T\mapsto q^{1/2} T$ to find (with $t=q^{1/2}$)
$$(T-t)(T+t^{-1}) =0. $$
The goal of this rescaling is to make this relation "nice", i.e. transforming nicely under $t\mapsto t^{-1}$. We could have alternatively rescaled $T\mapsto iq^{1/2}$ to get
$$(T-t)(T-t^{-1})=0. $$
These are the only two rescalings if we want the relation to be nice. Thus a priori the only possible values of $\sqrt{\lambda_1\lambda_2}$ for a nice Hecke algebra presentation are $1$ or $i$, explaining your observation. (We pick the $i$ presentation over the $1$ presentation to preserve the behavior of the Hecke algebra as $t\rightarrow 1$.)
Thus the fundamental reason for the similarity between the Hecke algebra presentation and the trace relation in $SL_2$ is that the Hecke algebra is two dimensional. But the dimension of the Hecke algebra is the size of the Weyl group $S_2$. The fact that $|S_2|=2=\mathrm{rank}(SL_2)$ is a coincidence.<|endoftext|>
TITLE: Parametrization of real-valued SU(N)
QUESTION [5 upvotes]: I want to construct a $SU(N)$ matrix $V$, with the following property:
All the elements of the first row are given, i.e. $V_{1,j}=a_i$ (with $\sum_i a_i^2=1$)
All matrix elements are real, i.e. $V_{i,j} \in \mathbb{R}$
How can I find a matrix $V$ that satifies the criteria? Specifically, how can I find the matrix elements as a function of $a_i$, i.e. $V_{i,j}(a_i)$?
Special case: SU(2)
$$
V=
\left[ {\begin{array}{cc}
a_1 & a_2 \\
V_{2,1} & V_{2,2} \\
\end{array} } \right]
$$
We easily find $V_{2,1}=-a_2$ and $V_{2,2}=a_1$.
Special case: SU(3)
$$
V=
\left[ {\begin{array}{cc}
a_1 & a_2 & a_3 \\
V_{2,1} & V_{2,2} & V_{2,3} \\
V_{3,1} & V_{3,2} & V_{3,3} \\
\end{array} } \right]
$$
Here already I cannot find any feasible way to represent $V_{i,j}$ as a function of $a_1, a_2, a_3$. I have tried to use the generators of SU(3), the Gell-Mann matrices $\lambda_i$. In particular, $\lambda_2$, $\lambda_5$, $\lambda_7$ are the generators for real-valued SU(3) matrices. However, the resulting equation system involves multiple trigonometric functions for which I cannot solve $V_{i,j}(a_1, a_2, a_3)$.
The matrix $V$ is not unique, I just want any solution.
REPLY [3 votes]: In addition to the comments I made above about continuous solutions, I thought I'd point out a solution that works for all $n$ with only one point of discontinuity, namely
$$
(a_1\ a_2\ \ldots\ a_n) = (1\ 0\ \ldots\ 0).
$$
Away from this point, one can start with the following formulae:
$$
V_{i,1}= V_{1,i} = a_i\qquad\text{and}\qquad
V_{i,j} = \delta_{ij} - \frac{a_ia_j}{(1-a_1)}\quad\text{when}\ 1
TITLE: Why is modular forms applicable to packing density bounds from linear programming at $n\in\{8,24\}$?
QUESTION [10 upvotes]: Sphere packing problem in $\mathbb R^n$ asks for the densest arrangement of non-overlapping spheres within $\mathbb R^n$. It is now know that the problem is solved at $n=8$ and $n=24$ using modular forms. I understand some sphere packing and issues going with it but my understanding is most upperbounds come from linear programming and the bounds that are currently proven optimal (including at $n=8$ and $n=24$) already come from linear programming bounds.
How do modular forms become part of the story that provide the lower bound (do they arise naturally from some packing related structure)?
Is there a bigger story that this is just a chapter of that may apply to other upper bounds generated from linear programming? What makes modular forms click for this class of linear programming bounds (perhaps Sphere packing and quantum gravity is of utility?)?
REPLY [12 votes]: This is a tough question, and I don’t think there’s a definitive answer yet. For some mathematical details, see the following survey articles:
https://arxiv.org/abs/1611.01685
https://arxiv.org/abs/1603.05202
Instead, I’ll focus on the big picture here. Why modular forms? I can see a couple of potential answers:
(1) Why not modular forms? Before Viazovska’s proof, numerical experiments indicated that there were remarkable special functions in 8 and 24 dimensions that would prove the optimality of $E_8$ and the Leech lattice. However, nobody had any idea how to construct them explicitly, or prove existence at all. Modular forms are by far the most important class of special functions related to lattices (in higher dimensions, since arguably trig functions and the exponential function are the most important special functions related to lattices), so lots of people expected that the magic functions for sphere packing should be connected somehow to modular forms. The proof had to wait for Viazovska to come up with a beautiful integral transform, but the fact that it used modular forms wasn’t such a great surprise. I.e., her contribution wasn’t the idea that modular forms should play a role, but rather figuring out how to use them, which was quite subtle and ingenious.
You’re right that nobody has any idea how to use modular forms to optimize the linear programming bound in other dimensions. However, it’s possible that they will continue to play a role. For example, see the example Felipe Gonçalves and I found at the end of Section 2.1 of our paper https://arxiv.org/abs/1712.04438 (which is not a sphere packing bound, but closely related). It really looks like a small perturbation of a function based on modular forms (see https://arxiv.org/abs/1903.05737), and I wouldn’t be surprised if the optimal function has a nice series expansion based in some way on modular forms. From this perspective, the remarkable thing about 8 and 24 dimensions wouldn’t be the appearance of modular forms, but rather the fact that the series collapses to a single term, with a matching sphere packing. However, this is all speculative.
(2) The other perspective is that we have very little understanding of why 8 and 24 dimensions are special in the first place. For example, why shouldn’t sphere packing in 137 dimensions also admit an exact solution via linear programming bounds? It sure doesn’t look like it does, but perhaps we just don’t know the right sphere packing to use, and some currently unknown packing might match the upper bound. That would be very surprising, since our experience is that exceptional phenomena occur in clusters. We would expect to see some sort of remarkable symmetry group, probably a finite simple group, and there aren’t any candidates acting on 137 dimensions. However, this expectation is just based on our limited experience, and mathematics can confound our expectations. So far, nobody has found even a convincing heuristic argument for why there shouldn’t be an exact solution in 137 dimensions, and that’s a major gap in our understanding. The most we can say is that it would have to differ in some important ways from 8 and 24 dimensions, which is far from an explanation of why it can’t happen.
I guess I’d summarize it like this. If you accept that lattices in 8 and 24 dimensions play a special role, then modular forms feel naturally connected. However, we’re missing a deeper explanation of the role of these special dimensions.
Let me add one more specific mathematical comment. The magic functions in 8 and 24 dimensions fit into a general picture of building radial functions that vanish on all but finitely many vector lengths in a lattice, and whose Fourier transforms vanish on all but finitely many vector lengths in the dual lattice. If you can do this in full generality, then Poisson summation lets you solve for the number of lattice vectors of each length. These are the coefficients of a modular form, namely the theta series of $E_8$ or the Leech lattice, so the conclusion is that this family of functions somehow “knows about” the theta series. In other words, you can’t expect to construct the whole family without running into modular forms in some way. This leaves a couple of possibilities: maybe the magic functions for sphere packing are simpler than most functions in this family, and could be constructed without modular forms, or maybe these functions are deeper than modular forms (and require some mysterious special functions not yet known to mathematicians). What we know now is that modular forms suffice, and in a sense are necessary because the magic functions are unique.<|endoftext|>
TITLE: How flexible is the infinite-dimensional torus?
QUESTION [14 upvotes]: Let $\mathbb T=\mathbb R/\mathbb Z$ be the circle group and $\mathbb T^\omega$ be the infinite-dimensional torus, considered as an abelian compact topological group.
Problem 1. Is it true that for any finite set $F\subset\mathbb T^\omega$ and any neighborhood $U\subseteq \mathbb T^\omega$ of zero there exists an automorphism $\alpha$ of $\mathbb T^\omega$ such that $\alpha(F)\subset U$?
This problem can be reformulated in the language of the special linear groups $SL(n,\mathbb Z)$.
Problem 2. Is it true that for any $n\in\mathbb N$, neighborhood of zero $U$ in $\mathbb R^n$ and vectors $x_1,\dots,x_n$ in $\mathbb R^\omega$ there exists $m>n$ and a matrix $A\in SL(m,\mathbb Z)$ such that $\mathrm{pr}_n\circ A\circ \mathrm{pr}_m(x_i)\in U$ for all $i\in\{1,\dots,n\}$?
Here $\mathrm{pr_k}:\mathbb R^\omega\to\mathbb R^k$, $\mathrm{pr}_k:x\mapsto x{\restriction}k$, is the projection onto the first $k$ coordinates.
Remark 1. For any field $\mathbb F$ and vectors $x_1,\dots,x_n\in\mathbb F^{2n}$ there exists a linear transformation $A\in SL(2n,\mathbb F)$ of $\mathbb F^{2n}$ such that $A(\{x_1,\dots,x_n\})\subset\{0\}^n\times\mathbb F^n$.
Remark 2. For any vector $(x,y)\in\mathbb R^2$ and any $\varepsilon>0$ there exists a matrix $A\in SL(2,\mathbb Z)$ such that $(x,y)\cdot A\in (-\varepsilon,\varepsilon)\times\mathbb R$. Such matrix $A$ can be constructed by finding relatively prime integer numbers $p,q$ such that $|xp+yq|<\varepsilon$ and then finding integer numbers $a,b$ such that $pb-qa=1$ (using the extended Euclidean algorithm). Then the matrix $A=\left(\begin{array}&p&a\\q&b\end{array}\right)\in SL(2,\mathbb Z)$ has the required property.
Taking into account Remarks 1 and 2, I would expect that the following stronger form of Problem 2 has an affirmative answer.
Problem 3. Is it true that for any $n\in\mathbb N$ and vectors $x_1,\dots,x_n\in\mathbb R^{2n}$ there exists a linear transformation $A\in SL(2n,\mathbb Z)$ such that $A(\{x_1,\dots,x_n\})\subset(-\varepsilon,\varepsilon)^n\times\mathbb R^n$?
REPLY [4 votes]: This is a draft proof of an affirmative answer to Problem 3.
Proposition. For any $n\in\mathbb N$, $\varepsilon>0$ and vectors $x_1,\dots,x_n\in\mathbb R^{2n}$ there exists a linear transformation $A\in SL(2n,\mathbb Z)$ such that $A(\{x_1,\dots,x_n\})\subset[-\varepsilon,\varepsilon]^n\times\mathbb R^n$.
Proof. Let $m=2n$, $x_i=(x_{i1},\dots,x_{im})$ for each $i$ and $X=\|x_{ij}\|$. We shall call a column
of a matrix small, provided all its entries have absolute value at most than $\varepsilon$, and big, otherwise. Let $k$ be the maximal number of small columns in a matrix $XB$, where $B\in SL(m,\mathbb Z)$ and $C$ be an arbitrary matrix in $SL(m,\mathbb Z)$ such that $XC$ has $k$ small columns. It suffices to show that if $k(2lM)^n$ and define a map $f$ from the subset $Q^l$ of points of the set $[-K, K]^l$ with all integer coordinates to $\mathbb R^n$ by putting $f(d)=d_1y_1+\dots + d_ly_l$ for each $d=(d_1,\dots,d_l)\in Q^l$. Since all $|d_i|\le K$ and all entries of columns $y_i$ have absolute value at most $M\varepsilon$, each coordinate of a vector $f(d)$ is at most $lKM\varepsilon$. Therefore the image $f(Q^l)$ can be covered by $(2lKM)^n$ axis-parallel cubes with side $\varepsilon$. Since $|Q^l|=(2K+1)^l>(2K)^{n+1}>(2lKM)^n$, there exist two distinct elements $d’$ and $d’$ of $Q^l$ such that each coordinate of a vector $y=f(d’)-f(d’’)$ has an absolute value at most $\varepsilon$. Put $d=d’-d’’$. Dividing entries of $d$ by their greatest common divisor, if needed, we can assume that the greatest common divisor of entries of $d$ is $1$. It is well-known (see, for instance, this MSE thread]) that there exists a matrix $D’\in SL(l,\mathbb Z)$ whose first column is $d$. Put $D=\begin{pmatrix} D’ & 0\\ 0 & I\end{pmatrix}$, where $I$ is the $(m-l)\times (m-l)$ identity matrix. Then the first column of the matrix $XCD$ is small, whereas its last $k$ columns are the same as in the matrix $XC$. $\square$<|endoftext|>
TITLE: Non-polynomial splines, a non-linear problem
QUESTION [5 upvotes]: I'm looking for references on how to construct spline-like functions from a basis that does not include piecewise polynomials.
To be specific, given a class of functions such as "decaying exponentials" or "sines and cosines" (which are parameterized by a single parameter, e.g. the decay rate or the frequency), is there an efficient and numerically stable method to construct a function that is piecewise a linear combination of $N$ such functions (whose parameters are to be determined), interpolates given data $(x_k,f_k)$ [which are assumed to be such that they can be interpolated using the given function class, i.e. e.g. monotonically decreasing for decaying exponentials] and has continuous derivatives at $x_k$ up to order $2N-2$ (in order to fix $N$ linear coefficients and $N$ non-linear parameters)?
I can of course write down the explicit equations needed to satisfy these conditions, but directly solving those using a non-linear equation system solver does not look too promising as an approach.
What literature I could find so far on splines with non-polynomial components referred to spaces spanned by polynomials and some given non-polynomial. Here I'm looking for the case where there are no polynomials and the non-polynomial functions are parameterized by a parameter whose values are to be determined by the interpolation and smoothness conditions.
REPLY [2 votes]: The interpolant
$$A\left(e^{\frac{a}{A}\xi}-1\right)+B\cdot\left(\cosh\left(\frac{a}{A}\xi\right)-1\right)\ =\ (A+B)\mathbf{e^{\frac{a}{A}\xi}}+B\cdot \mathbf{e^{-\frac{a}{A}\xi}}-(A+B)\\
a=\frac{d}{dx}S(x_i),\\
\xi=x_{i+1}-x_i,\\
\eta=y_{i+1}-y_i,\\
B=\frac{\eta-A\left(e^{\frac{a}{A}\xi}-1\right)}{\cosh\left(\frac{a}{A}\xi\right)-1}$$
The construction of the spline $S(x)$ happens from left to right and requires knowldege of the slope at $x_0$ to be able to calculate the Interpolant that connects $(x_0,y_0)$ with $(x_1,y_1)$; knowing the interpolant we can determine the slope at $x_1$ and we are in the same situation as before so that eventually $S(x)$ is determined.
The underlying ideas that led to identifying the interpolant for $C^1$ continuous interpolation can most likely be generalized to higher degrees of smoothness but I'm still on my way.
Addendum:
parameter $A$ provides limited control over the shape of the interpolant:
if we want the "purest" expontial functions, the $A$ should be chosen to minimize $B$
if we strive for preserving shape we can chose $A$ to control the abscissa of the (unique) local extremum in cases of $y_i\lt y_{i+1}\land y_{i+2}\lt y_{i+1}$
align it with the vertex of the parabola through $\left(\,(x_i,y_i),\,(x_{i+1},y_{i+1}),\,(x_{i+2},y_{i+2})\,\right)$
align it with the abscissa of the intersection of the line with slope $a$ that contains $(x_i,y_i)$ and the line through $(x_{i+1},y_{i+1})$ and $(x_{i+2},y_{i+2})$<|endoftext|>
TITLE: What's an illustrative example of a tame algebra?
QUESTION [7 upvotes]: A finite-dimensional associative $\mathbf{k}$-algebra $\mathbf{k}Q/I$ is of tame representation type if for each dimension vector $d\geq 0$, with the exception of maybe finitely many dimension vectors $d$ representations*, the indecomposable representations of $\mathbf{k}Q/I$ with that dimension vector can be described up to isomorphism as finitely many one-parameter families, the parameter coming from $\mathbf{k}$.
What is an illustrative example of a tame algebra? Specifically, what's an example of a quiver $Q$ and admissible ideal $I$ such that (1) for some dimension vectors $d$ the indecomposables can't be described as finitely many one-parameter families*, and (2) for some dimension vectors there is more than just one one-parameter family.
I ask because my go-to example of a tame algebra now is the path algebra of the Jordan quiver, the quiver having one vertex and one loop, over an algebraically closed field. But this example doesn't utilize all the wiggle-room that the definition of a tame algebra allows. So I'm hoping there is a better quintessential example to keep in mind.
* Note that, as it was originally written, this was not the correct definition of an algebra having tame representation type, and actually condition (1) is not possible. See the comments below for the correct definition.
REPLY [6 votes]: I think the following is an example of a tame algebra where there is more than one component to a moduli space of fixed dimension. I don't know any examples where there are dimension vectors with moduli of dimension $>1$.
Take a quiver with two vertices $1$ and $2$, two arrows $x_1$ and $x_2$ from $1$ to $2$ and two arrows $y_1$ and $y_2$ from $2$ to $1$. Impose the relations $x_i y_j = 0$ and $y_j x_i = 0$ for $1 \leq i,j \leq 2$. I believe every indecomposable representation either satisfies $x_1=x_2=0$ or $y_1=y_2=0$. Thus, every indecomposable representation is a representation of either the Kronecker quiver $1 \rightrightarrows 2$ or else $1 \leftleftarrows 2$, both of which are tame, so this is tame.
For each dimension vector of the form $(n,n)$, we get two families of indecomposable representations, coming from choosing whether to make $x_1=x_2=0$ or else $y_1=y_2=0$.
REPLY [5 votes]: For the Kronecker quiver (two vertices, two arrows in the same direction) and dimension vector (1,1), over an algebraically closed ground field, the indecomposables are naturally parameterized by points in $\mathbb P^1(k)$. (The representation with the two maps given by $a$ and $b$ is sent to $[a:b]$.
For other tame quivers with no relations over an algebraically closed ground field, the situation is slightly worse: the natural indexing set for the representations whose dimension vector is the null root is $\mathbb P^1(k)$ with some points (up to three of them) counted more than once (but finitely many times). This happens in the example Bugs gave: there are three inhomogeneous tubes, each of width two, each containing two representations of dimension vector the null root, whereas the other points of $\mathbb P^1(k)$ each correspond to one representation. (With the all-inward orientation, the reason for the indexing by $\mathbb P^1(k)$ is that the moduli space of 4 points on $\mathbb P^1$—equivalent to representations with dimension vector $(1,1,1,1,2)$, i.e., the null root—is again $\mathbb P^1$.)
I am not quite sure what you mean by the extra wiggle room of type (1). Are these supposed to be dimension vectors that have only finitely many indecomposables? I would usually think that in that case, they can also be described by one-parameter families: just make the families constant.<|endoftext|>
TITLE: Fermat's Last Theorem for integer matrices
QUESTION [41 upvotes]: Some years ago I was asked by a friend if Fermat's Last Theorem was true for matrices. It is pretty easy to convince oneself that it is not the case, and in fact the following statement occurs naturally as a conjecture:
For all integers $n,k\geq 2$ there exist three square matrices $A$, $B$ and $C$ of size $k\times k$ and integer entries, such that $\det(ABC)\neq 0$ and:
$$A^n + B^n = C^n$$
Of course, the case $k=1$ is just Fermat's Last Theorem, but in that case the conclusion is the opposite for $n>2$.
I think that I read somewhere that it is known that the above assertion is true (I do not remember exactly where, and haven't seen anything on Google, but this old question on MSE, on which there is an old reference, that I think does not answer this).
Two observations that are pretty straightforward to verify are the following: the case $2\times 2$ and $3\times 3$ solve the general case $k\times k$ by putting suitable small matrices on the diagonal.
Also, as it is stated in a comment on that question, if the exponent $n$ is odd then the case $2\times 2$ can be solved by this example:
$$\begin{bmatrix} 1 & n^\frac{n-1}{2}\\ 0 & 1\end{bmatrix}^n + \begin{bmatrix} -1 & 0\\n^\frac{n-1}{2} & -1\end{bmatrix}^n = \begin{bmatrix} 0 & n\\1& 0\end{bmatrix}^n$$
Does anybody know of such examples in the $2\times 2$ case for even $n$, and the general $3\times 3$ case?
More clearly: are there easy and explicit examples for each $n$ and $k$ for the above conjecture?
REPLY [18 votes]: This problem is addressed in "On Fermat's problem in matrix rings and groups," by Z. Patay and A. Szakács, Publ. Math. Debrecen 61/3-4 (2002), 487–494, which summarizes previous work on the topic and gives some new results. It seems that the problem is not completely solved.
When $k=2$, Khazonov showed that there are solutions in $SL_2(\mathbb Z)$ if and only if $n$ is not a multiple of 3 or 4, but I couldn't immediately find any statement anywhere about the case $4\mid n$ and $2\times 2$ integer matrices with nonzero determinant.
Khazanov also proved that $GL_3(\mathbb Z)$ solutions do not exist if $n$ is a multiple of either 21 or 96, and $SL_3(\mathbb Z)$ solutions do not exist if $n$ is a multiple of 48.
Patay and Szakács give explicit solutions for $SL_3(\mathbb Z)$ when $n=\pm 1\pmod 3$ as well as for $n=3$. Here's a solution for $n=3$:
$$\pmatrix{0& 0&1\\ 0 &-1& 1\\ 1 & 1 & 0}^3 + \pmatrix{0&1&0\\ 0&1&-1\\ -1&-1&0}^3 = \pmatrix{0&1&1\\0&0&1\\1&0&0}^3.$$<|endoftext|>
TITLE: Curious identity between the two kinds of Chebyshev polynomials
QUESTION [14 upvotes]: I have found, by accident, an identity that relates a sum of Chebyshev polynomials of the first kind to a Chebyshev polynomial of the second kind. It goes as follows:
Given an integer partition of $n$, let $g_a$ be the number of times $a$ appears in said partition.${}^1$ Then the following identity holds for all $n \in\mathbb{N}$:
$$
U_n(x) = \sum_{\substack{n_i>0\\ \sum_i n_i = n}} \frac{1}{\prod_{a\in \{n_i\}} g_a!} \prod_{i=1} \frac{2}{n_i} T_{n_i}(x)\,.
$$
The sum is over all integer partitions of $n$, the product is on all $n_i$'s in the partition, with repetitions.
I have a very roundabout way to prove this identity (I'll skip the details). The left hand side is obtained by contracting two symmetric traceless tensors of $SO(4)$. That is, letting $|x|=|y|=1$ and $x,y\in \mathbb{R}^4$ then
$$
(x^{i_1}\cdots x^{i_n} - \mathrm{traces}) (y_{i_1}\cdots y_{i_n} - \mathrm{traces}) \propto U_n(x\cdot y)\,.
$$
The right hand side instead comes from the same contraction but in spinor notation. Namely we let
$$
\mathrm{x} = \left(\begin{matrix}x_3-x_4 & x_1 - i x_2 \\ x_1 + i x_2 & -x_3-x_4\end{matrix}\right)\,,\quad \bar{\mathrm{x}} = \epsilon\, \mathrm{x}\, \epsilon^T\,,
$$
and $\mathrm{y}$ in a similar way ($\epsilon$ is the Levi Civita tensor). Then we introduce two dimensional spinors $\eta,\tilde{\eta}$ let $\partial_{\eta^\alpha}\eta^\beta = \delta_\alpha^\beta$ (similar for $\tilde{\eta}$) and finally
$$
(\partial_\eta \mathrm{x} \partial_{\tilde{\eta}})^n (\eta \mathrm{y}\tilde{\eta})^n \sim \sum_{\substack{n_i>0\\ \sum_i n_i = n}} \# \prod_{i} \mathrm{tr}\,((\mathrm{x}\bar{\mathrm{y}})^{n_i})\,.
$$
The sum over partitions comes from a combinatoric argument. Then it's a simple exercise to show that $\mathrm{tr}\,((\mathrm{x}\bar{\mathrm{y}})^n) \propto T_n(x\cdot y)$.
My questions are
Is this identity known already?
If not, could you come up with some more direct argument to prove it?
$\;{}^1$ For example, $(1,1,1,2,2,3)$ is an integer partition of $n=10$ with $g_1 =3,\, g_2=2,\,g_3=1$.
REPLY [5 votes]: I think I can sketch a shorter proof.
Let $z_j = x_j+x_j^{-1}$, and let $p_m$ and $h_m$ denote the power-sum and complete homogeneous symmetric polynomial.
Then (see e.g p.3 in this preprint)
$$
2 T_m(z_j/2) = p_m(x_j,x_j^{-1})
\text{ and }
U_m(z_j/2) = h_m(x_j,x_j^{-1})
$$
Now, we can use the Newton identities, to express $h_m$
in terms of the power-sum symmetric functions.
This gives a relation between the $U_m$ and the $T_m$.
Looking at your formula, it is very similar to the Newton identity.<|endoftext|>
TITLE: Resolution graphs in the sense of Némethi
QUESTION [7 upvotes]: The following definitions are from lecture notes of Némethi. A surface singularity $(X,0)$ is defined by $$(X,0) = (\{ f_1 = \ldots = f_m=0 \}) \subset \mathbb (\mathbb{C}^n,0),$$ where $f_i : (\mathbb{C}^n ,0) \to (\mathbb{C},0)$ are germs of analytic functions with $$r(p) = \mathrm{rank} \left [ \frac{\partial f_i}{\partial z_i} (p) \right ]_{i=1, \ldots, m; j=1, \ldots, N} = N-2$$ for any generic or smooth point $p$ of $X$.
If $r(0) = N-2$, then $(X,0)$ is analytically isomorphic to $\mathbb (\mathbb{C}^2,0)$. The singularity $(X,0)$ is called normal, if any bounded holomorphic function $f: X - \{ 0\} \to \mathbb C$ can be extended to a holomorphic function defined on $X$.
Then there is an algorithm in Appendix 1 for finding the resolution graphs of singularities in $\mathbb{C}^3$ with equation of the form $g(x,y) + z^n$ (i.e. suspensions of curve singularities in $(\mathbb C^2,0)$).
I couldn't understand the steps of that algorithm. My question is that
Is there anyone to describe this algorithm for example Brieskorn sphere $\Sigma(2,3,4)$?
Is it possible to find any Magma or Sage code for this purpose?
REPLY [3 votes]: About question 2: I think that the software Singular has this feature; it's well-documented, and if you look for resolution graph you should find the reference.
About question 1: well, I must admit that that algorithm is not pleasant, and that it took me a while to work out the case of $\Sigma(2,3,4)$.
Anyway, here we go. I can't draw graphs here (but if anyone can tell me that we can embed \xygraphs, next time I can make an effort), so things will have to be less pictorial than I'm comfortable with. I'll use greek letters for vertices to avoid clashing with Némethi's letters. So, we fix $g(x,y) = x^2+y^3$ and $n=4$.
First off, the resolution graph of $g$ has three vertices $\alpha, \beta, \gamma$, and an arrowhead $\delta$. The self-intersections are $e_\alpha = -3$, $e_\beta=-2$, $e_\gamma = -1$. ($\gamma$ is the central vertex, corresponding to the last blow-up, $\alpha$ was the first blow-up, and $\beta$ the second.)
From the formula (*) in the appendix, relating multiplicities with self-intersections, and using that $m_\delta = 1$, we obtain that $(m_\alpha, m_\beta, m_\gamma) = (2,3,6)$. We the compute $(d_\alpha, d_\beta, d_\gamma) = (2,1,3)$.
Step 1(a): from the computations above, $\alpha$ is covered by two vertices of multiplicity 1, while $\beta$ and $\gamma$ are covered by one vertex of multiplicity 3 each. Call them $\tilde\alpha_1, \tilde\alpha_2, \tilde\beta, \tilde\gamma$.
The genus formula says that they're all rational ($\tilde g = 0$).
Step 1(b): the edge $(\alpha,\gamma)$ lifts to strings connecting $\tilde\alpha_i$ to $\tilde\gamma$, each of type $G(1,3,2)$; which means just a single vertex $\varepsilon_i$ with $(e_{\varepsilon_i}, m_{\varepsilon_i}) = (-2,2)$ (and two arrowheads ending at $\tilde\alpha_i$ and $\tilde\gamma$). The edge $(\beta,\gamma)$ lifts to a string of type $G(6,3,4)$, which is a single vertex $\zeta$ with $(e_{\zeta}, m_{\zeta}) = (-2,3)$ (and two arrowheads ending at $\tilde\beta$ and $\tilde\gamma$.
Step 1(c): the arrowhead $\delta$ lifts to a string of type $G(6,1,4)$, which is again a single vertex $\eta$ with $(e_{\eta}, m_{\eta}) = (-2,2)$ (and two arrowheads, one ending at $\tilde\gamma$ and the other one free).
At this point, the graph is star-shaped with four legs: its center is at $\tilde\gamma$ (with weight unspecified); two legs with weights $(-2,?)$ (ending at $\tilde\alpha_1, \tilde\alpha_2$), one with weights $(-2,?)$ (ending at $\tilde\beta$), one with weight $-2$ and an arrowhead.
Step 2: we now compute the missing self-intersections using (*). We obtain $e_{\tilde\alpha_i} = -2$, $e_{\tilde\beta} = e_{\tilde\gamma} = -1$.
Step 3: we drop the arrowhead, and blow down $\tilde\gamma$ and then $\zeta$ (which, after blowing down $\tilde\gamma$, can be blown down). The graph we obtain is $E_6$, as expected.<|endoftext|>
TITLE: Gaussian measure on function spaces
QUESTION [6 upvotes]: I'm reading this classic work and I'd like to get deeper inside some of its techniques. In particular, the authors state: "We construct a Gaussian measure $d\mu_{0}(\phi)$ on a measure space of continuous functions $\phi(x), x\in \Lambda \subset \mathbb{R}^{3}$ with covariance $u$:
\begin{eqnarray}
\int d\mu_{0}(\phi)e^{i\int f\phi} = e^{-\frac{1}{2}\int f u f} \tag{1}\label{1}
\end{eqnarray}
It is then straightforward to show that:
\begin{eqnarray}
e^{-\beta U} = \int d\mu_{0}(\phi) e^{i\sqrt{\beta}\sum_{\alpha}e_{i(\alpha)}\phi(x_{\alpha})}" \tag{2}\label{2}
\end{eqnarray}
First of all, how to construct such a Gaussian measure $d\mu_{0}$ on a space of continuous functions? Is it defined by condition (\ref{1}) or does (\ref{1}) follow as a consequence? Besides, how can we prove existence? Does anyone know any reference on this construction?
Second, equation (\ref{2}) seems to follow by taking $f = \sum e_{i(\alpha)}\delta(x_{\alpha})$. But how can we take such an $f$ is $f$ must be a continuous function rather than a distribution?
REPLY [4 votes]: You should have a look at the book by Gelfand and Vilenkin
Generalized functions. Vol. 4: Applications of harmonic analysis
where they describe how to construct Gaussian measures on (duals) of nuclear spaces.
Thus, given an open set in $\newcommand{\bR}{\mathbb{R}}$ $D\subset \bR^n$ one begins by constructing a measure on the space $C^{-\infty}(D)$ of generalized functions on $D$. If the covariance kernel is sufficiently regular then this measure is concentrated one on a much smaller subspace.
Also, if you read French, I recommend this 1967 paper by Xavier Fernique. It is not the most comprehensive but I found it very helpful.
Finaly, there is V. Bogachev's book Gaussian Measures.<|endoftext|>
TITLE: Distribution of signs of automorphic forms
QUESTION [6 upvotes]: Let's say we have an automorphic form $f$ on $GL(2)$ that is self-dual. In particular, the associated L-function $L(s,f)$ satisfies a functional equation with sign $\varepsilon_F = \pm 1$.
Is it known that the proportion of such automorphic forms with given sign (say $-1$) is exactly $1/2$?
I know many results about distributions of signs for coefficients and eigenvalues of automorphic forms, however when I think of this question I wonder whether it is well-known or difficult?
REPLY [8 votes]: For simplicity, let's consider the case of holomorphic modular forms over $\mathbb Q$ of squarefree level and trivial nebentypus. Then one knows from
Iwaniec, Henryk; Luo, Wenzhi; Sarnak, Peter. Low lying zeros of families of $L$-functions. Publications Mathématiques de l'IHÉS, Tome 91 (2000) pp. 55-131.
an asymptotic formula for the dimensions of the subspaces of cusp forms with root number $+1$ and root number $-1$. In particular, the proportion of forms with root number $+1$ tends to $\frac 12$ as you take some combination of the weight and the level to infinity.
As Peter mentions in the comments, I also consider this in my paper
Refined dimensions of cusp forms, and equidistribution and bias of signs. Journal of Number Theory, Vol. 188 (2018), 1-17.
Specifically, I get an exact formula for the dimensions of subspaces with prescribed root number (or prescribed Atkin-Lehner signs), and observe a "strict bias" phenomenon for root number +1: while the proportion of newforms with root number +1 is asymptotically $\frac 12$, in any given space, there are always at least as many newforms with root number +1 as with -1, and it is strictly greater except in a few special situations.
One should similarly be able to prove that the proportion is $\frac 12$ in more general families of automorphic forms, say using a trace formula and simply bounding error terms, though I don't know the most general situation in which this has already been done in the literature. Essentially one needs to know that the trace of the Fricke involution is not large compared to the dimension.<|endoftext|>
TITLE: Foliation of $\mathbb R^n$ by connected compact manifolds
QUESTION [7 upvotes]: Does there exist a smooth nontrivial fiber bundle $p: F \hookrightarrow \mathbb R^n \to B$ such that $F$ and $B$ are connected manifolds with $F$ compact? "Nontrivial" here means the fiber $F$ is not a point.
REPLY [14 votes]: On the other hand, if you only mean "foliation" as in your title, and not "fibration", then there is Vogt's foliation of R^3 by circles! (But it is not C^1, only differentiable).
Vogt, Elmar, "A foliation of R3 and other punctured 3-manifolds by circles",
Publications Mathématiques de l'IHÉS, Tome 69 (1989), p. 215-232
http://www.numdam.org/item/PMIHES_1989__69__215_0/<|endoftext|>
TITLE: Area of a elliptic surface confined by a sphere
QUESTION [5 upvotes]: Let $S$ be a surface enclosed inside the unit sphere in $R^3$. If every point of S is elliptic, then must $\operatorname{Area}(S)≤\operatorname{Area}(S^2)$?
REPLY [4 votes]: I do not think so. Just imagine that you peel a large orange whose surface area is much bigger than that of a unit sphere. Then you "spiral" the peel to make it arbitrarily small and place it inside a unit sphere. Imagine a surface that looks more or less as this one:
It has positive curvature so very point is elliptic.
Here you remove a small cylinder around the vertical axis so there is no problems with the curvature.
Since you can have as many "turns" as you want, its surface area can be arbitrarily large while the surface occupies a small region in space.<|endoftext|>
TITLE: Positively curved metric with uniformly positive scalar curvature
QUESTION [5 upvotes]: Can we find a complete noncompact Riemannian manifold $(M^n,g)$ with bounded geometry satisfying the following conditions?
the curvature operator $Rm>0$;
the scalar curvature $R \ge 1$.
Notice that any such manifold must be diffeomorphic to $\mathbb R^n$.
REPLY [4 votes]: Yes, this is possible. Note that a strictly convex hypersuface in $\mathbb R^{n+1}$ has positive $Rm$.
To get an example, consider the following graph $H\subset \mathbb R^{n+1}$ over the open unit $n$-disk in $\mathbb R^n$:
$$H:=\left(x_{n+1}=\frac{1}{1-\sum_{i=1}^n{x_i^2}}\right).$$
Clearly, $H$ is convex. Moreover, the scalar curvature of $H$ tends to the scalar curvature of the unit $n-1$-sphere as $x_{n+1}$ tends to infinity. In particular the scalar curvature is greater than a certain positive $c>0$ on $H$. So if one scales down this hypersurface by a constant (i.e takes the hypersurface $\varepsilon\cdot H\subset \mathbb R^{n+1}$), we get $R(\varepsilon\cdot H)>1$. The example works if $n\ge 3$.<|endoftext|>
TITLE: Primes mod 4 and integer polynomials
QUESTION [6 upvotes]: I have asked these questions as comments here (these are related to the question there). The questions are: Let $S$ be one of the following sets of primes:
All primes of the form $4k+1$ ;
All primes of the form $4k+3$;
All primes of the form $4k+1$ except $5, 13$;
Is there a monic integer polynomial which is reducible mod prime $p$ iff $p\in S$.
REPLY [5 votes]: Here is a way to argue without showing directly that the polynomial must have degree $2$. It was explained to me by Borys Kadets (all further mistakes are, of course, my contribution).
Lemma. If a set of primes $S$ of density $\frac{1}{2}$ admits such polynomial then some subset $S'\subset S$ with $\#(S\setminus S')<\infty$ admits a monic quadratic polynomial that is reducible precisely at $S'$.
Proof. Suppose that $f$ is a polynomial of degree $n$ satisfying the condition for the set $S$. Let $G$ be the Galois group of its splitting field coming with an embedding $G\subset S_n$. By Chebotarev density, exactly $\frac{1}{2}\# G$ elements of this group must be cycles of length $n$.
Since the centralizer of a length $n$ cycle $\sigma\in S_n$ is the subgroup generated by $\sigma$, the number of conjugacy classes of length $n$ cycles in $G$ is $\frac{n}{2}$. In particular, $n$ is even and $G\cap A_n$ has index $2$ in $G$ with cycles of length $n$ forming the non-trivial coset.
The subgroup $G\cap A_n\subset G$ corresponds to a degree $2$ extension $K/\mathbb{Q}$. If a prime $p$ is ramified in the splitting field of $f$ then $f$ is reducible modulo $p$. For any unramified prime $p$ the polynomial $f$ is reducible modulo $p$ iff the Frobenius element of a prime above $p$ in the splitting field is not a length $n$ cycle, the latter condition being equivalent to the fact that $p$ is split in $K$. Thus, the set of primes split (including ramified) in $K$ is equal to $S$ with the possible exception of a finite set of ramified primes.
The minimal polynomial of a generator of $\mathcal{O}_K$ satisfies the conclusion of the lemma. $\square$
Starting with any of the three sets $S$ the lemma gives a quadratic polynomial $x^2+ax+b$ with $a,b\in \mathbb{Z}$ that is reducible precisely at the primes from a set $S'$. Since we want it to be irreducible mod $2$, both $a$ and $b$ have to be odd.
This polynomial is irreducible modulo $p>2$ if and only if $D:=a^2-4b$ is not a square mod $p$.
Set 2: The number $(-D)$ is supposed to be a non-residue modulo all but finitely many primes, but that's impossible. This can be shown by a counting argument: if there was a finite set of primes $p_1,\dots, p_k$ such that $D+n^2$ is a product of powers of $p_i$'s then there would be $O((\log N)^k)$ numbers of the form $D+n^2$ in the interval $[1,\dots, N]$.
Sets 1 and 3: Here we want $(-D)$ to be a square modulo all but finitely many primes. That forces it to be a square in $\mathbb{Z}$. However, setting $-D=c^2$ gives $a^2+c^2=4b$. That is impossible for odd $a$.<|endoftext|>
TITLE: Pulling back a functor, it becomes monadic
QUESTION [8 upvotes]: $\require{AMScd}$I am in the following situation: the diagram
$$
\begin{CD}
\cal M @>r>> [{\cal B},Set] \\
@VuVV @VVf^*V \\
\cal D @>>N_g> [{\cal A}^\text{op},Set]
\end{CD}
$$
is a (strict) pullback in $\bf Cat$; moreover, $f : \cal A^\text{op}\to B$ is bijective on objects (and $f^*$ is the "inverse image" functor given by precomposition). In turn, $g$ is the "nerve" induced by a functor $g : \cal A \to D$. $N_g$ is fully faithful because $g$ happens to be dense, and so $r$ is f.f. as well. If needed, $\cal D$ is complete and cocomplete.
This gives $\cal M$ a very explicit description: it is a reflective subcategory of $[{\cal B},Set]$ made by the functors $F : {\cal B}\to Set$ such that $F(fA)={\cal D}(gA,D)$ for some $D\in\cal D$.
Is all this sufficient to imply that $u$ is monadic? If not, what additional assumptions are needed?
REPLY [10 votes]: If $A$ and $B$ are small and $D$ is locally presentable then $u$ is a monadic right adjoint. More generally, what is non-trivial is the construction of a left adjoint of $u$. As soon as $u$ has a left adjoint, $u$ is monadic.
The proof is relatively simple: the forgetful functor $f^*$ is monadic, so it satisfies all the conditions of Beck monadicity criterion, and all the conditions of the Beck criterion except the existence of an adjoint are automatically satisfied for its pullback ($f^*$ is an isofibration, so the square is both a strict and pseudo-pullback).
Now when all the category involved are locally presentable as both $f^*$ and $N_g$ are accessible right adjoint functors, the square is also a pullback in the category of locally presentable categories and accessible right adjoint functor (which is known to have all limits, and these limits are preserved by the forgetful functor to set by an old results, which appears in Bird's phd thesis, though I think it was known before)
For more details, you can also have a look to John Bourke and Richard Garner Monads and theories this is how they construct their functor from "pre-Theory to Monads" (which is left adjoint to the Kleisli category functor)<|endoftext|>
TITLE: The "Chaos Game" as a particular series of i.i.d. random variables
QUESTION [6 upvotes]: Fix a parameter $\alpha\in(0,1)$ and take an i.i.d. sequence $X_0,X_1,\ldots$ of $\mathbb{R}^n$ valued random variables. Construct the limiting random variable
$X_\infty = (1-\alpha)\sum_{k=0}^\infty \alpha^k X_k.$
Is any general result known about this kind of limit? What if the $X_i$ follow a well known distribution like uniform/Rademacher?
I was motivated by this sum after running into: https://en.wikipedia.org/wiki/Chaos_game. For example if the $X_i$ are uniformly distributed on the 3 vertices of a triangle and $\alpha = 1/2$ the limiting distribution is supported on the associated Sierpinski Triangle. The fact that finite support distributions can give fractal shapes from this construction leads me to believe this is a non-trivial question.
My apologies if this ends up being an exercise in some well known textbook on probability theory. If it is, I'd appreciate a reference for that textbook.
Edit: I was able to locate http://u.math.biu.ac.il/~solomyb/RESEARCH/Bernotes.pdf
REPLY [10 votes]: $\newcommand\al{\alpha}$Let us drop the factor $1-\al$, by considering
$$Y:=X_\infty/(1-\al)=\sum_{k=0}^\infty\al^k X_k.$$
By Kolmogorov's three-series theorem, this series will converge almost surely (a.s.) unless at least one of the tails of the distribution of $X_0$ is too heavy.
Assume that the series indeed converges a.s.
Then, obviously,
$$Y\overset D=X+\al Y,$$
where $\overset D=$ denotes the equality in distribution and $X$ is an independent copy of the $X_k$'s. So, we have the functional equation for $F_Y$:
$$F_Y(y)=\int_{-\infty}^\infty F_Y((y-x)/\al)\,dF_X(x)$$
for real $y$, where $F_Z$ denotes the cdf of $Z$. Equivalently, we have the functional equation for $f_Y$:
$$f_Y(t)=f_Y(\al t)\,f_X(t)$$
for real $t$, where $f_Z$ denotes the characteristic function of $Z$. Of course, we can also write
$$f_Y(t)=\prod_{k=0}^\infty f_X(\alpha^k t)$$
for real $t$.
In the particular case when $X$ is Rademacher, the distribution of $Y$ is the well-studied Bernoulli convolution.
In the particular case when $X$ is $U(0,1)$ and $\alpha=1/2$, $F_Y$ is the well-studied Fabius function.<|endoftext|>
TITLE: Pseudo-intersections, splitting families, and ultrafilters
QUESTION [7 upvotes]: Suppose $U$ is a non-principal ultrafilter on $\omega$, and let us define $\tau(U)$ to be the minimum cardinality of a family $\mathcal{X}\subseteq U$ such that $\mathcal{X}$ does not have an infinite pseudo-intersection, that is, there is no infinite $A$ such that $A\setminus B$ is finite for all $B\in \mathcal{X}$.
Claim: $\tau(U)\leq\mathfrak{s}$ for any $U$.
The point is that $U$ will contain a splitting family of cardinality $\mathfrak{s}$, and this splitting family has no infinite pseudo-intersection. (To see the first statement, note that if $\mathcal{X}$ is a splitting family and we replace some of the elements of $\mathcal{X}$ by their complements, then the resulting collection is still a splitting family. Since an ultrafilter will contain one of $X$ and $\omega\setminus X$ for each $X\in\mathcal{X}$, we may as well assume $\mathcal{X}\subseteq U$.)
Question: Is there (in ZFC) an ultrafilter $U$ on $\omega$ for which $\tau(U)=\mathfrak{s}$?
I conjecture that the answer is "no", and that this negative answer will be witnessed in the original Blass-Shelah model of NCF from the paper below.
Blass, Andreas; Shelah, Saharon, There may be simple $P_{\aleph _ 1}$- and $P_{\aleph _ 2}$-points and the Rudin-Keisler ordering may be downward directed, Ann. Pure Appl. Logic 33, 213-243 (1987). ZBL0634.03047.
REPLY [7 votes]: The answer is no -- it is consistent that every $U \in \omega^*$ has $\tau(U) < \mathfrak{s}$.
I had an idea for proving this earlier today, using the Mathias model. I couldn't quite make things work, and I ended up talking about the problem with Alan Dow for a good part of the afternoon. (1) We still think the Mathias model could work, but it seems tricky. (2) There's a fix: if you interleave Laver forcings with Mathias forcings in a certain way, then the resulting iteration does work. (I'll sketch this below.) (3) I learned from Alan that your question has been studied already. The consistency of "every $U \in \omega^*$ has $\tau(U) < \mathfrak{s}$" is already known (via a different argument than the Mathias-Laver iteration sketched below), and the characteristic $\tau(U)$ has been studied quite a bit.
A rich source of information on $\tau(U)$ is the following paper by Brendle and Shelah:
Jörg Brendle and Saharon Shelah, ``Ultrafilters on $\omega$ -- their ideals and their characteristics,'' Transactions of the AMS 351 (1999), pp. 2643-2674. (available here)
What you call $\tau(U)$ is in this paper called $\pi \mathfrak{p}(U)$. They prove, among other things, that
$\bullet$ If $U$ is not a $P$-point, then $\tau(U) \leq \mathfrak{b}.$
$\bullet$ If $U$ is not a $P$-point, then the cofinality of $\tau(U) \leq \mathfrak{b}$ is uncountable. (This provides a partial answer to Santi's question in the comments.)
$\bullet$ A characterization of $\tau(U)$ is given in terms of an ideal defined from Ramsey-null sets.
$\bullet$ It is consistent that $\tau(U) < \mathfrak{s}$ for all $U \in \omega^*$.
The first two results are in Section 2, the next in Section 3, and the last in Section 7. The last result answers your question, of course, but I should also mention another relevant paper:
Alan Dow and Saharon Shelah, ``Pseudo P-points and splitting number,'' Archive for Mathematical Logic 58 (2019), pp. 1005-10027. (available here)
In the Brendle-Shelah paper, they prove $\sup_{U \in \omega^*}\tau(U) < \mathfrak{s}$ is consistent, but the gap between $\sup_{U \in \omega^*}\tau(U)$ and $\mathfrak{s}$ is only one. In the Dow-Shelah paper, they use a more complicated matrix iteration to make the gap between $\sup_{U \in \omega^*}\tau(U)$ and $\mathfrak{s}$ arbitrarily large.
Finally, let me sketch the idea I mentioned above. The idea is to do a countable support iteration that uses Laver forcing at limit steps of cofinality $\omega_1$, and uses Mathias forcing everywhere else. The iteration is of length $\omega_2$ and CH holds in the ground model. Let $V[G]$ denote the result of such an iteration, and let $U \in \omega^*$ in $V[G]$. By reflection, there is some intermediate model $V[G_\alpha]$ with $\alpha < \omega_2$ where $U \cap V[G_\alpha]$ is an ultrafilter in $V[G_\alpha]$, and where $\alpha$ has cofinality $\omega_1$. At this stage, we force with Laver forcing, and this adds a length-$\omega_1$ tower to $U \cap V[G_\alpha]$. All the subsequent Laver forcings and Mathias forcings preserve the fact that this tower has no pseudo-intersection, and so this tower is a witness to the fact that $\tau(U) = \aleph_1$ in the final extension. (See Theorem 7.11 from this paper of Alan's for more detail on the last two sentences.) Finally, the Mathias forcings enable us to get $\mathfrak{s} = \mathfrak{c}$. This is well-known to be true in the Mathias model (although not in the Laver model), and it's true in this model for essentially the same reasons.<|endoftext|>
TITLE: Calabi-Yau threefold with an automorphism of infinite order
QUESTION [13 upvotes]: I am looking for a (hopefully simple) example of a Calabi-Yau threefold (projective, simply connected, with trivial canonical bundle) admitting an automorphism of infinite order.
REPLY [6 votes]: Another nice example is by Oguiso and Truong, "Explicit Examples of rational and Calabi-Yau threefolds with primitive automorphisms of positive entropy".
Briefly, take $E$ to be an elliptic curve with an order $3$ automorphism $\tau$ and form the abelian variety $A = E \times E \times E$. The quotient of $A$ by the diagonal action of $\tau$ is mildly singular, but it has only isolated singularities and it's easy to check/standard that there's a crepant resolution $X$ that's a CY3.
Then it's easy to get automorphisms of infinite order: $SL(3,\mathbb Z)$ acts on $X$ in an obvious way. In fact, [OT] show that some of these automorphisms are "primitive", meaning they don't preserve any nontrivial fibration (unlike in the elliptically fibered examples). This is done using the theory of dynamical degrees.
(Note that this obviously gives honest automorphisms, not just birational maps; there's nothing subtle to check on singular fibers, unlike what you usually run into looking at elliptically fibered examples, as Bort alludes in the comments above.)<|endoftext|>
TITLE: The abc-conjecture over the positive rationals and Levy-Schoenberg kernels?
QUESTION [6 upvotes]: I am continuing the "abc-adventure" and have a specific question, which needs some explanation:
In this paper by Gangolli, the term "Levy-Schoenberg" kernel is defined (Definition 2.3).
Consider the group of $G = (\mathbb{Q}_{>0},\times)$ of positive rationals.
Then, in this paper by Boudreaux & Beslin, the $\gcd$ is extended to $G$, which I will call for short $\gcd^*$
Having a multiplicative function $f:\mathbb{N} \rightarrow \mathbb{N}$, one might extend it to $G$ via:
$$f^*\left(\frac{a}{b}\right) = \frac{f\left(\dfrac{a}{\gcd(a,b)}\right)}{f\left(\dfrac{b}{\gcd(a,b)}\right)}$$
Using:
$$\gcd^*(a,b)=1 \iff a,b \in \mathbb{N} \text{ and } \gcd(a,b)=1$$
then $f^*$ is multiplicative on $G$.
I will look at $f=\operatorname{rad}$, hence $\operatorname{rad}^*$ is the extension to $G$.
Using these "extensions" one might formulate the abc-conjecture over $G$.
It is not difficult to show, that it is equivalent to the abc-conjecture of the natural numbers.
My question is, if $k(a,b) = \frac{\gcd^*(a,b)}{a+b}$ is positive definite $\ge 0$.
Let $d(a,b) = \sqrt{1-2k(a,b)}$ and
$$f(a,b) = \frac{1}{2}\big(d(a,1)^2+d(b,1)^2-d(a,b)^2\big)$$
If $k(a,b)$ is positive definite over $G$, then $d$ is an Euclidean metric, and by the characterization of Schoenberg, $f$ is posivite definite.
Furthermore:
$$f(a,b) = f(b,a)$$
$$f(a,1) = 0 \quad \forall a \in G$$
$$r(a,b) := f(a,a)+f(b,b)-2f(a,b) = d(a,b)^2$$
is invariant under the action of $G$:
$$r(qa,qb) = r(a,b) \quad \forall q,a,b \in G$$
This makes $f$ by the definition (2.3) of the paper at the begining of the question to a "Levy-Schoenberg" kernel.
Of course replacing $k(a,b)$ with $k(a,b) := \frac{1}{\operatorname{rad}^*\left(\frac{ab(a+b)}{\gcd^*(a,b)^3}\right)}$ and using the abstract invariance property:
$$
k(qa,qb) = k(a,b) \quad \forall q,a,b \in G
$$
we can construct more of these Levy-Schoenberg kernels, if the "rad"-function above is a positive definite kernel... which seems difficult to prove.
Why the question, if $k(a,b)$ is positive definite:
If we go back to the natural numbers, and define:
$X_a := $ set of divisors of $a$. Then $\mu(X) = \sum_{x \in X} \phi(x)$ for every finite subset $X \subset \mathbb{N}$, and hence $\mu(X_a) = a$ and $X_a \cap X_b = X_{\gcd(a,b)}$, where $\phi$ is the Euler totient function.
Using this, one can prove that over the natural numbers, with the help of this paper by Nader, Bretto, Mourad and Abbas, that:
$$ \frac{\gcd(a,b)}{a+b} = \frac{\mu(X_a \cap X_b)}{\mu(X_a)+\mu(X_b)}$$
is positive definite.
My idea was to do the same in the case $G$:
Let for $a \in G$ be defined $X_a := \{ d | \gcd^*(a,d) = d \}$ be the set of divisors of $a$, which does not need to be finite.
Then $X_a \cap X_b = X_{\gcd^*(a,b)}$.
Hence it remains (?) to find a measure $\mu$ on $G$ such that for all $X_a$ we have:
$$\mu(X_a) = a$$
Then we would have that $k(a,b)$ is positive definite!
Of course, one does not need to follow this route, to prove the positive-definiteness of $k$, this is just an idea.
Thanks for your help!
Related:
The abc-conjecture as an inequality for inner-products?
REPLY [5 votes]: I think I found an answer to the question above:
Let $k(a,b)$ be a (positive definite $\ge 0$, symmetric) kernel on $\mathbb{N}\times \mathbb{N}$ such that if $k^*(a,b)$ is a function on $G \times G$ then we have:
$$k^*(a,b) = k(a',b')$$
where $a'=\frac{a}{\gcd^*(a,b)}, b'=\frac{b}{\gcd^*(a,b)}$,
then $k^*$ is a kernel on $G\times G$.
Proof:
Since $k$ is positive definite on $\mathbb{N}\times \mathbb{N}$, it follows that for $a_i',b_i' \in \mathbb{N}$ (which are pairwise coprime), the matrix:
$$k(a_i',b_i')=k^*(a_i,b_i)$$
is positive definte ($i=1,\cdots,n$ for some $a_i,b_i \in G$, $n$ a natural number).
Hence $k^*$ is positive definite.
Since $k^*(a,b) = \frac{\gcd(a,b)}{a+b}$ satisfies the assumption $k^*(a,b) = k(a',b')$, it follows that $k^*$ is a positive definite kernel.
Thanks for your patience, with my never-ending questions! ;)<|endoftext|>
TITLE: Does this equation have more than one integer solution?
QUESTION [7 upvotes]: Consider the following diophantine equation
$$n = (3^x - 2^x)/(2^y - 3^x),$$ where $x$ and $y$ are positive integers and $2^y > 3^x$.
Does $n$ have any other integer solutions besides the case when $x=1$ and $y=2$, which give $n=1$?
REPLY [15 votes]: This follows quickly from the observations of user44191. Check each $1 \leq x \leq 66$ and note that, for $x \geq 67$, we have $y < 1.6x$. Applying lower bounds for linear forms in two complex logarithms (as in, say, Theorem 5.2 of de Weger's thesis), we have that
$$
2^y - 3^x \geq 3^{0.9x},
$$
since $3^x > 10^{15}$. From the fact that $2^y-3^x \mid 2^{y-x}-1$, it follows that
$$
2^{0.6x}-1 \geq 3^{0.9x},
$$
a contradiction.<|endoftext|>
TITLE: HNN-extension as a 2-colimit
QUESTION [9 upvotes]: In the spirit of this question, it would be interesting to give a characterization of HNN extensions as a 2-colimit. If $G$ is a group and $\alpha:H \xrightarrow{\cong} K$ is an isomorphism between two subgroups of $G$, then I think that the HNN extension $G_{\alpha}$ has the following universal property: if we let $i_1:H \hookrightarrow G,i_2:K \hookrightarrow G$ be the canonical inclusions, then the set of group homomorphisms $G_{\alpha} \to T$ are in natural bijection to pairs $(f,t)$ where $f:G \to T$ is a group homomorphism and $t:f \circ i_2 \circ \alpha \Rightarrow f \circ i_1$ is a 2-morphism. Here we're considering $\mathbf{Grp}$ as a full subcategory of $\mathbf{Cat}$ which carries a standard 2-category structure.
I'm just wondering if this universal property can be phrased as some kind of 2-colimit.
REPLY [10 votes]: Assuming your universal property is true, it exactly says that the HNN extension is the coinserter of $(i_2 \circ \alpha,i_1) : H \rightrightarrows G$.<|endoftext|>
TITLE: Conceptual reason that monadic functors create limits?
QUESTION [16 upvotes]: Let $U: Alg_T \to C$ be the forgetful functor from the category of algebras of $T: C \to C$ ($T$ could be a monad; I'm happy to think about the simpler case where $T$ is just an endofunctor or pointed endofunctor). Then a very simple diagram chase shows that
$U$ creates limits.
In other words, if I have a diagram in $Alg_T$ whose image in $C$ has a limit, there is an obvious way to lift the limit cone to $Alg_T$, and a straightforward diagram chase verifies that indeed, this is a limit in $Alg_T$.
This is all very easy, but I'm unhappy with the state of affairs for a few reasons:
I'm very lazy and I hate doing diagram chases.
This proof does not obviously generalize to other contexts. For one thing, I have to rehash variations on the same diagram chase for each of the cases where $T$ is a monad, an endofunctor, a pointed endofunctor, etc. For another, even if I stick with just monads, say, I have to rehash the same diagram chase if I want to generalize to other contexts such as enriched or internal category theory. Of course, doing the same work over and over is supposed to mean there's a bigger picture I'm missing.
I find it remarkable that in order for $U$ to create limits, one need not assume any kind of limit-preservation hypotheses about $T$. There's something to be explained, and the proof via diagram chase doesn't accomplish that.
The second point may have real weight -- I haven't checked very diligently, but it seems that it might not actually be known whether this this theorem remains true in full generality in the enriched context, for example! (Although a moment's reflection makes me think I could run the same diagram chase with no fuss if it weren't for point (1) above.) So here's my
Question: What is the conceptual reason for which monadic functors (and other forgetful functors from "categories of algebras") create limits?
REPLY [6 votes]: From an abstract point of view, the reason is that the monad $T$ always preserves any limits that exist colaxly and colax preservation is what is required.
(This answer is closely related to Peter's answer, but describes some published results on the topic.)
The following is Proposition 4.11 of Limits for Lax morphisms by Steve Lack.
If $T$ is a $2$-monad on a $2$-category $C$ then the forgetful $2$-functor $U:T-Alg_c \to C$ from strict algebras and colax morphisms to the base creates lax limits.
Note the switch between lax limits and colax morphisms. The sense of creation is that the projections from the limit should be strict maps.
The Eilenberg-Moore object of a monad or the category of algebras for a pointed endofunctor are both examples of lax limits.
One can take $T$ the $2$-monad for categories with a class $D$ of limits and the result then applies to your setting.
Another instance would be to take as $T$ the $2$-monad for monoidal categories. Then the result becomes that if you have an opmonoidal monad on a category, it lifts to a monoidal structure on the category of algebras.
REPLY [3 votes]: This is just to flesh out the approach using inserters and equifiers discussed in the answers, in a way that doesn't quite go down to the level of diagram chasing. I fear, however, that some things implicit in this argument woule require a diagram chase to carefully check.
Also, there's something I still find mysterious: what is it about inserters and equifiers which makes it so there is a particular leg of the limit cone and particular elements of the diagram such that certain hypotheses on these elements in the diagram ensure the forgetful functor down the special leg creates limits? To put a finer point on it: can we give a better description of the class of restriction functors among limits which (under certain partial limit preservation conditions) create limits? A better description than "whatever can be built up from inserters and equifiers"?
Lemma: Let $F,G: C\rightrightarrows D$ be functors, and let $Ins(F,G)$ be the inserter. Then the forgetful functor $Ins(F,G) \to C$ creates any limits that $G$ preserves.
For the proof, note that in in general, if $(c',\phi'), (c,\phi) \in Ins(F,G)$, then
$$ Hom((c',\phi'), (c,\phi)) = Hom(c',c) \times_{Hom(Fc',Gc)^2} Hom(Fc',Gc)$$
where the pullback is over the diagonal map.
Proof: Consider a diagram $(c_i, F(c_i) \xrightarrow {\phi_i} G(c_i))_{i \in I}$ in $Ins(F,G)$ such that $G(\varprojlim_i c_i) = \varprojlim_i G(c_i)$. Then $(F(\varprojlim_i c_i) \to F(c_i) \xrightarrow{\phi_i} G(c_i))_{i \in I}$ is a cone, and so induces a map $\phi: F(\varprojlim_i c_i) \to \varprojlim_i G(c_i) = G(\varprojlim_i c_i)$. We claim that $(\varprojlim_i c_i , \phi)$ is a limit of our diagram. Indeed,
$$Hom((c',\phi'), (c,\phi)) = Hom(c',c) \times_{Hom(Fc',Gc)^2} Hom(Fc',Gc) \\
\qquad \qquad \qquad \qquad \qquad \qquad \qquad = \varprojlim_i Hom(c',c_i) \times_{\varprojlim_i Hom(Fc',Gc_i)^2} \varprojlim_i Hom(Fc',Gc_i) \\
\qquad \qquad \qquad \qquad \qquad \qquad = \varprojlim_i (Hom(c',c_i) \times_{Hom(Fc',Gc_i)^2} Hom(Fc',Gc_i)) \\
\qquad \quad = \varprojlim_i Hom((c',\phi'),(c_i,\phi_i))$$
where we have used that limits commute with limits.
Lemma: Let $\phi,\psi: F \rightrightarrows G : C \rightrightarrows D$ be a diagram of categories, and let $Eq(\phi,\psi)$ be its equifier. Then the full subcategory inclusion $Eq(\phi,\psi) \to C$ is closed under any limits preserved by $G$.
Proof: This boils down to the limit of equal morphisms being equal.<|endoftext|>
TITLE: Approximating ring maps of finite Tor-dimension
QUESTION [6 upvotes]: Let $R$ be a commutative ring, and let $S$ be a finitely presented $R$-algebra of finite Tor-dimension over $R$. Can $R \to S$ be realized as the base change, along some ring map $R_0 \to R$, of a finitely presented ring map $R_0 \to S_0$ of finite Tor-dimension with $R_0$ Noetherian? Can we furthermore arrange that $R_0$ is actually of finite type over $\mathbf{Z}$?
REPLY [2 votes]: First of all, I strongly suspect that the answer to the question is: no.
Secondly, I want to give an example to show that the answer by Riza Hawkeye is incorrect as it stands (see also the comment by Denis Nardin for why the result is quoted incorrectly from Lurie). Namely, consider the system of rings
$$
A_0 \to A_1 \to A_2 \to A_3 \to \ldots
$$
where $A_i = \mathbf{Z}[x, z_i]/(xz_i)$ and where the maps send $x$ to $x$ and $z_i$ to zero. Then we see that the colimit of the system is $A = \mathbf{Z}[x]$. Now set $B_0 = A_0/xA_0$. Clearly, we see that $B_i = B_0 \otimes_{A_0} A_i$ is equal to $A_i/xA_i$ which has infinite tor dimension over $A_i$ for all $i$. On the other hand, we have $B = A/xA$ and this has tor dimension $1$ as a module over $A = \mathbf{Z}[x]$.<|endoftext|>
TITLE: Intuition about how Voronoi formulas change lengths of sums
QUESTION [6 upvotes]: In reading the literature one encounters countless examples of Voronoi formulas, i.e., formulas that take a sum over Fourier coefficients, twisted by some character, and controlled by some suitable test function, and spits out a different sum over the same Fourier coefficients, twisted by some different characters, and this time controlled by some integral transform of the test function.
The reason one wants to do this in practice is that the second sum is somehow better, of course, which in my (admittedly limited) experience tends to boil down to the length of the second sum having changed significantly to the better.
I'll give an example (from Xiaoqing Li's Bounds for GL(3)×GL(2) L-functions and GL(3) L-functions, because it is what I happen to have in front of me).
In this case we have the GL(3) Voronoi formula
$$
\sum_{n > 0} A(m, n) e\Bigl( \frac{n \bar d}{c} \Bigr) \psi(n) \sim \sum_{n_1 \mid c m} \sum_{n_2 > 0} \frac{A(n_2, n_1)}{n_1 n_2} S(m d, n_2; m c n_1^{-1}) \Psi \Bigl( \frac{n_2 n_1^2}{c^3 m} \Bigr),
$$
where $\psi$ is some smooth, compactly supported test function, $\Psi$ as suggested above is some integral transform of it, $A(m, n)$ are Fourier coefficients of (in this case) an SL(3) Maass form, $(d, c) = 1$, and $d \bar d \equiv 1 \pmod{c}$.
(I've omitted lots of details here, but the details, I think, aren't relevant to my question.)
Doing so essentially transforms the $n$-sum into the $n_2$-sum, where, as is evident in the formula, the $n_2$-sum has a very different argument in its test function.
What happens in practice now is that, once we get to a point where applying the Voronoi formula is appropriate, we transform the sum and study the integral transform, chiefly by means of stationary phase analysis in order to find what length of the new $n_2$-sum is.
In the particular example at hand, this, after identifying the stationary phase and playing along, this takes us from an $n$-sum on $N \leq m^2 n \leq 2 N$, i.e., $n \sim \frac{N}{m^2}$, to an $n_2$-sum on
$$
\frac{2}{3} \frac{N^{1/2}}{n_1^2} \leq n_2 \leq 2 \frac{N^{1/2}}{n_1^2},
$$
i.e., $n_2 \sim \frac{N^{1/2}}{n_1^2}$, which then means that the arguments in the test function are now of size $\frac{N^{1/2}}{c^3 m}$.
I can go through the motions of performing this stationary phase analysis and so on, but my question is this: is there any intuition to be had about how and in what way these Voronoi formulas alter the lengths of sums?
REPLY [4 votes]: First of all, the description of $\psi$ after the first display is confusing (assuming OP meant $\psi$ is supported around $N$, otherwise conclusion form the first display does not make sense). I went to the relevant part (end of p.318) of Li's paper and found that $\psi$ is not just any test function, it has a weight of $n^{-3/4}$, and, more importantly, it has oscillation. A necessary display of the LHS would be (taking $m=d=1$)
$$\sum_{n}A(1,n)e\left(\frac{n}{c}+2\sqrt{n}-\frac{1}{\sqrt{n}}\right)\psi(n/N),$$
where $\psi$ is a test function supported on $[1,2]$.
To have an intuition about what the length of the dual sum would be one can follow the general heuristic formula (HF) below.
$$\text{length of the dual sum} = \frac{\text{total conductor}}{\text{length of the original sum}}.$$
Here total conductor is the conductor of the oscillating object taken all the twisting into account. For example, the total conductor of $L(1/2+it,\pi'\otimes\pi)$ where $\pi$ and $\pi'$ are fixed $\mathrm{GL}(n)$ and $\mathrm{GL}(m)$ automorphic representations, is $t^{nm}$.
Conductor of $e_q(x):=e(x/q)$ is $q$. In this case the denominator in the entry of $e()$ is of size $c\sqrt{N}$. But there is twist by a $\mathrm{GL}(3)$ Hecke eigenvalue. So the total conductor is $c^3N^{3/2}$. Applying the above formula one obtains the length of the dual sum.
One can check that the above HF also occurs in the formula of the approximate functional equation of the central $L$-value. There are two sums in the formula corresponding to the representation and its contragredient. One can check that the product of the length of the sums equals to the conductor of the $L$-function.
(The main reason of this HF to work is the automorphy under some suitable Weyl element of the underlying automorphic form, which is, indeed, the key ingredient to prove the approximate functional equation and Voronoi formula.)<|endoftext|>
TITLE: Moments of a positive random variable
QUESTION [7 upvotes]: Suppose one is handed a list of $K$ numbers, with a claim that these numbers are the first $K$ moments of a positive random variable $X$ (meaning there is 0 probability that $X<0$).
What is the strongest possible test that one could run on this list to test this claim? (We do not know any additional information about $X$.)
The most obvious thing to check first is that all the moments are positive.
A better test would involve checking that Jensen’s inequalities are satisfied. What is the most powerful test?
In general, there is a convex "allowed region” in the $K$-dimensional space of possible moments of $X$. Is there a good way to characterize this space?
REPLY [6 votes]: This is known as the truncated Stieltjes moment problem, and there is a necessary and sufficient condition taking the form of a semidefinite program. See Section 5 of the classic paper by Curto and Fialkow.<|endoftext|>
TITLE: Examples of plane algebraic curves
QUESTION [12 upvotes]: There are many interesting sequences of polynomials which contain
polynomials of arbitrarily high degree, for example classical
orthogonal polynomials. Most of them arise as characteristic polynomials
of some sequences of operators, or as polynomial solutions
of some differential equations.
What are some natural specific
sequences of plane (affine or projective) algebraic
curves which contain curves of arbitrarily high degree and genus?
One such example is Fermat's curves $x^n+y^n=1$. Lissajous
(a.k.a. Chebyshev)
curves are of arbitrary degree but they have zero genus. Sequences of hyperelliptic curves occur in the theory of integrable systems. What else?
I looked to the Catalog of Plane curves by D. Lawrence (Dover, 2014)
and to the book of Brieskorn and Knörrer, Plane algebraic curves,
and found only Lissajous curves, epitrochoids and
hypotrochoids (all of genus zero) as examples of arbitrarily high degree.
I understand that many examples can be constructed. But I am asking on some naturally occuring sequences, whatever it can mean. Of some historical significance or appearing in applications.
EDIT. Thanks to all who answered or commented. I am not marking this question as "answered" for a while, hoping for more examples. Of course,
TITLE: Lifting a complete intersection in $\mathbb{P}^n_{\mathbb{F}_p}$ to $\mathbb{Z}_p$
QUESTION [7 upvotes]: Suppose that you are given a (not necessarily smooth) projective variety $X \subseteq \mathbb{P}^n_{\mathbb{F}_p}$ of codimension $d$ that is a complete intersection. In other words, it can be defined by exactly $d$ homogeneous polynomials and no set of polynomials of cardinality less than $d$ has $X$ as its zero set. Let $f_1$, ..., $f_d$ be a set of polynomials defining $X$. If I lift these polynomials arbitrarily to $\mathbb{Z}_p$ (call the lifts $F_1$, ..., $F_d$), I will get a projective algebraic set $X' \subseteq \mathbb{P}^n_{\mathbb{Z}_p}$.
My question is this: will this $X'$ be flat over $\mathbb{Z}_p$? Equivalently, is the module $\mathbb{Z}_p[x_0,...x_n]/(F_1, ..., F_d)$ flat over $\mathbb{Z}_p$? I know that $\mathbb{Z}_p$ is a DVR, so flatness is equivalent to being torsion-free but I just cannot see how to prove that it is either. If $X$ is smooth, then $X'$ is definitely flat: $X'$ will also be smooth by the Jacobian criterion and hence flat.
REPLY [3 votes]: This is an expanded version of my comment:
The main claim is that the question can be answered from basic facts in (local) commutative algebra. In particular, the same statement holds for a complete intersection in affine space, which I explain below.
We work over any dvr $R$ with residue field $k$. Let $X$ be of codimension $d$ in $\mathbb{A}^n_k$ defined by equations $f_1,f_2,\dots,f_d$. Let $F_1,F_2,\dots,F_d$ be elements of $R[x_1,x_2,\dots,x_n]$ such that $F_i$ is a lift of $f_i$ and let $X'$ be the subscheme of $\mathbb{A}^n_R$ defined by the ideal $(F_1,F_2,\dots,F_d)$. Then $X'$ is a local complete intersection scheme since the uniformizer of $R$ is a nonzero divisor in the polynomial ring and $X$ is a local complete intersection. In particular, $X'$ is Cohen-Macaulay, so it has no embedded points.
Now suppose the coordinate ring $A$ of $X'$ has non-zero $R$-torsion elements. The set of all such elements is a non-zero ideal in $A$ and the support of this ideal (as an $A$-module) is contained in $X'$ (viewed as a subset of $X$). This implies that $A$ has an embedded prime, so we get a contradiction.<|endoftext|>
TITLE: Greatest common divisor in $\mathbb{F}_p[T]$ with powers of linear polynomials
QUESTION [11 upvotes]: Let $n>1$ and $p$ be an odd prime with $p-1 \mid n-1$ such that $p^k - 1 \mid n-1$ does not hold for any $k>1$. Notice that, since $p-1 \mid n-1$, we have $T^p - T \mid T^n-T$ in $\mathbb{F}_p[T]$ and hence also $T^p - T = (T+u)^p - (T+u) \mid (T+u)^n - (T+u)$ for all $u \in \mathbb{F}_p$.
Question. Is $T^p - T$ actually the gcd of $\{(T+u)^n - (T+u) : u \in \mathbb{F}_p\}$ in $\mathbb{F}_p[T]$?
I have verified this with computer algebra software for $n \leq 7000$ (code link). For many $n$ actually $u=0,1$ are sufficient.
I tried to find a proof, but my first idea didn't work. The only thing I know so far is that the gcd is invariant under $T \mapsto T+1$ and therefore contained in $\mathbb{F}_p[T^p-T]$. I expect that there are two proofs (if the statement is true at all), namely one using finite fields $\mathbb{F}_{p^m}$, and one using a direct calculation with polynomials. I am more interested in a direct calculation here. The background is a new proof of Jacobson's theorem I am working on. Notice that the statement is false for $p=2$ (but still true for many $n$ in this case) and that it is clearly false without the $p^k-1$-requirement.
REPLY [11 votes]: This is false.
Let $p$ be an odd prime, let $\ell$ be another prime, and let $m$ be a small prime divisor of $p^{\ell}-1$, that doesn't divide $p-1$. Let $n= 1 + \frac{ p^{\ell}-1}{m}$.
Then $n-1$ is a multiple of $p-1$, is not a multiple of $p^{\ell}-1$, and is not a multiple of $p^{k}-1$ for any other $k$ because $p^{\ell}-1$ is not a multiple of $p^{k}-1$ for any $1 < k < \ell$.
Then $x \in \mathbb F_{p^\ell}$ is a root of $T^{n } - T$ if and only if $x$ is an $m$'th power in $\mathbb F_{p^\ell}$. So roots of the gcd of $(T+u)^n - (T+u)$ for all $u$ in $\mathbb F_p$ are exactly those $x \in \mathbb F_{p^\ell}$ such that $x = y_0^m, x+1 = y_1^m, \dots, x+p-1 = y_1^{m}$ for some $y_0, \dots, y_{p-1}$ in $\mathbb F_{p^\ell}$.
Thus, to find a counterexample, it suffices to check that the number of $\mathbb F_{p^\ell}$ points of the curve $C_m$ with variables $x,y_0,\dots, y_{p-1}$ and equations $x +i = y_i^m$ is greater than the number $p m^{p-1}$ of solutions with $x \in \mathbb F_p$. Then the $x$ coordinates of the extra points will be roots of the gcd but not roots of $T^p- T$.
By Riemann-Hurwitz, the genus $g$ of $C$ satisfies $$2-2g = 2 m^p - (p+1) m^{p-1} (m-1)$$ since the degree over $\mathbb P^1$ (under the map $x$) is $m^p$, there are $p+1$ branch points $0,1\dots, \infty$, and each branch point has ramification of order $m$ on each point lying above it. There are $m^{p-1}$ missing points at $\infty$.
So by Weil's theorem, the number of $\mathbb F_{p^\ell}$-points of $C$ is at least $$p^\ell - p^{\ell/2} ((p+1) m^{p-1} (m-1) + 2 - 2 m^p) +1 - m^{p-1}$$
with the first term the main term, the second term coming from the Frobenius eigenvalues, and the last term coming from the missing points.
Thus, as long as
$$ p^\ell > p^{\ell/2} ((p+1) m^{p-1} (m-1) + 2 - 2 m^p) + m^{p-1} + p m^{p-1},$$ there is a counterexample.
Plugging into Wolfram Alpha, this inequality fails for $p=3, \ell=11, m=23$ but succeeds for $p=3,\ell=23, m=47$ and $p=3, \ell=29, m=59$.
I found these by looking at the sequence of multiplicative orders of $3$ modulo primes and looking for prime values, which became $\ell$, with the modulus prime becoming $m$.
It seems there are many examples with $\ell$ not much smaller than $m$, which as long as both are much larger than $p$, means this inequality is easily satisfied, since anything of the form $p^\ell$ beats anything of the form $m^p$.
In the comments, François Brunault found an explicit example: The polynomial $$T^{23}-T^{22}-T^{21}-T^{20}-T^{19}+T^{18}-T^{16}+T^{13}+T^{12}+T^{11}-T^{10}+T^8+T^6+T^4-T^2-T-1$$ divides $$\gcd( T^{n} - T, (T-1)^n - (T-1) , (T+1)^n - (T+1))$$ in $\mathbb F_3[T]$ when $n= 1 + \frac{3^{23}-1}{47}$ and provided the following Pari/GP code to check it:
P = Mod(x^23-x^22-x^21-x^20-x^19+x^18-x^16+x^13+x^12+x^11-x^10+x^8+x^6+x^4-x^2-x-1, 3); n = 1 + (3^23-1)/47; t = Mod(x, P); print(t^n == t & (t+1)^n == t+1 & (t-1)^n == t-1);<|endoftext|>
TITLE: Is this a new strange attractor?
QUESTION [7 upvotes]: I recently made some experiments in programming strange attractors, and I found this (very simple) equations, which create a nice strange attractor:
xn=x+dt*(z-y)
yn=y+dt*(x/2-1)
zn=z+dt*(-xy/2-z)
You can see it in action on my Youtube channel:
https://youtu.be/Bm_M6mUGjtg
My question: Is this a variation of the Lorenz- or Rössler attractor - or did I stumble upon something new?
EDIT:
Meanwhile I programmed a little 3D View for this attractor:
You can see/move the view on this applet (Java needed):
https://cerumen.de.cool/attractor/index.html
(Here you also find the source code for Processing)
And here the Javascript-Version: https://cerumen.de.cool/attractor/js/index.html (with processing.js... bit slow)
Perhaps my question was also asked too amateurishly.
I was simply surprised by the simplicity of the system of equations I have found.
Therefore I would like to know if this strange attractor is a descendant of one of the well known ones (Lorenz / Rössler).
Edit 2:
I have now brought the system of equations into a more general form:
xn=x+dt*(z-y)
yn=y+dt*(ax-b)
zn=z+dt*(-axy-z)
with a in range [0 to 1], b in range [0.5 to 1]
This makes it more interesting.
Here some sample images for different values for a and b:
Edit 3:
Here a video with the generalized equations and constantly changing parameters a and b: https://youtu.be/gxusM8pmNwU
I think you can see here quite well how the system goes from order through bifurcation into chaos...
REPLY [3 votes]: In another forum a user drew my attention to the publication of
J. C. Sprott, Some simple chaotic flows, Phys. Rev. E 50, R647-R650 (1994), doi:10.1103/PhysRevE.50.R647, author pdf.
This shows that there are many very simple chaotic systems of equations.
I guess this answers my question...
You can see some of the equations here, and here are the corresponding graphs.
But thank you all for your interest.<|endoftext|>
TITLE: Significance of the length of the Perron eigenvector
QUESTION [5 upvotes]: Let $A$ be a positive square matrix. Perron-Frobenius theory says that there exist $\lambda,v$ with $Av=\lambda v$ and $\lambda$ equals the spectral radius of $A$, $\lambda$ is simple, and $v$ is positive.
Now consider also the left Perron eigenvector $u^T A=\lambda u^T$. Another result of Perron-Frobenius theory is that
$$\lim_{m\to \infty} \frac{A^m}{\lambda^m} = \frac{v u^T}{u^T v}.$$
Suppose $\|v\|=1$. The above result says that the "correct" normalization for u is $u^T v=1$ rather than the more usual $u^T u=\|u\|^2=1$. This motivates the question: what is the significance of the ratio
$$\frac{u^T v}{u^T u} ?$$
Are there matrices $A$ for which this ratio is arbitrarily large? Arbitrarily small? Does this ratio determine any properties of $A$? Note that if $A$ is symmetric, then $u=v$ and this ratio is always equal to $1$, but that's not the case in general for arbitrary $A$. Could it be the case that this ratio is measuring how far $A$ is from being symmetric?
Note too that this normalization is necessary so that the limit $\frac{v u^T}{u^T v}$ is a projection matrix (i.e. that its only non-zero eigenvalue is one). In this context, I understand why the normalization is necessary, but I'm interested in the amount of normalization necessary with respect to the length of $u$.
Any pointers appreciated. Thanks!
EDIT In the comments, it is argued that the real quantity of interest in this setup is
$$\frac{\left( u^T v \right)^2}{\left(u^T u \right) \left( v^T v \right)}.$$
This quantity is also of interest to me, and an acceptable replacement for my original question.
REPLY [3 votes]: That quantity $s = \frac{|u^Tv|}{\|u\|\|v\|}$ is the inverse of the eigenvalue condition number. The smaller it is, the more sensitive to perturbation the Perron value is.
More precisely, any perturbed matrix $A+E$ with $\|E\| \leq \varepsilon$ has a Perron value $\tilde{\lambda}$ that satisfies $|\tilde{\lambda}-\lambda| \leq \frac{\varepsilon}{s} + \mathcal{O}(\varepsilon^2)$. See e.g. Section 7.2.2 of Golub and Van Loan's Matrix Computations 4th ed.
In addition, note that if $A$ is normal then $s=1$ (its maximum possible value) and the Perron value is perfectly conditioned; while if $\lambda$ is a defective eigenvalue (e.g. $A = \begin{bmatrix}1 & 1 \\ 0 & 1\end{bmatrix}$) then $s=0$. So rather than a "distance from symmetric" I'd say that $1-s$ is a "distance from normal" or $s$ is a "distance from defective".<|endoftext|>
TITLE: Calculate the $\downarrow$, $\downarrow\uparrow$ and $\uparrow\downarrow$ cofinalities of the poset of nontrivial finitary partitions of $\omega$
QUESTION [5 upvotes]: Let $(P,\le)$ be a poset. For a point $x\in P$ let
$${\downarrow}x=\{p\in P:p\le x\}\quad\text{and}\quad{\uparrow}x=\{p\in P:x\le p\}$$be the lower and upper sets of the point $x$, and for a subset $S\subset P$, let
$${\downarrow}S=\bigcup_{s\in S}{\downarrow}s\quad\text{and}\quad{\uparrow}S=\bigcup_{s\in S}{\uparrow}s$$be the lower and upper sets of the set $S$ in $P$.
Now consider the following cardinal characteristics of $P$:
$\bullet$ the $\downarrow$-cofinality ${\downarrow}(P)=\min\{|C|:C\subseteq P\;\wedge \;{\downarrow}C=P\}$;
$\bullet$ the $\uparrow$-cofinality ${\uparrow}(P)=\min\{|C|:C\subseteq P\;\wedge\;{\uparrow}C=P\}$;
$\bullet$ the $\uparrow\downarrow$-cofinality ${\uparrow}{\downarrow}(P)=\min\{|C|:C\subseteq P\;\wedge \;{\uparrow\downarrow}C=P\}$;
$\bullet$ the $\downarrow\uparrow$-cofinality ${\downarrow}{\uparrow}(P)=\min\{|C|:C\subseteq P\;\wedge\;{\downarrow\uparrow}C=P\}$.
Proceeding in this fashion, we could define the $\downarrow\uparrow\downarrow$-cofinality ${\downarrow\uparrow\downarrow}(P)$ and $\uparrow\downarrow\uparrow$-cofinality ${\uparrow\downarrow\uparrow}(P)$ and so on.
It is clear that $$\max\{{\uparrow\downarrow}(\mathfrak P),{\downarrow\uparrow}(\mathfrak P)\}\le \min\{{\downarrow}(\mathfrak P),{\uparrow}(\mathfrak P)\}.$$
I would like to know the values of the $\downarrow$, $\downarrow\uparrow$ and $\uparrow\downarrow$ cofinalities of the poset $\mathfrak P$ of nontrivial finitary partitions of $\omega$.
By a partition I understand a cover $\mathcal P$ of $\omega=\{0,1,2,\dots\}$ by pairwise disjoint sets.
A partition $\mathcal P$ is defined to be
$\bullet$ finitary if $\sup_{P\in\mathcal P}|P|$ is finite (i.e., the cardinalities of the cells of the partition are upper bounded by some finite cardinal);
$\bullet$ nontrivial if the subfamily $\{P\in\mathcal P:|P|=1\}$ is finite (i.e., $\mathcal P$ contains infinitely many cells of cardinality $\ge 2$).
The family $\mathfrak P$ of all nontrivial finitary partitions of $\omega$ is endowed with the refinement partial order $\le$ defined by $\mathcal P_1\le\mathcal P_2$ if each cell of the partition $\mathcal P_1$ is contained in some cell of the partition $\mathcal P_2$.
It can be shown that $${\uparrow\downarrow\uparrow}(\mathfrak P)=1={\downarrow\uparrow\downarrow}(\mathfrak P),$$ so only four cofinalities (with at most two arrows) can be infinite.
Using almost disjoint families of cardinality continuum, it can be shown that ${\uparrow}(\mathfrak P)=\mathfrak c$.
Problem 1. Calculate the $\downarrow$-cofinality ${\downarrow}(\mathfrak P)$ of the poset $\mathfrak P$. In particular, is ${\downarrow}(\mathfrak P)=\mathfrak c$? Or ${\downarrow}(\mathfrak P)=\mathfrak d$?
Remark 1. It can be shown that ${\downarrow}(\mathfrak P)\ge\mathfrak d$.
Problem 2. Evaluate the cardinal characteristics ${\downarrow\uparrow}(\mathfrak P)$ and ${\uparrow\downarrow}(\mathfrak P)$ of the poset $\mathfrak P$.
REPLY [2 votes]: At the moment we have the following information on the cofinalities of the poset $\mathfrak P$ (see Theorem 7.1 in this preprint).
Theorem.
1) ${\downarrow}\!{\uparrow}\!{\downarrow}(\mathfrak P)={\uparrow}\!{\downarrow}\!{\uparrow}(\mathfrak P)=1$.
2) ${\downarrow}(\mathfrak P)={\uparrow}(\mathfrak P)=\mathfrak c$.
3) ${\downarrow}\!{\uparrow}(\mathfrak P)\ge \mathrm{cov}(\mathcal M)$.
4) $\mathsf \Sigma\le{\uparrow}\!{\downarrow}(\mathfrak P)\le\mathrm{non}(\mathcal M)$.
Here $\mathrm{non}(\mathcal M)$ is the smallest cardinality of a nonmeager set in the real line, and
$\mathsf \Sigma$ is the smallest cardinality of a subset $H$ in the permutation group $S_\omega$ of $\omega$ such that for any infinite sets $A,B\subseteq \omega$ there exists a permutation $h\in H$ such that $h(A)\cap B$ is infinite.
By Theorem 3.2 in this preprint, $$\max\{\mathfrak b,\mathfrak s,\mathrm{cov}(\mathcal N)\}\le\mathsf\Sigma\le\mathrm{non}(\mathcal M).$$
The cardinal $\mathsf\Sigma$ is equal to the cardinal $\mathfrak j_{2:2}$, discussed in this MO-post.
However I do not know the answer to the following
Problem. Is ${\downarrow\!\uparrow}(\mathfrak P)\le\mathrm{non}(\mathcal N)$?
Here $\mathrm{non}(\mathcal N)$ is the smallest cardinality of a subset of the real line, which is not Lebesgue null.<|endoftext|>
TITLE: Linear combination of sine and cosine
QUESTION [26 upvotes]: I was explaining to my students the other day why $\cos(2x)$ is not a linear combination of $\sin(x)$ and $\cos(x)$ over $\mathbb{R}$. Besides the canonical method of using special values of sine and cosine, I noticed something interesting. In the following, all vector spaces are over $\mathbb{R}$.
Consider the linear space $C^\infty_b(\mathbb{R})$ of real-valued bounded smooth functions on $\mathbb{R}$, and take any $c > 0$. We say a function $f \in C^\infty_b(\mathbb{R})$ has property $P(c)$, if for all $k \in \mathbb{N}$ (including $0$), we have
$$\sup f^{(k+1)} = c \sup f^{(k)} = -\inf f^{(k+1)} = -c \inf f^{(k)}.$$
Here, the supremum and infimum are of course taken over $\mathbb{R}$, and $f^{(k)}$ is the $k$-th derivative of $f$, with the convention that $f^{(0)}=f$.
Define
$$S(c) = \{f \in C^\infty_b(\mathbb{R}) \,\vert\, f \text{ has property } P(c)\}.$$
Since for fixed $a,b$ and all $x$, we have $a \sin(x) + b \cos(x) = \sqrt{a^2 + b^2} \sin(x + \theta) $ for some fixed $\theta$, it is clear that all linear combinations of $\sin(cx)$ and $\cos(cx)$ belong to $S(c)$. In particular, linear combinations of $\sin(x)$ and $\cos(x)$ are all in $S(1)$, while $\cos(2x)$ is not.
Question: is it true that $S(c) = \operatorname{Vect}\bigl(\sin(cx), \cos(cx)\bigr)$?
If $f \in S(1)$ and $f$ is periodic, then using Fourier series, I can prove that $f$ is indeed $2 \pi$-periodic and $f \in \operatorname{Vect}\bigl(\sin(x), \cos(x)\bigr)$ with some work. Although I haven't checked this yet, I also believe that the periodic case for $S(c)$ where $c>0$ is arbitrary could be established by a more elaborate Fourier series argument (of course, I could be wrong). So the real interest lies in treating the non-periodic case, i.e., answering the following
Special case: does $f \in S(c)$ imply $f$ is periodic?
At first, I suspect the answer to the above special case is negative. But after some experiment, I am not so sure. Note that the radius of convergence of the Taylor series (say around $0$) for all $f \in S(c)$ is infinite, so the Taylor series of $f$ converges to $f$ itself. In particular, all functions in $S(c)$ are automatically analytic, so one does not have much freedom when trying to construct a (counter-)example.
If the answer turns out to be negative, then can one at least assert that $S(c)$ is a linear subspace? What if we only consider periodic functions for some fixed period in case $S(c)$ is not a linear subspace? Of course, these probably depend on the explicit form of the answer which is not yet known to me, and all of these are just some (perhaps stupid and naive) speculation on an old exercise of a first-year undergraduate. But it seems interesting, and any thought is appreciated.
Edit: I was a bit careless in formulating the question since the questions for all different $c$ are equivalent merely by rescaling, so one can simply assume $c = 1$, in which case we still have much work to do.
Edit 2: Proof of the periodic case can be found here in case anyone is interested.
REPLY [7 votes]: As noted by the OP we can replace $f$ by $af(bx)$ for suitable $a,b\in\mathbb{R}$ so that wlog we can take $c=1$ and ensure that $\sup f=-\inf f=1$.
Firstly we note that $f(z)$ is infinitely differentiable on $\mathbb{R}$ so we can form the taylor series at 0, $f(z)=\sum_{i=0}^{\infty}\frac{f^{(i)}(0)}{i!}z^i$. Since $|f^{(i)}(0)|\leq 1$ for all $i\geq 0$ we have $\sum_{i=0}^{\infty}|\frac{f^{(i)}(0)}{i!}z^i|=\sum_{i=0}^{\infty}\frac{|f^{(i)}(0)|}{i!}|z|^i\leq \sum_{i=0}^{\infty}\frac{|z|^i}{i!}$ which converges for all $z$ to $e^{|z|}$.
Hence $F(z)=\sum_{i=0}^{\infty}\frac{a_i}{i!}z^i$ is an absolutely convergent series defining an entire function on $\mathbb{C}$ agreeing with $f$ on $\mathbb{R}$ s.t. $\sup f^{(k)}=-\inf f^{(k)}=1$ for all $k\in \mathbb{Z}_{\geq0}$ and $|F(z)|\leq e^{|z|}$ for all $z\in \mathbb{C}$.
We now determine the form of $F$ given the preceding conditions.
First note that Bernstein proved the following (see Rahman and Tariq$^1$) as an extension of his related inequality for polynomials:
Theorem Let $g$ be an entire function of exponential type $\tau>0$ such that $|g(x)|\leq M$ on the real axis. Then $$\sup_{-\infty
TITLE: Is $\mathfrak j_{2:1}=\mathfrak{j}_{2:2}$ in ZFC?
QUESTION [11 upvotes]: A function $f:\omega\to\omega$ is called
$\bullet$ 2-to-1 if $|f^{-1}(y)|\le 2$ for any $y\in\omega$;
$\bullet$ almost injective if the set $\{y\in \omega:|f^{-1}(y)|>1\}$ is finite.
Let us introduce two critical cardinals, related to $2$-to-$1$ functions:
$\mathfrak{j}_{2:1}$ is the largest cardinal $\kappa\le\mathfrak c$ such that for any family $F\subset \omega^\omega$ of $2$-to-$1$ functions with $|F|<\kappa$ there exists an infinite subset $J\subset\omega$ such that for any $f\in F$, the restriction $f{\restriction}J$ is almost injective;
$\mathfrak{j}_{2:2}$ is the largest cardinal $\kappa\le\mathfrak c$ such that for any family $F\subset \omega^\omega$ of $2$-to-$1$ functions with $|F|<\kappa$ there are two infinite sets $I,J\subset\omega$ such that for any $f\in F$ the intersection $f(I)\cap f(J)$ is finite.
It can be shown that $\max\{\mathfrak s,\mathfrak b\}\le\mathfrak j_{2:1}\le\mathfrak j_{2:2}\le\mathrm{non}(\mathcal M)$.
I would like to have more information on the cardinals $\mathfrak j_{2:1}$ and $\mathfrak j_{2:2}$.
Problem 0. Is $\mathfrak j_{2:1}=\mathfrak j_{2:2}$ in ZFC?
Problem 1. Is $\mathfrak j_{2:2}=\mathrm{non}(\mathcal M)$ in ZFC?
Problem 2. What is the value of the cardinals $\mathfrak j_{2:1}$ and $\mathfrak j_{2:2}$ in the Random Model? (In this model $\mathfrak b=\mathfrak s=\omega_1<\mathfrak c=\mathrm{non}(\mathcal M)$, see $\S$11.4 in this survey paper of Blass).
Remark. It can be shown that the cardinal $\mathfrak j_{2:1}$ (resp. $\mathfrak j_{2:2}$) is equal to the smallest weight of a finitary coarse structure on $\omega$ that contains no infinite discrete subspaces (resp. contains no infinite asymptotically separated sets). In this respect $\mathfrak j_{2:1}$ can be considered as an asymptotic counterpart of the cardinal $\mathfrak z$, defined as the smallest weight of an infinite compact Hausdorff space that contain no nontrivial convergent sequences. The cardinal $\mathfrak z$ was introduced by Damian Sobota and deeply studied by Will Brian and Alan Dow.
The similarity between $\mathfrak j_{2:1}$ and $\mathfrak z$ suggests another
Problem 3. Is $\mathfrak j_{2:1}=\mathfrak z$ in ZFC?
REPLY [2 votes]: I can answer problems 2 and 3, although I still don't know the answer to problems 0 and 1.
The main point is that
$\mathfrak{j}_{2:1} = \mathfrak{c}$ in the random model.
I'll sketch a proof of this below. (It's a bit long, but I've tried to make it readable.) The proof actually shows a little more: it gives you $\mathrm{cov}(\mathcal{N}) \leq \mathfrak{j}_{2:1}$.
This result also answers problem 3, because we know that $\mathfrak{z} = \aleph_1$ in the random model. (This was first proved by Alan Dow and David Fremlin here. It is also a corollary to Theorem 4.2 in this paper by me and Alan.) Therefore $\mathfrak{z} < \mathfrak{j}_{2:2},\mathfrak{j}_{2:1}$ is consistent. On the other hand, Koppelberg proved that $\mathfrak{z} \leq \mathrm{cov}(\mathcal{M})$. (Actually, she proved the dual statement in the category of Boolean algebras here. Stefan Geschke wrote a purely topological proof here.) Because you have proved that $\mathfrak{j}_{2:2},\mathfrak{j}_{2:1} \leq \mathrm{non}(\mathcal{M})$, and because $\mathrm{non}(\mathcal{M}) < \mathrm{cov}(\mathcal{M})$ in the Cohen model, it follows that $\mathfrak{j}_{2:2},\mathfrak{j}_{2:1} < \mathfrak{z}$ is consistent. Thus there is no inequality between $\mathfrak{z}$ and either of $\mathfrak{j}_{2:2}$ of $\mathfrak{j}_{2:1}$ that is provable in $\mathsf{ZFC}$.
(I know I gave a different argument for this in the comments. I don't like that argument as much because it relies on Alan's unpublished -- and mostly unwritten -- argument that $\mathfrak{z} = \aleph_1$ in the Laver model. I'm sure he's right. But I like that the argument here relies on the fact that $\mathfrak{z} = \aleph_1$ in the random model, and you can go read one or two proofs of this if you like.)
Now let's sketch the proof that $\mathfrak{j}_{2:1} = \mathfrak{c}$ in the random model. For the sake of clarity, I'm going to avoid forcing jargon and give a probabilistic argument that (I hope) will give you the right idea.
To show that $\mathfrak{j}_{2:1} = \mathfrak{c}$ in the random model, let's first recall how random real forcing works. Roughly, we imagine ourselves to live in a universe $V$ of sets, containing real numbers, subsets of $\mathbb N$, lots of $2$-to$1$ functions, and whatever else. But we know that our universe is about to get bigger -- this is the forcing -- by the introduction of a "truly random" real number $r$. The new, bigger universe is called $V[r]$.
The first observation I'd like to make is that all continuous measures on uncountable Polish spaces are essentially isomorphic. This means that it doesn't matter whether we view $r$ as a random element of $\mathbb R$, or of $[0,1]$, or of $2^\omega$ with the standard product measure, or whatever. For this problem, we want to view $r$ as an infinite sequence of random selections from larger and larger finite sets $I_n$, where $I_n$ has size $n!$. We select, at random, only a single element from each set. (This can be formalized by saying that we'd like $r$ to be a random element of the Polish space $\prod_{n = 0}^\infty \{1,2,\dots,n!\}$, equipped with the usual product measure. But let's keep it informal.) So our universe is about to get bigger by introducing a truly random sequence of selections from some sets $I_0, I_1, I_2, \dots$ with $|I_n| = n!$.
Within $V$, we can try to anticipate objects that will be constructible from $r$ in $V[r]$. For example, we can anticipate that once we get $r$, we can build a set $J \subseteq \mathbb N$ according to the following recipe: first identify $I_n$ with the interval $[1+1+2+\dots+(n-1)!,1+1+2+\dots+(n-1)!+n!) \subseteq \mathbb N$, and then let the $n^{\mathrm{th}}$ element of $J$ be whatever $r$ randomly selects from this interval.
Now I claim that this set $J$ described above has the following property: if $f$ is any $2$-to-$1$ function in the ground model $V$, then the restriction of $f$ to $J$ is almost-injective. To prove this, it suffices to argue that it's true with probability $1$, given that $r$ makes its selections randomly. This suffices because this is precisely what we mean when we say that $r$ is a "truly random" addition to $V$: if there is a randomness test defined in $V$ (such as one defined from any $f \in V$), then $r$ is random with respect to that test.
So let's argue probabilistically.
Fix a $2$-to-$1$ function $f \in V$.
If $f(a) = f(b)$, we may view this as a "guess" that $f$ is making about our set $J$: the guess is that $a$ and $b$ are both in $J$. In other words, $f$ gets to guess at pairs from $J$ infinitely many times, and it is our job to prove that, with probability $1$, only finitely many of these guesses are correct.
So what is the probability that $f$ correctly guesses a pair of elements from $J$?
If $f$ identifies a member of some $I_m$ with a member of some $I_n$, where $m \neq n$, then there is a probability of exactly $\frac{1}{m!n!}$ that $f$ will have correctly guessed a pair from $J$. When $f$ makes other kinds of guesses (not identifying some member of some $I_m$ with a member of some $I_n$, where $m \neq n$), then the probability is $0$ that $f$ will have correctly guessed a pair from $J$.
If $m < n$, then $f$ gets at most $|I_m| = m!$ chances to guess a pair from $J$ with one member in $I_m$ and the other in $I_n$. By the previous paragraph, the probability of one of these guesses being correct is $\leq\! m!\frac{1}{m!n!} = \frac{1}{n!}$. Summing over all $n > m$, it follows that the probability of $f$ correctly guessing any pair of elements from $J$ with one member in $I_m$ and the other in $I_n$ for some $n > m$ is
$$\leq\! \sum_{n = m+1}^\infty \frac{1}{n!} < \frac{1}{(m+1)!} \sum_{k = 0}^\infty \frac{1}{(m+1)^k} = \frac{1}{(m+1)!}\frac{m+1}{m} < \frac{1}{m!m}.$$
Now fix $k > 0$. Summing over all $m > k$, we see that the probability of $f$ correctly guessing a pair of elements from $J$ with one member in $I_m$ and the other in $I_n$ for some $n > m > k$ is
$$\leq\! \sum_{m = k+1}^\infty \frac{1}{m!m} < \sum_{m = k+1}^\infty \frac{1}{m!} < \frac{1}{k!k}.$$
Therefore the probability of $f$ correctly guessing a pair of elements from $J \setminus (I_0 \cup \dots \cup I_k)$ is at most $\frac{1}{k!k}$. For any fixed $\varepsilon > 0$, we can choose $K$ large enough that $\sum_{k = K}^\infty \frac{1}{k!k} < \varepsilon$. This means that for $K$ large enough, the probability of $f$ correctly guessing more than ${K+1} \choose 2$ pairs of elements of $J$ is less than $\varepsilon$. Therefore the probability of $f$ correctly guessing infinitely many pairs of elements of $J$ is less than $\varepsilon$. As $\varepsilon$ was arbitrary, the probability of $f$ correctly guessing infinitely many pairs of elements of $J$ is $0$.
This shows that our set $J$ in $V[r]$ "should" (probabilistically) have the property that $f \restriction J$ is almost injective for every $f \in V$. But as we said earlier, this means $J$ really does have this property.
Why does this mean $\mathfrak{j}_{2:1} = \mathfrak{c}$ in the random model? The random model is $V[G]$, where $G$ is a "random" element of the measure algebra $2^{\aleph_2}$. If $\mathcal F$ is any set of $\aleph_1$ $2$-to-$1$ functions in $V[G]$, then a standard "nice names" argument shows that there is some weight-$\aleph_1$ subalgebra $X$ of $2^{\aleph_2}$ such that $\mathcal F$ is already in the intermediate model $V[X \cap G]$. Because $|X| = \aleph_1$, there will be random reals added in moving from the intermediate model $V[X \cap G]$ to the final model $V[G]$ -- random over $V[X \cap G]$, not just over $V$. We've just showed that the addition of these random reals adds some $J$ that "works" for every $f \in \mathcal F$.
Why does this mean $\mathrm{cov}(\mathcal{N}) \leq \mathfrak{j}_{2:1}$? There are a few ways to see this. The easiest is probably just to go through the above argument and convince yourself that what we've really proved is that every $2$-to-$1$ function $f$ is "solved" by a measure-$1$ set of $J$'s in the Polish space $\prod_{n = 0}^\infty \{1,2,\dots,n!\}$. Equivalently, the set of $J$'s that fail to work for a given $2$-to-$1$ function $f$ is a null set $N_f$.
Therefore, if $\mathcal F$ is any size $<\! \mathrm{cov}(\mathcal{N})$ family of $2$-to-$1$ functions, $\bigcup_{f \in \mathcal F}N_f$ does not cover our Polish space, and so there is some $J$ that works for every $f \in \mathcal F$.<|endoftext|>
TITLE: Near permutation $n\mapsto n+1$ not conjugate to its inverse on the Stone-Čech remainder?
QUESTION [12 upvotes]: Let $\beta\omega$ be the Stone-Čech compactification of the discrete infinite countable space $\omega$, and $\beta^*\omega=\beta\omega\smallsetminus \omega$ is the Stone-Čech remainder.
The map $j:n\mapsto n+1$ extends to an self-injection of $\beta\omega$, which itself restricts to a self-homeomorphism $\phi$ of $\beta^*\omega$.
In ZFC+CH, is it true that $\phi$ and $\phi^{-1}$ are not conjugate in $\mathrm{Homeo}(\beta^*\omega)$?
Indeed in Shelah's model ("forcing axiom"), in which CH fails, there exists a homomorphism $\mathrm{Homeo}(\beta^*\omega)\to\mathbf{Z}$ mapping $\phi$ to $1$. So the non-conjugacy of $\phi$ with $\phi^{-1}$ is consistent. But under CH, the group $\mathrm{Homeo}(\beta^*\omega)$ is simple (Rubin) so the non-conjugacy couldn't be attested by a homomorphism to $\mathbf{Z}$ as above.
Note: Boolean algebraic translation through Stone duality: consider the endomorphism of the Boolean algebra $2^\omega$ of subsets of $\omega$ given by $A\mapsto \{a\in\omega:a+1\in A\}$. It induces an automorphism $\Phi$ of the quotient Boolean algebra $2^\omega/\mathrm{fin}$, where $\mathrm{fin}$ is the ideal of finite subsets. Is (under ZFC+CH) $\Phi$ non-conjugate to its inverse in $\mathrm{Aut}_{\mathrm{Ring}}(2^\omega)$?
Indeed Stone duality yields (in ZFC) an isomorphism $\mathrm{Homeo}(\beta^*\omega)\to\mathrm{Aut}_{\mathrm{Ring}}(2^\omega)$ mapping $\phi$ to $\Phi$.
Further comments:
A side question is whether it is consistent with ZFC that $\phi$ and $\phi^{-1}$ are conjugate, I don't know either (but I'm primarily interested in the CH case).
Also in ZFC it is easy to check that $\phi$ is not conjugate to $\phi^n$ for any $n\ge 2$.
REPLY [12 votes]: This is a great question -- and it's wide open. Here's what I know about it:
$\bullet$ As you mentioned, it is consistent that $\phi$ and $\phi^{-1}$ are not conjugate. This observation was first made by van Douwen, soon after the publication of Shelah's result that you mention in your question. You mentioned forcing axioms, so let me point out that the non-conjugacy of $\phi$ and $\phi^{-1}$ follows from $\mathsf{MA}+\mathsf{OCA}$, which is a weak form of $\mathsf{PFA}$. This is due to Boban Velickovic.
$\bullet$ If it is consistent with $\mathsf{ZFC}$ that $\phi$ and $\phi^{-1}$ are conjugate, then it is consistent with $\mathsf{ZFC}+\mathsf{CH}$. (Proof sketch: If $\phi$ and $\phi^{-1}$ are conjugate in some model, then force with countable conditions to collapse the continuum to $\aleph_1$ and make $\mathsf{CH}$ true. Because this forcing is countably closed, it won't change much about the Boolean algebra $\mathcal P(\omega)/\mathrm{fin}$, and will preserve the fact that $\phi$ and $\phi^{-1}$ are conjugate.)
$\bullet$ Even better, the existence of certain large cardinals implies that if it is possible to force "$\phi$ and $\phi^{-1}$ are conjugate" then this statement is already true in every forcing extension satisfying $\mathsf{CH}$. This follows from a theorem of Woodin concerning what are called $\Sigma^2_1$ statements about the real line (explained further here). The assertion "$\phi$ and $\phi^{-1}$ are conjugate" is an example of such a statement. (Very roughly, this theorem seems to suggest that if this statement is consistent, then it should follow from $\mathsf{CH}$. At any rate, trying to prove it from $\mathsf{CH}$ seems like a reasonable strategy.)
$\bullet$ In fact, Paul Larson has pointed out to me that the statement "$\phi$ and $\phi^{-1}$ are conjugate" is a now very rare example of a $\Sigma^2_1$ statement about the real line whose status we do not know under $\mathsf{ZFC}+\mathsf{CH}$ (plus large cardinal axioms).
$\bullet$ I proved a partial result a few years ago, showing that $\mathsf{CH}$ implies $\phi$ and $\phi^{-1}$ are semi-conjugate:
$\qquad$Theorem: Assuming $\mathsf{CH}$, there is a continuous surjection $Q: \omega^* \rightarrow \omega^*$ such that $$Q \circ \phi = \phi^{-1} \circ Q.$$
The paper is "Abstract $\omega$-limit sets," Journal of Symbolic Logic 83 (2018), pp. 477-495, available here. In the same paper, I show that the forcing axiom $\mathsf{MA}+\mathsf{OCA}$ implies $\phi$ and $\phi^{-1}$ are not semi-conjugate. (Or rather, I show that this is a corollary to a deep structure theorem of Ilijas Farah.)
$\bullet$ Finally, in a more recent paper (to appear in Topology and its Applications, currently available here), I show that there is no Borel set separating the conjugacy class of $\phi$ and the conjugacy class of $\phi^{-1}$ (in the space of self-homeomorphisms of $\omega^*$ endowed with the compact-open topology). Roughly, this shows that if $\phi$ and $\phi^{-1}$ fail to be conjugate, it's not "for any real reason" -- or at least not for any nicely definable topological reason.<|endoftext|>
TITLE: Are $f\sqrt{1+g^2}$ and $fg\sqrt{1+g^2}$ smooth if $f,fg,fg^2$ are smooth?
QUESTION [6 upvotes]: Suppose that $f$ and $g$ are functions from $\mathbb R$ to $\mathbb R$ such that the functions $f,fg,fg^2$ are smooth, that is, are in $C^\infty(\mathbb R)$. Does it then necessarily follow that the
functions $f\sqrt{1+g^2}$ and $fg\sqrt{1+g^2}$ are smooth?
Of course, the problem here is that the function $g$ does not have to be smooth, or even continuous, at zeroes of the function $f$.
One may also note that the continuity of the
functions $f\sqrt{1+g^2}$ and $fg\sqrt{1+g^2}$ (at the zeroes of $f$ and hence everywhere) follows easily from the inequalities $|f\sqrt{1+g^2}|\le|f|+|fg|$ and $|fg\sqrt{1+g^2}|\le|fg|+|fg^2|$.
REPLY [14 votes]: No.
Set
$$ f(x) = \exp(-2/|x|^2) \operatorname{sign} x, \qquad g(x) = \exp(1/|x|^2) \sqrt{|x|} \operatorname{sign} x $$
for $x \ne 0$, and, of course, $f(0) = g(0) = 0$. Then clearly
$$ \begin{aligned}
f(x) & = \exp(-2/|x|^2) \operatorname{sign} x , \\
f(x) g(x) & = \exp(-1/|x|^2) \sqrt{|x|} , \\
f(x) (g(x))^2 & = x
\end{aligned} $$
are infinitely smooth, but
$$ f(x) g(x) \sqrt{1 + (g(x))^2} = \sqrt{|x| \exp(-2/|x|^2) + |x|^2} = |x| (1 + o(1)) $$
is not even differentiable at $0$.<|endoftext|>
TITLE: Homotopy Gerstenhaber algebras: description via operads vs derivations
QUESTION [7 upvotes]: There are at least a couple of definitions in the literature for an $E_2$-algebra, also known as a homotopy Gerstenhaber algebra, also known as $G_{\infty}$-algebra.
Suppose $V$ is a graded vector space. Let $S(V), \text{Lie}(v)$ denote the graded symmetric algebra and the graded free Lie algebra of $V$ respectively and $S_+(V)$ be the symmetric algebra in strictly positive degree.
$\textbf{Definition (Tamarkin)}$ A $G_{\infty}$-algebra structure on $V$ is a degree $+1$ map $$ \delta: S_+(\text{Lie}(V^*[1])[1]) \rightarrow S_+(\text{Lie}(V^*[1])[1]) $$ which behaves as a derivation of both $\cdot$ and $[\,,]$ such that $\delta^2 = 0$.
Unpacking this leads to saying that we have a collection of maps $$ m_{k_1, \dots, k_n}: V^{\otimes k_1} \otimes \dots \otimes V^{\otimes k_n} \rightarrow V$$ of degree $3-(k_1 + \dots k_n + n)$ obeying appropriate symmetry and associativity relations.
The second definition pertains to algebras over the little disk operad. Let $D_2(k)$ denote the configuration space of $k$ little disks inside a big disk and let $\text{Chains}_{\bullet}(D_2(k))$ be the singular chain complex. Letting $\mathcal{P}(k) = \text{Chains}_{\bullet}(D_2(k))$, one can define an operadic structure on this collection of vector spaces. This leads one to the
$\textbf{Definition (Getzler-Jones?)}$ An $E_2$-algebra is an algebra over the operad $\text{Chains}(D_2)$.
How can one show that these two definitions are equivalent?
Short of a full proof of equivalence, it would be nice to understand a description of the cycle in $D_2(k_1 + \dots +k_n)$ which corresponds to the map $m_{k_1, \dots, k_n}$. For example: if one were working in $H_{\bullet}(D_2)$, the homology operad, one associates to the point class in configuration space of two disks the operation $\cdot = m_2$, and to the cycle involving one little disk going around the other the bracket $[\,,] = m_{1,1}$ in the Gerstenhaber algebra. Is there an explicit description of the cycle corresponding to $m_{k_1, \dots, k_n}$?
REPLY [4 votes]: Here are the operads that are involved in that game:
the operad $D_2$ of little disks, which is a topological operad.
its chain operad $C_{-*}(D_2,\mathbb{k})$, which is an operad in cochain complexes (of $\mathbb{k}$-modules).
its homology operad $H_{-*}(D_2,\mathbb{k})$, which is known to be isomorphic to the Gerstenhaber operad $G^{\mathbb{k}}$, which is itself a binary quadratic operad satisfying the Koszul property.
the minimal resolution of $G^{\mathbb{k}}$ is the operad $G^{\mathbb{k}}_\infty$ governing ($\mathbb{k}$-linear) $G_\infty$-algebras.
Being a resolution, $G^{\mathbb{k}}_\infty$ is obviously quasi-isomorphic to $G^{\mathbb{k}}$. As Phil Tosteson mentions in his answer, the difficult part relies on proving that $C_{-*}(D_2,\mathbb{k})$ is formal. It's only proven over a field $\mathbb{k}$ of characteristic zero, and it's actually not formal for $\mathbb{k}=\mathbb{F}_p$ (see e.g. the introduction of https://arxiv.org/pdf/1903.09191.pdf). To my knowledge, there are essentially two different proofs:
one by Tamarkin: https://sites.math.northwestern.edu/~tamarkin/Papers1/Formality.pdf.
another one by Kontsevich, which generalizes to higher dimension: https://arxiv.org/abs/math/9904055 (see also https://arxiv.org/abs/0808.0457 for more details).
Note that the formality quasi-isomorphisms from these two proofs happen to coincide, after the correct "choice of associator" has been made. See https://arxiv.org/abs/0905.1789.
Finally, I don't think it is meaningful to ask which cycle corresponds to the map $m_{k_1,\dots,k_n}$. The reason is that $m_{k_1,\dots,k_n}$ is not closed. You may ask if they are represented by nice chains... I don't know the answer to that question (and I suspect that it is close to be as hard as proving the formality itself), but for the $m_{1,\dots,1}$ the answer is known:
first observe that $D_2$ is weakly homotopy equivalent to the operad of compactified configuration spaces of points in the plane.
$m_{1,\dots,1}$ (with $n$ "$1$"s) can be represented by the top-dimensional/fundamental cell of the compactified configuration space $\overline{C}_n\simeq D_2(n)$ of $n$-points in the plane.<|endoftext|>
TITLE: Existence of a strange measure
QUESTION [36 upvotes]: The answer to this question must be known, but I do not know where to find it. It is related to the Ulam measures I believe.
Question. Is there a finitely additive measure defined on all subsets of positive integers $\mathbb{N}$, with values into $\{0,1\}$, (only two values) that is
$$
\mu:2^{\mathbb{N}}\to \{0,1\}
$$
such that $\mu(\{n\})=0$ for every $n\in\mathbb{N}$ and $\mu(\mathbb{N})=1$ ?
I believe such a result is needed in a proof of the co-area inequality that is stated in Coarea inequality, Eilenberg inequality. I am working with my student on some generalizations of that result so this is a research related question.
REPLY [5 votes]: This can be proved without introducing ultrafilters by name, by doing "finitary measure theory" and using Zorn's lemma.
An algebra $A$ on a set $X$ is just a $\sigma$-algebra without the $\sigma$, i.e. $\newcommand{\powerset}{\mathcal{P}}A \subseteq \powerset(X)$ and is closed under finite unions and complements (and therefore all other Boolean operations).
Let $\newcommand{\N}{\mathbb{N}}F \subseteq \powerset(\N)$ be the set of finite sets and their complements (so-called cofinite sets). This is an algebra. Furthermore, we can define a 2-valued finitely-additive measure $\mu : F \rightarrow \{0,1\}$ to be $0$ on the finite sets and $1$ on the cofinite sets. The existence of the required 2-valued finitely-additive measure on $\powerset(\N)$ then follows from:
Proposition For any algebra $A \subseteq \powerset(X)$ and finitely-additive 2-valued measure $\mu : A \rightarrow \{0,1\}$, there exists a finitely-additive measure $\overline{\mu} : \powerset(X) \rightarrow \{0,1\}$ extending $\mu$.
Proof: Most of the difficulty is in believing that it's true. We use Zorn's lemma. The poset consists of pairs $(B,\nu)$ where $B \supseteq A$ is an algebra of sets, and $\nu : B \rightarrow \{0,1\}$ is a finitely-additive measure extending $\mu$. The order relation $(B_1,\nu_1) \leq (B_2,\nu_2)$ is defined to hold when $B_1 \subseteq B_2$ and $\nu_2$ extends $\nu_1$. Every chain in this poset has an upper bound - we just take the union of algebras (this is the step that fails for $\sigma$-algebras) and define the measure on the union in the obvious way.
Let $(B,\nu)$ be a maximal element in the poset. Suppose for a contradiction that $B \neq \powerset(X)$, so there is some $U \in \powerset(X) \setminus B$. We contradict the maximality of $B$ by extending $\nu$ to a larger algebra $B'$ including $U$. Define $B' = \{ (U \cap S_1) \cup (\lnot U \cap S_2) \mid S_1, S_2 \in B \}$. It is clear that $B \subseteq B'$ and $U \in B'$, and with a little Boolean reasoning we can prove that for all $S_1,S_2,T_1,T_2 \in B$:
$$
((U \cap S_1) \cup (\lnot U \cap S_2)) \cup ((U \cap T_1) \cup (\lnot U \cap T_2))\\ = (U \cap (S_1 \cup T_1)) \cup (\lnot U \cap (S_2 \cup T_2))
$$
and
$$
\lnot ((U \cap S_1) \cup (\lnot U \cap S_2)) = (U \cap \lnot S_1) \cup (\lnot U \cap \lnot S_2)
$$
This proves that $B'$ is an algebra.
Now, define $d \in \{0,1\}$ to be the "outer measure" of $U$, i.e. $d = 0$ if there exists $S \in B$ such that $U \subseteq S$ and $\nu(S) = 0$, otherwise $d = 1$. Without loss of generality we can take $d = 1$, because we can exchange the roles of $U$ and $\lnot U$. We define $\nu'((U \cap S_1) \cup (\lnot U \cap S_2)) = \nu(S_1)$. This is well-defined because if $(U \cap S_1) \cup (\lnot U \cap S_2) = (U \cap T_1) \cup (\lnot U \cap S_2)$, then $U \cap S_1 = U \cap S_2$, so $U \subseteq \lnot (S_1 \triangle S_2)$, so as $d = 1$, $\nu(\lnot (S_1 \triangle S_2)) = 1$, and therefore $\nu(S_1) = \nu(S_2)$. The identities we used to prove that $B'$ is an algebra can then be used to prove that $\nu'$ is finitely additive, and it follows directly from the definition that it extends $\nu$. So we successfully contradicted the maximality of $B$. $\square$
Of course, I actually think ultrafilters are a good thing to know about, both in topology and logic. There is also no metamathematical benefit in doing it this way - over ZF the existence of a non-principal ultrafilter on $\N$ and a finitely-additive 2-valued measure on $\powerset(\N)$ are equivalent. The above proof is based on something I came up with while reproving Stone duality in the case where the points of the Stone space are defined to be Boolean homomorphisms into $2$, rather than ultrafilters. The proposition above is a special case of the fact that complete Boolean algebras (such as $2$) are injective objects in the category of Boolean algebras.<|endoftext|>
TITLE: Number of regions formed by $n$ points in general position
QUESTION [9 upvotes]: Given $n$ points in $\mathbb{R}^d$ in general position, where $n\geq d+1$. For every $d$ points, form the hyperplane defined by these $d$ points. These hyperplanes cut $\mathbb{R}^d$ into several regions. My questions are:
(1) is there a formula in terms of $d$ and $n$ that describes the number of regions?
(2) the same question for the number of bounded regions?
I tried many key words on google but found nothing helpful. Any reference or ideas will be appreciated. Thanks
REPLY [2 votes]: This non-answer completes Joseph O'Rourke's nice non-answer, for the case of $n$ hyperplanes in $\mathbb{R}^d$ in general position. But it also suggests that the OP situation may also well have unique answers.
Define:
$U_{d,n}=$ number of unbounded regions cut by $n$ hyperplanes in $\mathbb{R}^d$
$B_{d,n}=$ number of bounded regions
$T_{d,n}=$ total number of regions $=U_{d,n}+B_{d,n}$
$S_{d,n}=$ number of regions cut on the sphere $S^d$ by $n$ great $S^{d-1}$-circles
Then $U_{d,n}, B_{d,n}$, $T_{d,n}$ and $S_{d,n}$ are unique with these formulas:
$U_{d,0}=1$, $\quad B_{d,0}=0$, $\quad T_{d,0}=1$, $\quad S_{d,0}=1$
$U_{1,n}=2$, $\quad B_{1,n}=n-1$, $\quad T_{1,n}=n+1$, $\quad S_{1,n}=2n$
and for $n>0$
$U_{d+1,n}=S_{d,n}$
$S_{d,n}=U_{d,n}+2B_{d,n}$
$T_{d,n+1}=T_{d,n}+\sum_{i=0}^{i=d-1}{n\choose i}$
Proof.
In $\mathbb{R}^{d+1}$ take a huge and growing $S^d$ sphere, so that all the bounded regions zoom down to a point at the center of the sphere, the hyperplanes become great circles on the sphere and the unbounded regions corresponds to regions cut by the circles on the sphere. Therefore if the numbers are unique (as will be proved at the end) 1. follows.
Centrally project $\mathbb{R}^d$ onto a half $S^d$ (tangent to it). Complete the semisphere to a sphere by central symmetry. Then the hyperplanes become great circles, the $B_{d,n}$ bounded regions in $\mathbb{R}^d$ become $2B_{d,n}$ regions in $S^d$ and the $U_{d,n}$ unbounded ones become $U_{d,n}$ regions stretching across the suture line (equator) of the sphere. Again by unicity 2. follows.
Start with $\mathbb{R}^d$ and $n$ hyperplanes in generic position inside it. Now add a new hyperplane in generic position the following way:
first chose a point inside one region: no matter how that point is eventually stretched to a hyperplane, to it will split the region in two, for a gain of 1, or $n \choose 0$. Now stretch that point to a line: since it is a generic line it will meet each of the $n$ hyperplanes once and at each meeting the line will cross into one one new region and split it - with a gain of $n \choose 1$ new regions. Next stretch the line to a generic 2-plane, which will meet once each of the $(d-2)$-dimensional intersections of two hyperplanes; at each meeting the growing plane will arrive from having already crossed 3 of the 4 regions, to cross into the fourth and cut it; this a gain of another $n \choose 2$ regions.
In general as a generic $m-1$-plane grows to a generic $m$-plane it will meet all the $n \choose m$ $(n-m)$-dimensional intersections of $m$ hyperplanes, and each time it will go from cutting $2^m-1$ regions before crossing the intersection to cutting all $2^m$ after crossing, for a total gain of $n \choose m$ regions. This continues up to $m=d-1$, proving 3.
Proof of Unicity.
By induction:
$U_{d,0}$, $\quad B_{d,0}$, $\quad T_{d,0}$, $\quad S_{d,0}$ are unique;
$T_{d,n}$ unique $\implies$ $T_{d,n+1}$ unique (by the proof of 3.);
$U_{d,n}$ and $B_{d,n}$ unique $\implies$ $S_{d,n}$ unique (by the proof of 2. and the fact that the construction can be reversed in a non-unique way to show that $S_{d,n}=U_{d,n}+2B_{d,n}$ for some values of $U_{d,n}$ and $B_{d,n}$);
$S_{d,n}$ unique $\implies$ $U_{d+1,n}$ unique (by the proof of 1.);
$U_{d+1,n}$ and $T_{d+1,n}$ unique $\implies$ $B_{d+1,n}$ unique (as $B=T-U$).<|endoftext|>
TITLE: The meaning and purpose of "canonical''
QUESTION [23 upvotes]: This question is jointly formulated with Neil Barton. We want to know about the significance of canonicity in mathematics broadly. That is, both what it means in some detail, and why it is important.
In several mathematical fields, the term 'canonical' pops up with
respect to objects, maps, structures, and presentations. It's not
clear if there's something univocal meant by this term across
mathematics, or whether people just mean different things in different
contexts by the term. Some examples:
In category theory, if we have a universal property, the relevant
unique map is canonical. It seems here that the point is that the map
is uniquely determined by some data within the category. Furthermore, this kind of scheme can be used to pick out objects with certain properties that are canonical in the sense that they are unique up to isomorphism.
In set theory, L is a canonical model. Here, it is unique and definable. Furthermore, its construction depends only on the ordinals-- any two models of ZF with the same ordinals construct the same version of L.
In set theory, other models are termed 'canonical' but it's not
clear how this can be so, given that they are non-unique in certain
ways. For example, there is no analogue of the above fact for L with respect to models of ZFC with unboundedly many measurable cardinals. No matter how we extend the theory ZFC + "There is a proper class of measurables," there will not be a unique model of this theory up to the specification of the ordinals plus a set-sized parameter. See here.
Presentations of objects can be canonical: The most simple being
that of fractions, whose presentation is canonical just in case the
numerator and denominator have no common factors (e.g. the canonical
presentation of 4/8 is 1/2). But this applies to other areas too; see here.
Sometimes canonicity seems to be relative. Given a finite-dimensional vector space, there is a canonical way of defining an isomorphism between V and its dual V* from a choice of a basis for V. This determines a basis for V*, and thus the initial choice of basis for V yields a canonical isomorphism from V to V**. But two steps can be more canonical than one: The resulting isomorphism between V and V** does not vary with the choice of basis, and indeed can be defined without reference to any basis. See here.
Other examples can be found here.
Our soft questions:
(a) Does the term 'canonical' appear in your field? If so what is the
sense of the term? Is it relative or absolute?
(b) What role does canonicity play in your field? For instance, does it help to solve problems, help set research goals, or simply make results more interesting?
REPLY [2 votes]: Some of the examples given in the question are a careless misuse of the word. Who writes "canonical basis" for $K^n$ when they mean "standard basis", and who writes of a "canonical presentation of a fraction" when they mean a "fraction in its lowest terms", which isn't even canonical unless everything is positive.
In my field, arithmetic geometry, "canonical" has a well-understood meaning even if it is difficult to write down a precise definition. In his 1980 book, Milne was comfortable assuming that his readers would know what it meant (in his later writings, he has switched to using $\simeq$ for "canonically isomorphic"). Roughly, it means that the object can be constructed without making any arbitrary choices. There is a huge difference between saying two objects are isomorphic and saying they are canonically isomorphic. Barr botched this in his translation of Grothendieck's Tohoku paper by replacing "=" (meaning canonically isomorphic) with isomorphic.
I agree that the use of "canonical" is problematic in the Langlands program. There are major conjectures saying that some set (of representations) is bijective to some other set. After Serre pointed out that this only means that the two sets have the same cardinality, the word "canonical" was added. It is part of the problem to figure out what that means.<|endoftext|>
TITLE: Brown representability in slice category
QUESTION [7 upvotes]: Brown's representability theorem gives us a very nice set of conditions to check that a (contravariant) functor $Hot^{op}\rightarrow Set$ is representable. Choose an object $X$ in $Hot$. Then it is seems natural to ask whether or not an analogue of the Brown representability theorem is true for the slice category, i.e. if there exists a nice set of conditions to check whether or not a contravariant functor
$$F:(Hot/_X)^{op} \rightarrow Set$$
is representable. Is there such an analogue? I've searched online, but couldn't find an comments on the subject. My hope would be that this functor is representable if $F$ respects coproducts and sends homotopy pushouts to weak pullbacks.
One way I would hope to recover such a criterion is to consider the functor $$Hot\rightarrow Hot/_X, Y\mapsto (Y\rightarrow \{*\})$$
which by composition gives us a functor $Hot^{op}\rightarrow Set$, for which we know we can apply Brown's Theorem. I don't know however if this is enough to test representability of the functor $F$.
REPLY [6 votes]: Edgar Brown wrote two papers on this topic: one in the Annals in 1962 that focused on the category of topological spaces, and a second one "Abstract homotopy theory"
Trans. Amer. Math. Soc. 119 (1965).
His second paper is very axiomatic, and I believe your situation is easily checked to satisfy his properties. (Check this!)
This paper obviously predated Quillen's work on model categories (it likely partially inspired Quillen), so of course he doesn't use that language. I confess that I have always been a bit disturbed by the number of papers on Brown Representability written by authors who show no indication that they have ever looked at the original papers. Yes, people have written about more general versions (and less general, when they assume a triangulated category!) but I encourage folks to use their library resources to look at Brown's own work.<|endoftext|>
TITLE: Positively curved manifold with collapsing unit balls
QUESTION [7 upvotes]: Can we find a complete connected noncompact Riemannian manifold $(M^n,g)$ such that the curvature operator $Rm>0$ and
$$
\inf_{p \in M} \text{Vol}_gB(p,1)=0?
$$
REPLY [6 votes]: The answer is negative if $\dim M=2$ and positive otherwise, as shown in the paper:
Croke, C. B., & Karcher, H. (1988). VOLUMES OF SMALL BALLS ON OPEN MANIFOLDS: LOWER BOUNDS AND EXAMPLES. AMERICAN MATHEMATICAL SOCIETY (Vol. 309). https://www.ams.org/journals/tran/1988-309-02/S0002-9947-1988-0961611-7/S0002-9947-1988-0961611-7.pdf
The negative result is an immediate consequence of Theorem A, while the positive one follows from Example 1. Their construction for the latter is as follows:
Consider the $d$-dimensional hypersurface given by the paraboloid $\mathcal P = \{x\in \mathbb R^{d+1}\mid x_{d+1}=x_1^2+\ldots+x_d^2\}$. At any point $A$ of $\mathcal P$ one can consider its tangential cone $\mathcal C$, i.e., the Euclidean cone of vertex $V$ with axis along the line $AV$ and that intersects $\mathcal P$ tangentially. One then can consider $\tilde {\mathcal P}$ to be $\mathcal P$ with $\mathcal C$ "attached" (see the Figure at p.765).
This is a piecewise $C^\infty$ hypersurface, with a conical singularity at $V$ and that is $C^1$ at the intersection $\mathcal P\cap \mathcal C$. Outside of these regions, the curvature of $\tilde {\mathcal P}$ is $K>0$, since the curvature of $\mathcal P$ is positive and the one of the cone is zero. Moreover, it is always possible to smooth a conical singularity preserving the $K>0$ bound, and the same is true for the $\mathcal P\cap \mathcal C$ part. (You can maybe look at this thesis, e.g., Lemma 3.2.4.) So we have constructed a smooth hypersurface with $K>0$, and where we can estimate the volume of balls contained in the smoothed conical region by the volumes of the corresponding regions in the non-smoothed cones. Moreover, this operation can be done with a sequence of tangent cones $\mathcal C_n$, as soon as they are sufficiently distant one from the other.
The estimation of the volumes of the balls on the tangent cones is the object of the claim at the end of p.765. In particular, there the authors show that if the vertex of the cone is $V = (k+1,\ldots, 2k+1)$ for $k\ge 5$ (I'm fixing $r=1$), this volume is bounded by the volume of a "spherical spindle" :
$$
\operatorname{vol}(B(V,1))\le C_d\sin^{d-2}\theta ,
$$
where $\theta$ is an angle bounded by $\tan^2\theta\le 6/k$. (Here I chose $\varepsilon =1\le k^2/(2k+2)$.)
This shows that it is possible to attach to $\mathcal P$ a sequence of cones $\mathcal C_n$ with vertices $V_n\to +\infty$, and smooth them out so that
$$
\lim_{n\to +\infty}\operatorname{vol}(B(V_n,1))\le (C_d+1) \lim_{n\to +\infty} \sin^{d-2}\left(\theta_n\right) = 0.
$$
This proves that the obtained hypersurface (which has positive curvature) satisfies your requirement.<|endoftext|>
TITLE: What is the etale homotopy type of the Witt group of braided fusion categories?
QUESTION [6 upvotes]: The Witt group $\mathcal{W}$ of braided fusion categories (see also the sequel paper) can be defined over any field; I am happy to restrict to characteristic $0$ if it matters.
Is $\mathbb k \mapsto \mathcal W(\mathbb k)$ an (affine?) algebraic group scheme?
Assuming $\mathcal W$ is sufficiently scheme-like for the following question to make sense, what I really want to know is:
What is the etale homotopy type of $\mathcal{W}$?
REPLY [2 votes]: The answers to your questions are essentially in the first paper you cite. The second paper has more information on finer structure, like torsion, but the basic properties are all we need.
In the first paragraph of the introduction, the authors mention that any embedding of algebraically closed fields of characteristic zero yields an isomorphism on rational points, so it is geometrically a zero dimensional object. In particular, it is perhaps more profitable to think of it as a big Galois module. By Corollary 5.23, $\mathcal{W} \otimes \mathbb{Q}$ is a vector space of countably infinite dimension, so (assuming one manages to define $\mathcal{W}(R)$ for commutative $\mathbb{Q}$-algebras $R$) it is in fact an ind-affine group ind-scheme, rather than an affine group scheme.<|endoftext|>
TITLE: Volume of solution sets for polynomials in $\mathbb{C}[x]$
QUESTION [7 upvotes]: Denote $\pmb{a}=(a_1,\dots,a_d)\in\mathbb{R}^d$ and consider the set
$$\mathcal{E}_d=\{\pmb{a}\in\mathbb{R}^d: \text{each root $\xi$ of $x^d+a_dx^{d-1}+\cdots+a_2x+a_1=0$ lies in $\vert\xi\vert<1$}\}.$$
In the reference shown below, Fam proved that the $d$-dimensional Lebesgue measure satisfies
$$\lambda_d(\mathcal{E}_d)=2^d\prod_{k=1}^{\lfloor\frac{d}2\rfloor}\left(1+\frac1{2k}\right)^{2k-d}.$$
I'd like to propose a complex version here. Denote $\pmb{c}=(c_1,\dots,c_d)\in\mathbb{C}^d$ and consider the set
$$\mathcal{S}_d=\{\pmb{c}\in\mathbb{C}^d: \text{each root $\xi$ of $x^d+c_dx^{d-1}+\cdots+c_2x+c_1=0$ lies in $\vert\xi\vert<1$}\}.$$
Now, let's ask:
QUESTION. What is the $2d$-dimensional Lebesgue measure
$$\lambda_{2d}(\mathcal{S}_d)?$$
For contrast, $\lambda_1(\mathcal{E}_1)=2$ while $\lambda_2(\mathcal{S}_1)=\pi$.
Reference.
A. T. Fam,The volume of the coefficient space stability domain of monic polynomials, Proc. IEEE Int. Symp.Circuits and Systems, 2 (1989), pp. 1780–1783.
REPLY [5 votes]: $\def\CC{\mathbb{C}}\def\RR{\mathbb{R}}$The answer is $\tfrac{\pi^n}{n!}$. It is certainly a surprise to have the answer come out so simple!
Let $\phi : \CC^n \to \CC^n$ be the map which takes $(z_1, z_2, \ldots, z_n)$ to the elementary symmetric functions $(e_1, e_2, \ldots, e_n)$ where $e_k = \sum_{1 \leq i_1 < i_2 < \cdots < i_k \leq n} z_{i_1} z_{i_2} \cdots z_{i_n}$. Let $D$ be the unit disc in $\CC$. So you want to compute the volume of $\phi(D^n)$, also known as $\int_{\phi(D^n)} \mathrm{Vol}$.
The map $D^n \to \phi(D^n)$ is $n!$ to $1$ and, since $\phi$ is complex analytic, $\phi$ is orientation preserving. So
$$\int_{\phi(D^n)} \mathrm{Vol} = \frac{1}{n!} \int_{D^n} \phi^{\ast}(\mathrm{Vol}) = \frac{1}{n!} \int_{D^n} \det J^{\RR}_{\phi}$$
where $J^{\RR}_{\phi}$ is the Jacobian of $\phi$ considered as a smooth map $\RR^{2n} \to \RR^{2n}$. I will write $J^{\CC}_{\phi}$ when I instead want the $n \times n$ matrix of complex numbers coming from thinking of $\phi$ as a complex analytic map $\CC^n \to \CC^n$.
The relation between these two notions is this: $\det J^{\RR}_{\phi} = |\det J^{\CC}_{\phi}|^2$. (This is just linear algebra -- if $L/K$ is a degree $d$ field extension, $f: L^n \to L^n$ is a linear map and $g: K^{dn} \to K^{dn}$ is the linear map gotten by identifying $L$ with $K^d$, then $\det g = N_{L/K}(\det f)$.) We have the well known identity
$$\det J^{\CC}_{\phi}(z_1, \ldots, z_n) = \prod_{i
TITLE: Relative logarithmic cotangent bundle
QUESTION [5 upvotes]: Let $\mathcal X \rightarrow S$ be a flat family of projective varieties over a discrete valuation ring $S$ such that the generic fibre $\mathcal X_{\eta}$ (say) is smooth projective variety and the special fibre $\mathcal X_0$ (say) is a normal crossing divisor in $\mathcal X$.
Question: Does there exist a vector bundle $\mathcal E$ over $\mathcal X$ such that $\mathcal E|_{_{\mathcal X_{\eta}}}\cong \Omega^1_{_{\mathcal X_{\eta}}}$ and $\mathcal E|_{_{\mathcal X_0}}\cong \Omega^1_{_{\mathcal X_0}}(\mathrm{Log} D)$, where $D$ is the singular locus of $\mathcal X_0$?
It will be very helpful if anyone can explain this construction or provide a reference. Thank you..
REPLY [6 votes]: First of all, it's unclear what you mean by $\Omega^1_{X_0}(\log D)$ since $X_0$ is singular.
Second, if you make up such a definition then most probably such a vector bundle will not exist. Note that the Euler characteristics $\chi(X_\eta, \Omega^1_\eta)$ and $\chi(X_0, \Omega^1_{X_0}(\log D)$ have to agree. I suggest checking this for an elliptic curve degenerating to a node.
Finally, what people usually consider in your situation is the vector bundle $\Omega^1_{X/S}(\log X_0)$, so differentials on $X$ with log poles along the components of $X_0$, divided by the pull-back $t^{-1}dt$ where $t$ is a uniformizer of $S$. This is indeed locally free and restricts to $\Omega^1_{X_\eta}$.<|endoftext|>
TITLE: What is known about the non-existence of strongly regular graphs srg(n,k,0,2)?
QUESTION [6 upvotes]: Only few strongly regular graphs with parameters $\lambda=0$ (triangle-free) and $\mu=2$ (any two non-adjacent vertices
have exactly two common neighbors) are known, see the wikipedia page: the 4-cycle, the Clebsch graph and the Sims-Gewirtz graph.
I am looking for any information about the potential existence of more such graphs. For which values of $n$ and $k$ are they known not to exist?
REPLY [5 votes]: Example 1 in A.Neumaier paper says in partcular that the vertex degree in this case must be $k=t^2+1$, for $t$ not divisible by 4. As well, the number of vertices is $v=1+k+\binom{k}{2}$. The examples you list correspond to $t=2,3$. The next possible parameter set corresponds to $t=5$, so you have $v=352$, $k=26$. A.Brouwer's database lists this tuple of parameters as feasible, but no examples known. Similarly for $t=6,7$ you have feasible sets of parameters $v=704,1276$, resp. $k=37,50$, but no examples known.
To see that $k=t^2+1$, note that the 2nd eigenvalue of the adjacency matrix is
$$
r:=\frac{1}{2}\left[(\lambda-\mu)+\sqrt{(\lambda-\mu)^2 + 4(k-\mu)}\right]=-1+\sqrt{k-1},\quad \text{i.e. $t^2:=(r+1)^2=k-1.$}
$$
Similarly, the 3rd eigenvalue is $s:=-1-\sqrt{k-1}$, and one can compute their multiplicites, see e.g. Brouwer-van Lint, p.87
to rule out the case $t$ divisible by 4.
Namely, the multiplicity of $r$ is given by
$$
-\frac{k(s+1)(k-s)}{(k+rs)(r-s)}=\frac{k\sqrt{k-1}(k+1+\sqrt{k-1})}{4\sqrt{k-1}}=\frac{(t^2+1)(t^2+2+t)}{4},
$$
which cannot be an integer if $4|t$.<|endoftext|>
TITLE: Conceptual proof of classification of surfaces?
QUESTION [28 upvotes]: Every compact surface is diffeomorphic to $S^2$, $\underbrace{T^2\#\ldots \#T^2}_n$, or $\underbrace{RP^2\#\ldots \#RP^2}_n$ for some $n\ge 1$.
Is there a conceptual proof of this classification theorem?
Here, the term "conceptual" a little bit up for interpretation...
One may take it to mean: that doesn't rely on unintuitive facts.
For example, a proof that doesn't need $T^2\#RP^2\cong RP^2\#RP^2\#RP^2$ as an ingredient, but which has the property that this diffeomorphism comes out as a consequence would definitely count as conceptual.
REPLY [12 votes]: This is more an extended comment than an answer to the question. The first thing to note is that there are different strenghts of the classification theorem for surfaces. Of course, there are the differentiable, triangulated and topological setting. But even if we choose such a setting, there are two statements one has to prove (at least in one approach):
Every closed surface is isomorphic to a sphere with handles or cross-caps attached.
The isomorphism type only depends on the number of handles and cross-caps attached.
Both the Zip-proof and the Zeeman proof refered to above in the comments only prove the first part and not the second part. The second part is essentially equivalent to the well-definedness of the connected sum. Especially in the topological setting, this is a subtle point, requiring even in dimension 2 a kind of Schönflies theorem (and being a very difficult theorem in dimension 4). Out of frustration about this state of affairs I wrote up a variant of Zeeman's proof (based on a treatment by Thomassen), but including the well-definedness of attaching handles/cross-caps. (It took me a long time to thus actually understand the argument of Zeeman.)
This is not really an answer to the original question, both because this proof is in the triangulated setting and does not avoid the isomorphism of the original question. But I really want to stress the point that well-definedness of attaching handles or connected sum is something one has to prove and not just hide by abuse of notation.<|endoftext|>
TITLE: Geometry of algebraic curve determined by point counts over all number fields?
QUESTION [24 upvotes]: Let $C$ be a smooth (geometrically irreducible) projective curve of genus $g>1$ over a number field $K$. The Mordell conjecture (first proved by Faltings) says that for any finite field extension $L/K$ (say, inside a fixed algebraic closure $\overline{K}$) there is a function $\{$(isomorphism classes of) finite extensions $L/K$ inside $\overline{K}\} \rightarrow\ \{$non-negative integers$\}$ defined by $L \mapsto \#C(L).$ Does this function determine (the isomorphism class of) $C/\overline{K}$, or otherwise any geometric properties of $C$ such as the genus?
REPLY [6 votes]: Heuristically, we can compute the genus. This follows essentially the srategy of damiano and David Lampert in the comments.
First note that it is possible to compute from this function the number $\#'C(L)$ of points over $L$ with field of definition exactly $L$. (Here the field of definition is the smallest field containing $K$ that the point is defined over.) This follows from inclusion exclusion.
Now observe that if we let $$ r_d = \lim \sup_{X\to \infty} \frac{\log \sum_{L/K, |\Delta_{L/K}|0$, I think it is reasonable to expect that $d$ is the gonality of $C$ and that $r_d = \frac{1}{ g+ d-1}$, so that the genus of $C$ is $\frac{1}{r_d} +1-d$.
Why do I think this? Consider first a degree $d$ map $\pi: C \to \mathbb P^1$. Associated to a point of $x\in \mathbb P^1(K)$, of which there are $\approx Y^2$ of height $
TITLE: What types are to mathematical proofs as types à la Martin-Löf are to constructive proofs, and what's wrong with them?
QUESTION [7 upvotes]: The question is motivated by this surprising sentence from Freek Wiedijk's The QED Manifesto Revisited.
I agree that the QED-like systems that exist today are not good enough
to start developing a library as is described in the QED manifesto.
Surprising to me, that is, who had not heard of the QED project before.
(Edited after A. Bauer's answer) OK, time I mopped up.
I was 1st introduced to the Curry-Howard correspondence back in school days long, loooooong ago; I understood it as s/th along the lines of "Any computer program is a constructive proof that inhabitation of its input type entails inhabitation of its output type"; my very next question was, "OK then, if instead of any program we restrict ourselves to programs that model mathematical proofs in classical logic, what would be the proper type system(s) for them?". I guessed it had to be existential types, guarded subtypes (meaning types restricted by any formula; so that you would write e. g. "let x: {r : Real | !(r : Rational)}" if all you mean is to express that x is irrational), or such like stuff; then I let the matter rest and turned to more pressing ones, to wit, getting a degree.
If I had pressed it further at the time &, instead of the title question, asked "What types are to classical logic as simple types are to constructive logic?", maybe I'd have been answered "There are no such types: there is no need to extend or otherwise modify simple types for classical logic, since it is encompassed in constructive logic. Go get some understanding of type theory, and learn how to do with them. "
If I had asked again at the time the QED manifesto was issued, a plausible answer might have turned to "They're called refinement types. What's wrong with them is, they're a pointless gadget: the problem they solved was not a foundational one, and introducing them did not present type theory with any new challenge. So all the papers they deserve have been written already. "
Returning to it now, it seems to me the natural way to attack the QED challenge would be: 1. to set forth a serious answer to my old question, 2. design the corresponding language, 3. build the proper language tools, including, in the end, a full-fledged proof assistant (probably posing as a type checker), 4. look what happens. In a programming language with refinement types, the C-H correspondence now works as: subprograms returning (refinements of) void model conjectures, their bodies model proof outlines, assertions in the bodies model steps in the proof, Skolem functions arise naturally if handlers for failed assertions are allowed. Then, verified theorems in predicate logic are those subprograms in which the type checker has been able to reduce all assertions to tautologies.
What hopefully would happen if a suitable extension to such a language had become popular is, useful tools could start developing, such as doclets to turn fragments of source code to academic papers; people might get used to writing in it just to convert formulas from .pdf to .ps, or to save themselves the hassle of adjusting to the style guides of every other journal they submit to; later, we might dream of replacing arXiv with a global GitHub repository of their works. This could take place long before any proof assistant worth mentioning existed for the language.
Given Wiedijk's paper though, the real story must have been quite different. It reads like people trying to endow dependent types with refinement soon hit some snag and, instead of developing proof assistants (Edit: read formal languages and tools) for classical logic, they went happily to rewrite classical math for constructive logic.
(Edit) With predictable results: mathematicians have been accepting a certain style of proof outline for some time now and non-mathematicians are used to it, so if it is too arcane work to make it understandable by a automated checker, they will rather dispose of the proof checker than of the proof.
Turn away for a minute from what dependent types mean, and look how they would be used in a language purporting to be strongly typed. The user is student X with a major in control systems, a hobby in Java programming, currently at grips with an intro to topological groups. So far, he has managed to translate "pick any dyadic integer " as "let x: Integer(2)" for proof-checking freeware he downloaded a minute ago, then "consider first the case of a rational integer" to "let x1:Integer(2)*Rational"; now attacking "pick some irrational number x and consider the fractional parts of the p.x's, p ranging over Z: obviously, blah blah blah". Not understanding a word of type theory, he first looks for a way to say "let x: Real - Rational", finds none, looks harder, still finds none, then let go & turns back to more pressing matters, to wit, control systems.
Of course, if student X had access to such a language as I sketched, he would have given up long before he got any proof automated. No harm done, though: what he wants is to understand TG's, not that a computer program understand it in his stead, nor to understand type theory. All he needs is to keep his fingers busy while reading, and I advocate giving him a productive way to do just that.
My read of Wiedijk's paper is, if you bar student X from modelling things the way he understands them, he will turn to what he considers more pressing matters, regardless of how pressing your need for computer-checkable proofs, and how deep your understanding of what the modelling language guarantees. It explains how we still lack a free-access, widely-known, low-quality, multi-audience, computer-interpreted codebase of mathematical theories the way we have a free-access, widely-known, low-quality, multi-audience, human-readable textbase of common lore.
Edit Just to clarify: the codebase of mathematical theories in question would still be as far away from being computer-checked, as is WP from auto-generating ontologies for the subject matter of its articles; light-yrs away. The (hopeful) progress lies in the rest of the road being mostly for metamathematicians & software engineers to tread, with the part of "ordinary" mathematicians perceived as non-problematic.
Hence my question: what type systems are the natural ones to express assertions in classical logic? Or maybe I should have titled this post "What types are to dependent types, as refinement types à la Freeman-Pfenning are to simple types?". Anyway, what have they that makes their theory so unsavory?
REPLY [12 votes]: A good starting point to learn about type theories for classical logic is the $\lambda\mu$-calculus introduced in 1992 by Parigot in λμ-Calculus: An algorithmic interpretation of classical natural deduction, which extend the $\lambda$-calculus to give a computational interpretation of classical natural deduction.
For the next step, I would recommend reading the research summary on Hugo Herbelin's home page. Chasing some of the references listed there, you will learn how incorporating control operators into type theory yields a computational understanding of excluded middle.
For a modern take on the role of excluded middle in Martin-Löf type theory I recommend reading at least Section 3.4 of the HoTT book. The important bit to take away from there is the fact that excluded middle can coexist with the rest of homotopy type theory quite naturally.
You speak of proving correctness of programs, possibly using classical logic. For that you can use a system based on refinement types, such as F*. Also note that proving correctness of programs is an activity unlike formalization of traditional mathematics, so do not expect a single tool to work equally well for both of them.
In my opinion the presence or lack of excluded middle in a proof assistant has very little to do with it being useful for formalization. Of course, if you want to formalize classical mathematics then you need excluded middle (but much less frequently than most mathematicians expect). Most modern proof assistants let you postulate excluded middle quite directly and use it to your heart's content, so that cannot be the show stopper. One of the most popular proof assistants is Isabelle/HOL, which has excluded middle built in, but that does not make it significantly more successful in comparison with other formalization tools.
It is a bit difficult to understand what precisely you are getting at in your long discussion, but I would venture to say that you've misidentified negation and excluded middle as the culprit. I am not even sure whether you think that Martin-Löf type theory has no notion of negation – of course it does! In fact, the problems faced by your student X has nothing whatsoever to do with type theory. Student X would face essentially the same obstacles if they used any other formal system, such as first-order logic and set theory, or higher-order classical logic. Your student X is quite naive to think that formalization of mathematics is just a simple exercise in translation from English prose. It may be true that usability is decided by the users, but the users' abilities play a role as well.<|endoftext|>
TITLE: Precise probability that $m$ random vectors in $n$ dimensional space are nearly-orthogonal
QUESTION [5 upvotes]: Consider $m$ vectors $v_1,\dots,v_m$ in $\mathbb R^n$, drawn uniformly and independetly from unit sphere. It is pretty straightforward from Chebyshev inequality that
$$
\mathrm P (\forall i\ne j \ |v_i \cdot v_j|\leq \varepsilon) \to 1\ \text{as} \ n \to \infty.
$$
But what about quantitative version of this limit, i.e. if we define
$$
f(m, \varepsilon, \delta) = \min\{n : \mathrm P(\forall i\ne j\ |v_i \cdot v_j| \leq \varepsilon)\geq 1 - \delta\}
$$
what can we say about asymptotic behavior of $f$?
REPLY [4 votes]: $\newcommand{\ep}{\varepsilon}
\newcommand{\Ga}{\Gamma}
\newcommand{\de}{\delta}$
For $\ep\in(0,1)$, let
\begin{equation*}
P_{m,n}:=P\Big(\bigcap_{1\le i\ep\}\Big).
\end{equation*}
By Bonferroni inequalities,
\begin{equation*}
Mp\ge Q_{m,n}\ge Mp-R/2,
\end{equation*}
where
\begin{equation*}
p:=P(|v_1\cdot v_2|>\ep),
\end{equation*}
\begin{equation*}
M:=m(m-1)/2,
\end{equation*}
\begin{equation*}
R:=\sum_{1\le i\ep,|v_k\cdot v_l|>\ep).
\end{equation*}
If $\{i,j\}\cap\{k,l\}=\emptyset$, then $P(|v_i\cdot v_j|>\ep,|v_k\cdot v_l|>\ep)=P(|v_i\cdot v_j|>\ep)\,P(|v_k\cdot v_l|>\ep)=p^2$.
If $\{i,j\}\cap\{k,l\}\ne\emptyset$ but $\{i,j\}\ne\{k,l\}$, then, using the iid condition on the $u_i$'s and the spherical symmetry, for (say) the unit vector $e_1$ of the standard orthonormal basis of $\mathbb R^n$, we have
\begin{multline}
P(|v_i\cdot v_j|>\ep,|v_k\cdot v_l|>\ep)
=P(|v_1\cdot v_2|>\ep,|v_1\cdot v_3|>\ep) \\
=P(|e_1\cdot v_2|>\ep,|e_1\cdot v_3|>\ep)
=P(|e_1\cdot v_2|>\ep)\,P(|e_1\cdot v_3|>\ep)=p^2.
\end{multline}
So, $P(|v_i\cdot v_j|>\ep,|v_k\cdot v_l|>\ep)=p^2$ for any $i,j,k,l$ such that $1\le i\ep)=K_nI_n,
\end{equation*}
where, with $n\to\infty$,
\begin{equation*}
K_n:=\frac{\Ga(n/2)}{\Ga(1/2)\Ga((n-1)/2)(n-1)^{1/2}}\to1/\sqrt\pi,
\end{equation*}
\begin{equation*}
I_n:=(n-1)^{1/2}\int_{\sqrt c}^\infty(1+t^2)^{-n/2}\,dt=e^{-nc/(2+o(1))},
\end{equation*}
\begin{equation*}
c:=\frac{\ep^2}{1-\ep^2}.
\end{equation*}
Collecting the pieces, we see that
\begin{equation}
Q_{m,n}=Me^{-nc/(2+o(1))}.
\end{equation}
Setting now $\de=Q_{m,n}\to0$, we find the asymptotics of the needed $n$:
\begin{equation}
n\sim2\frac{1-\ep^2}{\ep^2}\,\ln\frac{m(m-1)}{2\de}.
\end{equation}<|endoftext|>
TITLE: Free product decompositions of the fundamental group of Hawaiian Earrings
QUESTION [11 upvotes]: This is a spin-off of my question here, separated from the older question following Jeremy's suggestion.
Definition. Call a group $G$ essentially freely indecomposable if in every free product decomposition $G\cong G_1\star G_2$, one of the free factors $G_1, G_2$ is free of finite rank.
Question. Let ${\mathbb H}$ denote the Hawaiian Earrings. Is the fundamental group $\pi_1({\mathbb H})$ essentially freely indecomposable?
The only relevant result I could find in the literature is the theorem (due to Higman) which (according to "The combinatorial structure of the Hawaiian earring group" by Cannon and Conner) implies that that every freely indecomposable free factor of $G=\pi_1({\mathbb H})$ is either trivial or infinite cyclic. Maybe Higman's methods prove more, but his paper ("Unrestricted Free Products, and Varieties of Topological Groups", Journal of LMS, 1952) is behind the paywall.
REPLY [9 votes]: This answer is courtesy of Sam Corson who kindly pointed out the following.
Theorem: The Hawaiian earring group $\pi_1(\mathbb{H})$ is essentially freely indecomposable, i.e. if $\pi_1(\mathbb{H})\cong G_1\ast G_2$, then one of $G_1$ or $G_2$ must be a finitely generated free group.
The key is to apply a special case of Theorem 1.3 in
K. Eda, Atomic property of the fundamental groups of the Hawaiian earring and wild locally path-connected spaces, J Math. Soc. Japan 63 (2011), 769-787.
Let $C_n$ be the $n$-th circle of $\mathbb{H}$. Take $\mathbb{H}_{\geq n}=\bigcup_{k\geq n}C_k$ to be the smaller copies of the Hawaiian earring and $\mathbb{H}_{\leq n}=\bigcup_{k=1}^{n}C_k$ to be the union of the first $n$-circles. One of the defining properties of the Hawaiian earring group is that there is a canonical isomorphism $\pi_1(\mathbb{H})\cong \pi_1(\mathbb{H}_{\geq n+1})\ast \pi_1(\mathbb{H}_{\leq n})$ for all $n\in\mathbb{N}$.
Suppose that $\phi:\pi_1(\mathbb{H})\to G_1\ast G_2$ is an isomorphism. In the case of the Hawaiian earring, Eda's theorem cited above implies that for any homomorphism $\phi:\pi_1(\mathbb{H})\to G_1\ast G_2$, there exists an $n\in\mathbb{N}$, $i\in\{1,2\}$, and $w\in G_1\ast G_2$ such that $\phi(\pi_1(\mathbb{H}_{\geq n+1}))\leq w G_i w^{-1}$. Suppose, without loss of generality, that $i=1$. Now if $\gamma(g)=w^{-1}gw$ is conjugation in $G_1\ast G_2$, then $\gamma\circ \phi:\pi_1(\mathbb{H})\to G_1\ast G_2$ is an isomorphism mapping $\pi_1(\mathbb{H}_{\geq n+1})$ into $G_1$. Let $\psi:G_1\ast G_2\to G_2$ be the projection, which is surjective. Then $\psi\circ\gamma\circ\phi:\pi_1(\mathbb{H})\to G_2$ is a surjection and since $\pi_1(\mathbb{H})\cong \pi_1(\mathbb{H}_{\geq n+1})\ast \pi_1(\mathbb{H}_{\leq n})$ and $\psi\circ\gamma\circ\phi(\pi_1(\mathbb{H}_{\geq n+1}))=1$, we must have that $\psi\circ\gamma\circ\phi$ maps $\pi_1(\mathbb{H}_{\leq n})$ onto $G_2$. Therefore, $G_2$ is finitely generated. Moreover, since $G_2$ is isomorphic to a subgroup of the locally free group $\pi_1(\mathbb{H})$, it follows that $G_2$ is free.<|endoftext|>
TITLE: Maximal dimension of abelian subalgebra of exceptional simple Lie algebra in positive characteristic
QUESTION [6 upvotes]: For complex semisimple Lie algebras, the maximal dimension of an abelian subalgebra was determined by Mal'cev in 1945. For $E_7$, for example, it is $27$, and is the radical of the $E_6$ parabolic.
What about in characteristic $p$ for $p>0$? I suspect the answer is nearly, but not quite, the same. Maybe you have that it is bounded by $29$ or something. Is there any literature on this? All of the papers and references I have found so far are in characteristic $0$.
I don't need the exact bound, just something close will do. (I have a Lie algebra $L$ with an abelian subalgebra of dimension, say, $43$, and I want to know that $L$ cannot be embedded in $E_7$, and in particular is not $E_7$. But I'm in characteristic $7$, for example, or worse, $2$ or $3$.)
REPLY [4 votes]: Let $\ell\ge0$ be the characteristic of the (algebraically closed) ground field. Let $G$ be semisimple with Lie algebra $\mathfrak g$.
First, the maximal dimension of an abelian subalgebra of $\mathfrak g$ can increase for small $\ell$. Let, e.g., $\mathfrak g=sl(2)$ and $\ell=2$. Then the Borel subalgebra is abelian, hence the maximal dimension is $2$ instead of $1$. The same holds for $\mathfrak g=pgl(2)$ and $\ell=2$ where $\langle e,f\rangle$ is abelian.
Secondly, this is a phenomenon of very small characteristics. The following statement is not the most general one but covers most cases:
Assume that $G$ is of adjoint type and $\ell>3$. Then the dimension of a maximal abelian subalgebra is the same as in characteristic $0$.
Proof: Key is the following deformation argument: Being an abelian subalgebra is a closed condition in the Grassmannian of $\mathfrak g$. In other words, there is a closed subscheme $A_d\subseteq Gr_d(\mathfrak g)$ classifying abelian subalgebras of dimension $d$. The group $G$ acts on $A_d$ by conjugation. Assume that $A_d\ne\emptyset$. Since $A_d$ is projective, any Borel subgroup $B$ of $G$ has a fixed point. This shows, that if $\mathfrak g$ contains an abelian subalgebra $\mathfrak a$ of dimension $d$ then it will also contain one which is normalized by $B$. Assume from now on that this is the case.
Then, in particular, $\mathfrak a$ will be normalized by a maximal torus $T\subseteq B$. Let $\mathfrak g=\mathfrak t\oplus\bigoplus_\alpha\mathfrak g_\alpha$ be the root space decomposition. Then any $T$-stable subspace, so also $\mathfrak a$, has the form
$$
\mathfrak a=\mathfrak a_0\oplus\bigoplus_{\alpha\in S}\mathfrak g_\alpha
$$
where $S$ is a set of roots.
Since $\mathfrak a$ is ablian, its subset of semisimple elements $\mathfrak a_0$ inside $\mathfrak b$ is also normalized by $B$. Hence $\mathfrak a_0$ consists of fixed points of $B$ and therefore of $G$. So $\mathfrak a_0$ sits in the schematic center of $\mathfrak g$. Because $G$ is of adjoint type we infer $\mathfrak a_0=0$.
Thus everything depends on $S$. I argue that the condition on $S$ making $\mathfrak a=\bigoplus_{\alpha\in S}\mathfrak g_\alpha$ abelian is independent of $\ell$, proving our assertion.
For this let $e_\alpha\in\mathfrak g_\alpha$ be a Chevalley generator. Then $[e_\alpha,e_\beta]$ must be zero for all $\alpha,\beta\in S$. If $\alpha+\beta=0$ then $h_\alpha=[e_\alpha,e_\beta]\ne0$ since $\ell\ne2$. So this case must not occur. If $\gamma=\alpha+\beta$ is a root then a famous formula of Chevalley asserts
$$
[e_\alpha,e_\beta]=\pm N_{\alpha\beta}e_\gamma\quad\text{with }N_{\alpha\beta}\in\{1,2,3\}.
$$
Because $\ell>3$, by assumption, we get $[e_\alpha,e_\beta]\ne0$. So this case must not occur either. In the remaining cases we have $[e_\alpha,e_\beta]=0$. The condition on $S$ is therefore: $\mathfrak a$ is an abelian subalgebra if and only if $\alpha+\beta$ is not zero and not a root for all $\alpha,\beta\in S$. This condition is clearly characteristic free.<|endoftext|>
TITLE: Characterisation of essentially algebraic theories as monads
QUESTION [7 upvotes]: The following correspondence between algebraic theories and monads on $\mathbf{Set}$ is well-known (see, for example, Algebraic Theories: A Categorical Introduction to General Algebra).
The category of (finitary) $S$-sorted algebraic theories is equivalent to the category of (finitary) monads on $\mathbf{Set}/S$. The category of models for a fixed algebraic theory $T$ is equivalent to the category of algebras for the corresponding monad.
I would like to know if there is a known analogous result in the setting of essentially algebraic (i.e. finite limit) theories. That is, some statement like the following.
The category of (finitary) $S$-sorted essentially algebraic theories is equivalent to the category of [some class of] monads on [some category]. The category of models for a fixed algebraic theory $T$ is equivalent to the category of algebras for the corresponding monad.
There are many characterisations of categories of models of essentially algebraic theories (i.e. locally presentable categories), but I have not been able to find one in terms of a category of algebras for some monad. Given the many generalisation of Linton's result, for example in Notions of Lawvere theory and Monads with arities and their associated theories, it would seem like this result should be a straightforward application of an existing theorem, but, as far as I can tell, the case of essentially algebraic theories is never explicitly treated. (I originally read Theorem 6.7 of Notions of Lawvere Theory as stating that one-sorted essentially algebraic theories also correspond to finitary monads on $\mathbf{Set}$, but this seems unlikely to be correct.)
If the $S$-sorted case is unknown, I am also interested in the correspondence specifically in the one-sorted setting.
REPLY [3 votes]: I'm going to give a partial answer to my question, which addresses a misconception I had and illustrates why many of the existing generalisations of theory–monad correspondence are not sufficient to provide a monadic correspondence with essentially-algebraic theories. As far as I know, this indicates that a correspondence is open.
Let $\mathbb C$ be a small category, and denote by $\widehat{\mathbb C}$, $\mathbf{Rex}(\mathbb C)$, $\mathbf{Ind}(\mathbb C)$ and $\mathbf{Lex}(\mathbb C)$ the free cocompletion, finite cocompletion, filtered cocompletion and finite completion of $\mathbb C$, respectively. We have $\mathbf{Ind}(\mathbf{Rex}(\mathbb C)) \simeq \widehat{\mathbb C}$.
We'll instantiate the results of Bourke & Garner's Monads and theories for $K : \mathbf{Rex}(\mathbb C) \hookrightarrow \widehat{\mathbb C}$. This is an example of the so-called "presheaf context". The results follow directly, so I won't spell everything out. $K$ preserves finite colimits and so a $\mathbf{Rex}(\mathbb C)$-theory is a finite colimit-preserving identity-on-objects functor $J : \mathbf{Rex}(\mathbb C) \to \mathcal T$. A model is a finite-limit preserving functor $F : \mathcal T^{\mathrm{op}} \to \mathbf{Set}$. A $\mathbf{Rex}(\mathbb C)$-nervous monad is a monad on $\mathbf{Rex}(\mathbb C)$ whose underlying endofunctor preserves filtered colimits. The categories of $\mathbf{Rex}(\mathbb C)$-nervous monads and $\mathbf{Rex}(\mathbb C)$-theories are equivalent by Theorem 17 of ibid. (and similarly their categories of algebras and categories of models by Theorem 34 of ibid.)
We can now take opposites appropriately to generalise the classical algebraic theory–finitary monad correspondence: the categories of finite limit-preserving identity-on-objects functors from $\mathbf{Lex}(\mathbb C^{\mathrm{op}})$ (which I'll dub "category-sorted lex theories") are equivalent to finitary monads on $\widehat{\mathbb C}$. When $\mathbb C = S$ is discrete, category-sorted lex theories are equivalent to sorted algebraic theories (this can be seen as a consequence of sifted colimits and filtered colimits coinciding on indexed sets, or of the coincidence of finite completion and finite product completion on sets). When $\mathbb C$ is a non-discrete category, the two constructions differ. This essentially gives a classification of finitary and sifted colimit-preserving monads on presheaf categories (at least on small categories), which was suggested by Simon Henry.
I was previously assuming that category-sorted lex theories (at least for discrete categories) was the right notion of essentially algebraic theory. However, I was mistaken: category-sorted lex theories are less expressive than sorted algebraic theories. In particular, the requirement for theories to be identity-on-objects is too restrictive, as the codomains may no longer be finitely complete. An analogous definition of sorted essentially algebraic theory to sorted algebraic theory will look different (as far as I'm aware, no such characterisation has been given in the literature) and, as such, the existing theory–monad correspondences are insufficient to describe a correspondence for essentially algebraic theories. I'll continue to pursue this question according to this line of thinking, but unless anyone can see that I've missed something obvious, I'm satisfied that such a correspondence is at least not known (or easily derivable from results in the literature).<|endoftext|>
TITLE: An "analytic continuation" of power series coefficients
QUESTION [43 upvotes]: Cauchy residue theorem tells us that for a function
$$f(z) = \sum_{k \in \mathbb{Z}} a(k) z^k,$$
the coefficient $a(k)$ can be extracted by an integral formula
$$a(k) = \frac{1}{2\pi i}\oint f(z) z^{-k-1},$$
with a contour around zero. Now, there is nothing to prevent us from thinking of $a\colon \mathbb{Z} \to \mathbb{C}$ as a function over the complex domain, defined by the above integral. In this way, we have naturally "continued" the function $a(k)$ over the integers to one over the complex numbers. Is there any meaning to this beyond just something amusing?
For example, consider $f(z)=\exp(z)$. In this case, $a(k)=1/k!$ which one would think would be continued to $1/\Gamma(1+k)$. But at least if you attempt to plot what happens above, this is not the case. We get a function with Bessel-like oscillations (blue) which nevertheless coincides with $1/\Gamma(1+k)$ (orange) on integral points!
One can experiment with other choices of $f(z)$. When $f(z)=(z+1)^n$, it seems like the continuation actually coincides with what one would naturally think (the Beta function). Which is actually quite strange. For example for $n=5$ and $f(z) = 1 + 5 z + 10 z^2 + 10 z^3 + 5 z^4 + z^5$, how does this computation "learn" that the sequence $1,5,10,10,5,1$ actually corresponds to the binomial coefficients and thus should be continued to the Beta function? If we perturb one of the coefficients, say change $5 z^4$ to $2 z^4$, the result would look like a "perturbed" Beta function (blue below vs the Beta function in orange).
Any explanation (or references in the literature) for what is going on here?
REPLY [6 votes]: Edit (1/21/21): (Start)
For your first example, a classic method of interpolation is related to fractional calculus.
$$k!\; a(k) = k! \; \oint_{|z|=r} \frac{e^z}{z^{k+1}} \; dz = e^{-1}k! \; \oint_{{|z|=r}} \frac{e^{z+1}}{z^{k+1}} \; dz $$
$$= e^{-1}k! \; \oint_{|z-1|=1} \frac{e^{z}}{(z-1)^{k+1}} \; dz =e^{-1} D^k_{z=1} e^z.$$
Interpolating using a standard fractional integroderivative, of which there are several reps,
$$\lambda! \; a(\lambda) = \; e^{-1} D_{z=1}^{\lambda} \; e^z = e^{-1} \; \sum_{n \ge 0} \frac{z^{n-\lambda}}{(n-\lambda)!}\; |_{z=1}$$
$$ = e^{-z} \; \sum_{n \ge 0} \frac{z^{n-\lambda}}{(n-\lambda)!}\; |_{z=1} = e^{-z} z^{-\lambda}\; E_{1,-\lambda}(z) \; |_{z=1},$$
where $E_{\alpha,\beta}(z)$ is the Mittag-Leffler function (general definition in Wikipedia, MathWorld; some history and applications), encountered very early on by anyone exploring fractional calculus.
This method of interpolation gives the entire function
$$ a(\lambda) = e^{-1} \; E_{1,-\lambda}(1) \frac{1}{\lambda!} = e^{-1} \; \sum_{n \ge 0} \frac{1}{(n-\lambda)!} \; \frac{1}{\lambda!}, $$
which matches the OP's first graph and extends it to any real or complex argument, giving, of course, $a(k) = 1/k!$ for $k=0,1,2, ...$.
Similarly, the fractional calculus can be applied to usefully interpolate coefficients generated by $ D_z^n \; f(z)$ to those of $D_z^{\lambda} f(z)$, with care taken with defining branch cuts and contours of integration for the complex function for the point of evaluation. The fundamental example is Euler's integral rep of the beta function as depicted below. Another is the interpolation of the associated Laguerre polynomials to the confluent hypergeometric functions and, therefore, interpolations of the associated coefficients. (See links below.)
Other productive methods of interpolation--all can be related to fractional calculus--are sketched below.
(End)
Edit 1/23/21: (Start)
The interpolation you desire is obtained by applying Ramanujan's favorite Master Formula, a.k.a. Mellin transform interpolation, as discussed below and illustrated in four other MO-Qs--Q1, Q2, Q3, and Q4. This, naturally, amounts to replacing $k$ by $s$ only outside your Cauchy integral rather than within for the reason Terry Tao notes, which is equivalent to noting the Cauchy contour integration is not equal to the Mellin transform integration.
(End)
The Mellin transform pair allows for interpolation of the coefficients of generating functions, often directly connected to sinc and/or Newton interpolation. Basically, the following is a sketch of the analytics of Ramanujan's Master Formula/Theorem, which he so profoundly and elegantly wielded.
Here $(a.)^n := a_n$ in the Taylor series $e^{a.x}$ is interpolated as $a_{-s}$ via the Mellin transform of the Taylor series $f(x) = e^{-a.x}$.
First consider the normalized Mellin transform and its inverse
$$F(s) = MT[f(x)] = \int_{0}^{\infty} f(x) \; \frac{x^{s-1}}{(s-1)!} \; dx$$
$$f(x) = MT^{-1}[F(s)] = \frac{1}{2 \pi i} \int_{\sigma - i \infty}^{\sigma + i \infty} \frac{\pi}{\sin(\pi s)} F(s) \frac{x^{-s}}{(-s)!} \; ds .$$
Then the RMT holds for a class of functions such that the simple poles of the inverse sine in the inverse Mellin transform give
$$f(x) = e^{-a.x} = \sum_{n \geq 0} \frac{(-a.x)^n}{n!} = \sum_{n=0} a_n \frac{(-x)^n}{n!} =
\sum_{n=0} F(-n) \frac{(-x)^n}{n!} \; ,$$
that is, such that we may close the complex contour to the left (e.g., in the sense of the limit of a semicircle with its radius expanding to infinity) for $0 < \sigma < 1$ and $0 < x < 1$ when $F(s)$ has no singularities/poles within the contour. This rep allows an extension of the RMT (and the Mellin transform) to cases in which poles are present in $F(s)$ and to other ranges of $x$.
Also note (see, e.g., Gelfand and Shilov's "Generalized Functions") the relation
$$D_x^{m+n+1} \; H(x) \frac{x^m}{m!} = H(x) \frac{x^{-n-1}}{(-n-1)!} = \delta^{(n)}(x),$$
reflected in the two (of several) reps of the fractional differintegro op equivalent under analytic continuation
$$\frac{x^{\alpha-\beta}}{(\alpha-\beta)!} = \frac{d^{\beta}}{dx^\beta}\frac{x^{\alpha}}{\alpha!}=\int_{0}^{x}\frac{z^{\alpha}}{\alpha!}\frac{(x-z)^{-\beta-1}}{(-\beta-1)!} dz = \frac{1}{2\pi i} \oint_{|z-x|=|x|}\frac{z^{\alpha}}{\alpha!}\frac{\beta!}{(z-x)^{\beta+1}}dz ,$$
with $H(x)$ the Heaviside step function and $\delta(x)$, the Diraac delta. (This is equivalent through binomial expansion to a sinc function/cardinal series interpolation of the binomial coefficients $\binom{s}{\alpha} = \sum_{n \geq 0} \binom{s}{n} \frac{\sin(\pi(\alpha-n))}{\pi(\alpha-n)}=\sum_{n \geq 0} \binom{s}{n} \binom{0}{\alpha-n} $, an instance also of the Chu-Vandermonde identity. The contour integral for this beta function integral is easily transformed into a bandlimited Fourier transform. See, e.g., the relation to Newton interpolation in the MSE_Q "Why is the Euler gamma function the best extension of the factorial function to the reals?" For more detail and connections, see my post "Fractional calculus and interpolation of generalized binomial coefficients." The integral reps of the confluent (see this MO-Q) and non-degenerate hypergeometric functions can be expressed effectually as this or easily related differintegral ops acting on simple functions. Years ago I found the Danish masters Niels Nielsen and Niels Norlund to be most informative on contour integrals in interpolation. There is also a connection to Riemann surfaces via Pochhammer's contour for the beta function integral.)
So, under the conditions above,
$$F(-n) = \int_{0}^{\infty} f(x) \; \frac{x^{-n-1}}{(-n-1)!} \; dx = \int_{0}^{\infty} e^{-a. x} \; \delta^{(n)}(x) \; dx = a_n,$$
and this suggests the analytic continuation and relation to umbral calculus
$$F(s) = \int_{0}^{\infty} f(x) \; \frac{x^{s-1}}{(s-1)!} \; dx = \int_{0}^{\infty} e^{-a.x} \; \frac{x^{s-1}}{(s-1)!} \; dx = (a.)^{-s} = a_{-s}.$$
The iconic guiding example is the Euler gamma function integral rep with $(a.)^n = a_n = c^n$
$$ (a.)^{-s} = a_{-s} = c^{-s} = F(s) = MT[f(x)= e^{-c\; x}] = \int_{0}^{\infty} e^{-c \; x} \; \frac{x^{s-1}}{(s-1)!} \; dx = \frac{1}{c^{s}},$$
giving the interpolation of the coefficients of the Taylor series of $e^{cx}$, i.e., $a_n = c^n$, as $a_{-s}=c^{-s}=F(s)$.
Another useful example, which vividly illustrates the relation to the Appell Sheffer sequences of umbral calculus (of which the $x^n$ with e.g.f. $e^{x}$ is the basic example), is the integral rep for (what I call) the Bernoulli function, simply related to the Hurwitz zeta function and generalizing the Bernoulli polynomials,
$$ B_{-s}(z) = (B.(z))^{-s} = \int_{0}^{\infty} e^{-B.(z)t} \; \frac{t^{s-1}}{(s-1)!} \; dt $$
$$ = \int_{0}^{\infty} \frac{-t}{e^{-t}-1} \; e^{-zt} \frac{t^{s-1}}{(s-1)!} \; dt = s \; \zeta(s+1,z)$$
where the e.g.f. for the Bernoulli polynomials with $(b.)^n = b_n$ the Bernoulli numbers is
$$e^{B.(x)t} = e^{(b.+x)t} = e^{b.t} e^{xt} = \frac{t}{e^t-1} \; e^{xt}.$$
Note that
$$B_n(z) = -n \; \zeta(1-n,z),$$
$$B_n(1) = -n \; \zeta(1-n,1) =-n \; \zeta(1-n) (Riemann) = (-1)^n B_n(0) = (-1)^n b_n.$$
Through this characterization, it is not too difficult to show that the Bernoulli function inherits all the elegant properties of a regular Appell sequence, such as $D_z \; B_{s}(z) = s \; B_{s-1}(z)$.
Hankel contour deformations, Hadamard finite part regularization of the Mellin transform guided by the inverse Mellin transform, the Mellin-Barnes contour integral, and other methods of analytic continuation can be used to extend the range of $s$ and other parameters, as have been done for the integral reps of the Euler gamma and beta functions and Riemann and other zeta functions and their generalizations. In addition, interchanging summation and integration often gives rise to useful asymptotic expansions of functions a la Borel, Heaviside, Hardy, and Poincare.
Riemann knew all this stuff. Ramanujan intuited it. Hardy formalized it. (I stumbled across it on a journey starting from the ladder ops of QM and a brief comment by my old math prof Stallybrass about the sequence $D^{m+n} H(x) \frac{x^m}{m!}$ in his integral transforms class an eon ago.)
For application to defining fractional powers of operators, see my answer and comments therein to the MO-Q "What does the inverse Mellin transform really mean?" and several of my blog posts, such as "The Creation / Raising Operators for Appell Sequences."<|endoftext|>
TITLE: Smallest $S\subset \mathbb C$ on which no degree $k$ polynomial always vanishes?
QUESTION [21 upvotes]: Say $p$ is a polynomial of degree $k$ in $\mathbb C[x]$. Then $p$ can have at most $k$ distinct roots. A somewhat obtuse way to state that is to say that among any set of $k+1$ distinct complex numbers, there must exist a value $a$ for which $p(a)\neq 0$.
The question here has to do with generalizing the above fact to multivariate polynomials.
Given some $p \in \mathbb C[x_1,\ldots,x_n]$, it turns out that whenever we have enough different values to choose from, we can always pick from these some $a_1,\ldots,a_n$ so as to give $p(a_1,\ldots,a_n)\neq 0$. Moreover, how many is "enough" is a function just of $n$ and the degree of $p$.
I want to emphasize that, in the above and in what follows, the $a_1,\ldots,a_n$ so chosen are distinct.
To make things more precise and specific, we have the following.
Theorem. Let $p \in \mathbb C[x_1,\ldots,x_n]$ be a polynomial of total degree $k$. Then any set of $n+k$ distinct complex numbers contains a subset of $n$ that can be assigned to the variables $x_1,\ldots,x_n$ so as to give a nonzero value for $p$. (Again, we are talking about choosing from the set $n$ distinct values that we assign to $x_1,\ldots,x_n$.)
The proof of the above is a not-too-difficult exercise.
But the question is:
Question. In the above theorem, can the $n+k$ be replaced with some other function of $n$ and $k$ so as to give a better upper bound?
In fact, let's define $M(n,k)$ to be the smallest positive integer such that for every $p\in \mathbb C[x_1,\ldots,x_n]$ of total degree $k$, every set $S$ of at least $M(n,k)$ distinct complex numbers contains some subset $\{a_1,\ldots,a_n\}$ (of cardinality $n$, so that these are distinct values) such that $p(a_1,\ldots,a_n)\neq 0$.
The theorem above states that $M(n,k) \le n+k$, but I don't even see how to meet that bound for $n=k=2$.
REPLY [11 votes]: This is probably just another way to present Fedor Petrov's solution: Expand
$$\frac{(1-t \alpha_1) (1-t \alpha_2) \cdots (1-t \alpha_{n+k-1})}{(1-t \beta_1)(1 - t \beta_2) \cdots (1-t \beta_n)}$$
as a formal power series in $t$. The coefficient of $t^k$ is a degree $k$ polynomial in the $\beta$'s, which vanishes whenever $\{ \beta_1, \beta_2, \ldots, \beta_n \}$ is a $n$-element subset of the $\alpha$'s.<|endoftext|>
TITLE: Internal vs. external definability of inner models
QUESTION [6 upvotes]: Suppose $\kappa$ is an inaccessible cardinal. Is the following situation consistent?
There is $p \in V_\kappa$ and a formula $\phi(x)$ such that there is exactly one $M \subseteq V_\kappa$ such that $M$ is a transitive set of size $\kappa$ and $M \models \phi(p)$.
The $M$ above is not a definable class in $V_\kappa$, meaning there is no $q \in V_\kappa$ and formula $\psi(x,y)$ such that $M = \{ x \in V_\kappa : V_\kappa \models \psi(x,q) \}$.
REPLY [5 votes]: I will try to partially answer your question. I claim that if $\kappa$ is weakly compact then this situation is inconsistent: I will show that such an $M$ is necessarily definable in $V_\kappa$.
Define the finite set of formulae $\Lambda:=\{\phi(x)\} \cup \text{tc}(\phi(x))$, where tc denotes the transitive closure (with respect to the 'proper sub-formula' relation). Use reflection in $M$ to find a transitive $q \prec_{\Lambda} M$ such that $p \in q$ and $q \in M$.
For every $x \in V_\kappa$ we shall inductively construct a ${<}\kappa$-branching tree $T_x$. It will consist of sequences (always with a last element) of transitive, $\Lambda$-elementary submodels containing $x$ and satisfying $\phi(p)$:
Set $\langle q \rangle$ to be the root of $T_x$. Assume that $\bar{y} \in T_x$ and $\bar{y}=\langle y_0, ... , y_\alpha\rangle$. Define the set of successors of $\bar{y}$ in $T_x$ as follows: $$\text{succ}_{T_x}(\bar{y}):=\{\bar{y}^{\frown} z \,\,\colon \,z \in V_{f(\bar{y})} \land y_\alpha \prec_{\Lambda} z \land x, y_\alpha \in z \land z \, \text{is transitive} \, \}$$ where $f(\bar{y}):=\max(\vert y_\alpha \vert^+ , \vert x\vert^+)$. In the limit case let $(\bar{y}_\alpha)_{\alpha < \gamma}$ ($\bar{y}_\alpha$ has length $\alpha +1$) be an increasing chain and set $\bar{y}_\gamma:=\langle y_0,...,y_\alpha,... \rangle ^\frown y_\gamma$, where $y_\gamma:= \bigcup_{\alpha < \gamma} y_\alpha$.
I claim that $x \in M \Longleftrightarrow T_x \,\, \text{has height} \,\, \kappa$.
Assume that $x \in M$. Using reflection in $M$, Löwenheim-Skolem and Mostowski collapse (and $q \in M$) one can easily show that $T_x$ must have height $\kappa$.
On the other hand, assume that $T_x$ has height $\kappa$. Any branch through $T_x$ defines an increasing chain of transitive, $\Lambda$-elementary submodels. The direct limit of this chain is a transitive model $M' \subseteq V_\kappa$ of size $\kappa$, satisfying $\phi(p)$ and containing $x$. By your assumption $M'=M$ follows and so $x \in M$.<|endoftext|>
TITLE: Nonbraided rigid monoidal category where left and right duals coincide
QUESTION [5 upvotes]: In a braided rigid monoidal category $(\mathcal{M},\otimes)$ left and right duals coincide. What is an example of a rigid monoidal category where left and right duals coincide but there exist no braiding for the category?
REPLY [6 votes]: The simplest example is G-graded vector spaces where G is a non-abelian group.<|endoftext|>
TITLE: An upper bound for the G.C.D. of $\binom{a}{3}$ and $\binom{b}{3}$
QUESTION [6 upvotes]: I can't seem to find anything in the literature on how to estimate the g.c.d. of $\binom{a}{k}$ and $\binom{b}{k}$. In particular, I would like to know why $\gcd(\binom{a}{3}, \binom{b}{3})\leq b \binom{a-b}{3}$ for $a-b\geq 4$. I'm confident it's true (computer search).
REPLY [2 votes]: I will hopefully prove this statement below modulo a finite check (I believe this will also work to show that $\gcd(\binom{b}{3}, \binom{a}{3}) \le \varepsilon b(a-b)^3$ for any $\varepsilon > 0$ except for a finite set of counterexamples).
Denote for simplicity $a - b = c$ and $\gcd(\binom{b}{3}, \binom{a}{3}) = d$.
First we will show that $d \ll c^5$. Indeed, each divisor of $d$ comes from one of the pairs $(a-k, b - l)$, where $k, l = 0,1, 2$. Each such pair gives us as a contribution a divisor of $a - b + l - k$ and there are five such numbers from $a - b - 2$ to $a - b + 2$. Note also that (up to a finite number of divisors like extra $6$ or so) different pairs with the same $l - k$ will contribute to the different prime divisors of $a - b + l - k $ because it will otherwise divide $\gcd(a - k, a - k_1) | (k - k_1)$. So we get $\ll (a - b - 2)\ldots (a - b + 2) \ll c^5$.
Therefore, if $c \le \delta \sqrt{a}$, then we are done. Indeed, in that case $d \ll c^5 \le c^3\delta^2 a$ while $b\binom{c}{3} \asymp ac^3$. Thus, $c \ge \delta \sqrt{a}$ for some fixed positive $\delta$.
Now assume that $c \ge C a^{2/3}$ for big enough $C$. Then $b\binom{c}{3} \gg C^3ba^2 \gg C^3b^3$ which is bigger than $\binom{b}{3}$ for big enough $C$ and we are done (here we used the obvious fact that $d\le \binom{b}{3}$). So $c \le Ca^{2/3}$ for some fixed positive $C$. In particular, $c = o(a)$ so $b \sim a$.
Now we will use the ingenious observation of GH from MO that
$$(2a - b - 1)\binom{b}{3} - (2b - a - 1)\binom{a}{3} = (a + b - 2)\binom{c + 1}{3}.$$
Note that if $\gcd(2a-b-1, 2b - a - 1) \ge C$ then we are done. Indeed, in that case we can divide by this $\gcd$ and get that $d \ll \frac{(a+b-2)c^3}{C}$. Since now $a\sim b$ we get that this is $\ll \frac{bc^3}{C}$ which is what we need for big enough $C$ (being a bit more careful we can show that it is enough to consider cases with $\gcd(2a-b-1, 2b-a-1) \le 2$).
We have $$\gcd(2a-b-1, 2b - a - 1) = \gcd(2a-b-1, 3(a-b)) \ge \gcd(2a-b-1, a-b)=\\ =\gcd(a-1, a - b) =\gcd(a-1, b-1).$$
Thus, $a-1$ and $b-1$ are almost coprime.
Similar to the above computation we can get that (up to the division by some uniformly bounded number) $d = (a+b-2)\binom{c+1}{3}$. We have here four factors: $(c-1), c, c+1, a+b-2$.
$c-1$ can come from gcd of $a-1,b$ or gcd of $a-2, b-1$.
$c$ can come from gcd of $a, b$; $a-2, b-2$ or $a-1, b-1$, but the last case can be excluded since $\gcd(a-1, b-1) \le C$.
$c + 1$ can come from gcd of $a, b-1$ or gcd of $a-1, b-2$.
Finally, $a+b-2$ can come from gcd of $a, b-2$; $a-2, b$ or $a-1, b-1$, but the last case we can again exclude.
Assume that $c-1$ splits between gcd of $a-1, b$ and gcd of $a-2, b-1$ as $r_1$ and $s_1$. Similarly for the next ones we would have $r_2, s_2$, $r_3, s_3$ and $r_4, s_4$.
Observe that different numbers among $r_1, \ldots , s_4$ corresponding to the same number among $a, a-1, a-2, b, b-1, b-2$ will be almost coprime in a sense that their $gcd$ is at most some constant (for $r_1, \ldots , s_3$ it is obvious because $c-k, c-l$ are almost coprime and with $r_4, s_4$ e.g. when we look at $r_1$ and $s_4$ corresponding to $b$ we have $\gcd(r_1, s_4) \mid \gcd(a-b-1, b, a+b-2) = 1$. Other cases are similar).
Note that $r_1s_1 \sim r_2s_2 \sim r_3s_3 \sim c$ and $r_4s_4 \sim a$.
Let's look at the numbers $b, b-2, a, a-2$:
For $b$ we have by almost coprimeness $r_1r_2s_4 \ll a$,
For $b-2$ we have $s_2s_3r_4 \ll a$,
For $a$ we have $r_2r_3r_4 \ll a$,
For $a-2$ we have $s_1s_2s_4 \ll a$.
Multiplying these inequalities we get $c^4a^2 \ll a^4$, that is $c \ll \sqrt{a}$.
So, after all this reasoning, we got that $\sqrt{a} \ll c \ll \sqrt{a}$ so $c\sim \sqrt{a}$! I guess one can reach the same conclusion from the assumption $d \ll \varepsilon b\binom{c}{3}$.
Now, we have two numbers which are (almost) divisible by our gcd: $\binom{c+2}{5}$ and $(a+b-2)\binom{c+1}{3}$. Note that both of them are proportional to the required $b\binom{c}{3}$. Thus, they are almost the same in a sense that $n\binom{c+2}{5} = m(a+b-2)\binom{c+1}{3}$ for some integers $n, m \le C$. Dividing by $\binom{c+1}{3}$ we get
$r(c+2)(c-2) = (a+b-2)$ for some $r\in \mathbb{Q}$, $r > 0$ with bounded denominator and numerator. So $a$ and $b$ are some second degree polynomials of $c$:
$$a = \frac{r(c+2)(c-2) + c + 2}{2}, b = \frac{r(c+2)(c-2)-c+2}{2}.$$
Therefore $\binom{a}{3}$ and $\binom{b}{3}$ are polynomials of degree $6$ in $c$. It remains to show that their $\gcd$ (as polynomials in $\mathbb{Q}[x]$) has degree at most $4$ – in that case $\gcd(\binom{a}{3}, \binom{b}{3}) \ll c^4$ while $b\binom{c}{3} \sim c^5$.
Since $\binom{a}{3}$ is a product of three quadratic polynomials, if $\gcd$ has degree at least $5$ then all factors should be used, in particular $a-1 = \frac{r(c+2)(c-2) + c}{2}$. If it is not coprime to some $b-k$ then their $\gcd$ divides their difference which is $\frac{2c + 2k - 2}{2}$. Thus, $c = 1 - k$ is a root of $a-1$ for $k = 0$, $k = 1$ or $k = 2$. Case $c = 0$ gives us $r = 0$, $c = -1$ gives us $r = -\frac{1}{3}$ and $c = 1$ gives us $r = \frac{1}{3}$. In all three cases the degree of gcd is at most $4$ and the proof is complete.<|endoftext|>
TITLE: Inner automorphisms of group algebras vs. inner automorphisms of the group
QUESTION [6 upvotes]: In a recent question on MSE I asked about conditions under which the canonical morphism $Out(G) \to Out(k[G])$ is injective.
Is it true that this morphism is indeed injective if $G$ is finite and $k=\mathbb{Z}$ ?
What I already know:
It's not injective if $k$ is a field in non-modular characteristic. The kernel is $Out_c(G)$, the group of conjugacy class preserving automorphisms. That is Captain Lama's answer to my question.
It is true if $k=\mathbb{Z}$ and $G$ is nilpotent. That's my own answer to the question.
In the mean time I have also found out that the morphism is injective if $G$ is rational, i.e. if $g$ is cojugated to $g^k$ for every $g\in G$ and every $k\in\mathbb{Z}$ with $gcd(k,ord(g))=1$, e.g. $G$ a symmetric group.
Proof: If $\alpha\in Aut(G)$ is in the kernel of the morphism, say $\alpha(g)=ugu^{-1}$ for some $u\in\mathbb{Z}G^\times$, then
$$u^\ast u g=u^\ast \alpha(g) u = u^\ast \alpha(g^{-1})^{-1} u = (u^\ast \alpha(g^{-1}) u)^\ast = (u^\ast u g^{-1})^\ast = g u^\ast u$$
where $^\ast$ is the antiautomorphism of the group algebra with $g\mapsto g^{-1}$. Hence $u^\ast u \in\mathbb{Z}G$. We want to prove that $u^\ast u = 1$.
Consider all the complex characters $\chi\in Irr(G)$ and their central characters $\omega_\chi: Z(\mathbb{C}G) \to \mathbb{C}$. We can pick a matrix representations $\rho_\chi: G\to GL_n(\mathbb{C})$ affording $\chi$ with $\rho(G)\subseteq U_n(\mathbb{C})$ so that $u^\ast u$ is mapped to some self-adjoint and positive definite matrix so that $\omega_\chi(u^\ast u)\in\mathbb{R}_{>0}$. Furthermore $u^\ast u\in Z(\mathbb{Z}G)$ is integral over $\mathbb{Z}$ so that $\omega_\chi(u^\ast u)$ is an algebraic integer. Moreover it must be in $\omega_\chi(Z(\mathbb{Q}G)) =\mathbb{Q}(\chi)$. Now if $G$ is rational, then all characters have rational values so that $\omega_\chi(u^\ast u)$ is a real, positive, rational, invertible integer. In other words $\omega_\chi(u^\ast u) = 1$, i.e $\rho_\chi(u^\ast u) = 1_{n\times n}$. Since $\chi$ was arbitrary, $u^\ast u=1$.
The only units of $\mathbb{Z}G$ with $u^\ast u=1$ are elements of the form $\pm g$ so that $\alpha\in Inn(G)$ as we wanted. QED.
Note that we can get the same conclusion under weaker conditions. For example if $\mathbb{Q}(\chi)=\mathbb{Q}(i)$, then the only real, positive, integral unit is also 1.
REPLY [4 votes]: The question for finite $G$ and $k = \mathbb{Z}$ is the normalizer problem, see [1, Section 1]. By a result of Jan Krempa, the kernel of the cannonical morphism is in that case always an elementary abelian $2$-group. As far as I know, there is basically only one example known where the kernel is non-trivial [1, Theorem A]. This example was constructed by Martin Hertweck and used to provide a counterexample to the isomorphism problem for integral group rings.
[1] Martin Hertweck; A counterexample to the isomorphism problem for integral group rings. Ann. of Math. (2) 154 (2001), no. 1, 115–138, MR1847590.<|endoftext|>
TITLE: Tight apartness relations in toposes
QUESTION [8 upvotes]: A tight apartness relation on a set is a binary relation $\#$ such that the following conditions hold:
$x = y$ if and only if $\neg (x \# y)$.
If $x \# y$, then $y \# x$.
If $x \# z$, then either $x \# y$ or $y \# z$ for every $y$.
I want to understand this notion better. Classically, it is completely trivial. Thus, it makes sense to look at it in various toposes. I tried to use the Kripke-Joyal semantics to get the external interpretation of an arbitrary object of a topos with a tight apartness relation, but it seems that it does not give anything particularly interesting in general. Thus, I've got the following question:
Question: What are examples of objects in toposes with a tight apartness relation which externally correspond to some interesting or useful notion?
Since constructively, a set can have more than one tight apartness relation on it, I'd like to see examples of an object with two different tight apartness relations which both have interesting interpretations.
Edit: There are several "generic" examples of objects with tight apartness relations, i.e., objects that can be defined in every topos (e.g., Dedekind reals). I'm particularly interested in "non-generic" examples, i.e., objects that can be constructed only in a specific topos.
REPLY [9 votes]: I'm not precisely sure what you're looking for. Here is an example for the external interpretation of an apartness relation:
Recall that the object of Dedekind reals $\mathbb{R}$ in a sheaf topos $\mathrm{Sh}(X)$ is the sheaf $\mathcal{C}$ of continuous (Dedekind-)real-valued functions on $X$.
The apartness relation on $\mathbb{R}$, defined by $x \mathrel{\#} y \Longleftrightarrow x - y \text{ is invertible}$, is then the following subsheaf $\mathcal{E}$ of $\mathcal{C} \times \mathcal{C}$:
$$ \mathcal{E}(U) = \{ (f,g) \,|\, \text{for all $x \in U$: $f(x) - g(x) \in \mathbb{R}$ is invertible} \} $$<|endoftext|>
TITLE: Does every $SL_2\mathbb{C}$ representation of a closed oriented surface extend over a compact oriented three-manifold?
QUESTION [14 upvotes]: Let $F$ be a compact oriented surface and $\rho:\pi_1(F)\rightarrow SL_2\mathbb{C}$ be a representation. Does there exist a compact oriented three-manifold $M$ with $\partial M=F$ and a homomorphism $\tilde{\rho}:\pi_1(M)\rightarrow SL_2\mathbb{C}$ so that the restriction of $\tilde{\rho}$ to $\pi_1(F)$ is equal to $\rho$?
If not is there an obstruction that allows you to identify the representations that do extend?
For instance let $BSL_2\mathbb{C}^\delta$ denote the classifying space of $SL_2\mathbb{C}$ as a discrete group. Corresponding to $\rho:\pi_1(F)\rightarrow SL_2\mathbb{C}$ is a continuous map $f:F\rightarrow BSL_2\mathbb{C}^\delta$. If $\rho$ extends over a three-manifold $M$ then the homology class represented by $f_*[F]$ is zero. Is there a computable way to detect this?
REPLY [4 votes]: This is an extended comment on the algorithmic question.
As Moshe points out, the lack of realization as a boundary follows from the Baire category theorem. On the other hand, how does one recognize when an element is not in this infinite countable union of subspaces?
Let's consider the genus 1 case $F=T^2$. Then a rep $\rho:\pi_1(F)\to SL_2(\mathbb{C})$ is determined by two eigenvalues of generators $(\mu, \lambda)$ (assuming the representation is not unipotent). In turn if $F=\partial M$, the boundary of a 3-manifold, then there is an associated $A$-polynomial $A(x,y) \in \mathbb{Z}[x,y,x^{-1},y^{-1}]$ such that $A(\mu,\lambda)=0$. I suspect that $[\rho]=0\in H_2(SL_2(\mathbb{C})^\delta)$ iff $[\rho]$ extends to a representation of a 3-manifold, but I haven't checked this. In any case, one sees that $\mu, \lambda$ are algebraically related in the bounding case. However conversely, if $\mu,\lambda$ satisfy an algebraic relation, it's not clear to me that this implies that $[\rho]=0$, since in general A-polynomials satisfy some non-trivial conditions.
I suspect there could be a formulation in terms of algebraic K-theory, but I don't know enough about this.
One may also ask if $H_2(SL_2(\mathbb{C})^\delta)$ is generated by representations of $T^2$? I suspect this might be true. It's not hard to see that a representation of a closed surface of genus $>2$ is cobordant to a sum of representations of genus 2 surfaces (since the commutator map in $SL_2(\mathbb{C})$ is onto). Then I think that genus 2 reps. may be cobordant to a pair of genus 1 reps. (at least the numerology works out, but I haven't checked it). Then one could ask for when a sum of genus 1 reps. is homologically trivial? In turn, this should be realized by a zero of an A-variety. But I'm not sure how one recognizes such points.<|endoftext|>
TITLE: Geometry of Level sets of elliptic polynomials in two real variables
QUESTION [5 upvotes]: Updated:
A polynomial $P(x,y)\in \mathbb{R}[x,y]$ is called an elliptic polynomial if its last homogeneous part does not vanish on $\mathbb{R}^2\setminus\{0\}$.The two answers to this post provide a proof for the following theorem:
Theorem: If $p$ is an elliptic polynomial whose last homogeneous part is positive definitive, then for $c$ sufficiently large
, $p^{-1}(c)$ is a simple closed curve. Moreover if the centroid of interior of $p^{-1}(c)$ is denoted by $e_c$ then $e_c$ is convergent as $c$ goes to $+\infty$. The limit $\lim_{c\to \infty} e_c$ can be written in terms of coefficients of $p$. If we drop the ellipticity condition then this convergence result is not necessarily true.
The previous version of the post:
Is there a polynomial function $P:\mathbb{R}^2 \to \mathbb{R}$ with the following property?
For sufficiently large $c>0$, $P^{-1}(c)$ is a simple closed curve $\gamma_c$, homeomorphic to $S^1$, but as $c$ goes to $+\infty$. the centroid $e_c$ of the interior of $\gamma_c$
does not converge to any point of $\mathbb{R}^2$.
Motivation: The answer is negative if we consider this question for polynomials $p:\mathbb{R} \to \mathbb{R}$ whose eventual level sets are $2$-pointed set, i.e. $S^0$.(Namely a polynomial of even degree). The motivation comes from line -3, item III, page 4 of Taghavi - On periodic solutions of Liénard equations, which can be generalized to every even degree polynomial with one variable.
REPLY [7 votes]: Concerning homogeneous polynomials: Let $P(x,y)=\sum_{j=0}^n a_j x^j y^{n-j}$ be such a polynomial, of degree $n$ such that $C:=P^{-1}(\{c\})$ is a simple closed curve for all large enough $c>0$.
If $n$ is odd, then every line through the origin will have at most one point of intersection with $C$. So, then $C$ cannot be a simple closed curve for any real $c$ -- because every line through any point interior to a simple closed curve must intersect the curve in at least two points.
It remains to consider the case when $n$ is even. Then $C$ is symmetric about the origin, and hence so is the interior of $C$. Then the centroid of the interior is the origin, and it does not depend on the level $c$.
Consider now the case of an elliptic polynomial
\begin{equation*}
P(z)=P(x,y)=\sum_{j=0}^n a_j x^j y^{n-j}+\sum_{j=0}^{n-1}b_j x^j y^{n-1-j}+K|z|^{n-2}
\end{equation*}
of (necessarily even) degree $n$,
where $z:=(x,y)$ and $K=O(1)$ (as $|z|\to\infty$). The ellipticity here is understood as the following condition:
\begin{equation*}
\min_{|z|=1}\sum_{j=0}^n a_j x^j y^{n-j}>0.
\end{equation*}
For any $d_*\in(0,1)$ and any real $D>0$, let $\mathcal P_{n,d_*,D}$ denote the set of all polynomials $p(x)=\sum_{j=0}^n d_j x^j$ such that $d_n\ge d_*$ and $\sum_{j=0}^n|d_j|\le D$.
Then it is not hard to see that there is a real $c_*(n,d_*,D)>0$, depending only on $n,d_*,D$, such that
for any polynomial $p(x)=\sum_{j=0}^n d_j x^j$ in $\mathcal P_{n,d_*,D}$ and for all real $c\ge c_*(n,d_*,D)$ the equation $p(x)=c$ has exactly two roots $x_\pm:=x_\pm(c)$ such that $x_-<00.
\end{equation*}
So, by (1.6), wlog (1.5) holds indeed.
Let us now turn back to the elliptic polynomial $P(x,y)$. For each real $t$ consider the polynomial
\begin{equation*}
p_t(r):=P(r\cos t,r\sin t).
\end{equation*}
By the ellipticity of the polynomial $P(x,y)$, there exist $d_*\in(0,1)$ and a real $D>0$ such that $p_t\in\mathcal P_{n,d_*,D}$ for all real $t$. Take now any real $c\ge c_*(n,d_*,D)$. Then, by the paragraph right above, for each real $t$ the equation $p_t(r)=c$ has exactly two roots $r_\pm(t):=r_\pm(c;t)$ such that
$r_-(t)<00&\text{ for }t\in[0,\pi],\\
|r_-(t-\pi)|>0&\text{ for }t\in[\pi,2\pi].
\end{cases}
\end{equation*}
So, the level curve $C$ is closed and simple, and its interior is
\begin{equation*}
I(c):=\{r\,(\cos t,\sin t)\colon0\le r0$ between the rays $t$ and $t+dt$ is at distance $\frac23\,r$ from the origin, (ii) the area of this sector is $\frac12\,r^2\,dt$, and (iii) the centroid of the union of the two sectors is the weighted average of the centroids of the two sectors, with weights adding to $1$ and proportional to the areas of the sectors, and thus proportional to the squared radii of the sectors.
Simplifying (2), we get
\begin{equation*}
d(t)\sim-\frac{2b(t)}{na(t)}.
\end{equation*}
Averaging now over all the pairs of opposite infinitesimal sectors, we see that the centroid converges to
\begin{align*}
&-\int_0^\pi dt\,\frac{2b(t)}{na(t)}(\cos t,\sin t)\frac12\,\Big(\frac c{a(t)}\Big)^{2/n}
\Big/\int_0^\pi dt\,\frac12\,\Big(\frac c{a(t)}\Big)^{2/n} \\
&=-\int_0^\pi dt\,\frac{2b(t)}{na(t)}(\cos t,\sin t)\Big(\frac1{a(t)}\Big)^{2/n}
\Big/\int_0^\pi dt\,\Big(\frac1{a(t)}\Big)^{2/n}. \tag{3}
\end{align*}
I have checked this result numerically for $P(x,y)=x^4 + y^4 + 3 (x - y)^4 + y^3 + x y^2 + 10 x^2$, getting the centroid $\approx(-0.182846, -0.245149)$ for $c=10^4$ and $\approx(-0.189242,-0.25)$ for the limit (as $c\to\infty$) given by (3). From the above reasoning, one can see that the distance of the centroid from its limit is $O(1/c^{1/n})$; so, the agreement in this numerical example should be considered good, better than expected.
One may also note that in general the level sets $P^{−1}([0,c])$ will not be convex, even if $P$ is a positive elliptic homogeneous polynomial. E.g., take $P(x,y)=(x−y)^2(x+y)^2+h(x^4+y^4)$ for a small enough $h>0$. Here is the picture of this level set for $c=1$ and $h=1/10$:
Clearly, the shape of this level set does not depend on $c>0$.
This non-convexity idea can be generalized, with
$$P(x,y)=P_{k,h}(x,y)
:=\prod_{j=0}^{2k-1}\Big(x\cos\frac{\pi j}k-y\sin\frac{\pi j}k\Big)^2+h(x^{4k}+y^{4k})$$
for natural $k$ and real $h>0$. Here is the picture of the curve $P_{k,h}^{-1}(\{1\})$ for $k=5$ and $h=(3/10)^{4k}$:<|endoftext|>
TITLE: Embedding of a group into a simple group in which every element is a commutator
QUESTION [8 upvotes]: It is known that any group $G$ can be embedded into a simple group $S$, see, e.g., the discussion at Can any group be embedded in a simple group?
My question is whether one can get an embedding such that the ambient group $S$ is of commutator width $1$ (i.e., each element of $S$ can be represented as a single commutator).
REPLY [12 votes]: Here is a construction. Every finite group with $n$ elements embeds into $A_{2n}$ (and even $A_{n+2}$) which is simple if $n>2$ and of commutator width 1 (as any other finite simple group by the Ore conjecture proved by Martin W. Liebeck, E. A. O'Brien, Aner Shalev, Pham Huu Tiep, although for $A_n$ it was probably known to Frobenius and certainly to Ore in 1951). Suppose that the group $G$ is infinite. Using HNN extensions embed your group into a group $S$ where all pairs of elements of the same order are conjugate (this is the standard application of HNN extensions: use HNN extensions with cyclic associated subgroups to make $G
TITLE: Is $\mathbb{C}^n$ rigid?
QUESTION [8 upvotes]: Let $\pi:X\to S$ be a smooth family of complex manifolds (in the sense of deformation theory) such that $\pi^{-1}(0)\cong\mathbb{C}^n$ and $S\subset \mathbb{C}$ is a neighborhood of $0$. Is $\pi$ trivial? That is, is $X\cong \mathbb{C}^n\times S$ possibly after shrinking $S$?
I know that a smooth family of compact complex manifolds with $H^1(M,\mathcal{T}_M)=0$ is trivial (where $M=\pi^{-1}(0)$) but I'm not sure whether this extends to this non-compact situation.
REPLY [17 votes]: Example. $\pi: \{(z,w)\in {\mathbb C}^2: |zw|<1\}\to {\mathbb C}$, $\pi(z,w)=z$.
Edit. Similarly, to get a nontrivial deformation of ${\mathbb C}^n$, consider
$$
X=\{(z_0, z_1,...,z_n)\in {\mathbb C}^{n+1}: |z_0 z_1|<1\}
$$
and let $\pi$ be the projection of $X$ to ${\mathbb C}$ which the 1-st coordinate line in ${\mathbb C}^{n+1}$. Then $\pi^{-1}(t)$ (for $t\ne 0$) will be biholomorphic to $B \times {\mathbb C}^{n-1}$, where $B\subset {\mathbb C}$ is an open disk. Hence, these fibers are not biholomorphic to $\pi^{-1}(0)={\mathbb C}^{n}$.
A better question, I think, is if every Stein manifold of positive dimension admits a nontrivial deformation. I suspect that this is known.<|endoftext|>
TITLE: Every complex number has a square root via LLPO without weak countable choice
QUESTION [6 upvotes]: Is it possible to prove that every complex number has a square root using analytic LLPO, but avoiding Weak Countable Choice or Excluded Middle? Unique Choice is allowed.
(Analytic LLPO is the statement that given any pair of real numbers $x$ and $y$, either $x \leq y$ or $x \geq y$. This statement is non-constructive, but still weaker than other statements like LPO or Excluded Middle.)
It is true in Johnstone's Topological Topos. This is because the Fundamental Theorem of Algebra is true for Cauchy Real numbers, and Cauchy reals are isomorphic to Dedekind reals in the Topological Topos. But this reasoning doesn't seem to work unless Cauchy is iso to Dedekind.
REPLY [7 votes]: Yes it is but there is no extensional square root function unless we also have LPO.
Note that the squaring function is a bijection from $Q_{+} = \{x + iy \mid x \geq 0, y \geq 0\}$ onto $H_{+} = \{x + iy \mid y \geq 0\}$. Similarly it is a bijection from $Q_{-} = \{x + iy \mid x \geq 0, y \leq 0\}$ onto $H_{-} = \{x + iy \mid y \leq 0\}$. Given LLPO, $H_{+} \cup H_{-} = \mathbb{C}$ and that is enough to prove the existence of square roots. The confusing part is that for a negative real number $x$, we obtain the square root $+ i\sqrt{-x}$ or $-i\sqrt{-x}$ depending on whether LLPO gives $x \in H_{+}$ or $x \in H_{-}$. So this does not give an extensional square root function.
Interestingly, this argument gives the stronger conclusion that every complex number has a square root in $$Q_{+} \cup Q_{-} =\{ x + iy \mid x \geq 0\}. $$ This stronger statement turns out to be equivalent to LLPO. Indeed, given a small enough real number $x$, if the square root of $-1+ix$ in $Q_{+} \cup Q_{-}$ is close to $i$ then $x \geq 0$ and similarly if that square root is close to $-i$ then $x \leq 0$.<|endoftext|>
TITLE: Is there a strictly increasing differentiable function maps positively measurable set to zero measure set?
QUESTION [10 upvotes]: Let $g(t)$ be a strictly increasing differentiable function. Can it map positively measurable set to zero measurable set?
It's obviously that $\{g'>0\}$ is dense. If I can prove that the Lebesgue measure $m(\{g'=0\}) = 0$, then for every set with positive measure, there will be a positively measurable subset with $g'>\epsilon$ on it, and consequently maps the set to nonzero measure set(It's a theorem and I forget it's name).
The question derives from my textbook, which says if $g(t)$ is a strictly increasing differentiable function and Riemann integrable, and $f(x)$ is Riemann integrable, then
$$\int f(x) = \int f(g(t))g'(t)$$
All functions defined on suitable closed set.
It seems that $f(g(t))$ may even be not integrable and that is totally a typo. But with my intuition of measure theory, this might be true since $g$ is differentiable.
REPLY [17 votes]: There are strictly increasing $C^1$ functions that map sets of positive measure to sets of measure zero. Here is a construction:
Let $C\subset [0,1]$ be a Cantor set of positive measure. For a construction, see https://en.wikipedia.org/wiki/Smith-Volterra-Cantor_set. Let $g(x)=\operatorname{dist}(x,C)$. The function $g$ is clearly continuous and equal zero on $C$. In fact $g$ is a $1$-Lipschitz function. Let
$$
f(x)=\int_0^x g(t)\, dt.
$$
The function $f$ is $C^1$ and it is strictly increasing. Indeed, if $y>x$, then
$$
f(y)-f(x)=\int_x^y g(t)\, dx>0
$$
because the interval $[x,y]$ is not contained in the Cantor set $C$ and therefore it contains an interval where $g$ is positive.
On the other hand $f'=g=0$ on $C$ which has positive measure and $f(C)$ has measure zero since $m(f(C))=\int_C f'(t)\, dt=\int_C g(t)\, dt=0$.
As was pointed out by Mateusz Kwaśnicki in his comment, this construction gives the following result:
Theorem. Let $f$ be as above. Then there is a Riemann integrable function $h$ such that $h\circ f$ is not Riemann integrable.
Proof. The set $f(C)$ is homeomorphic to the Cantor set ($f$ is strictly increasing so it is a homeomorphism) and has measure zero as explained above. Let
$$
h(x)=\begin{cases}
1 & \text{if $x\in f(C)$}\\
0 & \text{if $x\not\in f(C)$.}
\end{cases}
$$
The function $h$ is Riemann integrable with the integral equal zero since it is bounded and continuous outside the set $f(C)$ of measure zero (because $\mathbb{R}\setminus f(C)$ is open and $h=0$ there). However,
$$
(h\circ f)(x)=\begin{cases}
1 & \text{if $x\in C$}\\
0 & \text{if $x\not\in C$.}
\end{cases}
$$
is not Riemann integrable since it is discontinuous on a set $C$ of positive measure. $\Box$<|endoftext|>
TITLE: When are projective modules closed under highly-filtered colimits?
QUESTION [5 upvotes]: Let $R$ be a ring. Let $Mod(R)$ be the category of left $R$-modules, and let $Proj(R) \subseteq Mod(R)$ be the full subcategory of projective $R$-modules. Let's say that $R$-projectives are closed under highly-filtered colimits if there exists a cardinal $\kappa$ such that $Proj(R)$ is closed under $\kappa$-filtered colimits in $Mod(R)$. For example,
When $R$ is a field (or division ring), we have $Proj(R) = Mod(R)$, and so vacuously we have that $R$-projectives are closed under highly-filtered colimits.
When $R = \mathbb Z$, the question (perhaps surprisingly) depends on set theory
Under the anti-large cardinal hypothesis $V=L$, Eklof and Mekler showed that there exist aribtrarily large non-free abelian groups all of whose smaller subgroups are free and it follows that $\mathbb Z$-projectives are not closed under highly-filtered colimits.
Whereas if there exists a strongly compact cardinal, then it follows from the powerful image theorem of Makkai and Pare that free abelian groups are an accessible, accessibly embedded subcategory of all abelian groups, and in particular $\mathbb Z$-projectives are closed under highly-filtered colimits.
The positive result of Makkai and Pare generalizes immediately to show that if $R$ is a PID (in the noncommutative case I'm not sure of the terminology, but the hypothesis is that every submodule of a projective $R$-module is free), then assuming the existence of a sufficiently-large strongly compact cardinal (I think the cardinal just has to be bigger than the cardinality of $R$) we have that $R$-projectives are closed under highly-filtered colimits.
In fact, Rosicky and Brooke-Taylor's "$\lambda$-pure" version of the powerful image theorem shows that if there is a sufficiently large strongly compact cardinal, then projective $R$-modules are closed under highly-filtered colimits provided that $R$ is of global projective dimension $\leq 1$ (every submodule of a projective module is projective rather than free), so this result holds even for some $R$ which are not domains.
Question:
What are some other rings $R$ for which we can say (perhaps under set-theoretical hypotheses) whether $R$-projectives are closed under highly-filtered colimits?
Are there any rings $R$ other than division rings for which the question "are $R$-projectives closed under highly-filtered colimits?" is decidable in ZFC?
Is the precise large-cardinal strength of "$\mathbb Z$-projectives are closed under highly-filtered colimits" -- or (equivalently, I think) "any sufficiently-large abelian group all of whose smaller subgroups are free, is free" -- known?
REPLY [8 votes]: Let $R$ be a ring and $\kappa$ be a strongly compact cardinal such that $|R|<\kappa$. Then the class of all projective $R$-modules is closed under $\kappa$-filtered colimits.
This is Theorem 3.3 in the recent preprint of J. Šaroch and J. Trlifaj "Test sets for factorization properties of modules", https://arxiv.org/abs/1912.03749
For any left perfect ring $R$ (which includes both all left Artinian and all right Artinian rings), all flat left $R$-modules are projective, so the class of projective left $R$-modules is closed under $\aleph_0$-filtered colimits. For example, this applies to all the rings $\mathbb Z/n\mathbb Z$, $n\ge2$ (many of which are not division rings).<|endoftext|>
TITLE: Unexpected behavior involving √2 and parity
QUESTION [24 upvotes]: This post makes a focus on a very specific part of that long post. Consider the following map:
$$f: n \mapsto \left\{
\begin{array}{ll}
\left \lfloor{n/\sqrt{2}} \right \rfloor & \text{ if } n \text{ even,} \\
\left \lfloor{n\sqrt{2}} \right \rfloor & \text{ if } n \text{ odd.}
\end{array}
\right.$$
Let $f^{\circ (r+1)}:=f \circ f^{\circ r}$, consider the orbit of $n=73$ for iterations of $f$, i.e. the sequence $f^{\circ r}(73)$: $$73, 103, 145, 205, 289, 408, 288, 203, 287, 405, 572, 404, 285, 403, 569, 804, 568, 401, \dots$$
It seems that this sequence diverges to infinity exponentially, and in particular, never reaches a cycle. Let illustrate that with the following picture of $(f^{\circ r}(73))^{1/r}$, with $2000$, together with the miniman such $r$ (in red):
In fact all these numbers (as first terms) reach the following cycle of length $33$:
$$(15,21,29,41,57,80,56,39,55,77,108,76,53,74,52,36,25,35,49,69,97,137,193,272,192,135,190,134,94,66,46,32,22)$$
except the following ones: $$7, 8, 9, 10, 12, 13, 14, 18, 19, 20, 26, 27, 28, 38, 40, 54,$$ which reach $(5,7,9,12,8)$, and that ones $1, 2, 3, 4, 6$ which reach $(1)$, and $f(0)=0$.
If the pattern continues like above up to infinity, they must have infinity many such $n$.
Bonus question: Are there infinitely many $n$ reaching a cycle? Do they all reach the above cycle of length $33$ (except the
few ones mentioned above)? What is the formula of these numbers $n$?
Below is their counting function (it looks logarithmic):
REPLY [4 votes]: Here is a heuristic answer inspired by this comment of Lucia.
First, let assume that the probabilty for an integer $n$ to be odd is $\frac{1}{2}$, and that the probabilty for $f(n)$ to be odd when $n$ is even (resp. odd) is also $\frac{1}{2}$. We will observe that (surprisingly) it is no more $\frac{1}{2}$ for $f^{\circ r}(n)$ when $r \ge 2$ (in some sense, the probability does not commute with the composition of $f$ with itself).
if $n$ and $m=f(n)$ are even: note that $\frac{n}{\sqrt{2}} = m+\theta$ (with $0 < \theta < 1$) so that $m=\frac{n}{\sqrt{2}}- \theta$, then $$f^{\circ 2}(n) = f(m) = \left \lfloor{\frac{m}{\sqrt{2}}} \right \rfloor = \left \lfloor{\frac{\frac{n}{\sqrt{2}}- \theta}{\sqrt{2}}} \right \rfloor = \left \lfloor \frac{n}{2} - \frac{\theta}{\sqrt{2}}\right \rfloor$$ but $\frac{n}{2}$ is even with probability $\frac{1}{2}$, so in this case, $f^{\circ 2}(n)$ is odd with probability $\frac{1}{2}$.
if $n$ is even and $m=f(n)$ is odd: $$f^{\circ 2}(n) = f(m) = \left \lfloor\sqrt{2}m \right \rfloor = \left \lfloor \sqrt{2}(\frac{n}{\sqrt{2}} - \theta) \right \rfloor = \left \lfloor n - \sqrt{2} \theta) \right \rfloor$$ but $n$ is even and the probability for $0<\sqrt{2} \theta<1$ is $\frac{\sqrt{2}}{2}$ (because $\theta$ is assumed statistically equidistributed on the open interval $(0,1)$), so $f^{\circ 2}(n)$ is odd with probability
$\frac{\sqrt{2}}{2}$.
if $n$ is odd and $m=f(n)$ is even:
$$f^{\circ 2}(n) = f(m) = \left \lfloor{\frac{m}{\sqrt{2}}} \right \rfloor = \left \lfloor{\frac{\sqrt{2}n-\theta}{\sqrt{2}}} \right \rfloor = \left \lfloor n - \frac{\theta}{\sqrt{2}} \right \rfloor $$
but $n$ is odd and $0 < \frac{\theta}{\sqrt{2}}<1$, so $f^{\circ 2}(n)$ is even.
if $n$ is odd and $m=f(n)$ is odd:
$$f^{\circ 2}(n) = f(m) = \left \lfloor \sqrt{2} m \right \rfloor = \left \lfloor \sqrt{2} (\sqrt{2}n-\theta) \right \rfloor = \left \lfloor 2n - \sqrt{2} \theta \right \rfloor $$
but $2n$ is even and the probability for $0<\sqrt{2} \theta<1$ is $\frac{\sqrt{2}}{2}$, so $f^{\circ 2}(n)$ is odd with probability $\frac{\sqrt{2}}{2}$.
By combining these four cases together, we deduce that the probability for $f^{\circ 2}(n)$ to be odd is $$\frac{1}{2} \times \frac{1}{2} \times (\frac{1}{2} + \frac{\sqrt{2}}{2} + 0 + \frac{\sqrt{2}}{2}) = \frac{2\sqrt{2}+1}{8}$$
By continuing in the same way, we get that the probability for $f^{\circ 3}(n)$ to be odd is:
$$ \frac{1}{4} (\frac{1}{2}\frac{1}{2} + \frac{1}{2}\frac{\sqrt{2}}{2} + \frac{\sqrt{2}}{2}\frac{\sqrt{2}}{2} + 1\frac{1}{2} + \frac{\sqrt{2}}{2}\frac{\sqrt{2}}{2}) = \frac{\sqrt{2}+7}{16}$$
For $2 \le r \le 24$, we computed the probability $p_r$ for $f^{\circ r}(n)$ to be odd (see Appendix). It seems (experimentally) that $p_r$ converges to a number $\simeq 0.532288725 \simeq \frac{8+3\sqrt{2}}{23}$ by Inverse Symbolic Calculator. This leads to the following question/conjecture:
$$\lim_{r \to \infty}p_r = \frac{8+3\sqrt{2}}{23} \ \ ?$$
If so, consider the number $\alpha$ mentioned in the main post, then $$\alpha = 1-\frac{8+3\sqrt{2}}{23} = \frac{15-3\sqrt{2}}{23} \simeq 0.467711,$$ which matches with the computation in the main post. And next, we would have:
$$ \delta = \frac{\sqrt{2}}{2^{\alpha}}= 2^{\frac{1}{2}-\alpha} = 2^{\frac{6\sqrt{2}-7}{46}} \simeq 1.022633$$
Appendix
Computation
sage: for i in range(3,26):
....: print(sq2(i))
....:
[1/4*sqrt(2) + 1/8, 0.478553390593274]
[1/16*sqrt(2) + 7/16, 0.525888347648318]
[3/32*sqrt(2) + 13/32, 0.538832521472478]
[15/64*sqrt(2) + 13/64, 0.534581303681194]
[5/128*sqrt(2) + 61/128, 0.531805217280199]
[39/256*sqrt(2) + 81/256, 0.531852847392776]
[93/512*sqrt(2) + 141/512, 0.532269260352925]
[51/1024*sqrt(2) + 473/1024, 0.532348527032254]
[377/2048*sqrt(2) + 557/2048, 0.532303961432938]
[551/4096*sqrt(2) + 1401/4096, 0.532283123258685]
[653/8192*sqrt(2) + 3437/8192, 0.532285334012406]
[3083/16384*sqrt(2) + 4361/16384, 0.532288843554459]
[3409/32768*sqrt(2) + 12621/32768, 0.532289246647030]
[7407/65536*sqrt(2) + 24409/65536, 0.532288816169701]
[22805/131072*sqrt(2) + 37517/131072, 0.532288667983386]
[24307/262144*sqrt(2) + 105161/262144, 0.532288700334941]
[72761/524288*sqrt(2) + 176173/524288, 0.532288728736551]
[159959/1048576*sqrt(2) + 331929/1048576, 0.532288729880941]
[202621/2097152*sqrt(2) + 829741/2097152, 0.532288725958633]
[639131/4194304*sqrt(2) + 1328713/4194304, 0.532288724978704]
[1114081/8388608*sqrt(2) + 2889613/8388608, 0.532288725350163]
[1825983/16777216*sqrt(2) + 6347993/16777216, 0.532288725570602]
[5183461/33554432*sqrt(2) + 10530125/33554432, 0.532288725561857]
Code
def sq2(n):
c=0
for i in range(2^n):
l=list(Integer(i).digits(base=2,padto=n))
if l[-1]==1:
cc=1/4
for j in range(n-2):
ll=[l[j],l[j+1],l[j+2]]
if ll==[0,0,0]:
cc*=1/2
if ll==[0,0,1]:
cc*=1/2
if ll==[0,1,0]:
cc*=(1-sqrt(2)/2)
if ll==[0,1,1]:
cc*=sqrt(2)/2
if ll==[1,0,0]:
cc*=1
if ll==[1,0,1]:
cc=0
break
if ll==[1,1,0]:
cc*=(1-sqrt(2)/2)
if ll==[1,1,1]:
cc*=sqrt(2)/2
c+=cc
return [c.expand(),c.n()]<|endoftext|>
TITLE: Evaluation of the quality of research articles submitted in mathematical journals: how do they do that?
QUESTION [25 upvotes]: I would like to know as curiosity how the editorial board or editors* of a mathematical journal evaluate the quality, let's say in colloquial words the importance, of papers or articles.
Question. I would like to know how is evaluate the quality of an article submitted in a journal. Are there criteria to evaluate it? Many thanks.
I think that it must be a difficult task to evaluate the quality of a mathematical paper due how is abstract (the high level of abstraction of research in mathematics) the work of professional mathematicians. Are there criteria to evaluate it, or is it just the experience, knowledges and good work of the people working as publishers?
I'm curious about it but I think that this is an interesting and potentially useful post for other. As soon as I can I should accept an answer.
If there is suitable references in the literature about how they do this work, feel free to refer the literature, answering this question as a reference request, and I try to search and read it from the literature.
*I don't know what is the role of each person working in the edition of a mathematical journal.
REPLY [35 votes]: Assume we are talking about a good journal with a large editorial board representing a wide scope of mathematical interests. I will describe both the role of the editors and the role of the referees. This is my personal viewpoint and others might have different opinion/experience.
The role of the editors.
Good journals can accept only about 20% of submitted papers. This is not easy to reject 80% of papers and it often results in rejecting really good papers. Everyone understands that. The procedure of evaluating the papers by the editors is more or less as follows:
The editors have many years of research experience and (hopefully) developed a good mathematical taste. If the paper is close to the research interest of the editor then he or she can relatively easily identify the papers that are not particularly interesting. The reason for not being interesting can be based (for example) on the following criterion:
The result is not well motivated. It is very technical and follows more or less standard arguments. The authors simply take a known result, and prove a new result by slightly modifying the given assumptions. Usually it means that they make the statement more complicated and in a sense more general. Often, they neither have interesting examples supporting such generalizations nor indication of possible applications.
Unfortunately, most of the papers fell into this category. If the editor is sure that this is the case, then he or she rejects the paper without sending it to a referee. Then the authors usually get a rejection notice similar to this one:
We regret that we cannot consider it, in part because at present
we have a large backlog of excellent articles awaiting publication.
We are thus forced to return articles that might otherwise be considered.
If the editor is not sure about the quality of the paper, then they ask an expert (or several experts) for a quick opinion:
I wonder if you could make a quick, informal assessment of it. Are the results strong enough to warrant sending the article to a referee? Because of our backlog, we like to send to referees only articles that appear to be of very good to outstanding quality.
In that case the expert evaluating the paper is not asked to check all the proofs but to make a judgement based on the criterion explained above. This is an easy task for an expert. If the expert writes a negative opinion, then the authors receive a rejection notice often phrased the way as the rejection notice listed above.
For top journals all experts have to write a positive opinion before the paper is sent to referees.
If the experts' opinion is positive, then the paper is sent to a referee or to many referees. The most extreme case that I know of was a panel of 12 referees who took several years to evaluate the paper (this was when Thomas Hales proved the famous Kepler conjecture). For top quality journals all referees must write positive reports before the paper is accepted (once I received 6 reports, 4 positive and 2 not so positive and the paper was rejected).
Let me also add that the editorial boards are structured basically in two different ways. (1) The authors are asked to choose an editor from the list of editors and submit the paper directly to them. Then the editor who receives the paper handles the submission process according to the rules explained above. (2) The authors submit the paper to the main editor or just to the journal and then the main editor either rejects the paper by themselves or he/she sends it to one of the editors from the editorial board and that editor applies the rules listed above.
Of course some of the journals might have a slightly different approach than the one explained here. There is no a canonical solution and what I wrote is a somewhat a simplified version of the process that is applied in reality.
The role of referees. A paper passed through an initial screening and it was sent to a referee. This is the most unpleasant part of the process. A referee spends a lot of time to read the paper, they are not paid for this job and since their work is anonymous, they do not get any recognition for what they do.
What is the referee required to do? First of all, the referee has to assess originality of the results and whether the results are interesting enough. This part is the same as the one in the initial screening when the paper is sent to an expert for a quick opinion. Secondly, the referee is required to read the paper and check details. Let's be clear about that. Unless the paper is directly related to the research of the referee and he or she really wants to understand the details, there is no way the referee can check all details. Since I cannot speak for other people, let me say what I do in this situation.
My answer will only be a simplified version of the real process of the refereeing a paper, just a main idea of what I do.
I go through the whole paper (or most of the paper) to have a good idea of what it is all about, to see a big picture not only of the meaning of the theorems, but also a big picture of the techniques used in the proof. Then I check carefully details of many/some arguments while for other arguments I briefly skim over. If the argument seems reasonable and believable to me I do not bother checking it very carefully. If all details that I check are correct and if all other arguments seem reasonable I am content. In this case, if I like the statement of the main result, I accept the paper. If however, some arguments seem fishy to me, then I check them carefully. This is a point where often I ask the authors for further clarifications. If I really cannot pass through the paper, because I think it has mistakes or if it is written in an unreadable way, I often reject the paper.
The biggest problem is when I am convinced that the result proved in a paper is of an outstanding quality, but the paper is very difficult and for that reason not very easy to read. Then, hmm... Then, it is not easy and I often struggle with making a right decision.
REPLY [6 votes]: (i) I think almost every journal has a declared "Aims and scope" policy, and how well the submitted paper fits that policy is perhaps the important criterion in the editors' judgment of how good the paper would be for the readers of the journal. (ii) Obviously, for any reputable mathematical journal, the mathematics in the paper must be correct and nontrivial.
Nowadays, in the era of citation indexes, the main criterion, after the criteria (i) and (ii) above, seems to be how "topical" the paper is, that is, how high is the level of current interest the paper may attract.<|endoftext|>
TITLE: Generalized "Homology Whitehead" -- How much does stabilization remember?
QUESTION [8 upvotes]: Classically, the (non-local-coefficients) homology Whitehead theorem says that if $X \xrightarrow f Y$ is a map of simple spaces, and if the induced map $H_\ast(X;\mathbb Z) \to H_\ast(Y;\mathbb Z)$ is an isomorphism, then $f$ is a weak homotopy equivalence.
Conceptually for me, the essence of this theorem is that we have a stable invariant ($H_\ast(-;\mathbb Z)$), and we identify a (reasonably large) class of spaces (the simple spaces) such that our invariant detects equivalences when restricted to this class. I'm wondering how generally a statement of this form holds in a fairly general $\infty$-category $\mathcal C$ in place of $Spaces$.
For my purposes, I'm not particularly concerned with which stable invariant we use, so we might as well restrict attention to the universal case. Moreover, there are two general forms of "stabilization" which come to mind -- the category of spectrum objects $Sp(\mathcal C) = \varprojlim (\cdots \xrightarrow \Omega \mathcal C_\ast \xrightarrow \Omega \mathcal C_\ast)$, and the Spanier-Whitehead category $SW(\mathcal C) = \varinjlim (\mathcal C_\ast \xrightarrow \Sigma \mathcal C_\ast \xrightarrow \Sigma \cdots)$ (I use $\mathcal C_\ast$ to denote the $\infty$-category of pointed objects in $\mathcal C$). But we're eventually passing to some subcategory anyway, so we can reduce the $SW$ notion to the $Sp$ notion if we start out by replacing $\mathcal C$ with $Ind(\mathcal C)$ via the equation $Ind(SW(\mathcal C)) = Sp(Ind(\mathcal C))$.
Thus we are led to the following formulation:
Question: Let $\mathcal C$ be a presentable $\infty$-category. Can we identify a (reasonably large) full subcategory $\mathcal D \subseteq \mathcal C$ such that the composite functor $\mathcal D \to \mathcal C \xrightarrow {\Sigma^\infty_+} Sp(\mathcal C)$ is conservative? In particular, is this the case for $\mathcal D$ being one of the following?
The 1-fold suspension objects?
The 1-fold loop objects?
The 1-connected objects?
Here, a 1-fold suspension object is simply an object of the form $X = \Sigma Y$ for some $Y \in \mathcal C$; a 1-fold loop object is an object of the form $X = \Omega Y$ where $Y \in \mathcal C_\ast$ is a pointed object of $\mathcal C$. A 1-truncated morphism $W \to Z$ is a morphism such that for every $C \in \mathcal C$, the map $\mathcal C(C,W) \to \mathcal C(C,Z)$ has 1-truncated fibers, a morphism is 1-connected if it is left orthgonal to the 1-truncated morphisms, and an object $X$ is 1-connected if the map $X \to 1$ is 1-connected, where $1$ is the terminal object.
As a sanity check, I think each of my candidates for $\mathcal D$ are trivial when $\mathcal C$ has discrete hom-spaces, which is a good thing because in this case $Sp(\mathcal C)$ is also trivial.
REPLY [5 votes]: Putting together my and Dan's comments deserves to be called an answer. Namely:
If $\mathcal{C}$ is an $(\infty,1)$-topos, then the statement is true when $\mathcal{D}$ is the class of hypercomplete, pointed, nilpotent objects. Hypercompleteness is the usual $(\infty,1)$-categorical notion, pointed is obvious, while "nilpotent" here means that the internal group object $\pi_1(X)$ is nilpotent and acts nilpotently on each internal abelian group object $\pi_n(X)$, in an appropriate internal sense.
For a proof, see section 3 of Nilpotent Types and Fracture Squares in Homotopy Type Theory by Luis Scoccola, which proves in homotopy type theory that any cohomology isomorphism between pointed nilpotent types is $\infty$-connected — hence an equivalence if the types are hypercomplete. Then we get the result for $(\infty,1)$-toposes by interpreting homotopy type theory internally therein, as shown here for universes and here for higher inductive types. (Those papers don't yet quite complete the interpretation by showing that the universe is closed under HITs, but I doubt that this proof depends crucially on that.)
Note that the assumption used here is weaker than yours, namely that an isomorphism is induced only on internal cohomology group objects with coefficients in all internal abelian group objects. It seems likely that with your stronger assumption hypercompleteness could be removed from the result, but probably a different method would be required.<|endoftext|>
TITLE: Birkhoff's completeness theorem put into practice
QUESTION [8 upvotes]: Birkhoff's completeness theorem (see here, Theorem 14.19) states that an equation which is true in all models of an algebraic theory can be proven in equational logic.
Question. Does the proof of Birkhoff's completeness theorem actually produce for each specific equation a proof in equational logic? If yes, can you please demonstrate this with an instructive example?
Actually, I suspect that the answer is "No", but I am not entirely sure.
Let us look at the following well-known statement: If $R$ is a ring in which every element $r \in R$ satisfies $r^2=r$ (i.e. $R$ is boolean), then $R$ is commutative. There is the following overkill proof: $R$ is reduced, hence a subdirect product of domains $R_i$. Since the maps $R \to R_i$ are surjective, each domain $R_i$ satisfies the same equation and then must be isomorphic to $\mathbb{F}_2$. In particular, $R_i$ is commutative. Since $R \to \prod_{i \in I} R_i$ is injective, it follows that $R$ is commutative. So by Birkhoff's theorem, there must be some equational proof of this. My question is not how an equational proof looks like - this is just a basic algebra exercise. I would like to see how (if possible) it can be extracted from the overkill proof.
I think the proof of Birkhoff's theorem in this special case works as follows: Consider the free ring on two generators $\mathbb{Z}\langle X,Y\rangle$ and take the quotient with respect to the relations $r^2=r$ for all elements $r$. This is the free boolean ring $R$ on two generators. By the overkill proof, $XY=YX$ holds in $R$. This means that $XY=YX$ can be derived from the relations $r^2=r$ in $\mathbb{Z}\langle X,Y\rangle$. But we don't get a derivation, right? How to produce, for example, the following equation?
$$\begin{align*}
XY - YX &= \bigl((X+Y)^2 - (X+Y)\bigr) - \bigl(X^2-X\bigr) - \bigl(Y^2-Y\bigr) \\
&\phantom{=}+ \bigl((YX)^2 - (YX)\bigr) - \bigl((-YX)^2 - (-YX)\bigr),
\end{align*}$$
As far as I can tell, we are not even guaranteed a priori to have a proof which works without the axiom of choice, since this is used in the structure theorem for reduced rings in the overkill proof? (At first this might be confusing since the axiom of choice is surely not allowed in an equational proof, but actually the axiom of choice is used to show the existence of an equational proof.)
I am interested in more complicated applications of Birkhoff's theorem. This here is just an example to get started. You can also choose other examples if they are more instructive.
REPLY [10 votes]: Let me try to restate the question.
I consider an identity to be a pair, written $(s,t)$ or $s\approx t$. I also consider a set of identities to be a set of pairs.
Birkhoff's Theorem compares three things, namely
(1) $\Sigma\models s\approx t$,
(2) The pair $(s,t)$ belongs to the fully invariant congruence $\Theta^{T(X)}(\Sigma)$ on the term algebra $T(X)$, and
(3) $\Sigma\vdash s\approx t$.
I think the question is asking whether it is possible to extract from the proof of Birkhoff's Theorem and from a specific instance of $\Sigma\models s\approx t$ a proof witnessing that $\Sigma \vdash s\approx t$.
The proof of Birkhoff's theorem does explain why (1) $\Leftrightarrow$ (2). Also, if we are told HOW the pair $(s,t)$ is generated as a member of $\Theta^{T(X)}(\Sigma)$, that can be translated into a $\Sigma$-proof of $s\approx t$. What is missing is: given the knowledge that
$(s,t)\in\Theta^{T(X)}(\Sigma)$, do we know how the pair $(s,t)$ got into $\Theta^{T(X)}(\Sigma)$?
The proof of Birkhoff's Theorem does not require this missing ingredient, it only requires that if $(s,t)$ is in $\Theta^{T(X)}(\Sigma)$, then it was generated in some way. So the answer to the question in the form it was asked (can we extract a $\Sigma$-proof of $s\approx t$ from the proof of Birkhoff's Theorem?) has to be No.
You could try asking a slightly different question: Suppose, given finite $\Sigma$, that we are told that $\Sigma \models s\approx t$. By B's Theorem we know there must exist a $\Sigma$-proof of $s\approx t$. Question: can we estimate an upper bound on the complexity of such a proof?
An affirmative answer would imply that any finitely axiomatizable variety has a decidable equational theory. But we know this to be false, since in the 1940's Tarski found a finitely axiomatizable variety of relation algebras with an undecidable equational theory. Many other examples are known now.<|endoftext|>
TITLE: Lipschitz-continuity of convex polytopes under the Hausdorff metric
QUESTION [6 upvotes]: Recently, I proved the following Lipschitz-continuity like result for convex polytopes:
Let $A\in\mathbb R^{m\times n}$ and $b,b'\in\mathbb R^m$ be given such that $\{x\,:\,Ax\leq 0\}=\{0\}$ (which is equivalent to boundedness of all induced polytopes) and that $\{x\in\mathbb R^n\,:\,Ax\leq b\},\{x\in\mathbb R^n\,:\,Ax\leq b'\}$ are non-empty. Then
$$
\delta\big(\{x\in\mathbb R^n\,:\,Ax\leq b\},\{x\in\mathbb R^n\,:\,Ax\leq b'\}\big)\leq \Big( \max_{A_0\in\operatorname{GL}(n,\mathbb R),A_0\subset A}\|A_0^{-1}\| \Big)\|b-b'\|_1
$$
where the operator norm $\|\cdot\|$ as well as the Hausdorff metric $\delta$ are taken with respect to $(\mathbb R^n,\|\cdot\|_1)$. Also $A_0\subset A$ is short for "every row of $A_0$ is also a row of $A$" so the above maximum is taken over all invertible submatrices of $A$.
What this intuitively means is that if two polytopes have parallel faces (i.e. they are both described by the same $A$ matrix), but the location of these faces differs ($b\neq b'$), then the distance between the polytopes is upper bounded by the distance between the vectors $b,b'$ times a "geometrical" constant coming from $A$.
Hence the function $b\mapsto\{x\in\mathbb R^n:Ax\leq b\}$ (with suitable domain such that the co-domain equals all non-empty subsets of $\mathbb R^n$) is Lipschitz continuous with a constant determined by $A$.
This came up as a lemma to something only vaguely related which is why I don't have a problem with posting it publicly. Actually if this was a known result then my manuscript could be shortened by 3 pages. Thus my quesiton is:
Is the above result known and, if so, where can in be found in the (convex polytope-)literature?
I would be surprised if nobody has thought about this until now. While I haven't seen this result in the books of Grünbaum and Schrijver or the few papers on convex polytopes I am aware of, this is not the field I usually work in; hence this might very well be known but beyond my mathematical horizon. Thanks in advance for any answer or comment!
REPLY [2 votes]: This is a classic question in the literature on linear programming, since it is related to the stability of the feasible set (and hence the solutions) under perturbation of the parameters.
The classic work in this field is:
A. J. Hoffman, Approximate solutions of systems of linear inequalities, J. Res. Nat. Bur. Standards, 1952.
A more recent work, which essentially states your result as a special case (see Theorem 2.4 and the Lipschitz constant on p.19), is:
W. Li, The sharp Lifshitz constants for feasible and optimal solutions of a perturbed linear program, Linear Algebra and Its Applications, 1993.<|endoftext|>
TITLE: Is this subset of matrices contractible inside the space of non-conformal matrices?
QUESTION [5 upvotes]: Set $\mathcal{F}:=\{ A \in \text{SL}_2(\mathbb{R}) \, | \, Ae_1 \in \operatorname{span}(e_1) \, \, \text{ and } \, \, A \, \text{ is not conformal} \,\}$, and
$\mathcal{NC}:=\{ A \in M_2(\mathbb{R}) \, | \det A \ge 0 \, \,\text{ and } \, A \text{ is not conformal} \,\}$.
By a non-conformal matrix, I mean a matrix whose singular values are distinct. (i.e. I allow non-zero singular matrices in $\mathcal{NC}$).
Is each connected component of $\mathcal{F}$ contractible in $\mathcal{NC}$?
$\mathcal{F}$ has two connected components, both homeomorphic to an open half-plane with one point removed.
Indeed, $Ae_1 \in \operatorname{span}(e_1)$ and $A \in \text{SL}_2(\mathbb{R})$ imply that
$$ A=\begin{pmatrix} \lambda & y \\\ 0 & \lambda^{-1} \end{pmatrix} \, \, \, \text{for some }\, \lambda \neq 0.$$ $A$ is conformal if and only if $\lambda=\pm 1$ and $y=0$. So, $A$ is not conformal if f $\lambda \neq 1,-1,0$ or $\lambda=\pm 1$ and $y \neq 0$. Thus, one connected component of $\mathcal{F}$ is homeomorphic to
$$\{ 0<\lambda \neq 1\} \times \mathbb{R} \cup \{1\} \times \mathbb{R}\setminus\{0\}.$$
(The second component corresponds to $\lambda <0$.)
Here is what I know about the topology of $\mathcal{NC}$:
Let $\mathcal D=\{ (\sigma_1,\sigma_2) \, | 0 \le \sigma_1 < \sigma_2\}$. Then the map
\begin{align*}
\mu: SO_2\times \mathcal D\times SO_2\to \mathcal{NC}\\
(U,\Sigma,V)\mapsto U\Sigma V^T
\end{align*}
is a $2$-fold smooth covering map*. (i.e. $\mu(U,\Sigma,V)=\mu(-U,\Sigma,-V)$, and this is the only ambiguity in $U,V$ for a pre-image of a given point in $\mathcal{NC}$.
Since $SO_2 \cong \mathbb{S}^1$, and since after identifying antipodal points in $\mathbb{S}^1 \times \mathbb{S}^1$, we get the $2$-torus $\mathbb{T}^2$ again, it follows that $\mathcal{NC} \cong \mathbb{T}^2 \times D$.
*I am not entirely sure regarding the behaviour at the boundary points where $\sigma_1=0$, but I don't think this creates a serious problem.
REPLY [2 votes]: Edited. In the first version of the answer I was assuming that the space in which the contraction was taking place was not $\cal NC$ but the complement to non-conformal matrices in $SL(2,\mathbb R)$. I'll suggest a fix for this now.
Note, that we have a natural continuous map $u: {\cal NC}\to S^1=\mathbb RP^1$. Namely, to each matrix $A$ from ${\cal NC}$ we can associate the following one-dimensional subspace $u(A)\in \mathbb R^2$. Take the matrix $AA^{*}$ and take the eigenspace corresponding to the maximal eigenvalue of $AA^*$ (there will be two distinct eigenvalues since $A$ is not conformal).
So, if we find a closed path $\gamma$ in $\cal F$, such that its image $u(\gamma)\subset S^1$ is not contractible, we are done. How to find such a path is explained in the previous answer to this question, which the path $\gamma(t)$ constructed in the previous answer below. ( I believe that what I suggest works for several reasons but I don't have time to work out all the details now. By the way, it is also funny that $\pi_1(\cal NC)$ seem to be equal $\mathbb Z^2$, moreover it deformation retracts to $T^2$, I believe.)
Previous answer.
It is not contractible. Let us associate to each matrix $A\in SL_2(\mathbb R)$ the following vector $v(A)$. Take an orthogonal matrix $O\in SO_2(\mathbb R)$ such that $OA(e_1)$ is proportional to $e_1$ with a positive coefficient. Then set $v(A)=OA(e_2)$. We get a map to the upper half plane:
$$V:SL(2,\mathbb R)\to \{y>0\}$$
Note that the image of confromal matrices is the point $(0,1)$, and the image of any component $\cal F$ is the complement to $(0,1)$. And so each component can be identified with this puncutred half-plane. Hence it is enough to construct a path in $\cal F$ whose image under $V$ is not contractible in $\{y>0\}\setminus \{(0,1)\}$. This is easy, just take a non-contractible path $\gamma(t)\subset \{y>0\}\setminus \{(0,1)\}$ (that has a non-zero winding number around $(0,1)$), and consider the unique path of matrices $A_t\subset \cal F$ such that $A_t(e_2)=\gamma(t)$.<|endoftext|>
TITLE: Is the inclusion of its 2-skeleton into the walking idempotent homotopy cofinal?
QUESTION [6 upvotes]: Let $Idem = Idem^{(\infty)}$ be the walking idempotent [1], and let $Idem^{(n)}$ be its n-skeleton. Note that $Idem$ has one nondegenerate simplex in each dimension. Let $\iota_n^m: Idem^{(n)} \to Idem^{(m)}$ be the inclusion. Lurie has shown [2] the following:
If $X$ is a quasicategory, and if $Idem^{(3)} \xrightarrow f X$ is a map, then there exists a map $Idem \xrightarrow g X$ such that $g\iota_1^\infty = f\iota_1^3$.
That is, although a homotopy-coherent idempotent involves infinitely many pieces of coherence data corresponding to the infinitely many nondegenerate simplices of $Idem$, nevertheless all of this data is guaranteed to exist once the first 3 have been found. However, the resulting coherence data might be different from the original data in dimensions 2 and 3 [3].
The proof is a bit involved, and I have not studied it in detail, but what I do understand seems to suggest an affirmative answer to the following
Question: Is the inclusion $Idem^{(n)} \to Idem$ homotopy cofinal for certain $n$ (i.e. is it cofinal in the $\infty$-categorical sense -- depending on how one defines this, it may be necessary to Joyal-fibrantly replace $Idem^{(n)}$ before the question makes sense)? As noted in the comments, this is definitely not true for $n$ odd, nor is it true for $n=0$. Somehow it seems unlikely for $n=2$; hence the title question, which asks this for $n=4$.
I don't believe the inclusion $Idem^{(n)} \to Idem$ is left or right anodyne, so any proof will encounter some complications.
One idea would be to use the usual map $N \to Idem^{(n)}$, where $N \subseteq \mathbb N$ is the graph on which the poset $\mathbb N$ of natural numbers is free -- the 0-cells are natural numbers, and there is a 1-cell from $n$ to $n+1$ for each $n$. For the inclusion $N \to \mathbb N$ is a categorical equivalence, and it's easy to show that $\mathbb N \to Idem$ is homotopy cofinal using Quillen's Theorem A (since the relevant slice categories are just ordinary 1-categories). By composition, $N \to Idem$ is homotopy cofinal, but this factors through $Idem^{(n)}$. By a cancellation property of cofinality, in order to show that $Idem^{(n)} \to Idem$ is homotopy cofinal, it will suffice to show that $N \to Idem^{(n)}$ is homotopy cofinal, which sounds straightforward, since $N$ and $Idem^{(n)}$ are finite-dimensional. But I'm not sure the relevant slice objects are still finite-dimensional...
[1] That is, $Idem$ is the category with one object and one non-identity morphism $i$, satisfying the equation $i^2 = i$. In this question I identify 1-categories with their nerves, which are quasicategories.
[2] HTT 4.4.5.20 in the current version. This does not appear in the published version of HTT. It appears in older versions of HA as 7.3.5.14, but was moved to HTT when Lurie rewrote the section on idempotents in HTT.
[3] For example, consider the inclusion $Idem^{(3)} \to \widetilde{Idem^{(3)}}$ given by a Kan fibrant replacement such as $Ex^\infty$. This map is nontrivial on homology, so it cannot actually be extended to a map $Idem \to \widetilde{Idem^{(3)}}$ since this would amount to a trivialization. Thus the extension produced by Lurie's theorem must be changing the image of the 3-cell of $Idem^{(3)}$, at least. In particular, it's not the case that all quasicategories have the right lifting property with respect to $\iota_3^\infty$.
REPLY [2 votes]: I don't know what's wrong with the following computation, but the answer is clearly no: if there were a cofinal functor from a finite simplicial set to $Idem$, then any $\infty$-category with finite colimits would have split idempotents, which is not the case (witness finite spaces).
Somewhat surprisingly, this seems to work for even $n>0$, even for $n=2$! That is,
Claim: Let $n \in \mathbb N$. Then the inclusion $Idem^{(n)} \to Idem$ is homotopy cofinal (equivalently, since everything is self-dual: co-cofinal) if and only if $n$ is positive and even.
Proof: We have seen that this can't happen when $n=0$ or $n$ is odd. Otherwise, we verify the hypotheses of the Joyal-Lurie version of Quillen's Theorem A, i.e. we check that the simplicial set $X^{(n)} = Idem^{(n)} \times_{Idem} Dec(Idem)$ is weakly contractible. Here $Dec(Idem)$ is the decalage contruction $Dec(C)_n = C_{n+1}$ where we forget the 0th degeneracy. So in terms of simplices, we have $X^{(n)}_m = Idem^{(n)}_m \times_{Idem_m} Idem_{m+1}$. For $m \geq n+1$, an $m$-simplex in $X^{(n)}_m$ is a string of morphisms in $Idem$ (each either $i$ or $1$) of length $m+1$, such that among the last $m$ morphims in the string, all but at most $n$ are $1$. Any such simplex is a degeneracy of a simplex obtained by deleting one of the copies of $1$. That is, $X^{(n)}$ is $n$-skeletal. Now, the $n$-skeleton of $X^{(n)}$ agrees with that of $X^{(\infty)}$, which is weakly contractible. Therefore, since $n \geq 2$, we have $\pi_1(X^{(n)}) = \tilde H_{\leq n-1}(X^{(n)}) = 0$. So it will suffice to show that $H_n(X^{(n)}) = 0$. There are two nondegenerate simplices of degree $n$: the string $1,i,\dots,i$ (a $1$ followed by $n$ $i$'s) and the string $i,i,\dots,i$ (a string of $n+1$ $i$'s). The boundaries of these (remember that $n$ is even and we are omiting the $\partial_0$ term of the boundary map) are $1,i,\dots,i$ and $i,i,\dots,i$ (where now we have one fewer term in each string) respectively, which are linearly independent. Thus there are no nondegenerate cycles and $H_n(X^{(n)}) = 0$ as desired.<|endoftext|>
TITLE: Thickness and hierarchical hyperbolicity
QUESTION [7 upvotes]: Thick metric spaces were introduced by Behrstock, Drutu and Mosher, see here. Hierarchically hyperbolic spaces were introduced by Behrstock, Hagen and Sisto, see here.
I've heard that it is open whether hierarchically hyperbolic groups are thick. Of course, one has to restrict to hierarchically hyperbolic groups that are not non-elementary relatively hyperbolic (NERH for short), for it was proved in Behrstock, Drutu, Mosher that thick groups cannot be NERH. See also there, where Levcovitz proved that thick groups have trivial Floyd boundary, which cannot happen for NERH groups, since the Floyd boundary covers the Bowditch boundary.
Question : Do we know if this is true among all known examples of hierarchically hyperbolic groups (that are not NERH) ?
Note for example that we know that mapping class groups and Artin groups that are not NERH are indeed thick.
Special emphasis on CAT(0) cube complexes. As far as I know, we still don't know if all cubical groups are hierarchically hyperbolic, although to my knowledge, it's likely to be true. On the other hand, do we know if all (not NERH) cubical groups are thick ?
REPLY [4 votes]: This is not an answer, just some sketchy thoughts that are too long for the comment box. I and some other HHS enthusiasts are very interested in this question being answered; we've tried a fair bit and have set it aside, so I don't think they'll mind me trying to recall what some of the strategies and issues are.
It's indeed open for cocompact special groups, as far as I know. (Part of the motivation for the question comes from Coxeter groups, which are known to be hyperbolic relative to thick subgroups [Theorem VII here], and many of which are virtually compact special.)
There's a very similar question: for the classes of groups in question (HHG, cocompactly cubulated, virtually compact special), is it true that any group $G$ in the class is hyperbolic relative to a collection of subgroups, each of which has at most polynomial divergence (we're allowing $G$ to be hyperbolic relative to itself)?
In particular, does super-polynomial divergence imply exponential divergence for such $G$? (A nontrivially relatively hyperbolic group has at least exponential divergence [Theorem 6.13 here].)
It's also interesting to ask whether $G$ is hyperbolic relative to NRH subgroups (whether because the peripheral subgroups are thick, or NRH for some other reason).
Here is a naive approach that has been tried a couple of times (once for cubical groups, once for HHG) unsuccessfully. I think it's worth sketching because it's probably the first thing one might try, so it might save someone some work to see what the problems with it are. (Maybe they are surmountable.)
The idea is to build candidate thick/NRH subspaces inductively by "brute force", show that this construction terminates, and then show that the resulting subspaces give a valid peripheral structure.
More precisely: let $G$ act geometrically on a proper CAT(0) cube complex $X$, or a proper hierarchically hyperbolic space $X$. In either case, either $X$ is already hyperbolic, or there is a nontrivial product region $P$. (In the cubical case, $P$ is a convex subcomplex decomposing as the product of two unbounded CAT(0) cube complexes. In the HHS case, $P$ is a hierarchically quasiconvex subspace admitting a coarse-median-preserving quasi-isometry to the product of two unbounded hierarchically hyperbolic spaces. The HHS fact follows from Corollary 2.16 here.)
Let $\mathcal P_0$ be the ($G$--invariant) set of such product regions. Call $P,P'\in\mathcal P_0$ "elementary equivalent" if they have unbounded coarse intersection. Taking the transitive closure gives an equivalence relation on $\mathcal P_0$. For each equivalence class, one can take the union $U$ of the subspaces in the class. These $U$ are your candidate thick-of-order-at-most-$1$ pieces.
Now iterate this process: take the set of such $U$, impose the same equivalence relation, and make candidate thick-of-order-at-most-$2$ pieces, etc.
(If I remember right, the equivalence relation used at the $n^{th}$ stage, $n\geq 1$, is that $P,P'\in\mathcal P_{n-1}$ are elementary equivalent if their hierarchically quasiconvex hulls have unbounded coarse intersection. These are described nicely by Russell-Spriano-Tran in Section 5 of this paper; in the cubical case, you use cubical convex hulls.)
Suppose the process terminates, i.e. at some point you have a $G$--invariant collection $\mathcal P_n$ of subspaces, no two of which have hierarchically quasiconvex hulls with unbounded coarse intersection.
At this point, you should:
(1) Verify that each $P$ is "quasiconvex" in the sense appropriate to the category you're working in. In the cubical case, you should check that $P$ is at finite Hausdorff distance from its cubical convex hull; in the HHS case, you should check that $P$ is \emph{hierarchically quasiconvex} --- Proposition 5.11 in this paper by Russell-Spriano-Tran is probably the best way to go about this.
(2) Hope that the stabiliser of each $P\in\mathcal P_n$ acts coboundedly. ($G$ acts on $X$ coboundedly, $\mathcal P_n$ is $G$--invariant; you want to use properness of $X$ and boundedness of the coarse intersections between the elements of $\mathcal P_n$ to show that only finitely many elements of $\mathcal P_n$ intersect any given ball.)
Already there is something tricky here.
(3) Verify that adding combinatorial horoballs over the elements of $\mathcal P_n$ in $X$ gives you something hyperbolic.
I think one approach was to verify hyperbolicity by finding a hierarchically hyperbolic structure with no interesting "product regions".
(4) Hope that, more or less by construction, each $P\in\mathcal P_n$ is thick of order at most $n$.
At that point, you'd have what you want.
Difficulties include:
(a) It's not remotely clear that this process terminates. (But I think it's worth nailing down whether or not it does).
(b) If it doesn't terminate, one can still hope to be able to pass to some sort of limit, and find a collection $\mathcal P_\infty$ of subspaces, all with bounded coarse intersection, to get candidate peripherals in a relatively hyperbolic structure. If this works, one doesn't get that the peripherals are thick, but the hope is that they are at least not relatively hyperbolic.
(c) If I remember correctly, one big difficulty is that, at each (finite) stage of the induction, one might have to pass to larger and larger neighbourhoods to get something convex/hierarchically quasiconvex, and this creates problems if the process doesn't terminate.
More vaguely, at each step, there are various constants to control, and they blow up if the process doesn't terminate.
There was a quite different approach (for CAT(0) groups) discussed by many people at the AIM in 2016. There was some subsequent work on it and some useful ideas, but I'll have to check with those involved to see if it's okay to mention the idea/subsequent developments here --- it's possible that one or more of them are still actively working on it, although I'm not.
So, the short answer, is that it's a very interesting question for which the naive approach is a bit of a mess.
My feeling is that if there's a counterexample, then there's a counterexample where $G$ is $\pi_1$ of a CAT(0) square complex.<|endoftext|>
TITLE: Maximal subgroups of odd index in $\mathrm{PSL}(3,q)$
QUESTION [5 upvotes]: Let $G = \mathrm{PSL}(3,q)$ for $q$ odd. I am trying to understand a question that involves understanding the subgroups that contain a Sylow $2$-subgroup, and in particular, are subgroups of odd index in $G$. I need to find a complete description of the maximal subgroups of odd index in the group $G = \mathrm{PSL}(3,q)$
REPLY [8 votes]: The subgroups of ${\rm PSL}_3(q)$ for odd $q$ were first enumerated by H.H. Mitchell in 1911. (The case $q$ even was done by R.W. Hartley in 1925/6.)
Table 8.3 of the book "The Maximal Subgroups of Low-Dimensional Finite Classical Groups" by Bray, Holt and Roney-Dougal provides a convenient list. Using that it is not hard to answer your question.
For $q$ odd, the maximal subgroups of ${\rm SL}_3(q)$ of odd index are as follows:
(i) Two classes of maximal parabolic subgroups with structure ${\rm E}_q^2:{\rm GL}_2(q)$ and index $q^2+q+1$, where ${\rm E}_q$ denotes an elementary abelian group of order $q$ (the additive group of the field). These two classes are interchanged by the graph (inverse-transpose) automorphism of ${\rm SL}_3(q)$.
(ii) When $q \equiv 1 \bmod 4$, we have one class of imprimitive subgroups with structure $(q-1)^2:S_3$.
(iii) When $q = q_0^r$ for some odd prime $r$, we have $\gcd(\frac{q-1}{q_0-1},3)$ classes of subgroups with the structure ${\rm SL}_3(q_0).\gcd(\frac{q-1}{q_0-1},3)$. When there are three such classes, they are all conjugate under a diagonal outer automorphism of ${\rm SL}_3(q)$.<|endoftext|>
TITLE: What is the winning strategy in this pebble game?
QUESTION [19 upvotes]: Consider the following two-player pebble game. We have finitely
many stones on a finite linear track of squares. We take turns, and
the allowed moves are:
move any one stone one square to the left, if that square is empty, or
remove any one stone, or
remove any two adjacent stones.
Whoever takes the last stone wins.
Question. What is the winning strategy? And which are the
winning positions?
The game will clearly end always in finitely many moves, and so by
the fundamental theorem of finite games, one of the players will
have a winning strategy. So of course, I know that there is a
computable winning strategy by computing with the game tree, and we have a computable algorithm to answer any instance of the question. What I am hoping for is that there will be a simple-to-describe winning strategy.
This is what I know so far:
Theorem. It is a winning move to give your opponent a position
with an even number of stones, such that the stones in each successive pair stand
at even distance apart.
By even-distance, I mean that there are an odd number of empty
squares between, so adjacent stones count as distance one, hence
odd. Also, I am only concerned with the even distance requirement within each successive pairs, not between the pairs. For example, it
is winning to give your opponent a position with stones at
..O...OO.O....O.....O...........
We have distance 4 in the left-most pair, distance 2 in the next pair, distance 6 in the third pair, ignoring the distances in front and between the pairs.
Proof. I claim that if you give your opponent a position like
that, then he or she cannot give you back a position like that, and
furthermore, you can give a position like that back again. If your
opponent removes a stone, then you can remove the other one in that
pair. If your opponent moves the lead stone on a pair, then you can
move the trailing stone. If your opponent moves the trailing stone
on a pair, then either you can move it again, unless that pair is
now adjacent, in which case you can remove both. And if your
opponent removes two adjacent stones, then they must have been from
different pairs (since adjacent is not even distance), which would
cause the new spacing to be the former odd number plus another odd
number plus 2, so an even number of empty squares between, and so
you can move the trailing end stone up one square to make an odd
number of empty squares between and hence an even distance between
the new endpoints. Thus, you can maintain this even-distance
property, and your opponent cannot attain it; since the winning
move is moving to the empty position, which has all even distances,
you will win. $\Box$
What I wonder is whether there is a similarly easy to describe
strategy that solves the general game.
REPLY [3 votes]: We can go further and find the nim-value of any position of this game.
Theorem. The nim-value of a position in this game is the Nim-sum of the contributions of its pebbles, where
Pebbles in odd-numbered squares contribute $1$.
Pebbles in even-numbered squares contribute $2$.
(Here we count the leftmost square as the $1^{st}$ square.)
Corollary. As in Gro-Tsen's answer, the winning positions (with nim-value $0$) are the ones with both an even number of pebbles in odd-numbered squares and an even number of pebbles in even-numbered squares.
Corollary. All positions have nim-value $0, 1, 2,$ or $3$.
The proof relies on the following idea, which is encoded in the lemma below.
Key Insight. A position in this pebble game is equivalent to the sum of an individual game for each pebble with its own private track.
Lemma. Given a position $P$ of pebbles $p_1, p_2, ... p_n$, consider the games $P_i$ which each consist of only the single pebble $p_i$ in the same square. Then $P$ is equivalent to $\sum_i P_i$.
Proof. To check this, we need only show that the game $P + \sum_i P_i$ is a zero game, by demonstrating a winning strategy for the second player. So consider the possible moves for the first player in the game $P + \sum_i P_i$:
If the first player removes two adjacent pebbles $p_i$ and $p_{i+1}$ in $P$, the second player can respond by moving the pebble in $P_{i+1}$ to the left, leaving behind two copies of $P_i$, which nim-cancel.
If the first player removes any one pebble, the second player responds by removing the corresponding pebble.
If the first player moves a pebble to the left in $P$, the second player responds by moving the pebble to the left in $P_i$.
If the first player moves a pebble to the left in a $P_i$, the second player responds by moving the corresponding pebble to the left in $P$. If that move is blocked by a pebble directly to the left, that's fine -- the second player instead removes those two pebbles, noting that there are now two copies of $P_{i-1}$ that nim-cancel out.
Proof of Theorem. Because of the Lemma, we need only consider the case of a single pebble in position $i$ (where position $1$ is the leftmost square). By induction on $i$,
There is always the option to remove the pebble, an option with nim-value $0$.
If $i$ is odd, there may also be the option to move the pebble to the left, which is an even-numbered square with nim-value $2$. Thus the nim-value in the odd case is $mex(0,2) = 1$.
If $i$ is even, there is always also the option to move the pebble to the left, which is an odd-numbered square with nim-value $1$. Thus the nim-value in the even case is $mex(0,1) = 2$.
How to Evaluate Positions.
Example 1. The nim-value of the position in the question
. . O . . . O O . O . . . O O . . . . . O
is found quickly by seeing that a pebble in an odd-numbered position contributes nim-value 1, and a pebble in an even-numbered position contributes nim-value 2. So using this mask:
. . O . . . O O . O . . . O O . . . . . O
1 2 1 2 1 2 1 2 1 2 1 2 1 2 1 2 1 2 1 2 1
yields the Nim-sum
1 +1+2 +2 +2+1 +1
which equals 2, so removing any of the even-numbered pebbles (the "2s") is a win.
Example 2. You are playing the sum of the following two positions:
O . O . . . O O . O + O . O O
Evaluate each one with the mask:
O . O . . . O O . O + O . O O
1 2 1 2 1 2 1 2 1 2 1 2 1 2
1 +1 +1+2 +2 + 1 +1+2
This has a Nim-sum of 3, so moving any pebble to the left or removing any two adjacent pebbles (in either component!) is a win.<|endoftext|>
TITLE: 2D closed convex shape which minimizes average distance between points
QUESTION [6 upvotes]: For a 2D closed convex shape, with metric $d$ and fixed area $A$, we can calculate the average distance between random (interior) points. For different shapes, we will get different values for this average. We can ask, for which shape is this average a minimum (in terms of $A$). For the Euclidean metric, the answer is the circle. If we pick a different metric, what shape do we get?
REPLY [3 votes]: For an arbitrary metric and measure, this is going to be a very difficult question to answer. The shape of the minimizer will depend on the choice of metric and measure, if it exists at all. Joseph O'Rourke discussed the $L^1$ distance already, so I'm going to focus on distance functions which are either symmetric, or else induced by a smooth Riemannian metric. These sorts of questions are known as extremal problems [1] and are closely related to isoperimetric inequalities, which are an active area of research.
As noted in the comments, we need to specify which measure we use to compute averages. It appears that there are at least two natural choices of measure.
the Lebesgue measure on $\mathbb{R}^2$
the measure induced by a Riemannian metric
As I mentioned earlier, the particular shape of the minimizer will depend on the choice of distance function and measure. However, from a variational standpoint, there is a fairly general principle which is very helpful.
A convex region $S$ is a critical point for the average distance
between interior points iff the integral $\int_S d(p,x) \, d \mu(x)$ is constant for all $p \in \partial S$. Here, $\mu$ is the
measure used to determine averages.
In other words, a region $S$ is a possible minimizer for the average distance between points if the average distance between a point $p$ on the boundary of $S$ and interior points in $S$ does not depend on $p$. It's possible to prove this rigorously using variational techniques or using Crofton's differential equation (for a good exposition, see [2]). However, the basic intuition is that if this property does not hold, we can slightly change the shape of $S$ so as to reduce the total average distance.
From this, there are a few immediate consequences.
If the distance function is translation and rotationally symmetric and the measure we use is the Lebesgue measure on $\mathbb{R}^2$, the disk is a critical point. Oftentimes, this will be the unique critical point (and thus also the minimum), but any proof of this seems likely to depend on the particular details of the distance function.
If we consider a surface of constant curvature with its natural distance function and induced measure, the critical points for the average distance between points are geodesic balls. There is no guarantee that geodesic balls will be convex in the choice of coordinates you use, but they will be the minimizers nonetheless.
For more general (non-symmetric) Riemannian metrics (with their induced distance function and measure), this question is much more difficult, and the variational principles are less useful. However, with a lower bound on the sectional curvature, it is possible to get a lower bound on the areas of geodesic balls. Using this, it should be possible to establish uniform lower bound on the average distance between points in a region of area $A$. However, these sorts of estimates don't tell you what the minimizer looks like (or even if it exists at all).
It is possible to construct distance functions where no minimizer exists, by incorporating patches where the curvature is more and more negative.
[1] Bauer, Christina; Schneider, Rolf, Extremal problems for geometric probabilities involving convex bodies, Adv. Appl. Probab. 27, No. 1, 20-34 (1995). ZBL0827.52004.
[2] Eisenberg, Bennett; Sullivan, Rosemary, Crofton’s differential equation, Am. Math. Mon. 107, No. 2, 129-139 (2000). ZBL0986.60011.<|endoftext|>
TITLE: Proving a specific case of Robin's Inequality
QUESTION [5 upvotes]: Edit: It turns out that this is equivalent to the RH which gives the idea that this might a a little difficult to show. As such we could consider an even simpler case in which the number $n$ is squarefree (all values $k_j$ are equal to $1$. In previous papers it has been shown that squarefree numbers satisfy Robin's Inequality, but is this still the case for $2^kn$? If we make this loose condition we find our simpler form of
$$
\dfrac{2^{k+1}-1}{2^{k}}\prod_{j=1}^m\dfrac{p_j+1}{p_j} < e^\gamma\log\log\left(2^k\prod_{j=1}^m p_j\right)
$$
with a minima of $f(k)$ at
$$
k_{min} = -\dfrac{W_{-1}\left(-e^\gamma\prod_{j=1}^m \frac{1}{p_j+1}\right)+\log\left(\prod_{j=1}^m p_j\right)}{\log2}
$$
if we plug this in to our inequality we need to show that
$$
e^\gamma\log\left(-W\left(-e^\gamma\prod_{j=1}^m\frac{1}{p_j+1}\right)\right) > \dfrac{e^{-W\left(-e^\gamma\prod_{j=1}^m\frac{1}{p_j+1}\right)-\log\left(\prod_{j=1}^m p_j\right)+1}-1}{e^{-W\left(-e^\gamma\prod_{j=1}^m\frac{1}{p_j+1}\right)-\log\left(\prod_{j=1}^m p_j\right)}}\prod_{j=1}^m\dfrac{p_j+1}{p_j}
$$
which I will admit is disgustingly messy, but looks (at least naively to me) potentially tractable since prime product and inverse prime product series are well studied.
One reformulation of the Riemann Hypothesis is Robin's Inequality which states that for $n>5040$ the following holds iff the RH holds:
$$
\sigma(n) < e^\gamma n\log\log(n)
$$
where $\sigma$ is the sum of divisors function and $\gamma$ is the Euler Mascheroni Constant. Now for my specific case I want to show that given given some number $n=p_1^{k_1}p_2^{k_2}\ldots p_m^{m} > 5040$ where $p_j \neq 2$ is a prime number, if Robin's Inequality holds for $n$, then it must also hold for $2^k\cdot n$. Performing some algebra on the inequality we can see that this is the same as showing that if the following inequality holds
$$
\prod_{j=1}^m\dfrac{p_j^{k_j+1}-1}{p_j^{k_j}(p_j-1)} < e^\gamma\log\log\left(\prod_{j=1}^m p_j^{k_j}\right)
$$
then this inequality must hold as well
$$
\dfrac{2^{k+1}-1}{2^{k}}\prod_{j=1}^m\dfrac{p_j^{k_j+1}-1}{p_j^{k_j}(p_j-1)} < e^\gamma\log\log\left(2^k\prod_{j=1}^m p_j^{k_j}\right)
$$
I'll admit that I am not well versed in Analytic Number Theory, so this might be obvious and I have no idea, but so far I have only been able to show three fairly trivial things
According to numerical computations this seems to hold true. For large values of $n$ it appears that the R.H.S. is strictly larger that $2$ times the L.H.S. in the assumed inequality.
Since the left side is bounded with respect to $k$ and the right side is not, there must exist some $N$ for which if $k\geq N$ then the inequality holds. Therefore there are only finite cases for which this inequality may not hold.
Taking the difference of the left and right sides as this
$$
f(k) = e^\gamma\log\log\left(2^k\prod_{j=1}^m p_j^{k_j}\right) - \dfrac{2^{k+1}-1}{2^{k}}\prod_{j=1}^m\dfrac{p_j^{k_j+1}-1}{p_j^{k_j}(p_j-1)}
$$
has a derivative where $f'(0) < 0$ and $f'(N) > 0$, as such there exists a local minima of $f(k)$ which we can find to be the following value
$$
k_{min} = -\dfrac{W_{-1}\left(-e^\gamma\prod_{j=1}^m \frac{p_j-1}{p_j^{k_j+1}-1}\right)+\log\left(\prod_{j=1}^m p_j^{k_j}\right)}{\log2}
$$
where $W_{-1}$ is the second, more negative, solution of the Lambert W function when the argument is between $0$ and $-\frac{1}{e}$. The derivative also appears to be strictly positive past $k_{min}$ as
$$
f'(k) = \dfrac{e^\gamma \log(2)}{k\log 2 + \log\left(\prod_{j=1}^m p_j^{k_j}\right)} - \dfrac{\log(2)}{2^k}\prod_{j=1}^m\dfrac{p_j^{k_j+1}-1}{p_j^{k_j}(p_j-1)}
$$
as this will behave like $\frac{1}{k} - \frac{1}{2^k}$ where the negative part decreases significantly faster than the positive part.
If anyone could offer some potential insight that would be much appreciated!
REPLY [6 votes]: By Theorem 1.2 in this paper, Robin's Inequality is true for every odd integer $n>10$. If we knew what the OP wants to prove, then we would also know Robin's Inequality for every integer $n$ whose odd part exceeds $5040$. In particular, we would know Robin's Inequality for every colossally abundant number exceeding $5040$, because each colossally abundant number divides the second next one (cf. Proposition 4 in this paper). So, by Proposition 1 in Section 3 of Robin's paper, we would know Robin's Inequality for every integer exceeding $5040$, which is equivalent to the Riemann Hypothesis.
In short, it is hopeless to prove what the OP wants to prove, because it implies the Riemann Hypothesis.<|endoftext|>
TITLE: Trivial homology with local system
QUESTION [5 upvotes]: Let $X$ be the classifying space of the Higman group $G$. It is well known that $G$ is an acyclic group
$$H_{\ast}(X;\mathbb{Z})=H_{\ast}(pt;\mathbb{Z}).$$
Now, suppose that $\mathcal{M}$ is a local system on the space $X$ such that
$$H_{i}(X;\mathcal{M})=0, \textrm{ for all $0\leq i$}.$$
Does such local system $\mathcal{M}$ on $X$ exist (other then $\mathcal{M}= 0$)
REPLY [14 votes]: For $X = BG$ local systems on $X$ can be identified with $G$-modules, and homology with the derived tensor product $-\otimes^L_{\mathbb ZG}\mathbb Z$, i.e. $H_i(X;M) \cong \operatorname{Tor}^i_{\mathbb Z G}(M,\mathbb Z)$. One way to see this is to take the definition $H_i(X;M):= H_i(\mathcal S_*(\widetilde X)\otimes_{\mathbb Z\pi_1(X)} M)$, where $\pi_1(X)$ acts on (singular) chains on the universal cover $\widetilde X$ via deck transformations, and replace $\mathcal S_*(\widetilde X)$ with the cellular complex $C_*(\widetilde X)$ of the CW structure of the realization of the nerve of the groupoid $G//G$; then $C_*(\widetilde X)\otimes_{\mathbb Z G} M = \dots\to \mathbb Z[G^2]\otimes M\to \mathbb Z[G]\otimes M\to M$ is the bar complex computing group homology.
Let $G$ be an arbitrary group, and let $M = \operatorname{ker}(\mathbb Z G\to \mathbb Z)$ be the reduced group algebra, so that we have a short exact sequence
$$
0\to M\to \mathbb ZG\to\mathbb Z\to 0
$$
of $\mathbb ZG$-modules. This gives rise to a long exact sequence of Tor-groups, in particular a boundary operator $\partial:\operatorname{Tor}^{i+1}_{\mathbb Z G}(\mathbb Z,\mathbb Z)\to \operatorname{Tor}^i_{\mathbb Z G}(M,\mathbb Z)$. Its kernel is the image of the map $0 = \operatorname{Tor}^{i+1}_{\mathbb Z G}(\mathbb Z G,\mathbb Z)\to \operatorname{Tor}^{i+1}_{\mathbb Z G}(\mathbb Z,\mathbb Z)$, so it is always injective; its cokernel is the kernel of the map $\operatorname{Tor}^{i}_{\mathbb Z G}(\mathbb Z G,\mathbb Z)\to \operatorname{Tor}^{i}_{\mathbb Z G}(\mathbb Z,\mathbb Z)$, which is an isomorphism for $i = 0$ and has the zero group as its codomain for $i > 0$, so it is always injective, so $\partial$ is always surjective and thus always an isomorphism.
In your example of the Higman group, we know that $H_i(X;\mathbb Z)$ is $\mathbb Z$ concentrated in degree $0$, so that $\partial$ is an isomorphism with the zero group, so that $H_i(X;M) = 0$ for all $i \ge 0$.<|endoftext|>
TITLE: Problems in advanced calculus
QUESTION [18 upvotes]: I have been teaching Advanced Calculus at the University of Pittsburgh for many years. The course is intended both for advanced undergraduate students and the first year graduate students who have to pass the Preliminary Exam. This is a difficult course as you can see from the problems that we have on our exam:
http://www.mathematics.pitt.edu/graduate/graduate-handbook/sample-preliminary-exams
What bothers me quite a lot is the lack of a good collection of problems for functions of several variables. There are plenty of excellent collections for functions of one variable and for metric spaces, but there is almost nothing regarding good problems for functions of several variables. The only exception that I know is:
P. N. de Souza, J.-N. Silva,
Berkeley problems in mathematics.
Third edition. Problem Books in Mathematics. Springer-Verlag, New York, 2004.
This is an amazing collection of problems covering many areas of mathematics and what is important the problems have complete solutions.
Question. Do you know a good collection of problems for functions of several variables?
By this I mean a collection of problems that require deep understanding of the problem rather than a standard application of formulas and theorems. I believe that most of the problems in our Preliminary Exam in Analysis fell into this category. Ideally, I would prefer to have a collection with solutions or hints, as it would be very helpful for students (and for me as well).
There are many non published collections of problems available online. I am also interested in links to such collections.
While this might seem as a question that is not research level, I think otherwise. We teach Advanced Calculus to students and if we want them to be ready to do research in Analysis, we need to teach them with such problems.
Edit. I actually knew all the references listed in the answers. Clearly, the answers show that there is no good source of "ready to use" problems in Advanced Calculus of several variables.
REPLY [2 votes]: By this assumptions, I think you can find many interesting problems in the real analysis books that have chapters about several variable calculus. For example, I suggest the book:
Problems and Solutions in Real Analysis, 2nd Edition, by Masayoshi Hata.
There are a lot of good problems with detailed solutions about several variable calculus in this book.
Also, you can see the section of problem and solution of the Mathematical Monthly journals and select the relevant problems.<|endoftext|>
TITLE: Motivation for Karoubi envelope/ idempotent completion
QUESTION [5 upvotes]: This is the second part of my venture to become more comfortable with the concept of idempotent elements and idempotent splittings from category theoretical viewpoint. In the first part we considered the interpretation of idempotent elements & splitting from viewpoint of commutative algebra. The most fruitful analogy (at least for me) was that if we consider the category $\text{$R$-ModFree}$
of free $R$-modules, taking its completion means making it closed under taking direct summands. As the direct summands of free modules are exactly the projective modules completing means "to add some objects" which occur naturally as building blocks.
Now in case of commutative algebra projective objects allows to deal with projective resolutions and provide a framework for direct calculations of derived functors of right exact functors.
I read that there are a lot of constructions spreaded in a relatively wide areas of mathematics where one starts with a certain category $C$, construct from this one another say $F(C)$, and then pass to its idempotent completion $\widehat{F(C)}$.
Probably the most prominent example is the construction of pure motives where we start with category $(\operatorname{Sm}/k)$ of smooth varieties over a field $k$, then pass to category of correspondences $\operatorname{Cor}_k$, build its idempotent completion $\widehat{(\operatorname{Cor}_k)} $ and go ahead with the construction to build the category of Motives $\operatorname{Mot}_k$ and then, by trying to mimic the procedure of building the derived category, we arrive at the category of pure motives (of course that's just a very coarse overview).
The point of my interest is the necessity of taking idempotent completion in the intermediate step.
Of course, that's just an example, but similar strategies occur for example in $K$-theory when one study vector bundles or in constructions dealing with triangulated categories.
My Question: Can there be extracted a common motivation in these examples making the step that takes idempotent completion necessary or does it in every construction almost everywhere strongly depend on "what one wants"?
The only one "general mantra" that I found up to now having the $\text{$R$-Mod}$ example in mind was the necessity of projective objects in order to study right exact functors.
Question: Is this the only motivation or are there some other common deep reasons for the importance of taking idempotent completions?
REPLY [3 votes]: My understanding of the use of Karoubian completion for motives is that one would really like to have an abelian category of pure motives (modulo homological equivalence, say). However, we don't know how to adjoin all kernels and cokernels, and the Karoubian completion is the best we can do.
There is a hope for an abelian category of pure motives that has all the nice properties we want. There are many flavours of motives around (Chow, André, Nori, Voevodsky, ...), and each of them satisfies some but not all of the desired properties. You use whichever one is most convenient for your problem.
(As Mikhail Bondarko pointed out: Chow motives modulo numerical equivalence¹ are semisimple abelian, and this is basically the only way we know how to prove Chow motives form an abelian category. However, this result of Jannsen was only proven in 1992, so I don't think it was the original motivation.)
¹The problem with Chow motives modulo numerical equivalence is that it does not have a cohomological realisation, unless we prove standard conjecture D.<|endoftext|>
TITLE: Homology of perfect complexes
QUESTION [6 upvotes]: I apologize in advance if this question is basic.
If $P_{\bullet}$ is a perfect complex over say a ring $R$ such that
$H_{i}(P_{\bullet})=0 $ if $i\neq n$
$H_{i}(P_{\bullet})=E$ if $i=n$
is $E$ a finitely generated $R$-module ?
What can we say about the homology of a generic perfect complex in general?
REPLY [2 votes]: It is even finitely presented
See Lemma 14.1.27 of the book Derived Categories (also available at the arXiv at https://arxiv.org/abs/1610.09640).<|endoftext|>
TITLE: Sum of squares and partitions
QUESTION [7 upvotes]: This is an off-shot from my previous post on MO.
Given an integer partition $\lambda=(\lambda_1,\dots,\lambda_{\ell(\lambda)})$ of $n$, denote $\ell(\lambda)$ to be the length of $\lambda$.
Let $r_2(n)$ denote the number of ways of expressing $n$ as a sum of two squares of integers, look it up on OEIS.
Here is perhaps a "new formulation" of $r_2(n)$:
QUESTION. Is this true? If it is, please either provide a reference or a proof.
$$r_2(n)
=\sum_{\lambda\vdash n}(-1)^{n-\lambda_1}{\prod_{j=1}^{\ell(\lambda)}} \,4\,(\lambda_j-\lambda_{j+1})$$
where $\lambda_{\ell(\lambda)+1}=0$ and the product excludes $\lambda_j=\lambda_{j+1}$.
Example. Take $n=4$. The solutions to $4=x^2+y^2$ are $(\pm2,0), (0,\pm2)$ and hence $r_2(4)=4$. On the other hand, $\lambda=(4,0), (3,1,0), (2,2,0), (2,1,1,0), (1,1,1,1,0)\vdash 4$ so that
$$(-1)^{4-4}4\cdot4+(-1)^{4-3}4\cdot2\cdot4\cdot1+(-1)^{4-2}4\cdot2
+(-1)^{4-2}4\cdot1\cdot4\cdot1+(-1)^{4-1}4\cdot1=4.$$
REPLY [15 votes]: Start by checking that the following formal product can be expanded as a sum over partitions
$$\prod_{i\geq 1}\left(1+\sum_{r\geq 1}a_r(x_1x_2\cdots x_i)^r\right)=\sum_{\lambda}\left(\prod_{j\geq 1}a_{\lambda_j-\lambda_{j+1}}\right)\left(\prod_{j\geq 1}x_j^{\lambda_j}\right)$$
with the convention that $a_0=1$. The proof of this is really just a weighted version of the usual generating function for partitions.
If we set $x_1=t$ and $x_i=-t$ for $i\geq 2$, and all $a_r=4r$ for $r\geq 1$ then the right side becomes
$$\sum_{n\geq 0} t^n\sum_{\lambda\vdash n}(-1)^{n-\lambda_1}{\prod_{j=1}^{\ell(\lambda)}} \,4\,(\lambda_j-\lambda_{j+1})$$
and the identity says that this is equal to the product
$$\prod_{i\geq 1} \left(1+\frac{4(-1)^{i-1}t^i}{(1-(-1)^{i-1}t^i)^2}\right)=\frac{(1+t)^2}{(1-t)^2}\cdot \frac{(1-t^2)^2}{(1+t^2)^2}\cdot \frac{(1+t^3)^2}{(1-t^3)^2}\cdots$$
$$=\prod_{k\geq 1}\frac{(1-t^{2k})^{10}}{(1-t^k)^4(1-t^{4k})^4}.$$
This last final expression is a well known product formula for $r_2(n)$:
$$\sum_nr_2(n)t^n=\frac{\eta(t^2)^{10}}{\eta(t)^4\,\eta(t^4)^4}, \qquad
\text{where $\eta(t)=t^{\frac1{24}}\prod_{k=1}^{\infty}(1-t^k)$ is the Dedekind eta function}.$$
So your identity follows.<|endoftext|>
TITLE: Is there such a thing as a weighted Kan extension?
QUESTION [6 upvotes]: The title pretty much sums it up.
More in detail. Let $C$, $D$ and $E$ be categories, let $F:C\to D$ and $G:C\to E$ be functors, and let $P:C^{op}\to \mathrm{Set}$ be a presheaf. The colimit of $F$ in $D$ satisfies
$$
D(\mathrm{colim} \,F, d) \cong [C,D](F, d)
$$
for each object $d$ of $D$, where in the right-hand side $d$ denotes the constant functor, and $[C,D]$ the functor category.
This can be seen as a special case of a Kan extension, which satisfies
$$
[E,D](\mathrm{Lan}_G F, K) \cong [C,D](F-,K\circ G-)
$$
for each functor $K:E\to D$. Namely, by setting $E$ the terminal category we get exactly a colimit.
Just as well, a colimit is a special case of a weighted colimit, which satisfies
$$
D(\mathrm{colim}_W \,C, d) \cong [C^{op}, \mathrm{Set}](W-, D(F-, d))
$$
for each object $d$ of $D$. We get an ordinary colimit by setting $W$ to be the constant presheaf at the singleton.
Now, is there a common generalization?
Note that
In the Kan extension, the "dependent variable" of $F$ is paired to $K\circ G$, while in the weighted colimit, it is paired to $W$. So it's unclear how to fit both dependencies together.
One can express Kan extensions as particular weighted colimits - this is not what I'm asking.
(I could ask the same question for the enriched case.)
Any reference would also be welcome.
REPLY [6 votes]: Yes. Given $F:C\to D$ and a profunctor $H:E$ ⇸ $C$, i.e. a functor $H : C^{\rm op}\times E\to \rm Set$ (or to the enriching category $V$), the $H$-weighted colimit of $F$ is the functor $L : E \to D$ such that each value $L(e)$ is the $W(-,e)$-weighted colimit of $F$ (in a coherent way).
Of course, if $E$ is the unit category this reduces to an ordinary weighted colimit.
On the other hand, if $G:C\to E$ and $H(c,e) = E(G(c),e)$ is the corresponding representable profunctor, this reduces to a (pointwise) Kan extension.
There are real advantages of viewing weighted colimits and Kan extensions in this profunctory light. In particular, this is the natural definition of "weighted (co)limit" that makes sense in the abstract generality of a proarrow equipment or a Yoneda structure. In this paper I found it very useful to obtain a good notion of (co)limit in a new kind of category. It also has good formal properties for relating limits and colimits; see for instance Prop. 8.5 of ibid.<|endoftext|>
TITLE: Understanding fundamental group of Poincare homology sphere
QUESTION [7 upvotes]: I'm currently reading Knots, Links, Braids, and 3-Manifolds by V. V. Prasolov and A. B. Sossinsky. I have trouble understanding the following picture. The dashed line denotes a trefoil whose tubular neighborhood is to be cut out, and the thickened line denotes a longitude with frame number 1 around the dashed trefoil. Therefore, after pasting back the solid torus, the meridian would be pasted onto the thickened line therefore bounding a disk in the new 3-manifold. Hence we would have a nontrivial relation in the fundamental group. What I don't understand is the specific order of elements depicted in the circle on the right.
Any help is greatly appreciated. Thank you very much.
REPLY [8 votes]: It turns out that the order is actually not that important: choose a vertex on the circle to start from and a direction to travel in, and then read letters. If the orientation of the edge disagrees with your direction of travel, invert the letter (so save yourself some trouble by traveling counter-clockwise).
If you and I happen to make identical choices except for a choice of starting vertex, then the words we write down will differ by conjugation (in the free group on the set of generators). If additionally we disagreed about the direction of travel, then one of us will have to invert our word. However, the relations we each come up with will be true in both presentations, because they only differ by conjugation and inversion in the free group.<|endoftext|>
TITLE: Sum of degree differences for simple graphs
QUESTION [8 upvotes]: For a simple graph $G$ on $n$ vertices, let us define
$$\mathcal{I}_{n}(G)=\sum_{i,j=1}^{n}|\deg\ x_{i}-\deg\ x_{j}|^{3}.$$
I know that there are many different topological indices defined and studied for graphs. Have You ever seen such that was defined similar as above? Can You provide any references?
I am highly interested in finding $\sup \mathcal{I}_{n}$ over all graphs with $n$ vertices (or at least some tight upper bound). What I have tried myself, was noticing that $\mathcal{I}_{n}$ must be maximized by a threshold graph - these graphs produce degree sequences that are extreme points of he convex hull of all degree sequences. But this didn't lead me too far. I will be glad for any insight.
REPLY [5 votes]: I will guess that the optimum occurs for $k$ isolated vertices and a complete graph on the other $n-k$ where $k=\lfloor\frac{n+1}5\rfloor.$ The same count occurs for $k$ vertices of degree $n-1$ and no other edges so the other $n-k$ have degree $k.$
Past that I have these observations:
A graph $G$ and the complement $\bar G$ give the same value to the sum.
If the maximum degree in an optimal $G$ is $\Delta$ then any degree $\Delta$ vertex is connected to any other. This is because connecting two such increases some of the $|\deg(x_i)-\deg(x_j)|$ but decreases none.
Similarly two vertices with the minimum degree are non-adjacent.
For the type of graph I defined above, the count is $k(n-k)(n-k-1)^3.$ The maximum over the reals occurs at $$k=\frac{3\,n-\sqrt {4\,{n}^{2}-n+1}-1}5\approx \frac{n}{5}-\frac3{20}.$$
As commented, the exponent of $3$ is relevant. Take the conjectured optimal case of a $K_{4t}$ and $t$ isolated vertices. Deleting one edge reduces $2t$ degree difference from $4t-1$ to $4t-2$ and increases $2(4t-2)$ other differences from $0$ to $1.$ If one is summing the square or cubes of the differences that is worse. But with exponent $1$ that is an improvement.
NOTE Based on limited calculations, The same things seem maximal if we replace the exponent of 3 by 2<|endoftext|>
TITLE: Inductive definition of Bernstein polynomials
QUESTION [8 upvotes]: For $n\in \mathbb{N}$ let $B_n$ be the linear operator taking a function $f$ on the unit interval $I=[0,1]$ to its $n$-th Bernstein polynomial $B_nf$,
$$ B_nf(x):=\sum_{k=0}^n\binom{n}{k} f\Big(\frac{k}{n}\Big)x^k(1-x)^{n-k}\label{1}\tag{1}$$
The polynomial $B_nf(x)$ has a natural probabilistic interpretation, namely, it is the expected value of $f(\xi)$, where $\xi=\frac{1}{n}\sum_{j=1}^n \omega_j$ is the average of $n$ independent random variables with identical Bernoulli distribution of parameter $x$, that is, $\mathbb{P}(\omega_j=1)=x$. In fact, this is the starting point in the beautiful Bernstein's proof of the Weierstrass' density theorem via the WLLN. However, this question is about an alternative definition of the sequence $(B_n)_{n\ge0}$.
Let $D:C^1(I)\to C^0(I)$ be the derivative operator, and for all $n\ge1$, let $D_n:C^0(I)\to C^0(I)$ be the approximate discrete derivative given by the incremental ratio
$$D_nf(x):=\frac{f\big( \frac{n-1}{n} x+\frac{1}{n}\big)-f\big( \frac{n-1}{n} x\big)}{\frac{1}{n}}, $$
(which is well-defined for $f\in C^0(I)$ and $x\in I$).
It is easy to check that definition \eqref{1} implies
$$DB_n=B_{n-1}D_n\label{2}\tag{2}$$
together with:
$$B_0f(x)=B_nf(0)=f(0)\label{3}\tag{3}$$
Conversely these two imply formula \eqref{1}, as it follows immediately by induction, at least, if we already have it (quite a common situation of formulas proven by induction). Thus, since \eqref{2} and \eqref{3} characterize $(B_n)_n$, we may even take them as an inductive definition of $(B_n)_n$. Note that replacing $D_n$ with $D$ in \eqref{2} gives the analogous inductive definition for the Taylor polynomials in $0$. (Incidentally, formula \eqref{2} is relevant in the approximation theory, in that it implies that for $f\in C^k(I)$ one has $B_nf\to f$ in $C^k$: this by induction from the case $k=0$, since $D_n$ converges strongly to $D$. Also, it says that if some derivative $f^{(k)}$ is non-negative on $I$, so is $(B_nf)^{(k)}$.)
Question: How can we deduce naturally formula \eqref{1} (i.e., assuming we don't know it, and we do not have a crystal ball to guess it) from \eqref{2} and \eqref{3}?
REPLY [4 votes]: A comment on Josif Pinelis' formula $(b)$ for $\Delta_{n-k+1} \dots\Delta_{n-1}\Delta_{n}$, which is a main point of the computation. Let $\{\tau_{a}\}_{a\in\mathbb{R}}$ and $\{\delta_{b}\}_{a\in\mathbb{R}_+}$ denote respectively the linear group of translations on functions (that we may think defined on the whole real line w.l.o.g.), $f(\cdot)\mapsto f(\cdot+a)$, and the linear group of dilations, $f(\cdot)\mapsto f(\cdot b)$. So $$\tau_{a+b}=\tau_a\tau_b,$$ $$\delta_{ab}=\delta_a\delta_b,$$ $$\tau_{ab}=\delta_b^{-1}\tau_a\delta_b$$
Since $\Delta_n:=\delta_{\frac{n-1}{n}}\big(\tau_{\frac{1}{n}}-\mathbb{1}\big)$, moving all dilations on the left by the above relations imply nicely
$$\Delta_{n-k+1} \dots\Delta_{n-1}\Delta_{n}=\delta_{\frac{n-k}{n}}\big(\tau_{\frac{1}{n}}-\mathbb{1}\big)^{k},$$
whence
$$\frac{1}{k!} D^kB_n=\frac{1}{k!}B_{n-k} D _{n-k+1} \dots D _{n-1} D _{n}=\Big({n\atop k}\Big)B_{n-k}\delta_{\frac{n-k}{n}}\big(\tau_{\frac{1}{n}}-\mathbb{1}\big)^{k},$$
which we can expand to formula $(b)$.
edit. In fact we may skip the last expansion too, keeping all Josif's formulas on the level of operators. Since the $D_k$'s lower the degree of polynomials, $(2)$ and $(3)$ imply that $B_n$ takes values on polynomials of degree less than or equal to $n$, as said. So, for any $x$, denoting $e_x$ the evaluation form,
$$ e_xB_n=e_0\bigg[\sum_{k=0}^n \frac{x^k}{k!}D^kB_n\bigg]=e_0\bigg[\sum_{k=0}^n x^k \Big({n\atop k}\Big)B_{n-k}\delta_{\frac{n-k}{n}}\big(\tau_{\frac{1}{n}}-\mathbb{1}\big)^{k}\bigg]=$$
$$=e_0\bigg[\sum_{k=0}^n \Big({n\atop k}\Big)x^k\big(\tau_{\frac{1}{n}}-\mathbb{1}\big)^{k}\bigg]=e_0\bigg( \mathbb{1} + x \big(\tau_{\frac{1}{n}}-\mathbb{1}\big) \bigg)^n =e_0\bigg( x \tau_{\frac{1}{n}} + (1-x)\mathbb{1} \bigg)^n$$
$$=e_0\bigg(\sum_{k=0}^n \Big({n\atop k}\Big)x^k(1-x)^{n-k}\tau_{\frac{k}{n}} \bigg) $$
which indeed takes $f$ to the original $(B_nf)(x)$ given by $(1)$.<|endoftext|>
TITLE: Example of closed 4 manifold with $\mathbb{S}^1$ action with 1 fixed point and free away from it
QUESTION [5 upvotes]: I am looking for a smooth closed 4-manifold $M$ with a distinguished point $x\in M$, endowed with an $\mathbb{S}^1$ action such that the stabilizer of $p\in M\setminus\{x\}$ is trivial and $x$ is fixed.
A naive attempt:
If we consider the action given by $\mathbb{S}^1$ on $\mathbb{S}^3\subset \mathbb{C}^2$ that gives the Hopf fibration we can extend this action to the 4-ball (it is an unitary action), and it will have the origin as single fixed point. Unfortunately this manifold is not closed.
REPLY [15 votes]: Such a closed $4$-manifold does not exist, and this follows from:
Church, P., & Lamotke, K. (1974). Almost free actions on manifolds. Bulletin of the Australian Mathematical Society, 10(2), 177-196
Let me present the argument anyway. The answer breaks down into a local and global part. The local question is to understand what happens near the single fixed point of the $S^1$-action. The global question is about whether we can find, for each local model near the fixed point, a $4$-manifold which closes off the boundary of the model.
Claim 1: The only local model near the fixed point is the "naive attempt" from your question statement, i.e. the standard $S^1$-action on $\mathbb{C}^2$.
Proof of Claim 1: Up to coordinate change, we may assume the $S^1$-action acts by orthogonal transformations on $\mathbb{R}^4$. In particular, it preserves and is free on $S^3$, and the only free $S^1$-action on $S^3$ is the Hopf (or anti-Hopf if we invert one coordinate) fibration. So your naive attempt really is the only local model.
Claim 2: There is no closed $4$-manifold with the property you ask for.
Proof of Claim 2: Suppose there were such a closed $4$-manifold. Removing a ball around $p$ corresponding to the local model, $\widetilde{M} := M \setminus B_p$. Then we obtain a fibration
$$S^1 \rightarrow \widetilde{M} \rightarrow X,$$
such that on the boundary the fibration is just the Hopf fibration $\partial\widetilde{M} = S^3$ over $\partial{X} = S^2$. The fibration over $X$ comes with its classifying map $X \rightarrow BS^1$, and the composition
$$S^2 = \partial X \hookrightarrow X \rightarrow BS^1$$
classifies the Hopf fibration. But the image of $S^2$ under the classifying map for the Hopf fibration $S^2 \rightarrow BS^1$ (which in more down-to-earth language is just $\mathbb{C}\mathbb{P}^1 \hookrightarrow \mathbb{C}{P}^{\infty}$) represents a nontrivial homology class, while $X \rightarrow BS^1$ is a nullhomology of this class. So we arrive at a contradiction.
If you prefer characteristic classes, it suffices to consider the first Chern class in this argument. In addition, we see that if we have a $4$-manifold with an "almost free" $S^1$-action, then the number of fixed points is even, since each fixed point either adds or subtracts $1$ from the first Chern class, depending upon orientations. Conversely, this argument can be boosted to prove you can find a closed $4$-manifold with any even number of fixed points.
EDIT (responding to comment)
You can ask this question for other group actions, and again, there’s the local and global parts to consider. Let me do this for the case of $\mathbb{Z}_p$ (the cyclic group, not the $p$-adics for anybody who might be confused) which was asked in a comment. We claim that in this case, again, there is no $4$-manifold $M$ with a $\mathbb{Z}_p$ action which is free except for a single fixed point. (Hopefully I haven't made a mistake, which is entirely possible, so feel free to be skeptical. I'm sure an algebraic topologist on this site has a better argument for the last part.)
Again, we start with the local models, which arise from the representation theory of $\mathbb{Z}_p$. For any finite cyclic group, the irreducible real representations are either 1-dimensional (act by the +1 or -1, the latter if p is even) or 2-dimensional (a rotation of order $p$). Since you want every point to be free except the fixed point, you can't use the 1-dimensional representations, since they have order 1 and 2 respectively, so you have a direct sum of two rotations by some 2$\pi k_1/p$ and $2\pi k_2/p$ where $k_1$ and $k_2$ are coprime to $p$. Up to $\mathrm{Aut}(\mathbb{Z}_p)$, we may assume $k_1 = 1$, and we will simply write $k_2 = k$. At the boundary $S^3$ of this local model, we obtain the fibration
$$\mathbb{Z}_p \rightarrow S^3 \rightarrow L(p;k)$$
where $L(p;k)$ is a Lens space. Now for the global part. Assume there exists such a $4$-manifold $M$. Then we have the classifying map for the bundle $S^3 = \partial \widetilde{M} \rightarrow \partial X = L(p;k)$ factors through the classifying map for $X$ (which is now itself a $4$-manifold):
$$\phi \colon L(p;k) = \partial X \hookrightarrow X \rightarrow B\mathbb{Z}_p = K(\mathbb{Z}_p,1).$$
The cohomology ring $H^*(K(\mathbb{Z}_p,1);\mathbb{Z}_p)$ can be completely understood in terms of Steenrod squares and the Bockstein homomorphism, and in particular, we find that for $u \in H^1(K(\mathbb{Z}_p,1);\mathbb{Z}_p)$ the fundamental class, the class
$$v:= u \smile \beta(u) \in H^3(K(\mathbb{Z}_p;1)\mathbb{Z}_p)$$
is a nontrivial element (in fact a generator). Then one can check (e.g. from the cohomology ring structure of $L(p;k)$) that $\phi^*v \neq 0$ as well. Dually, the pushforward of the mod $p$ fundamental class represents a nontrivial element
$$\phi_*[L(p;k)] \neq 0 \in H_3(K(\mathbb{Z}_p,1);\mathbb{Z}_p),$$
and so $\phi$ cannot factor through a $4$-manifold $X$. So no such $4$-manifold $M$ exists.<|endoftext|>
TITLE: Cofinality for coends?
QUESTION [13 upvotes]: Recall that a functor $I \xrightarrow u J$ is cofinal if it has the property that for any functor $J \xrightarrow F C$, we have that $\varinjlim F \cong \varinjlim Fu$ via the canonical map, either side of the equation existing if the other does. This notion is very useful: it admits various reformulations which can be checked directly; there are numerous practical examples, and once a functor is known to be cofinal, the property which I've just treated as a definition becomes a great tool for computing colimits.
When passing from colimits to coends, it would be nice to have similarly powerful tools available. Coends can be reduced to colimits -- e.g. we have $\int^{j \in J} F(j,j) = \varinjlim(Tw(J) \to J^{op} \times J \xrightarrow F C)$, where $Tw(J)$ is the twisted arrow category. But from examples I've tried in the past, my impression is that it's relatively uncommon for a functor $I \xrightarrow u J$ to induce a cofinal functor $Tw(I) \xrightarrow{Tw(u)} Tw(J)$.
Question 1: What are some examples of functors $I \xrightarrow u J$ which induce cofinal functors $Tw(I) \xrightarrow {Tw(u)} Tw(J)$? Are they really quite rare?
Even if I'm right in thinking that such functors are rare, we shouldn't be deterred. After all, a coend is not the colimit of an arbitrary functor out of $Tw(J)$ -- but rather one which factors through $J^{op} \times J$. So there may still be functors $I \xrightarrow u J$ out there which always induce equivalences of coends, even if $Tw(I) \xrightarrow{Tw(u)} Tw(J)$ is not cofinal.
Question 2: What are some examples of functors $I \xrightarrow u J$ such that for any $J^{op} \times J \xrightarrow F C$, we have $\int^{j \in J} F(j,j) \cong \int^{i \in I} F(ui,ui)$ (via the canonical map), either side existing if the other does?
Finally, a more systematic question about these functors:
Question 3: Is it possible to give a direct combinatorial characterization of functors $u$ of the form described in Question 1 or Question 2 (analogous to the usual characterization of cofinal functors via connectedndess of slice categories)?
I'm also interested in versions of these questions other settings like enriched category theory or $\infty$-category theory.
Obviously, everything should have a dual story about limits and ends, too.
REPLY [8 votes]: Offline, Alex Campbell independently suggested a similar approach to the one Roald mentions in the comments, and worked it out. Here are the results -- we work with ends rather than coends for simplicity:
We observe that if $I^{op} \times I \xrightarrow F C$ is a functor, then the end $\int_{i \in I} F(i,i)$ is precisely the limit $\varprojlim_{Hom_I} F$ of $F$ weighted by the Hom-functor $Hom_I: I^{op} \times I \to Set$. Thus, we can apply the weighted version of initiality (the "limit" version of cofinality -- the more modern thing seems to be to say "final" for what I called "cofinal" above), which says in general that
Initiality for Weighted Limits: Let $I \xrightarrow u J$ be a functor, let $\phi: I \to Set$ and $\psi: J \to Set$ be functors (which we regard as "weights" for weighted limits), and let $\eta: \phi u \Rightarrow \psi$ be a natural transformation. Then the following are equivalent:
For any $C$ and any functor $J \xrightarrow F C$, we have $\varprojlim_\psi F \cong \varprojlim_\phi F u$ via the canonical map induced by $\eta$, either side existing if the other does.
$\eta$ exhibits $\psi$ as the Left Kan extension $\psi = Lan_u \phi$ of $\phi$ along $u$.
In particular, we can apply this in the case where $\phi = Hom_I$, $\psi = Hom_J$, and $\eta$ is given by the action of the functor $u$. The left Kan extension can be computed explicitly via a coend formula, and the result is the following:
Proposition (Initiality for Ends): Let $I \xrightarrow u J$ be a functor. The following are equivalent:
For every functor $F: J^{\mathrm{op}} \times J \to C$, we have $\int_{j \in J} F(j,j) \cong \int_{i \in I} F(ui,ui)$ via the canonical map, either side existing if the other does.
For every $j,j' \in J$, the canonical map $\int^{i \in I}Hom_J(j,ui) \times Hom_J(ui,j') \to Hom_J(j,j')$ is an isomorphism.
There are various ways to reformulate (2). For instance,
The composite of profunctors $Hom_J(1,u) \circ_I Hom_J(u,1)$ is canonically isomorphic to $Hom_J$.
For every $j\xrightarrow \beta j' \in J$, the "category of $u$-factorizations" of $\beta$ -- whose objects consist of triples $i \in I, j \xrightarrow \alpha ui \xrightarrow {\alpha'} j'$ composing to $\beta$ (morphisms are the obvious thing) -- is connected.
[ABSV] For any $C$, the functor $Fun(u,C): Fun(J,C) \to Fun(I,C)$, given by precomposition with $u$, is fully faithful.
[ABSV again] The functor $u$ is absolutely dense, i.e. for any $j \in J$ we have $j = \varinjlim (u / j \to J)$ and the colimit is absolute.
In the ABSV paper linked to above, such functors are called "lax epimorphisms" in light of (5) above (the idea being that a "pseudo-epimorphism" is a functor $u$ such that $F(u,C)$ is always a pseudo-monomorphism, which has something to do with the core of the categories involved, but here we take into account non-invertible 2-cells of $Cat$). In light of (5) above, one might also say "co-fully-faithful" or something like that.
Any localization is an example of such a functor. So is any composite or transfinite composite of localizations. The transfinite composites of localizations form the left half of a factorization system on $Cat$ whose right half is the conservative functors, and it's not hard to see that if $u$ is co-fully-faithful, in the factorization $u = wv$ with $v$ being a transfinite composite of localizations and $w$ being conservative, both $v$ and $w$ are co-fully-faithful. Thus when we look beyond localizations, it seems the appropriate thing to ask is "which conservative functors are co-fully-faithful?". For example, I think the functor from a category to its idempotent completion is co-fully-faithful (while also being fully faithful and in particular conservative). I think that's about all there is to say about co-fully-faithful functors which are also fully-faithful -- any such functor induces an equivalence of idempotent completions (one way to see this is to use the absolute density condition above with the Yoneda embedding). But of course, there may be quite a lot of daylight between co-fully-faithful functors which are conservative and those which are fully faithful.
The co-fully-faithful functors also seem related to the "liberal" functors (functors $u$ such that $Fun(u,C)$ is always conservative) of CJSV: co-fully-faithful implies liberal but not conversely.
Of course, the enriched and $\infty$-categorical counterparts of all of this should be clear at a conceptual level, at any rate.
Note also that everything is self-dual: a functor $u$ is co-fully-faithful iff $u^{op}$ is, so the questions about ends and coends are actually equivalent (and not just dual).<|endoftext|>
TITLE: Necessary conditions for the existence of solution of Sylvester equation AX=XB
QUESTION [13 upvotes]: Let's consider square matrices $A_{n \times n}$, $B_{n \times n}$ and $X_{n \times n}$ with elements from $\mathbb{R}$. Could you tell me please, what would be the necessary conditions for the existence of solution (may be not unique) of Sylvester equation:
$$
AX=XB.
$$
As I know, sufficient condition looks like (but probably it is a necessary and sufficient condition)
$$
\sigma_p(A) \cap \sigma_p(B) \neq \varnothing,
$$
here $\sigma_p(A)$ and $\sigma_p(B)$ are the spectra of matrices $A$ and $B$.
REPLY [32 votes]: This equation always has a solution: $X = O$. I'll assume throughout this answer that you're interested in a non-zero solution.
The equation $AX = XB$ is equivalent to $(A \otimes I - I \otimes B^T)\mathbf{x} = \mathbf{0}$, where $\otimes$ denotes the Kronecker product and $\mathbf{x}$ is the vectorization of $X$. Your question is thus equivalent to asking when the matrix $A \otimes I - I \otimes B^T$ is not invertible (i.e., when $0$ is not an eigenvalue of $A \otimes I - I \otimes B^T$).
Since the eigenvalues of $A \otimes I - I \otimes B^T$ are exactly the sums of the eigenvalues of $A$ and $-B$, the condition that you wrote ($\sigma_p(A) \cap \sigma_p(B) \neq \varnothing$) is in fact both necessary and sufficient.<|endoftext|>
TITLE: Connectedness, loops and formal moduli problems
QUESTION [10 upvotes]: Let $k$ be an algebraically closed field of characteristic zero. Formalizing a classical folk concept, Pridham and (in a different way,) Lurie defined a formal moduli problem (over $k$) to be a functor from local Artin CDGA's to homotopy types satisfying a certain sheaf condition. If the commutativity condition is weakened to an $E_n$ condition, any formal moduli problem is (uniquely) representable by an $E_n$ algebra; in the commutative case, a formal moduli problem is not necessarily representable by a CDGA, but rather by a (derived) Lie algebra.
There's an intuitive picture I like for this in a special case, and I want to understand how it fits into the general picture. Namely, say that $G$ is an affine algebraic Lie group. Let $BG$ be its classifying stack. Then the deformation problem of maps to $BG$ (relative to a choice of point $*\to BG$) is classified by the Lie group $\mathfrak{g}$.
Of course this picture involves a group rather than a "formal stack", which is my intuition for (the opposite category to) formal moduli problems, and I am curious how hard it is for a formal moduli problem to be representable by a group object.
From the point of view of spaces, going from groups to spaces "is not very hard": the category of groups is equivalent (via the delooping functor) to the category of connected pointed spaces.
On the other hand, for $E_n$ algebras, the category of cogroup objects is equivalent to the category of (suitably defined) $(n,1)$-commutative Hopf algebras, which is more closely related (via Koszul duality) to the category of $E_{n+1}$ algebras than to $E_n$ algebras; in particular, the delooping functor from group objects to $E_n$ algebras is far from fully faithful.
Now by formal nonsense, there is a loop-deloop pair of adjoint functors $$B:GFMP\leftrightarrows FMP:\Omega,$$ where $FMP$ is the category of ($E_\infty$) formal moduli problems and $GFMP$ is the category of cogroup objects in $FMP$. It seems natural to ask the following questions, to which I don't know the answer:
Is $B$ fully faithful, and if so what is its image?
Is there an analogue on the level of (co)group objects in Lie algebras to the Koszul duality functor from $(n,1)$ Hopf algebras to $E_{n+1}$ algebras?
A natural further question, which I suspect is harder, is to ask whether there is an "algebraic point of view" via Lie-like objects for $E_\infty$ objects in $FMP$.
REPLY [10 votes]: The presentation of the formal moduli problems story in Gaitsgory-Rosenblyum A Study in Derived Algebraic Geometry, Vol 2 may be what you are looking for. We review it here (in the case over $\mathrm{Spec}\, k$ for a field $k$ of characteristic zero, that the question concerns):
1. Looping/delooping equivalence in formal DAG
Just like the familiar adjoint equivalence in the homotopy theory of spaces
$$\mathrm B: \mathrm{Grp}_{\mathbb E_1}(\mathcal S)\simeq \mathcal S_*^{\ge 1}:\Omega,$$
there is an analogous adjoint equivalence in formal DAG
$$\mathrm B:\mathrm{Grp}_{\mathbb E_1}(\mathrm{FMP}_k) \simeq \mathrm{FMP}_k:\Omega,$$
where the loop space functor is in both cases given by $\Omega X = \mathrm{pt}\times_X \mathrm{pt}$, as per usual in homotopical/$\infty$-categorical things. Note that formal moduli problems are already inherently pointed, by the assumption that $X(k)$ is contractible.
2. Formal groups and Lie algebras
Now, $\mathrm{Grp}_{\mathbb E_1}(\mathrm{FMP}_k) = \mathrm{FGrp}_k$ is (an incarnation) of the $\infty$-category of (derived) formal groups over $k$. Thanks to the characteristic zero assumption, there is a further equivalence
$$
\mathrm{Lie}:\mathrm{FGrp}_k \simeq \mathrm{LieAlg}_k : \exp
$$
with derived Lie algebras (as modelled for instance by dg Lie algebras). Just like expected, the derived Lie algebra $\mathfrak g$ corresponding to the formal group $G$ is $\mathfrak g = T_{G, e}$, the tangent fiber at the unit.
3. Formal moduli problems and Lie algebras
The celebrated Lurie-Pridham identification between formal moduli problems and derived Lie algebras is precisely the composite of these two equivalences of $\infty$-categories
$$
\mathrm{FMP}_k\simeq \mathrm{Grp}_{\mathbb E_1}(\mathrm{FMP}_k)=\mathrm{FGrp}_k\simeq \mathrm{LieAlg}_k.
$$
That is
It sends a formal moduli problem $X$ to
$$
\mathrm{Lie}(\Omega X) = T_{\Omega X, e} = T_{X, x_0}[-1],
$$
where $x_0$ is the base-point of $X$, unique up to a contractible space of choices $X(\kappa)$. This shifted tangent fiber carries a canonical Lie algebra structure coming from (i.e. as the Lie algebra of) the $\mathbb E_1$-group structure of $\Omega X$.
The inverse equivalence $\Psi: \mathrm{LieAlg}_k \simeq \mathrm{FMP}_k$ is then given by $\Psi(\mathfrak g)= \mathrm B\exp(\mathfrak g)$.
4. Lurie's formula for $\Psi$
You may justifiably complain that this description of the inverse functor $\Psi$ does not look the same as the one in Lurie's writing. Let's see how to get it in that form.
Let's assume that the formal moduli problem $\Psi(\mathfrak g)$ is formally affine (true under some finiteness assumptions on $\mathfrak g$), in the sense that
$$
\mathrm B\exp(\mathfrak g)= (\mathrm{Spf}\,k)/\exp(\mathfrak g)\simeq \mathrm{Spf} \,k^{\mathfrak g}.
$$
Here of course the formal spectrum is the usual functor $\mathrm{Spf}:(\mathrm{CAlg}^\mathrm{aug}_k)^\mathrm{op}\to \mathrm{FMP}_k$. The derived Lie algebra invariants may be computed via the Chevalley-Eilenberg complex, thus $k^\mathfrak g\simeq {C}^*(\mathfrak g)$.
Then for any Artinian $k$-algebra $A$ we have
$$ (\Psi(\mathfrak g))(A) \simeq \mathrm{Map}_{\mathrm{CAlg}^\mathrm{aug}_k}({C}^*(\mathfrak g), A). \qquad \quad(1)
$$
This is why Lurie tells us to consider the "Koszul duality functor" $\mathfrak D: (\mathrm{CAlg}_k^\mathrm{aug})^\mathrm{op}\to \mathrm{LieAlg}_k,$ right-adjoint to the Chevalley-Eilenberg cochains functor. Indeed, (through a little use of the finiteness of $\mathfrak g$) we get
$$(\Psi(\mathfrak g))(A) \simeq \mathrm{Map}_{\mathrm{LieAlg}_k}( \mathfrak D(A), \mathfrak g),\qquad \qquad(2)
$$
which is how Lurie tells us to define $\Psi$.
Note that this is going the other way than the $C^*\dashv\mathfrak D$ adjunction. Indeed, without finiteness assumptions on $\mathfrak g$, the formal moduli problem $\Psi(\mathfrak g)$ will not necessarily be formally affine, and the formula (1) will not necessarily work. On the other hand, as Lurie teaches us, formula (2) will always work.
5. Loose ends
If I understand the original question correctly, this is presenting the formal moduli problem story precisely like the intuitive picture mentioned. Indeed: any formal moduli problem may be written as $X\simeq \mathrm B G$ for a derived formal group $G$, and the formal moduli problem (of mapping into) $\mathrm BG$ is classified by the Lie algebra $\mathfrak g$.
That said, the question makes analogy with the $\mathbb E_n$-algebra version of this story too. I am a little confused about the points raised - in particular, it seems like the following is asserted: a formal moduli problem on $\mathbb E_n$-algebras is represented by an $\mathbb E_n$-algebra. That is, so far as I understand, incorrect. Instead, any FMP on $\mathbb E_n$-algebras in classified by an $\mathbb E_n$-algebra, in a way somewhat analogous to the way that the functor $\mathfrak D$ presents formal moduli problems in the commutative case with Lie algebras.
In particular, there seems to be no need to think about cogroup objects, as the equivalence of $\infty$-categories $\mathrm{FMP}_k^{\mathbb E_n}\simeq \mathrm{Alg}^\mathrm{aug}_{\mathbb E_n}$ is covariant.
$\qquad$
Edit: A question was raised in the comments whether group objects in $\mathrm{FMP}_k^{\mathbb E_n}\simeq \mathrm{Alg}_{\mathbb E_n}^\mathrm{aug}$ correspond to $\mathbb E_{n+1}$-algebras. Unless I am misunderstanding the question, the answer is negative.
A monoid structure (of which a group structure is a particular example of) on an $\mathbb E_n$-algebra $A$ is given by a map $A\times A\to A$, plus coherence data. On the other hand, an additional $\mathbb E_1$-algebra structure on $A$ (which is equivalent to making it into an $\mathbb E_{n+1}$-algebra by Dunn Additivity) is given by a map $A\otimes_k A\to A$, plus coherence data. So a group object in $\mathbb E_n$-algebras, and an $\mathbb E_{n+1}$-algebra are different structures.
This might feel a little weird because we're used to tensor products corresponding to products of schemes. Alas, unlike the contravariant $\mathrm{Spec}$, the equivalence $\Psi:\mathrm{Alg}^{\mathrm{aug}}_{\mathbb E_n}\simeq \mathrm{FMP}_k^{\mathbb E_n}$ is covariant. (PS: another difficulty: the tensor product only becomes the coproduct on the level of $\mathbb E_\infty$-algebras, not $\mathbb E_n$-algebras, so even a contravariant equivalence wouldn't necessarily do the trick).
Rather, the situation is the same as in the $\mathbb E_\infty$-case above: there is an equivalence
$$\mathrm B:\mathrm{Grp}_{\mathbb E_1}(\mathrm{FMP}_k^{\mathbb E_n})\simeq \mathrm{FMP}_k^{\mathbb E_n}:\Omega.$$
This is a special case of the following general phenomenon: for any operad $\mathcal O$, the loops functor $\Omega: \mathrm{Alg}_{\mathcal O}(\mathrm{Mod}_k)\to \mathrm{Grp}_{\mathbb E_1}(\mathrm{Alg}_{\mathcal O}(\mathrm{Mod}_k))$ is an equivalence of $\infty$-categories (A Sudy in Derived Algebraic Geometry, Vol 2, Chapter 6, Proposition 1.6.4). The above claims are special cases for $\mathcal O$ the Lie operad and the $\mathbb E_n$-operad respectively.<|endoftext|>
TITLE: Interpreting proper elementarily equivalent end extensions?
QUESTION [6 upvotes]: Is there a tuple of parameter-free formulas $\Phi$ and a nonstandard $M\models PA$ such that $\Phi^M\models PA$, the induced $M$-definable initial segment embedding $j_\Phi^M:M\rightarrow\Phi^M$ is non-surjective, and $M\equiv \Phi^M$?
(Here by "$\Phi^M\models PA$" I mean "$\Phi$ defines an interpretation of a structure in the language of arithmetic in $M$, and that structure satisfies $PA$." Per Joel's comment below, we may freely require additionally that $j^M_\Phi$ be elementary.)
If we allow parameters the answer is yes, but in both the arguments there parameters are absolutely essential. I vaguely recall$^1$ a not-too-hard negative proof via Kripke's notion of fulfillability (see Putnam or Quinsey) but I can't reconstruct it at the moment.
$^1$I also recalled seeing this about the version with parameters, which turned out to be bogus; I think the parameter-free version is what I was actually thinking of, but now I'm much less sure I'm not just making stuff up.
REPLY [4 votes]: This answer is an attempt at explaining my critical posted comments on Hamkins' proposed answer; it also expands my posted comments to the MO question. I will explain:
(a) the gap in Hamkins' answer, (b) how it can be fixed (at the cost of considerably strengthening the hypotheses of the question), and
(c) relevant history and literature.
(a) [the gap] Suppose $M$ is a model of $PA$ and $\Phi$ consists of pair of arithmetical ternary formulae $\phi_{\rm{add}}(x,y,z)$, $\phi_{\rm{mult}}(x,y,z)$ for how to re-interpret (the graphs of) addition and multiplication such that (1) through (3) below hold:
(1) The model $\Phi^M$ satisfies $PA$, i.e., ($M$, $\phi^{M}_{\rm{add}}(x,y,z)$, $\phi^{M}_{\rm{mult}}(x,y,z)) \models PA$.
(2) $M$ and $\Phi^M$ are elementarily equivalent.
(3) The $M$-induced initial segment embedding $j^M_{\Phi}:M \rightarrow \Phi^M$ is not surjective.
Then as observed by Hamkins in his answer, by replacing $M$ by its elementary submodel $M_0$ of $M$ consisting of definable elements of $M$, the above conditions remain true. So far, so good.
The gap in the proposed proof occurs at the end of its first paragraph, where it is asserted that $j^{M_0}_{\Phi}:M_0 \rightarrow \Phi^M_0$ is an elementary embedding. The reasoning given is: "Since $M_0$ is pointwise definable, however, elementary equivalence will imply full elementarity, since all parameters are definable". However, for $j^{M_0}_{\Phi}$ to be an elementary embedding of $M_0$ into $\Phi^{M_0}$ we need to know that if $a \in M_0$ is definable in $M$ (and also in $M_0$) as the unique $x$ satisfying $\psi(x)$, then $j(a)$ is the unique $x$ satisfying $\psi(x)$ in $\Phi^M_0$, which is not warranted by the mere assumption that $j$ is an initial embedding.
(b) [the fix]. The gap explained above can be readily seen to be fillable if WE ASSUME, FURTHERMORE, THAT $M$ DOES NOT CONTAIN ANY NONSTANDARD DEFINABLE ELEMENTS (equivalently: $M$ is elementarily equivalent to the standard model of arithmetic $\Bbb{N}$, which is often paraphrased as "$M$ is a model of true arithmetic"). The reasoning: initial embeddings are automatically $\Delta_0$-elementary, so this extra assumption guarantees that $ a\in M_0$ is definable in $M$ via "the least $x$ such that $\delta(x)$ holds" for some $\Delta_0$-formula $\delta(x)$, which in turn guarantees that $j(a)$ is the unique $x$ satisfying $\delta(x)$ in $\Phi^M_0$.
(c) [history and literature] A few years before the appearance of Tennenbaum's famous characterization of the standard model of arithmetic as the only model of $PA$ (up to isomorphism) whose addition and multiplication are recursive, Feferman published an abstract in 1958 (in the Notices of AMS) to announce his result that there is no nonstandard model of true arithmetic (i.e., a model elementarily equivalent to the standard model of arithmetic $\Bbb{N}$) whose addition and multiplication are arithmetically definable, i.e., he proved:
Theorem (Feferman, 1958) The standard model of arithmetic $\Bbb{N}$ cannot interpret a nonstandard model of true arithmetic.
An early exposition of the above theorem of Feferman can be found in this 1960 paper of Scott. The theorem is also exposited as Theorem 25.4a in this edition of the textbook Computability and Logic (by Boolos, Burgess, and Jeffrey), but mysteriously with no reference to Feferman.
Many years later, Feferman's theorem was rediscovered and fine-tuned in by K. Ikeda and A. Tsuboi, in their paper Nonstandard models that are definable in models of Peano Arithmetic, Math. Logic Quarterly, 2007. Theorem 1.1 of this paper says the following (in the language used here).
Theorem. (Ikeda and Tsuboi, 2007) Suppose $M$ is an elementary extension of $\Bbb{N}$ (i.e., $M$ is a model of true arithmetic), and suppose that $M$ interprets a model $\Phi^M$ such that $M$ and $\Phi^M$ are elementarily equivalent. Then the $M$-induced initial segment embedding $j^M_{\Phi}:M \rightarrow \Phi^M$ is an isomorphism.
The proof of Ikeda and Tsuboi for Theorem 1.1 is essentially the one obtained by gluing Hamkins' answer (to the MO question) to the fix (b) above.
Ikeda and Tsuboi also pose the MO question and answer discussed here as an open question in their paper in item 2 of Remark 3.6. To my knowledge the problem remains open.
Finally, let me emphasize that the above discussion all pertained to parameter-free interpretations, since it is well-known that if $M$ is a recursively saturated model of $PA$, then there is some $\Phi(x)$ and some parameter $m \in M$, such that $M$ and $\Phi(m)^{M}$ are elementarily equivalent, and the $M$-induced initial segment embedding $j^M_{\Phi(m)}:M \rightarrow \Phi(m)^{M}$ is a nonsurjective embedding. Here is the proof outline: by recursive saturation, Th($M$) is in the standard system of $M$, and coded by some $m \in M$ such that, in the eyes of $M$, $m$ codes a consistent theory. So by the arithmetized completeness theorem, $M$ can build a model of $m$ whose elementary diagram is definable in $M$; the induced initial embedding is not surjective by Tarski's undefinability of truth.<|endoftext|>
TITLE: Multiplying all the elements in a group
QUESTION [27 upvotes]: Let $G = \{ g_i | i = 1, ...,n \}$ be a finite group and denote by $G!$ the multiset consisting of all the products of all different elements of $G$ in any order, that is
$$ G! = [ \prod_i g_{\sigma(i)} | \sigma \in S_n] $$.
I'm interested in knowing how $G!$ behaves as a set, and also how often does every element appear (i.e., how it behaves as a multiset).
In the case of an abelian $G$, $G!$ is either 1 or (iff exists) the single element of order 2 in $G$.
We can use the abelian case to get a (seemingly tight) upper bound on $G!$ as a set, by considering modding by $[G,G]$: projecting every such product to the quotient, which is abelian, we get $a^{\#[G,G]} \in G/[G,G]$ where $a$ is the single element of order 2 in $G/[G,G]$ if it exists, and the identity otherwise. Thus, if either $a$ is the identity or $\#[G,G]$ is even, $G!$ is entirely contained in $[G,G]$, and otherwise contained in the coset corresponding to $a$.
The reason this bound seems tight is that it's generally easy to get elements in the commutator group (up to the element of order 2): if $a,b,a^{-1},b^{-1}$ are distinct, $[a,b] \in G!$ by putting every other element of $G$ right next to it's inverse except for $a^{\pm1}, b^{\pm1}$ and likewise for products of commutators and so on.
Is it always the case that $G!$ is a coset of the commutator group? How often does each element appear?
It might also be useful to look at the action of $Aut(G)$ on $G!$, but I'm not totally sure what can that tell us.
REPLY [41 votes]: Yes, your $G!$ (as a set) is always either $[G,G]$ (if the order of $G$ is odd, or its Sylow $2$-subgroup is non-cyclic) or $z[G,G]$ if $G$ has cyclic Sylow $2$-subgroup, where $z$ is the involution in the Sylow $2$-subgroup. This was apparently a question/conjecture of Golomb (see p. 973) and independently of Fuchs (in a 1964 seminar), proved by Dénes and Hermann.
(Addressing Mark Sapir's comment, I could not find a published reference for Fuchs, but did manage to track down this paper by Dénes and Keedwell which contains a discussion of some of the history of the question.)<|endoftext|>
TITLE: Aleksandrov's proof of the second order differentiability of convex functions
QUESTION [12 upvotes]: Aleksandrov [A], proved a remarkable property of convex functions.
Theorem. If $f:\mathbb{R}^n\to\mathbb{R}$ is convex, then for almost every $x\in\mathbb{R}^n$ there is $Df(x)\in\mathbb{R}^n$ and a symmetric $(n\times n)$ matrix $D^2f(x)$ such that
$$
\lim_{y\to x}
\frac{|f(y)-f(x)-Df(x)(y-x)-\frac{1}{2}(y-x)^TD^2f(x)(y-x)|}{|y-x|^2}=0.
$$
I know two proofs of this result. One based on the theory of maximal monotone functions and one based on the fact that the second order distributional derivatives of a convex function are Radon measure. Both proofs are mentioned in
Second order differentiability of convex functions. Since these proofs use relatively modern techniques not available during Aleksandrov's time, his argument must have been very different.
Question 1. Can you briefly explain what was the idea of the original proof due to Aleksandrov?
My guess would be that his proof was based on methods of differnetial geometry. What else could he use in those days?
Question 2. In there any textbook where I can find the original proof due to Aleksandrov?
[A] A. D. Alexandroff,
Almost everywhere existence of the second differential of a convex function and some properties of convex surfaces connected with it. (Russian)
Leningrad State Univ. Annals [Uchenye Zapiski] Math. Ser. 6, (1939), 3–35.
REPLY [10 votes]: The paper On the second differentiability of convex surfaces by Bianchi, Colesanti, and Pucci (Geometriae Dedicata volume 60, pages 39–48 (1996)) concerns the proof of the Busemann-Feller-Alexandroff Theorem on the second order differentiability of convex functions. Its introduction gives brief synopses on several different methods of proof, including the original argument of Busemann-Feller and later Alexandroff, the two methods you mentioned in the other question (the monotone operator method of Rockafeller (using a result of Mignot); and the measure/distribution method of Reshetnyak), as well as a different one by Bangert (using almost purely differential geometric methods).
The paper gives also a new proof of the theorem, which is claimed to be in the same spirit of the original arguments of Busemann-Feller and Alexandroff. The authors considered the second order difference quotient of the convex function based at a point $x$, which they show has a limit a.e. as a convex function. This new convex function is related to the argument of Busemann-Feller in that the indicatrices constructed by Busemann-Feller are the 1-level-sets of this limited convex function.<|endoftext|>
TITLE: Which free strict $\omega$-categories are also free as weak $(\infty,\infty)$-categories?
QUESTION [7 upvotes]: There are a number of formalisms available for presenting free strict $\omega$-categories -- Street's parity complexes, Steiner's directed complexes, computads, polygraphs,... Typically one has a certain category $\mathcal C$ of combinatorial data, a straightforward notion of "map" from $C \in \mathcal C$ to a strict $\omega$-category $X$, and a free functor $F: \mathcal C \to \omega Cat$ with an explicit combinatorial description of $F$. The free functor will have the universal property that strict $\omega$-functors $F(C) \to X$ are in bijection with "maps" $C \to X$ whenever $X$ is a strict $\omega$-category.
Depending on one's choice of model, there may still be a clear notion of "map" from $C \in \mathcal C$ to a weak $(\infty,\infty)$-category $Y$. I'm interested to know conditions on $C \in \mathcal C$ guaranteeing that "maps" $C \to Y$ are in bijection with morphisms $i(F(C)) \to Y$, where $i: \omega Cat \to (\infty,\infty)Cat$ is the inclusion from strict $\omega$-categories to weak $(\infty,\infty)$-categories. Of course, this may be model-dependent. I find myself needing a result of this form in a particular setting, but I'd be interested seeing results of this kind for any choice of $\mathcal C$ and any model of weak $(\infty,\infty)$-categories -- I'd be particularly happy if the model of weak $(\infty,\infty)$-categories is nonalgebraic in nature.
Question: What is an example of
a category $\mathcal C$ of "presentations of certain strict $\omega$-categories",
a free functor $F: \mathcal C \to \omega Cat$ from $\mathcal C$ to strict $\omega$-categories with an explicit combinatorial description,
a 1-category $(\infty,\infty)Cat$ which "models" the $\infty$-category of weak $(\infty,\infty)$-categories (e.g. via a model structure or whatever) with inclusion functor $i: \omega Cat \to (\infty,\infty)Cat$,
a straightforward notion of "map" from objects $C \in \mathcal C$ to objects of $(\infty,\infty) Cat$,
and a (not completely vacuous) condition $\Phi$ on the objects of $\mathcal C$
such that
Objects $C \in \mathcal C$ satisfying $\Phi$ have the property that $Hom(iF(C),Y)$ is naturally isomorphic to the set of maps from $C$ to $Y$, for all (suitably fibrant, perhaps) $Y \in (\infty,\infty)Cat$?
I'm happy to see quite restrictive conditions $\Phi$; in fact in my case I don't need to understand much more than Gray tensor powers of the arrow category $\bullet \to \bullet$. I suspect that something along the lines of "$F(C)$ is gaunt" or "loop-free" or something may often do the trick, but I'd be happy with something more or less restrictive.
I suppose I'd also be happy to see examples with "$n$" replacing "$\omega$" and "$(\infty,n)$" replacing "$(\infty,\infty)$".
And let me stress that I'm not looking for some kind of fancy $\infty$-categorical freeness -- when I say $Hom(i(F(C)),Y)$ above, I mean $Hom$ in whatever 1-category is being used to model $(\infty,\infty)Cat$. Although if there are results showing something fancier, that would be interesting to hear about too.
REPLY [2 votes]: Important progress on this question has been made by Yuki Maehara, who shows in Orientals as free $\omega$-categories that the complicial nerve of the $n$th oriental is a complicial anodyne extension of the $n$-simplex (providing a fibrant replacement thereof in Verity's model structure on complicial sets). In other words, at least in the complicial set model, the orientals -- which a priori have a strict universal property -- have the expected weak universal property too.<|endoftext|>
TITLE: How far does this restricted definition on $\mathcal{O}$ goes?
QUESTION [5 upvotes]: $\mathcal{O}$ notation describes an onto function $f:\mathcal{O} \rightarrow \omega_{CK}$. In calculating all values $n \in \mathbb{N}$ such that $f(n)=\alpha$, when $\alpha$ is a limit, all indexes $e$ of ordinary programs are considered such that $\phi_e(i)=n_i$ (with $i \in \mathbb{N}$). The values $n_i$ must satisfy the condition that $f(n_i)=\alpha_i$ with the $\alpha_i$'s forming a (fundamental) sequence for $\alpha$.
I am just interested in the variation where we don't consider indexes of normal programs but instead consider indexes of primitive recursive functions/programs (given a suitable indexing for them).
(Q1) How far would such a variation go? That is, what is the smallest ordinal it can't represent.
(Q2) Consider the notation system (described by a 1-1 function $g:\beta \rightarrow \mathbb{N}$) assigning a unique number to each ordinal in which on limit values $\alpha<\beta$ we seek the p.r. function with smallest index $e$ satisfying $\phi_e(i)=g(\alpha_i)$ where the $\alpha_i$'s must form a (fundamental) sequence for $\alpha$.
At what value $\beta$ would such a system stop?
I don't have proficiency with the underlying theory so if the question seems posed in a strange manner then that might be the reason.
REPLY [5 votes]: For Q2, the answer is $\omega^2$, for both recursive and primitive recursive notations. It's not hard to see that every ordinal below $\omega^2$ can be reached.
To show that $\omega^2$ cannot be reached, the argument is the same for both primitive recursive and full recursive. For each $a$, we construct a $b$ ensuring that $a$ is not a notation of this sort for $\omega^2$. We will construct $b$ knowing its own index. For recursive notations, this is an application of the recursion theorem. For p.r. notations, even though we don't have the recursion theorem, this is still possible so long as we're willing to specify an a priori p.r. bound on the runtime. Since we intend to kill time by enumerating successors, an exponential bound will be fine.
Our first goal is to arrange that $b$ codes $\gamma+\omega$, where $\gamma$ is greatest such that some $c < b$ with $c <_\mathcal{O} a$ codes $\gamma$. So we unwrap the notation $a$ while enumerating successors. As soon as we see some $c < b$ with $c <_{\mathcal O} a$ and $|c|$ larger than any of the others yet seen, a c.e. event, we include $c$ in the sequence we're enumerating.
As soon as we see $b <_\mathcal{O} a$, we do something to ensure that $b$ is not a valid notation, e.g. break the monotonicity of our sequence.
If $a$ is a notation for $\omega^2$ with $b \not <_\mathcal{O} a$, then $\gamma+\omega < |a|$, and $b$ is the least notation for $\gamma+\omega$ (for any smaller notation would be included in the definition of $\gamma$, contradicting our choice of $\gamma$). So if $a$ is a notation of the desired sort for $\omega^2$, then $b <_\mathcal{O} a$. But then $b$ is not a notation, by construction, so $a$ is not a notation.<|endoftext|>
TITLE: Finite dimensional algebras over $\mathbb{Q}$
QUESTION [6 upvotes]: It is known that a finite dimensional basic algebra over an algebraically closed field is isomorphic to the path algebra of a finite quiver modulo an admissible ideal.
Question 1: Is the same true for algebras over $\mathbb{Q}$? If not, are there suitable further assumptions that would guarantee this?
Question 2: Are there some general useful tools to compute the dimension of the algebra, given the quiver and the ideal? (In concrete, easy cases it might be doable by hand, but I am looking for methods that are more broadly applicable)
REPLY [7 votes]: Question 1: No, take any finite field extension of $\mathbb{Q}$. It is basic but has a simple module that is not 1-dimensional and thus it is not of the form $KQ/I$ for $I$ admissible (since all simple modules are 1-dimensional for algebras of the form $KQ/I$).
For any field $K$, a basic algebra is isomorphic to a quiver algebra with admissible ideal if and only if the algebra $A$ is split (also called elementary sometimes), meaning that $A/J$ is isomorphic to matrix algebras over $K$. You can find this result in most representation theory books (of finite dimensional algebras) such as in the book of Auslander,Reiten and Smalo or the book by Kirichenko and Drozd.
Question 2: The best (computer) tool is the GAP-package qpa: https://folk.ntnu.no/oyvinso/QPA/ . Given a quiver and admissible ideal, you can calculate many information for the algebra, including the dimension.<|endoftext|>
TITLE: Understanding the adjunction $\mathfrak{C}:\mathbf{Set}^{\Delta^{op}}\rightleftharpoons \mathbf{Cat}_\Delta:\mathcal{N}$
QUESTION [6 upvotes]: Let $\mathbf{Cat}_\Delta$ denote the category of simplicially enriched categories (ie simplicial objects in $\mathbf{Cat}$ with all face and degeneracy maps bijective on objects). There is a canonical functor $\mathfrak{C}:\Delta\rightarrow\mathbf{Cat_\Delta}$ that takes $[n]$ to the simplicial category $\mathfrak{C}[n]$ defined as follows:
For $i, j\in[n]$, let $P_{ij}$ be the poset with objects $J\subseteq[n]$ containing $i$ as a least element and $j$ as a greatest element. (In particular $P_{ij}$ is empty if $i>j$.) Then $\mathfrak{C}[n]$ has objects the set $[n]$, and mapping spaces $\text{Map}_{\mathfrak{C}[n]}(i, j)$ given by the nerve of $P_{ij}$. The morphisms
$\text{Map}_{\mathfrak{C}[n]}(i_0, i_1)\times \text{Map}_{\mathfrak{C}[n]}(i_1, i_2)\times \cdots \times \text{Map}_{\mathfrak{C}[n]}(i_{k-1}, i_k)\rightarrow \text{Map}_{\mathfrak{C}[n]}(i_0, i_k)$
are induced by the poset maps $P_{i_0 i_1}\times\cdots P_{i_{k-1} i_k}\rightarrow P_{i_0 i_k}$ given by $(J_0, \cdots, J_k)\mapsto J_0\cup\cdots\cup J_k$.
Finally, morphisms in $\Delta$ are mapped to morphisms in $\mathbf{Cat}_\Delta$ in the obvious way. We can now define a functor (the "homotopy coherent nerve") $\mathcal{N}:\mathbf{Cat_\Delta}\rightarrow \mathbf{Set}^{\Delta^{op}}$ as follows: for $\mathcal{C}$ a simplicially enriched category, let the $n$-simplices of $\mathcal{N}(\mathcal{C})$ be the set $\text{Hom}_{\mathbf{Cat_\Delta}}(\mathfrak{C}[n], \mathcal{C})$. Face and degeneracy maps are given by post-composition with the co-face and co-degeneracy morphisms on the $\mathfrak{C}[n]$ in $\mathbf{Cat_\Delta}$, and $\mathcal{N}(f)$ is given by pre-composition with $f$ for any morphism $f$ in $\mathbf{Cat_\Delta}$.
This functor is not too difficult to wrap one's head around; essentially the $n$-simplices of $\mathcal{N}(\mathcal{C})$ can be thought of as "homotopy-coherent diagrams" in $\mathcal{C}$. For instance, $0$-simplices are simply objects of $\mathcal{C}$, and $1$-simplices are vertices of the mapping spaces of $\mathcal{C}$ (ie "morphisms" of $\mathcal{C}$). $2$-simplices are specified by a trio $x, y, z$ of objects of $\mathcal{C}$, a pair of vertices (ie "morphisms") $f\in \text{Map}_\mathcal{C}(x, y)_0$ and $g\in \text{Map}_\mathcal{C}(y, z)_0$, and a 1-simplex $\sigma\in \text{Map}_\mathcal{C}(x, z)_1$ with one of its faces the composition $g\circ f$. $\sigma$ can be thought of as giving a "homotopy" from $g\circ f$ to some other morphism in $\text{Map}_\mathcal{C}(x, z)$. This idea generalizes naturally for higher values of $n$.
The issue for me now is in defining a left adjoint to $\mathcal{N}$. This is done is a purely formal way by extending the functor $\mathfrak{C}:\Delta\rightarrow\mathbf{Cat}_\Delta$. Indeed, every simplicial set $S$ is a colimit of a small diagram of $\Delta^i$s in $\mathbf{Set}^{\Delta^{op}}$, and the category $\mathbf{Cat}_\Delta$ admits small colimits, so define $\mathfrak{C}:\mathbf{Set}^{\Delta^{op}}\rightarrow\mathbf{Cat}_\Delta$ by letting $\mathfrak{C}(S)$ be the colimit of the corresponding diagram of $\mathfrak{C}[i]$s. It is immediate by construction that $\mathfrak{C}$ and $\mathcal{N}$ will be adjoint.
While I understand this on a formal level, I am having an enormous amount of difficult visualizing what the functor $\mathfrak{C}$ actually "looks like". In particular I am not able to follow computations involving $\mathfrak{C}$ at all. Does anyone know of a way of better understanding this functor on an intuitive level? I've tried to give a more explicit construction of a general $\mathfrak{C}(S)$ but have thus far failed; clearly it should have as its objects the vertices of $S$, but I have come up short in trying to describe its mapping spaces and composition laws. I know that morally $\mathfrak{C}$ should be the "free simplicial category generated by $S$", but it is really unclear to me how this should look. Any insight or references would be greatly appreciated, thank you so much.
(Also was not sure whether to post this here or on math.stackexchange; if it's better suited for the latter site feel free to close and I will repost there.)
Edit: in light of the comments below perhaps the right question to ask is this; what are some simplicial sets $S$ for which it will be particularly illuminating to compute $\mathfrak{C}(S)$ explicitly? (Rather than trying to do a general computation.)
REPLY [2 votes]: To a fairly crude approximation: $\newcommand{\C}{\mathfrak{C}}$think of the functor $\C(-)$ as like geometric realisation, but realising the basic simplices as the categories $\C[n]$ instead of as the topological simplices. Lots of functors out of $\hat{\Delta}$ are defined analogously by left Kan extension of some functor on $\Delta$, and I find thinking of such functors as “roughly like geometric realisation” is always a good first approximation.
Of course, this depends on first understanding the categories $\C[n]$ — but you should be already described how to do that, since you describe the simplices appearing in $\mathcal{N}(-)$ as “not too difficult to wrap one’s head around […] homotopy coherent diagrams”, and the categories $\C[n]$ are just the categories that represent such diagrams, i.e. $\C[n]$ consists of essentially just a “homotopy coherent $n$-diagram” and nothing more.
The other thing that can be helpful for concrete computations is noticing that the functor $\C(-)$ preserves cofibrations — in particular, it maps the the boundary inclusions of simplicial sets to injective-on-objects functors, and colimits of such functors are fairly well-behaved in $\mathrm{Cat}$. So given a small combinatorial simplical sets, write it out explicitly as a “cell complex”, i.e. a composite of pushouts of the boundary inclusions; then the realisation functor will take this to a corresponding “cell complex” in $\mathrm{Cat}$, which should typically give a clear combinatorial description of the realisation.<|endoftext|>
TITLE: Reason to apply the Koszul sign rule everywhere in graded contexts
QUESTION [5 upvotes]: The Koszul sign rule is a sign rule that arises from graded-commutative algebras. For instance, let $\bigwedge(x_1,\dots, x_n)$ be the free graded-commutative algebra generated by $n$ elements of respective degrees $\lvert x_i\rvert$. Then, the sign $\varepsilon(\sigma)$ of a permutation $\sigma$ on $(x_1,\dotsc, x_n)$ is given by
$$x_1\wedge\dotsb\wedge x_n=\varepsilon(\sigma)x_{\sigma(1)}\wedge\dotsb\wedge x_{\sigma(n)},$$
which comes from the fact that in a graded-commutative algebra one has by definition $a\wedge b = (-1)^{\lvert a\rvert\lvert b\rvert}b\wedge a$.
There is also an antisymmetric Koszul sign rule which arises from graded-anticommutative algebras and it's just the previous sign times the sign of the permutation. Both signs are used for instance in Lada and Markl - Symmetric brace algebras.
However, I've been seeing the Koszul sign rule used in any graded context and even for operations that are not products in some algebra. For example, from Roitzheim and Whitehouse - Uniqueness of $A_\infty$-structures and Hochschild cohomology, given graded maps of graded algebras $f,g:A\to B$, if we want to evaluate $f\otimes g$ in an element $x\otimes y$, apparently we need to apply the sign rule to get
$$(f\otimes g)(x\otimes y)=(-1)^{\lvert x\rvert\lvert g\rvert}f(x)\otimes g(y),$$
but I see no mathematical reason to do that, it just seems to be a convention.
A more complex example of application of the Koszul sign rule is in the definition of brace algebra (also in the Lada and Markl paper).
I could give many more examples. In some of them I can understand the reason. For instance, the differential of a tensor product of complexes $C$ and $D$ cannot simply be $d_C\otimes 1_D+ 1_C\otimes d_D$ (it can be defined this way if we use the sign rule when we apply it to elements, but in any case it needs the sign). But maps in general need not be differentials. In other cases, the signs appear in nature and one use this sign rule to justify them, as in $A_{\infty}$-algebras, but this feels too artificial for me and doesn't really explain why we should use that sign rule.
So, in the end, every time there is a sequence $(x_1,\dotsc, x_n)$ of graded objects of any kind and not necessarily all of them of the same kind (elements, maps, operations, …), and related in any way (they can be multiplied, or applied, or whatever), we use the Koszul sign rule to permute the sequence.
To me all of this seems more philosophical than mathematical, and as I said it feels to be just a convention. But, is there some general mathematical reason to use the sign rule in such an extensive way? And if it's just a convention, why should we use it? From my experience, it gets very messy when it comes to applying the sign rule to larger formulas, and in the end everything is just a $+$ or $-$ sign, so I see no advantage.
REPLY [16 votes]: A precise statement of the conventions (which Jesse is referring to) is that the authors are using the symmetric monoidal structure on graded vector spaces, where the braiding map,, $\tau: V \otimes W \to W \otimes V$, is given by $$v \otimes w \mapsto (-1)^{{\rm deg} ~w ~{\rm deg}~ v} w \otimes v.$$
Roughly, what it means to use this symmetric monoidal structure is that you have to make all of your definitions diagramatically, using only $\tau$ to exchange symbols.
For instance suppose we have two algebras $A, B$ and an $A$ module $M$ and a $B$ module $N$. Then if $A,B,M,N$ were ordinary vector spaces, we are used to the fact that $M \otimes N$ is an $A \otimes B$ module. In the graded context, under the Koszul conventions, we define the action $$A \otimes B \otimes M \otimes N \to A \otimes M \otimes B \otimes N \to M \otimes N,$$ where in the first step we have used $1 \otimes \tau \otimes 1.$ Something quite similar is happening in the example your mention.
So far, this is more of a unified answer to how rather than why people use this convention.
For the question of why, the main reason the Koszul convention is useful in homological algebra has to do with the origin of homological algebra---topology.
Consider $\mathbb R^{p +q}$ with its standard orientation. Then the switching map $$\tau: \mathbb R^p \times \mathbb R^q \to \mathbb R^{q} \times \mathbb R^p$$ multiplies this orientation by $(-1)^{p q}$. This fundamental fact manifests itself in several ways.
One is that homology functor $H_*(-, k)$ from topological spaces to graded vector spaces is symmetric monoidal, but only with respect to the Koszul sign rule. This means that if one has an algebraic structure on a topological space $X$, then $H_*(X)$ naturally carries the same algebraic structure, but with respect to the Koszul sign rule. For instance, $X$ is always a co-commutative coalgebra, so $H_*(X)$ becomes a graded co-commutative coalgebra (with sign conventions according to the Koszul rule).
Something similar happens with the $A_\infty$ operad. Namely, the $A_\infty$ operad is the $dg$ operad obtained by taking the cellular homology of a (cellular) operad in topological spaces. The orientations of the cells of this operad explain the signs which arise.
There is also the monoidal Dold Kan correspondence, which you can read about on the nLab.
At the end of the day, it is just a convention (and not always the right one) but the relation with topology explains why people like to use it systematically.<|endoftext|>
TITLE: Shrinking and stretching of vector bundles
QUESTION [5 upvotes]: Let $M$ be a manifold, $p:E\to M$ a rank $d$ vector bundle. Suppose that $U \subset E$ is an open subset such that $U \cap p^{-1}(x)$ is nonempty and convex for all $x \in M$. Is it true that $U \to M$ is a fiber bundle with fiber $\mathbb R^d$? And that $U \cong E$ as fiber bundles? We may assume with no loss of generality that $U$ contains the zero section.
This seems like a statement that could be a lemma in any number of textbooks (if true), e.g. in connection with the tubular neighborhood theorem, but I haven't seen it anywhere. Lang proves in his differential geometry book that any vector bundle over a manifold is what he calls compressible, meaning that any open neighborhood of the zero section of $E$ can be shrunk to a smaller open neighborhood which is diffeomorphic to $E$ as a bundle over $M$.
REPLY [3 votes]: Since there are no references so far, let me give a sketch proof along the lines of my comment. I'll assume that $M$ is compact.
Let's show first that there is a smooth section of $E$ lying in $U$. Indeed, for any point $x\in M$ there is a neighbourhood $U_x$ with a section $s_x$. Take a finite cover $U_i$ of $M$ by such neighbourhoods and take the corresponding partition $1=\sum f_i$ of unity. Then by convexity $\sum s_i f_i$ is a smooth section lying in $U$.
Clearly we can assume that $s$ is the zero section (by taking an appropriate fiberwise diffeo), we will assume this from now on.
Now we will construct an exhaustion of $U$ by an increasing sequence of fiber-wise compact convex subsets $0\subset {\cal B_1}\subset ... \subset {\cal B_i}\subset ...$ so that $U=\cup_i {\cal B_i}$.
Let me show first how to construct one such subset ${\cal B_1}\subset U$.
For every point $p\in M$ let us choose some covex compact subset $B_p$ with smooth boundary in the fiber $U_p$. Then, since $U$ is open, there is an open neighbourhood $V_p$ of $p$ in $M$ such that over this neighbourhood there is a smoothly varying family of $B_x$ ($x\in V_p$), such that $B_x\subset U_x$. Take a finite cover of $M$ by such $V_i's$, let $\phi_i$ be the partition of unity. Then the sum
$${\cal B_1}=\sum_i \phi_i B_i (x)$$
is the desired subset $B\subset U$. Here by sum I mean the Minkowski sum.
It is clear that the interior of $\cal B_1$ is diffeomorphic to the bundle of vectors of length less than $1$ in $E$ (for some fiber-wise Euclidean metric). So the only need to construct a family of $\cal B_i$ that will exhaust $U$. This can be done as in 1).<|endoftext|>
TITLE: Concerning $k \subset L \subset k(x,y)$
QUESTION [9 upvotes]: The following is a known result in algebraic geometry:
Let $k$ be an algebraically closed field of characteristic zero (for example, $k=\mathbb{C}$).
Let $L$ be a field such that $k \subset L \subset k(x,y)$
and $L$ is of transcendence degree two over $k$.
Then there exist $h_1,h_2 \in k(x,y)$ such that $L=k(h_1,h_2)$.
Is it possible to find $g_1,g_2 \in k[x,y]$ such that $L=k(g_1,g_2)$?
The motivation is the following result: If $k \subset L \subset k(x,y)$ is of transcendence
degree one over $k$, then $L=k(h)$, where $h \in k[x,y]$; see this answer. Perhaps the arguments in that answer are also applicable here?
I have asked the above question in MSE, with no comments. See also this question.
Thank you very much!
REPLY [5 votes]: No. $M\mathrel{:=}k(x,y)$ has a $k$-automorphism $\sigma:x\mapsto 1/x,\,y\mapsto 1/y$, of order 2. Let $G\mathrel{:=}\langle\sigma\rangle$, and put $L\mathrel{:=}M^{G}$, the fixed field. The elements $x+1/x$ and $y+1/y$ of $L$ are algebraically independent over $k$, hence $L$ has transcendency degree 2. However, no non-constant polynomial $g\in k[x,y]$ can be in $L$ (that is, be invariant under $\sigma$).
By the same token, the automorphism $\tau:x\mapsto 1/y,\,y\mapsto 1/x$, fixes the field $k(x/y)$, which has transcendency degree 1 over $k$, but it cannot fix any non-constant polynomial. So $k(x/y)\neq k(h)$ for every $h\in k[x,y]$. (Here, $M$ is again quadratic over the fixed field of $\tau$, so that the latter is of tr. deg. 2 over $k$ again. A transcendental element independent of $x/y$ is $x+1/y$, for instance).<|endoftext|>
TITLE: About the number of primes which are the sum of 3 consecutive primes (OEIS A034962)
QUESTION [13 upvotes]: I made some numerical simulations about the number of primes which are the sum of 3 consecutive primes (OEIS A034962), that is for instance:
$$5+7+11=23$$
$$7+11+13=31$$
$$11+13+17=41$$
$$17+19+23=59$$
$$19+23+29=71$$
$$23+29+31=83$$
$$29+31+37=97$$
$$...$$
The number of such triplets, till a certain integer n, seems to be well approximated by the following function:
$$\frac{e\cdot\pi(n)}{\log(n)}$$
If the previous was true, the density of such primes in the whole set
of prime numbers would be comparable to that of primes inside the set
of naturals.
I ask if this estimate is correct and some references about this matter.
Thanks.
REPLY [24 votes]: The question asks how many primes $p_n \le x$ are there such that $p_n + p_{n+1}+p_{n+2}$ is also prime. This is beyond our reach to answer, but one can use Hardy-Littlewood type heuristics to attack this. Since $p_n + p_{n+1}+ p_{n+2}$ is roughly of size $x$, it has about $1/\log x$ chance of being prime, and so one should expect the answer to be on the scale $\pi(x)/\log x \approx x/(\log x)^2$. But the asymptotic will be a little different, since $p_n + p_{n+1} +p_{n+2}$ is not quite random -- for example it will always be odd (omitting $2+3+5$).
Let's flesh out the usual heuristic. To be prime, a number must be coprime to each prime number $\ell$. A random integer has a chance $(1-1/\ell)$ of being coprime to $\ell$. What is the chance that $p_n+ p_{n+1} + p_{n+2}$ is coprime to $\ell$? Each of the primes $p_n$, $p_{n+1}$, $p_{n+2}$ itself lies in some reduced residue class $\mod \ell$. Assuming these possibilities are equally distributed, one should get the chance
$$
\frac{1}{(\ell-1)^3} \sum_{\substack{a, b, c=1 \\ (a+b+c,\ell)=1}}^{\ell-1} 1.
$$
If $a$ and $b$ are given with $a\not\equiv -b \mod \ell$, then $c$ has $\ell-2$ possible residue classes, and if $a\equiv -b \mod \ell$ then $c$ has $\ell-1$ possible residue classes. So the answer is
$$
\frac{1}{(\ell-1)^3} \Big( (\ell-1)(\ell-2)(\ell-2) + (\ell-1)(\ell-1)\Big)=
\frac{\ell^2-3\ell+3}{(\ell-1)^2}.
$$
So we adjust the random probability at $\ell$ by the factor
$$
\Big(1-\frac {1}{\ell}\Big)^{-1} \frac{(\ell^2-3\ell+3)}{(\ell-1)^2}= 1 +\frac{1}{(\ell-1)^3}.
$$
For example, when $\ell=2$, this adjustment factor is $2$ reflecting the fact that $p_n+p_{n+1}+p_{n+2}$ is guaranteed to be odd.
Then the analog of the Hardy--Littlewood conjecture will be
$$
\sim \prod_{\ell} \Big(1+\frac{1}{(\ell-1)^3} \Big) \frac{x}{(\log x)^2}.
$$
The constant in the product is approximately $2.3$; definitely not $e$.
There are interesting biases concerning why this conjecture would be an underestimate. Work of Lemke Oliver and Soundararajan makes precise conjectures on the distribution of consecutive primes in arithmetic progressions. For example, if we look at $\ell=3$, the sum $p_n + p_{n+1} +p_{n+2}$ can be a multiple of $3$ only if all three primes are congruent to each other $\mod 3$. This configuration is not preferred, and there are significant biases against it for small numbers! Similarly for other $\ell$ also, there are biases that indicate that the actual probability would be a tiny bit larger (with the effect wearing off slowly for large $x$). In other words, one might be able to identify lower order terms on the scale of $x(\log \log x)/(\log x)^3$, which might explain why the numerics suggest a larger value for the constant.<|endoftext|>
TITLE: Relevant mathematics to the recent coronavirus outbreak
QUESTION [16 upvotes]: I would like to ask about (old* and new) reliable mathematical literature relevant to various mathematical aspects of the recent coronavirus outbreak: In particular, standard statistical/mathematical models that are used to predict the spread, mathematical studies of effectiveness of various strategies, etc.
*(Added) By old I also mean well-established models.
REPLY [5 votes]: The following paper is extremely important because it has informed the decisions of the UK government that realised (announced) on Monday 16/03/2020 that it can not afford "Herd immunity". The paper only shows the outcomes of the model and speaks about its parameters. It would of course be extremely interesting to know what exactly is the mathematics behind it. Mathematicians should try to read it.
https://www.imperial.ac.uk/media/imperial-college/medicine/sph/ide/gida-fellowships/Imperial-College-COVID19-NPI-modelling-16-03-2020.pdf?fbclid=IwAR2Ca5Ki23DWn-EGWeB3yaNE4f9GmnUcEWU_S60lsDC230AKUg4v_w82qeE<|endoftext|>
TITLE: Which compact metrizable spaces have continuous choice functions for non-empty closed sets?
QUESTION [9 upvotes]: Let $X$ be a compact metrizable space and let $\mathcal{K}_{ne}(X)$ be the collection of non-empty closed subsets of $X$ with the Vietoris topology (i.e. the topology induced by the Hausdorff metric for any compatible metric on $X$).
Question: When does there exist a continuous function $f: \mathcal{K}_{ne}(X) \rightarrow X$ such that for every $G \in \mathcal{K}_{ne}(X)$, $f(G) \in G$?
This feels like it should have been studied before, but I am unable to find a reference.
Some easy observations:
If $X$ has a continuous choice function for non-empty closed sets and $Y$ is a closed subspace of $X$, then $Y$ has a continuous choice function for non-empty closed sets.
$\inf : \mathcal{K}_{ne}([0,1])\rightarrow [0,1]$ is a continuous choice function for non-empty closed subsets of $[0,1]$. So we also have this for any closed subspace of $[0,1]$, such as Cantor space and any countable compact metrizable space.
The circle and the tripod (three copies of $[0,1]$ glued together at $0$) both do not have continuous choice functions for non-empty closed sets (in both spaces given a set with two points there is a continuous path that makes the points switch places while keeping them separate). So no spaces in which these embed do either.
Any finite disjoint union of spaces with continuous choice functions for non-empty closed sets also has a continuous choice function for non-empty closed sets (having elements in a clopen subset is a clopen condition in $\mathcal{K}_{ne}(X)$, so we can patch together the choice functions by cases).
A reasonable conjecture is that any such space embeds into $[0,1]$, but I could also see something tricky like the pseudo-arc having a continuous choice function for non-empty closed sets.
REPLY [6 votes]: It's an old (1981) theorem by Jan van Mill and Evert Wattel (see this paper) that a compact space has a continuous selection iff it is orderable. (So has a linear order whose order topology is the topology on $X$). $F \to \min(F)$ and $F \to \max(F)$ are then the two only continuous selection functions IIRC. Even a continuous selecting function for $[X]^2$, the subspace of doubletons, is enough to get orderabilility.<|endoftext|>
TITLE: Solution to differential equation $f^2(x) f''(x) = -x$ on [0,1]
QUESTION [14 upvotes]: I'd like to solve a differential equation $$ f^2(x) f''(x)=-x $$ where $f(x)$ is defined on $[0,1]$ and has a boundary condition $f(0)=f(1)=0$.
I somehow found out that the solution is fairly close to $f(x) = x^{1/3} \phi^{2/3}(\Phi^{-1}(1-x))$ where $\phi$ and $\Phi$ are pdf and cdf of a standard normal distribution, but it fails to solve the differential equation exactly.
Thank for all comments!
Based on the solution structure of Emden–Fowler Equation, I was able to identify the values of constants that satisfy the boundary conditions. The followings are the details:
Define
\begin{equation}
Z_R(\tau) \triangleq \sqrt{3} J_{1/3}(\tau) - Y_{1/3}(\tau)
, \quad
Z_L(\tau) \triangleq - \frac{2}{\pi} K_{1/3}(\tau)
\end{equation}
where $J, Y, K$ are Bessel functions.
Further define
\begin{equation}
\bar{\tau} \triangleq \inf\{ \tau > 0; Z_R(\tau) = 0 \} \approx 2.3834
, \quad
a \triangleq \frac{1}{ \bar{\tau}^{4/3} Z_R'(\bar{\tau})^2 } \approx 0.2910
, \quad
b \triangleq a \left( \frac{9}{2} \right)^{1/3} \approx 0.1763.
\end{equation}
Then, the solution curve $\{ (x, f(x)) \}_{x \in [0,1]}$ is characterized by
\begin{equation}
\left\{ \left( x_R(\tau), y_R(\tau) \right) \right\}_{\tau \in [0, \bar{\tau}]} \bigcup \left\{ \left( x_L(\tau), y_L(\tau) \right) \right\}_{\tau \in [0, \infty]}
\end{equation}
where
\begin{equation}
x_R(\tau) \triangleq a \tau^{-2/3}\left[ \left( \tau Z_R'(\tau) + \frac{1}{3} Z_R(\tau) \right)^2 + \tau^2 Z_R(\tau)^2 \right]
, \quad
y_R(\tau) \triangleq b \tau^{2/3} Z_R(\tau)^2.
\end{equation}
\begin{equation}
x_L(\tau) \triangleq a \tau^{-2/3}\left[ \left( \tau Z_L'(\tau) + \frac{1}{3} Z_L(\tau) \right)^2 - \tau^2 Z_L(\tau)^2 \right]
, \quad
y_L(\tau) \triangleq b \tau^{2/3} Z_L(\tau)^2.
\end{equation}
In addition to this analytic solution, I also obtained a numerical solution by repeatedly computing
$$ f_{k+1}(x) \gets \left[ \left( f_k(x-2h) + f_k(x+2h) \right) + 4 \left(f_k(x-h)+f_k(x+h)\right) + \frac{8 x h^2}{f_k^2(x)} \right] \big/ 10 $$
on the grid $x \in \{2h,3h,\ldots,1-3h,1-2h\}$ for small $h$ with an initialization $f_0(x) \triangleq 0.5(1-(1-2x)^2)$.
The following figure shows these solutions:
REPLY [24 votes]: Surprisingly, this case of the Emden-Fowler equation is explicitly solvable:
see formula (2.3.27) in A. Polyanin and V. Zaitsev, Handbook of exact solutions
of ordinary differential equations, Chapman & Hill, 2003.
I copy the formula, without verifying it. Let
$$Z=C_1J_{1/3}(\tau)+C_2Y_{1/3}(\tau),$$
or
$$Z=C_1I_{1/3}(\tau)+C_2K_{1/3}(\tau),$$
where $J,Y$ are Bessel and $I$, $K$ are modified Bessel functions.
Then
$$x=a\tau^{-2/3}[(\tau Z^\prime+(1/3)Z)^2\pm\tau^2Z^2],\quad y=b\tau^{2/3}Z^2$$
satisfy $d^2y/dx^2=Axy^{-2}$ with $A=-(9/2)(b/a)^3.$
For the $+$ sign in $\pm$ take the first formula for $Z$, and for the $-$ the second one.
Remark. Emden-Fowler equation appears for the first time in the famous book by R. Emden, Gaskugeln (1907) and since then frequently arises in the study of stars and black holes.<|endoftext|>
TITLE: Does there exist a non-zero signed finite borel measure which is zero on all balls?
QUESTION [12 upvotes]: Let $(X,d)$ be a compact separable metric space. Let $\mu$ be a Borel, regular, finite, signed measure on $X$ such that for all $x\in X$, for all $r>0$, $\mu(B(x,r))=0$, where $B$ denotes the (either open or closed) ball w.r.t $d$.
Is $\mu$ zero?
If $\mu$ is positive one can show that $\mu=0$ using the Borel-Lebesgue theorem, but what if $\mu$ is signed?
REPLY [11 votes]: Stealing an answer from "user940" on Math.SE, the answer is yes, such measures can exist. In the paper
Davies, Roy O., Measures not approximable or not specifiable by means of balls, Mathematika, Lond. 18, 157-160 (1971). ZBL0229.28005.
the author constructs a compact metric space $X$ and two distinct Borel probability measures $\mu_1, \mu_2$ that agree on every closed ball. (They must therefore also agree on open balls, because an open ball is a countable increasing union of closed balls.) Taking $\mu = \mu_1 - \mu_2$ provides your desired signed measure.<|endoftext|>
TITLE: Algebraic operation corresponding to "taking residues at roots of unity"
QUESTION [7 upvotes]: I'm not sure if this is more appropriate for MO or MSE, this is a question that came up in actual research, but it's of a somewhat elementary nature.
Let $R$ be the ring of rational functions with poles at roots of unity, is there a sufficiently nice way to algebraically describe the operation on $R$ of "taking the sum of residues at roots of unity of $f(x)dx/x$? This operation takes $\frac{x^a}{x^m-1}$ to $1$ if $a$ is a multiple of $m$ and 0 otherwise, and this determines it on the entirety of $R$. By "sufficiently nice", I mean I'd like to be able to describe it purely algebraically, and hopefully without making reference to complex numbers.
What I envisioned was something like "taking the algebraic residue at $x^m-1$", where $m$ is the lcm of the orders of the roots of unity in the denominator of $f$. However this operation is not well defined, as to express an arbitrary polynomial as a series in $x^m-1$ I need to to take $m$th roots. For example, consider the case of $x/x^m-1$, after renaming $x^m-1$ to $\alpha$, the result becomes $Res_{\alpha}\frac{(\alpha+1)^{1/m}}{\alpha}\frac{d\alpha}{\alpha+1}$.
I still get the correct answer if I take the appropriate residues at each branch and sum them (resulting in 0), but this still requires complex analysis. I could alternatively just declare such functions have 0 residue, but I don't have any justification for doing that.
EDIT: Because of the particulars of my situation, I'm specifically looking for an interpretation somehow in terms of the functions $x^\alpha-1$.
REPLY [6 votes]: The sum of the (finite) residues of a rational function $F(x)$ is equal to
$$ -\mathrm{Res}(F(x),x=\infty) = \mathrm{Res}\left( \frac{1}{x^2}F(1/x), x=0\right). $$
This is just the coefficient of $x$ in the Laurent expansion with finitely many negative exponents of $F(1/x)$ at $x=0$.